Sie sind auf Seite 1von 279

IMMOVABLE AND MOVABLE PROPERTY (ART.

415-418)

JRPA
Lopez v. Orosa
G.R. Nos. L-10817-18, 103 SCRA 98

DOCTRINE: For while it is true that generally, real estate connotes the land and the building
constructed thereon, it is obvious that the inclusion of the building in the enumeration of what
may constitute real properties could only mean one thingthat a building is by itself an
immovable property

FACTS:
Lopez was engaged in business under the name Lopez-Castelo Sawmill. Orosa, who lived in
the same province as Lopez, one day approached Lopez and invited the latter to make an
investment in the theatre business. Orosa, his family and close friends apparently were forming
a corporation named Plaza Theatre. Lopez expressed his unwillingness to invest. Nonetheless,
there was an oral agreement between Lopez and Orosa that Lopez would be supplying the
lumber for the construction of the theatre. The terms were the following: one, Orosa would be
personally liable for any account that the said construction would incur; two, payment would be
by demand and not by cash on delivery.

Pursuant to the agreement, Lopez delivered the lumber for the construction. Lopez was only
paid one-third of the total cost. The land on which the building has been erected was previously
owned by Orosa, which was later on purchased by the corporation. Due to the incessant
demands of Lopez, the corporation mortgaged its properties. On an earlier relevant date, the
corporation obtained a loan with Luzon Surety Company as surety and in turn, the corporation
executed a mortgage over the land and building. In the registration of the land under Act 496,
such mortgage wasnt revealed. Also due to the demands of Lopez, Orosa issued a deed of
assignment over his shares of stock in the corporation. As there was still an unpaid balance,
Lopez filed a case against Orosa and Plaza theatre. He asked that Orosa and Plaza theatre be
held liable solidarily for the unpaid balance; and in case defendants failed to pay, the land and
building should be sold in public auction with the proceeds to be applied to the balance; or that
the shares of stock be sold in public auction. Lopez also had lis pendens be annotated in the
OCT. The trial court decided that there was joint liability between defendants and that the
material mans lien was only confined to the building.

ISSUE:
W/N the material mens lien for the value of the materials used in the construction of the building
attaches to said structure alone and doesnt extend to the land on which the building is adhered
to?

HELD:
The contention that the lien executed in favor of the furnisher of materials used for the
construction and repair of a building is also extended to land on which the building was
constructed is without merit. For while it is true that generally, real estate connotes the land and
the building constructed thereon, it is obvious that the inclusion of the building in the
enumeration of what may constitute real properties could only mean one thingthat a building
is by itself an immovable property. Moreover, in the absence of any specific provision to the
contrary, a building is an immovable property irrespective of whether or not said structure and
the land on which it is adhered to belong to the same owner.
Appellant invoked Article 1923 of the Spanish Civil Code, which providesWith respect to
determinate real property and real rights of the debtor, the following are preferred: xxx Credits
for reflection, not entered or recorded, and only with respect to other credits different from those
mentioned in four next preceding paragraphs. Close examination of the abovementioned
provision reveals that the law gives preference to unregistered refectionary credits only with
respect to the real estate upon which the refectionary or work was made. This being so, the
inevitable conclusion must be that the lien so created attaches merely to the immovable
property for the construction or repair of which the obligation was incurred. Therefore, the lien in
favor of appellant for the unpaid value of the lumber used in the construction of the building
attaches only to said structure and to no other property of the obligors.

ABB
Associated Insurance and Surety Company v. Iya
103 SCRA 972

DOCTRINE: A building is an immovable property irrespective of where or not said structure and
the land on which it is adhered to belong to the same owner.

FACTS:
Spouses Adriano Valino and Lucia A. Valino own a house of strong materials. They filed a bond
of P 11,000.00 subscribed by the Associated Insurance and Surety Co., Inc. and as a counter-
guaranty, the spouses Valino executed an alleged chattel mortgage on the aforementioned
house in favor of the surety company.

The parcel of land on which the house is erected was still registered in the name of the
Philippine Realty Corporation but was able to obtain the same from them after full payment of
the purchase price. The Valinos acquired another loan from Isabel Iya for P12,000.00, executing
a real estate mortgage over the house and lot. However, they were unable to pay off their other
loan which caused the foreclosure of the chattel mortgage. The surety company was awarded
the land as the highest bidder in the auction but later on discovered that the land was subject to
a real estate mortgage. The surety company then requested that the house and lot be excluded
from the real estate mortgage. Iya, in her answer, said that she had a real right over the
property and that the chattel mortgage on which the foreclosure was based should be declared
null and void for non-compliance with the form required by law. The CA ruled that the
foreclosure of the real estate mortgage is limited to the land alone and they awarded the
structure to the surety company saying that the house is a personal property and may be
subject to chattel mortgage.

ISSUE:
Which of the mortgages should have preference?

HELD:
It was held in Lopez vs. Orosa that the building is an immovable itself, separate and distinct
from the land. A building is an immovable property irrespective of whether or not said structure
and the land on which it is adhered to belong to the same owner.

Only personal properties can be the subject of a chattel mortgage and since the structure in this
case is an immovable, it cannot subject to a chattel mortgage. Therefore the chattel mortgage
and the sale on which it was based should be declared null and void. Also, while it is true that
said document was registered in the Chattel Mortgage Register of Rizal, this act produced no
effect whatsoever for where the interest conveyed is in the nature of a real property, the
registration of the document in the registry of chattels is merely a futile act which would produce
no legal effect insofar as the building is concerned.

FZC
Bicerra v. Teneza
G.R. No. L-16218, 6 SCRA 648

DOCTRINE: A house is classified as immovable property by reason of its adherence to the soil
on which it is built (Article 415, paragraph 1, Civil Code). This classification holds true
regardless of the fact that the house may be situated on land belonging to a different owner. But
once the house is demolished it ceases to exist, hence its character as an immovable likewise
ceases.

FACTS:
The Bicerras are supposedly the owners of the house (PhP 20,000) built on a lot owned by
them in Lagangilang, Abra which the Tenezas forcibly demolished in January 1957, claiming to
be the owners thereof. The materials of the house were placed in the custody of the barrio
lieutenant. The Bicerras filed a complaint claiming actual damages of P200, moral and
consequential damages amounting to P600, and the costs. The CFI Abra dismissed the
complaint claiming that the action was within the exclusive (original) jurisdiction of the Justice of
the Peace Court of Lagangilang, Abra.

The Supreme Court affirmed the order appealed. Having been admitted in forma pauperis, no
costs were adjudged.

ISSUE:
WON the house is immovable property even if it is on the land of another

HELD:
House is immovable property even if situated on land belonging to a different owner; Exception,
when demolished.

LNAC
Evangelista v. Alto Surety & Insurance Co., Inc.
G.R. No. L-11139

DOCTRINE: Sales on execution affect the public and third persons. The regulation governing
sales on execution are for public officials to follow. The form of proceedings prescribed for each
kind of property is suited to its character, not to the character, which the parties have given to it
or desire to give it. When the rules speak of personal property, property which is ordinarily so
considered is meant; and when real property is spoken of, it means property which is generally
known as real property. The regulations were never intended to suit the consideration that
parties may have privately given to the property levied upon.

FACTS:
Petitioner Santos Evangelista instituted a Civil Case for a sum of money. He obtained a writ of
attachment, which levied upon a house, built by Rivera on a land situated in Manila and leased
to him, by filing copy of said writ and the corresponding notice of attachment with the Office of
the Register of Deeds of Manila. Judgment was rendered in favor of Evangelista, who, bought
the house at public auction held in compliance with the writ of execution issued in said case.
The definite deed of sale was issued to him upon expiration of the period of redemption. When
Evangelista sought to take possession of the house, Rivera refused to surrender it, upon the
ground that he had leased the property to the Alto Surety & Insurance Co., Inc., respondent
herein, and that the latter is now the true owner of said property. It appears that a definite deed
of sale of the same house had been issued to respondent, as the highest bidder at an auction
sale held in compliance with a writ of execution. Hence, Evangelista instituted the present action
against respondent and Rivera, for the purpose of establishing his (Evangelista) title over said
house, securing possession thereof, apart from recovering damages.

In its answer, respondent alleged that it has a better right to the house, because the sale made,
and the definite deed of sale executed, in its favor, on September 29, 1950 and May 10, 1952,
respectively, precede the sale to Evangelista (October 8, 1951) and the definite deed of sale in
his favor (October 22, 1952). Rivera, in effect, joined forces with respondent.

CFI rendered judgment for Evangelista, sentencing Rivera and respondent to deliver the house
in question to petitioner herein and to pay him, jointly and severally from October, 1952, until
said delivery, plus costs.

On appeal taken by respondent, the above decision was reversed by the CA which absolved
Alto Surety from the complaint on account that although the writ of attachment in favor of
Evangelista had been filed with the Register of Deeds of Manila prior to the sale in favor of Alto
Surety, Evangelista did not acquire thereby a preferential lien, the attachment having been
levied as if the house in question were immovable property.

Evangelista now seeks a review by certiorari.

ISSUE:
Whether a house, constructed by the lessee of the land on which it is built, should be dealt with,
for purpose of attachment, as immovable property, or as personal property.

HELD:
Said house is not personal property, much less a debt, credit or other personal property not
capable of manual delivery, but immovable property. As explicitly held, in Laddera vs. Hodges,
"a true building (not merely superimposed on the soil) is immovable or real property, whether it
is erected by the owner of the land or by usufructuary or lessee.

It is true that the parties to a deed of chattel mortgage may agree to consider a house as
personal property for purposes of said contract. However, this view is good only insofar as the
contracting parties are concerned. It is based, partly, upon the principle of estoppel. Neither this
principle, nor said view, is applicable to strangers to said contract. Much less is it in point where
there has been no contract whatsoever, with respect to the status of the house involved, as in
the case at bar.

The rules on execution do not allow, and, we should not interpret them in such a way as to allow,
the special consideration that parties to a contract may have desired to impart to real estate, for
example, as personal property, when they are, not ordinarily so. Sales on execution affect the
public and third persons. The regulation governing sales on execution are for public officials to
follow. The form of proceedings prescribed for each kind of property is suited to its character,
not to the character, which the parties have given to it or desire to give it. When the rules speak
of personal property, property which is ordinarily so considered is meant; and when real
property is spoken of, it means property which is generally known as real property. The
regulations were never intended to suit the consideration that parties may have privately given
to the property levied upon. Enforcement of regulations would be difficult were the convenience
or agreement of private parties to determine or govern the nature of the proceedings.

The mere fact that a house was the subject of the chattel mortgage and was considered as
personal property by the parties does not make said house personal property for purposes of
the notice to be given for its sale of public auction. This ruling is demanded by the need for a
definite, orderly and well defined regulation for official and public guidance and would prevent
confusion and misunderstanding.

The foregoing considerations apply, with equal force, to the conditions for the levy of attachment,
for it similarly affects the public and third persons.

TKDC
Leung Yee v. Strong Machinery Co.
G.R. No. L-11658

DOCTRINE: The mere fact that the parties decided to deal with the building as personal
property does not change its character as real property. Neither the original registry in the
chattel mortgage registry nor the annotation in said registry of the sale of the mortgaged
property had any effect on the building.

FACTS:
Compaia Agricola Filipina bought several rice-cleaning machinery from a machinery company,
Frank L. Strong Machinery Company and executed a chattel mortgage to secure payment of the
purchase price. The deed of mortgage includes the building where the machinery was installed
without any reference to the land on which it stood. Since Compaia Agricola Filipina failed to
pay when due, the mortgaged property was sold by the sheriff and was bought by the
machinery company.

Few weeks later, Compaia Agricola Filipina executed a deed of sale of the land where the
building stood to the machinery company. In effect, the machinery company possessed the
building when the sale took place and continued its possession ever since.

When the chattel mortgage was executed, Compaia Agricola Filipina executed another
mortgage in favor of Yee over the building to pay its debt to the machinery company. Since
Compaia Agricola Filipina failed to pay when due, Yee secured a judgment to levy execution
upon the building and bought the building at the sheriffs sale; Yee secured the sheriffs
certificate of sale and registered it in the land registry.

When the execution was levied upon the building, the machinery company filed with the sheriff a
sworn statement setting up its claim of title and demanding the release of property from the levy.
On the other hand, Yee filed an action to recover possession of the building from the machinery
company. Trial court ruled in favor of the machinery company on the basis of Article 1473 of the
Civil Code; it ruled that the machinery company registered the title to the building prior to the
registration date of Yees certificate.

ISSUE:
Whether or not the nature of property is changed by its registration in the Chattel Mortgage
Registry. -- NO

HELD:
The registry under Article 1473 of the Civil Code refers to registry of real property and the
annotation or inscription of a deed of sale of real property in a chattel mortgage registry cannot
be given the legal effect of an inscription in the registry of real property.

The Chattel Mortgage Law contemplates mortgages of personal property. The sole purpose and
object of the chattel mortgage registry is the registration of personal property mortgages
executed in the manner and form prescribed in the statute.

In this case, the building where the rice-cleaning machinery was installed was real property. The
mere fact that the parties dealt with it as separate and apart from the land on which it stood
does not change its character as real property. Neither the original registry of the building in the
chattel mortgage nor the annotation of sale of the mortgaged property in the registry had any
effect on the buildings nature as immovable property.

AMD
Standard Oil Co. of New York v. Jaramillo
44 SCRA 630

DOCTRINE: The duties of a register of deeds in respect to the registration of chattel mortgage
are of a purely ministerial character; and no provision of law can be cited which confers upon
him any judicial or quasi-judicial power to determine the nature of any document of which
registration is sought as a chattel mortgage.

FACTS:
Gervasia de la Rosa, Vda. de Vera, was the lessee of a parcel of land situated in Manila and
owner of the house built thereon. She executed a chattel mortgage in favor of Standard Oil Co.
to convey both the leasehold interest in said lot and the building. After the document had been
duly acknowledge and delivered, petitioner presented it to the respondent, Joaquin Jaramillo, as
ROD of Manila, to be recorded in the book of record of chattel mortgages. Upon examination of
the instrument, Jaramillo was of the opinion that it was not a chattel mortgage as the interest
mortgaged did not appear to be personal property, within the meaning of the Chattel Mortgage
Law, and registration was refused on this ground only. So, petitioner sought for a peremptory
mandamus to compel the respondent to record the said document in the register. Jaramillo
interposed a demurrer before the SC.

ISSUE:
W/N the ROD can refuse the registration of a Chattel Mortgage? NO

HELD:
It is his duty to accept the proper fee and place the instrument on record. The duties of a
register of deeds in respect to the registration of chattel mortgage are of a purely ministerial
character; and no provision of law can be cited which confers upon him any judicial or quasi-
judicial power to determine the nature of any document of which registration is sought as a
chattel mortgage.

The original provisions touching this matter are contained in section 15 of the Chattel Mortgage
Law (Act No. 1508), as amended by Act No. 2496; but these have been transferred to section
198 of the Administrative Code. There is nothing in any of these provisions conferring upon the
register of deeds any authority whatever in respect to the "qualification, of chattel mortgage. His
duties in respect to such instruments are ministerial only. The efficacy of the act of recording a
chattel mortgage consists in the fact that it operates as constructive notice of the existence of
the contract, and the legal effects of the contract must be discovered in the instrument itself in
relation with the fact of notice. Registration adds nothing to the instrument, considered as a
source of title, and affects nobody's rights except as a specifies of notice.

Articles 334 and 335 of the Civil Code supply no absolute criterion for discriminating between
real property and personal property for purpose of the application of the Chattel Mortgage Law.
Those articles state rules which, considered as a general doctrine, are law in this jurisdiction;
but it must not be forgotten that under given conditions property may have character different
from that imputed to it in said articles. It is undeniable that the parties to a contract may by
agreement treat as personal property that which by nature would be real property; and it is a
familiar phenomenon to see things classed as real property for purposes of taxation which on
general principle might be considered personal property.

CRF
Sibal v. Valdez
G.R. No. L-27532

DOCTRINE: For the purpose of attachment and execution, and for the purposes of the Chattel
Mortgage Law, "ungathered products" have the nature of personal property. (batasnatin)

FACTS:
As a first cause of action the plaintiff alleged that the defendant Vitaliano Mamawal, deputy
sheriff of the Province of Tarlac, by virtue of a writ of execution issued by the Court of First
Instance of Pampanga, attached and sold to the defendant Emiliano J. Valdez the sugar cane
planted by the plaintiff and his tenants on seven parcels of land. That within one year from the
date of the attachment and sale the plaintiff offered to redeem said sugar cane and tendered to
the defendant Valdez the amount sufficient to cover the price paid by the latter, the interest
thereon and any assessments or taxes which he may have paid thereon after the purchase, and
the interest corresponding thereto and that Valdez refused to accept the money and to return
the sugar cane to the plaintiff.

One of the defenses of the defendant Emiliano J. Valdez is that the sugar cane in question had
the nature of personal property and was not, therefore, subject to redemption. The trial court
hold that the sugar cane in question was personal property and, as such, was not subject to
redemption.

ISSUE:
Whether the sugar cane in question is personal or real property under civil code? Under chattel
mortgage law?

HELD:
The court ruled that It is contended that sugar cane comes under the classification of real
property as "ungathered products" in paragraph 2 of article 334 of the Civil Code. Said
paragraph 2 of article 334 enumerates as real property the following: Trees, plants, and
ungathered products, while they are annexed to the land or form an integral part of any
immovable property."

We may, therefore, conclude that paragraph 2 of article 334 of the Civil Code has been modified
by section 450 of the Code of Civil Procedure and by Act No. 1508, in the sense that, for the
purpose of attachment and execution, and for the purposes of the Chattel Mortgage Law,
"ungathered products" have the nature of personal property. The lower court, therefore,
committed no error in holding that the sugar cane in question was personal property and, as
such, was not subject to redemption.

MPF
Tsai v. CA
G.R. No. 120098
DOCTRINE: Even if the properties are immovable by nature, nothing detracts the parties from
treating them as chattels to secure an obligation under the principle of estoppel.

FACTS:
EVERTEX secured a loan from PBC, guaranteed by real estate and chattel
mortgage over a parcel of land where the factory stands, and the chattels located therein, as
included in a schedule attached to the mortgage contract. another loan was obtained
secured by a chattel mortgage over properties with similar descriptions listed in the first
schedule.
During the date of execution of the second mortgage. EVERTEX purchased
machineries and equipment.
Due to business reverses, EVERTEX filed for insolvency proceedings. It failed to
pay its obligation and thus, PBC initiated extrajudicial foreclosure of the mortgages.
PBC was the highest bidder in the public auctions, making it the owner of the
properties. It then leased the factory premises to Tsai.
Afterwards, EVERTEX sought the annulment of the sale and conveyance of the
properties to PBC as it was allegedly a violation of the insolvency law.
The RTC held that the lease and sale were irregular as it involved properties not
included in the schedule of the mortgage contract.

ISSUE:
Whether or not the (immovable) properties in question can be entered into a chattel mortgage. --
YES

HELD:
An immovable may be considered a personal property if there is a stipulation as when it is used
as security in the payment of an obligation where a chattel mortgage is executed over it, as in
the case at bar. While it is true that the controverted properties appear to be immobile, a perusal
of the contract of real estate mortgage and chattel mortgage by the parties gives a contrary
indication. Both the trial and appellate courts show that the intention was to treat the
machineries as movables or personal property.

Assuming that the properties were considered immovables, nothing detracts the parties from
treating it as chattels to secure an obligation under the principle of estoppel.

AMDG
Yap v. Tanada
G.R. No. L-32917

DOCTRINE: The Civil code considers as immovable property among others, anything attached
to an immovable in a fixed manner, in such a way that it cannot be separated therefrom without
breaking the material or deterioration of the object

FACTS:
Goulds Pumps International (Phil.), Inc. (herein Goulds) filed a complaint against Yap and his
wife seeking to recover the balance of the price and installation of the water pump in the latters
residence. The city court declared Yap and his wife in default and rendered a judgment in favor
of Goulds. Yap appealed to the CFI wherein Judge Tanada was residing. Yap was again
declared in default and judgment was again rendered in favor of Goulds. Later on, Judge
Tanada granted Goulds Motion for Issuance of Writ of Execution. Subsequently, the water
pump was sold in a public auction in favor of Goulds being the highest bidder. Another writ of
execution was issued as regards the removal of the water pump and delivery of such to Goulds.
Yap is questioning validity of the auction sale and praying that it be annulled as well as the writ
of execution. Yap is alleging that the water pump is considered as an immovable property
because it is installed in his residence. He also argued that being an immovable property, a
notice must be made before the auction sale pursuant to the Rules of Court. It is noted that

Yap filed several motion of reconsideration to which all were denied.

ISSUE:
Whether or not the water pump is an immovable property -- NO

HELD:
The Civil code considers as immovable property among others, anything attached to an
immovable in a fixed manner, in such a way that it cannot be separated therefrom without
breaking the material or deterioration of the object

The water pump involved in this case does not satisfy the above description. It is highly possible
to remove the water pump without it breaking or deteriorating by simply loosening the bolts or
dismantling the fasteners that were used to attach or install it in his house.

GCG
Mindanao Bus Co. v. City Assessor and Treasurer
G.R. No. L-17870

DOCTRINE: Movable equipment, to be immobilized in contemplation of Article 415 of the Civil
Code, must be the essential and principal elements of an industry or works which are carried on
in a building or on a piece of land. Thus, where the business is one of transportation, which is
carried on without a repair or service shop, and its rolling equipment is repaired or serviced in a
shop belonging to another, the tools and equipment in its repair shop which appear movable are
merely incidentals and may not be considered immovables, and, hence, not subject to
assessment as real estate for purposes of the real estate tax.

FACTS:
Petitioner is a public utility solely engaged in transporting passengers and cargoes by motor
trucks, over its authorized lines in the Island of Mindanao, collecting rates approved by the
Public Service Commission.

The petitioner is the owner of the land where it maintains and operates a garage for its TPU
motor trucks; a repair shop; blacksmith and carpentry shops, and with these machineries which
are placed therein, its TPU trucks are made; body constructed; and same are repaired in a
condition to be serviceable in the TPU land transportation business it operates.

These machineries have never been or were never used as industrial equipments to produce
finished products for sale, nor to repair machineries, parts and the like offered to the general
public indiscriminately for business or commercial purposes for which petitioner has never
engaged in,

The City Assessor of CDO then assessed a P4,400 realty tax on said machineries and repair
equipment. This was then appealed to the Court of Tax Appeals (CTA) who sustained the
respondent city assessor's ruling.

ISSUE:
Whether or not the machineries and the equipments are considered immobilized and thus
subject to a realty tax. -- NO

HELD:
The Supreme Court held a decision for the petition for review to be set aside and the
equipments in question declared not subject to assessment as real estate for the purposes of
the real estate tax.

The law that governs the determination of the question at issue is as follows:

Art. 415. The following are immovable property:

x x x x x x x x x

(5) Machinery, receptacles, instruments or implements intended by the owner of the
tenement for an industry or works which may be carried on in a building or on a piece of
land, and which tend directly to meet the needs of the said industry or works; (Civil Code
of the Phil.)

Aside from the element of essentiality the above-quoted provision also requires that the industry
or works be carried on in a building or on a piece of land. Thus in the case of Berkenkotter vs.
Cu Unjieng, supra, the "machinery, liquid containers, and instruments or implements" are found
in a building constructed on the land. A sawmill would also be installed in a building on land
more or less permanently, and the sawing is conducted in the land or building.

But in the case at bar the equipments in question are destined only to repair or service the
transportation business, which is not carried on in a building or permanently on a piece of land,
as demanded by the law. Said equipments may not, therefore, be deemed real property.

Resuming what we have set forth above, we hold that the equipments in question are not
absolutely essential to the petitioner's transportation business, and petitioner's business is not
carried on in a building, tenement or on a specified land, so said equipment may not be
considered real estate within the meaning of Article 415 (c) of the Civil Code.

Said equipments are not considered immobilized as they are merely incidental, not essential
and principal to the business of the petitioner. The transportation business could be carried on
without repair or service shops of its rolling equipment as they can be repaired or services in
another shop belonging to another

VCL IV
Fels Energy, Inc. v. Province of Batangas, et al.
G.R. No. 168557

DOCTRINE: Article 415 (9) of the New Civil Code provides that docks and structures which,
though floating, are intended by their nature and object to remain at a fixed place on a river, lake,
or coast are considered immovable property. Thus, power barges are categorized as
immovable property by destination, being in the nature of machinery and other implements
intended by the owner for an industry or work which may be carried on in a building or on a
piece of land and which tend directly to meet the needs of said industry or work.

FACTS:
On January 18, 1993, National Power Corporation (NPC) entered into a lease contract with
Polar Energy, Inc. over 330 MW diesel engine power barges moored at Balayan Bay in Calaca,
Batangas. The contract, denominated as an Energy Conversion Agreement, was for a period of
five years. Article 10 states that NPC shall be responsible for the payment of taxes. (other than
(i) taxes imposed or calculated on the basis of the net income of POLAR and Personal Income
Taxes of its employees and (ii) construction permit fees, environmental permit fees and other
similar fees and charges. Polar Energy then assigned its rights under the Agreement to Fels
despite NPCs initial opposition.

FELS received an assessment of real property taxes on the power barges from Provincial
Assessor Lauro C. Andaya of Batangas City. FELS referred the matter to NPC, reminding it of
its obligation under the Agreement to pay all real estate taxes. It then gave NPC the full power
and authority to represent it in any conference regarding the real property assessment of the
Provincial Assessor. NPC filed a petition with the Local Board Assessment Appeals (LBAA).
The LBAA ordered Fels to pay the real estate taxes. The LBAA ruled that the power plant
facilities, while they may be classified as movable or personal property, are nevertheless
considered real property for taxation purposes because they are installed at a specific location
with a character of permanency. The LBAA also pointed out that the owner of the bargesFELS,
a private corporationis the one being taxed, not NPC. A mere agreement making NPC
responsible for the payment of all real estate taxes and assessments will not justify the
exemption of FELS; such a privilege can only be granted to NPC and cannot be extended to
FELS. Finally, the LBAA also ruled that the petition was filed out of time.

Fels appealed to the Central Board Assessment Appeals (CBAA). The CBAA reversed and
ruled that the power barges belong to NPC; since they are actually, directly and exclusively
used by it, the power barges are covered by the exemptions under Section 234(c) of R.A. No.
7160. As to the other jurisdictional issue, the CBAA ruled that prescription did not preclude the
NPC from pursuing its claim for tax exemption in accordance with Section 206 of R.A. No. 7160.
Upon MR, the CBAA reversed itself.

ISSUE: Whether or not barges are considered as real property, thus can be subject to real
property tax -- YES

HELD:
The CBAA and LBAA power barges are real property and are thus subject to real property tax.

In Consolidated Edison Company of New York, Inc., et al. v. The City of New York, et al., a
power company brought an action to review property tax assessment. On the citys motion to
dismiss, the Supreme Court of New York held that the barges on which were mounted gas
turbine power plants designated to generate electrical power, the fuel oil barges which supplied
fuel oil to the power plant barges, and the accessory equipment mounted on the barges were
subject to real property taxation.

Moreover, Article 415 (9) of the New Civil Code provides that docks and structures which,
though floating, are intended by their nature and object to remain at a fixed place on a river, lake,
or coast are considered immovable property. Thus, power barges are categorized as
immovable property by destination, being in the nature of machinery and other implements
intended by the owner for an industry or work which may be carried on in a building or on a
piece of land and which tend directly to meet the needs of said industry or work.

FXRL
Davao Sawmill Co. v. Castillo
G.R. No. 40411,

DOCTRINE: Generally, machinery becomes immobilized when placed by the owner of the plant
or property. This rule does not apply should the machinery be placed by any other person such
as a tenant or usufructuary.

FACTS:
The petitioner company operates a sawmill in barrio Tigatu, Davao.
Said facility contained both movable and immovable property (machines and
other such implements).
However, the land on which it is situated belongs to another person.
The parties executed a lease contract providing that upon the expiration or
termination of such lease, the following shall happen:
o The ownership of all structures and improvements introduced by the petitioner
company shall be transferred to the respondents without any cost or obligation to
pay.
o The machines and their accessories shall not be included in said transfer.
It was noted by the court that in a previous case between the two parties,
judgment was rendered against the petitioner company upon which a writ of execution
was brought against its machines (as personalty) in favor of Castilllo, et al.
Additionally, the records of the current case reflected that the petitioner company
had treated its machinery as personal property by executing chattel mortgages on them
in favor of third persons.
Petitioner company contends that its machines are immovable under the first and
fifth paragraphs of Article 334 (now Article 415) of the Civil Code.

ISSUE:
W/N the machines of the petitioner company are movable or immovable property.

HELD:
The machines are movable.

The court observed that the petitioner company failed to register its protest at the time its
machines were sold. Generally, this inaction would be inconclusive but it is indicative of the
intention impressed upon the property in question.

This is so because while machines are generally movable property, they may nevertheless be
immobilized by destination or purpose subject to several conditions.

This conclusion finds its ground under the fifth paragraph of Article 415. Here, machinery
becomes immobilized when placed by the owner of the plant or property. This rule does not
apply should the machinery be placed by any other person such as a tenant or usufructuary.

Applying the rule to the case on hand, the machinery was placed by the petitioner company who
was merely a lessee. As such, the equipment was never immobilized in the first place.

RSDM
Makati Leasing and Financial Corporation v. Wearever Textile Mills, Inc.
G.R. No. L-58469

DOCTRINE: If a house of strong materials, like what was involved in the above Tumalad case,
may be considered as personal property for purposes of executing a chattel mortgage thereon
as long as the parties to the contract so agree and no innocent third party will be prejudiced
thereby, there is absolutely no reason why a machinery, which is movable in its nature and
becomes immobilized only by destination or purpose, may not be likewise treated as such. This
is really because one who has so agreed is estopped from denying the existence of the chattel
mortgage.

FACTS:
The private respondent Wearever Textile Mills, Inc., discounted and assigned several
receivables with the former under a Receivable Purchase Agreement in order to obtain financial
accommodations from herein petitioner Makati Leasing and Finance Corporation. To secure the
collection of the receivables assigned, private respondent executed a Chattel Mortgage over
certain raw materials inventory as well as a machinery described as an Artos Aero Dryer
Stentering Range.

Upon default, petitioner filed a petition for extrajudicial foreclosure of the properties mortgage to
it. The Deputy Sheriff assigned to implement the foreclosure failed to gain entry into private
respondent's premises and was not able to effect the seizure of the aforedescribed machinery.
Petitioner thereafter filed a complaint for judicial foreclosure with the Court of First Instance of
Rizal.

Acting on petitioner's application for replevin, the lower court issued a writ of seizure, the
enforcement of which was however subsequently restrained upon private respondent's filing of
a motion for reconsideration. After several incidents, the lower court finally issued an order lifting
the restraining order for the enforcement of the writ of seizure and an order to break open the
premises of private respondent to enforce said writ. The lower court reaffirmed its stand upon
private respondent's filing of a further motion for reconsideration.
The Court of Appeals, in certiorari and prohibition proceedings subsequently filed by herein
private respondent, set aside the Orders of the lower court and ordered the return of the drive
motor seized by the sheriff pursuant to said Orders, after ruling that the machinery in suit cannot
be the subject of replevin, much less of a chattel mortgage, because it is a real property
pursuant to Article 415 of the new Civil Code, the same being attached to the ground by means
of bolts and the only way to remove it from respondent's plant would be to drill out or destroy the
concrete floor, the reason why all that the sheriff could do to enfore the writ was to take the main
drive motor of said machinery. The appellate court rejected petitioner's argument that private
respondent is estopped from claiming that the machine is real property by constituting a chattel
mortgage thereon.

ISSUE:
Whether or not the property in suit is real property NO. It is a personal property

HELD:
Examining the records of the instant case, We find no logical justification to exclude the rule out,
as the appellate court did, the present case from the application of the abovequoted
pronouncement. If a house of strong materials, like what was involved in the above Tumalad
case, may be considered as personal property for purposes of executing a chattel mortgage
thereon as long as the parties to the contract so agree and no innocent third party will be
prejudiced thereby, there is absolutely no reason why a machinery, which is movable in its
nature and becomes immobilized only by destination or purpose, may not be likewise treated as
such. This is really because one who has so agreed is estopped from denying the existence of
the chattel mortgage.

In rejecting petitioner's assertion on the applicability of the Tumalad doctrine, the Court of
Appeals lays stress on the fact that the house involved therein was built on a land that did not
belong to the owner of such house. But the law makes no distinction with respect to the
ownership of the land on which the house is built and We should not lay down distinctions not
contemplated by law.

It must be pointed out that the characterization of the subject machinery as chattel by the private
respondent is indicative of intention and impresses upon the property the character determined
by the parties. As stated in Standard Oil Co. of New York v. Jaramillo, 44 Phil. 630, it is
undeniable that the parties to a contract may by agreement treat as personal property that
which by nature would be real property, as long as no interest of third parties would be
prejudiced thereby.

MRAM
Board of Assessment Appeals v. MERALCO
10 SCRA 68


DOCTRINE: The steel towers or poles of MERALCO are not real properties because 1) they are
not adhered to the soil, 2) they are not attached to an immovable property and can be
dismantled without breaking or deteriorating the material and 3) they are not machineries nor
instruments or implements intended for the industry or works on the land

FACTS:
Generated by its hydroelectric plant, MERALCOs electric power is transmitted from Laguna to
Manila through electric transmission wires. These electric transmission wires which carry high
voltage current, are fastened to insulators attached on steel towers. MERALCO has constructed
40 of these steel towers within Quezon City, on land belonging to it.

Three steel towers were the subject of this dispute. When inspected, the findings disclose that
there was no concrete foundation but there was adobe stone underneath. Further, it could not
be ascertained whether said adobe stone was purposely or not.

From this, the City Assessor of Quezon City declared the steel towers subject to real property
tax. MERALCO, however, protested the assessment saying that the steel towers are considered
poles and according to their franchise, it is exempt from taxation

ISSUE:
Whether or not the steel towers or poles of the MERALCO are considered real properties,
hence subject to real property tax?

HELD:
The Supreme Court held in the negative. The Court said that the steel towers are personal
properties. The Court based their ruling on the enumeration of immovable properties in Art. 415
of the Civil Code.

First, the steel towers do not come within the objects mentioned in par. 1, because they do not
constitute buildings or constructions adhered to the soil.Moreover, they are not construction
analogous to buildings nor adhering to the soil because as per description, they are removable
and merely attached to a square metal frame by means of bolts, which when unscrewed could
easily be dismantled and moved from place to place.

Second, they can not be included under paragraph 3 since they are not attached to an
immovable in a fixed manner; they can be separated without breaking the material or causing
deterioration upon the object to which they are attached. In fact, each of these steel towers or
supports consists of steel bars joined together by means of bolts, which can be disassembled
by unscrewing the bolts and reassembled by screwing the same.

Lastly, they do not fall under paragraph 5, as they are not machineries, receptacles, instruments
or implements. SC said that even if they were machineries, receptacles, instruments or
implements, they are not intended for industry or works on the land. MERALCO is not engaged
in an industry or works in the land in which the steel supports or towers are constructed.

FMM
Machinery & Engineering Supplies, Inc. v. CA
G.R. No. L-7057

DOCTRINE: When the machinery and equipment in question appeared to be attached to the
land, particularly to the concrete foundation of said premises, in a fixed manner, in such a way
that the former could not be separated from the latter "without breaking the material or
deterioration of the object or that in order to remove said outfit, it became necessary, not only to
unbolt the same, but , also, to cut some of its wooden supports and when, said machinery and
equipment were "intended by the owner of the tenement for an industry" carried on said
immovable and tended, it becomes immovable property pursuant to paragraphs 3 and 5 of
Article 415 of Civil Code of the Philippines.

FACTS:
On 13 March 1953, Machinery & Engineering Supplies, Inc. (the
Petitioner) filed a complaint for replevin in the Court of First Instance (CFI) of Manila for
the recovery of the machinery and equipment sold and delivered to Ipo Limestone Co., Inc
and Dr. Antonio Villarama (the Respondents) at their factory in Barrio Bigti, Norzagaray,
Bulacan.

Upon application ex-parte of the Petitioner and upon approval of its bond
sum of P15,769.00, herein Respondent Judge issued an order directing the Provincial
Sheriff of Bulacan to seize and take immediate possession of the properties specified in the
said order.

On 19 March 1953, two deputy sheriffs of Bulacan, Ramon S. Roco and a
crew of technician and laborers proceeded to Bigti to carry out the CFIs order.

Leonardo Contreras, herein Respondent Companys Manager met the
sheriffs and handed the latter a letter addressed to Atty. Leopoldo C. Paled, ex-officio
Provincial Sheriff of Bulacan, signed by the Respondent Companys counsel, protesting
against the seizure of the properties on the ground that the same are not personal
properties.

Roco and the deputy sheriffs contended that their duty is ministerial and
went ahead to the factory. At the factory, Roccos attention was called to the fact that the
equipment could not possibly be dismantled without causing damages or injuries to the
wooden frames attached to them but Roco insisted in dismantling the same on his own
responsibility and alleged that the bond was posted for such eventuality. Thus, the deputy
sheriffs directed that some of the machines supports be cut.

On 20 March 1953, the Respondent Company filed an urgent motion, with
a counter-bond in the amount of P15,769 for the return of the properties seized by the
sheriffs. On the same day, the trial court issued an order, directing the Provincial Sheriff of
Bulacan to return the machinery and equipment to the place where they were installed at the
time of seizure.

On 2 March 1953, the deputy sheriffs returned the said properties by
depositing them along the road near the quarry of the Respondent Company, without
inventory and re-installation in its former position and replacing the destroyed posts, which
rendered its use impracticable.

On 23 March 1953, Respondents counsel asked the provincial sheriff if
the machinery and equipment dumped on the road would be re-installed to their former
position and condition. The next day, the provincial sheriff filed an urgent motion in court
manifesting the Roco had been asked to furnish the sheriffs office with the expenses,
laborers, technical men and equipment to carry into effect the courts order, among other
things but that Roco absolutely refused and asking the Court that Respondent Company be
ordered to provide the required aid or relieve the sheriff of the duty of complying to the said
order.

On 30 March 1953, the trial court ordered the provincial sheriff and the
Petitioner Company to reinstate the machinery and equipment removed by them in their
original condition. An urgent motion of the provincial sheriff dated 15 April 1953 requesting
for an extension was denied and on 4 May 1953, the trial court ordered the Petitioner
Company to furnish the provincial sheriff with the necessary funds and technical crew and
laborers to reinstate the machinery and equipment.

The case was appealed before the Court of Appeals but the latter
dismissed the same for lack of merit.

Hence this petition filed before the Supreme Court (the SC). The
Petitioner argued that the respondent judge had completely disregarded his manifestation
that the machinery and equipment seized were and still are the Petitioner's property until
fully paid for and such never became immovable. The question of ownership and the
applicability of Art. 415 of the new Civil Code are immaterial in the determination of the only
issue involved in this case.

ISSUE:
Whether the machineries and equipments can be considered as personal properties subject to
replevin. -- NO

HELD:
The SC held that the special civil action known as replevin, governed by Rule 62 of Court, is
applicable only to "personal property". When the sheriff repaired to the premises of respondent
company, the machinery and equipment in question appeared to be attached to the land,
particularly to the concrete foundation of said premises, in a fixed manner, in such a way that
the former could not be separated from the latter "without breaking the material or deterioration
of the object." Hence, in order to remove said outfit, it became necessary, not only to unbolt the
same, but, also, to cut some of its wooden supports. Moreover, said machinery and equipment
were "intended by the owner of the tenement for an industry" carried on said immovable and
tended." For these reasons, they were already immovable property pursuant to paragraphs 3
and 5 of Article 415 of Civil Code of the Philippines, which are substantially identical to
paragraphs 3 and 5 of Article 334 of the Civil Code of Spain. As such immovable property, they
were not subject to replevin.

RGGM
Punsalan, Jr. v. Vda. De Lacsamana
121 SCRA 331

DOCTRINE: Buildings are always immovable under the Civil Code. Separate treatment by the
parties of building from the land in which it stood does not change the immovable character of
the building.

FACTS:
Punsalan was the owner of a piece of land, which he mortgaged in favor of PNB. Due to his
failure to pay, the mortgage was foreclosed and the land was sold in a public auction to which
PNB was the highest bidder.

On a relevant date, while Punsalan was still the possessor of the land, it secured a permit for
the construction of a warehouse.

A deed of sale was executed between PNB and Punsalan. This contract was amended to
include the warehouse and the improvement thereon. By virtue of these instruments,
respondent Lacsamana secured title over the property in her name.

Petitioner then sought for the annulment of the deed of sale. Among his allegations was that the
bank did not own the building and thus, it should not be included in the said deed.

Petitioners complaint was dismissed for improper venue. The trial court held that the action
being filed in actuality by petitioner is a real action involving his right over a real property.

ISSUE:
Whether or not the warehouse is an immovable and must be tried in the province where the
property lies.

HELD:
Warehouse claimed to be owned by petitioner is an immovable or real property. Buildings
are always immovable under the Civil Code. A building treated separately from the land on
which it is stood is immovable property and the mere fact that the parties to a contract seem to
have dealt with it separate and apart from the land on which it stood did not change its character
as immovable property.

MCSS
Prudential Bank v. Panis
153 SCRA 390

FACTS:
Plaintiff-spouses Magcale secured two loans over a 2-storey residential building.

For failure of the plaintiffs to pay their obligation to defendant Bank after it became due, the
deed of the Real Estate Mortgage were extrajudicially foreclosed.

ISSUE: WON a valid real estate mortgage can be constituted on the building. -- YES

HELD:
Inclusion of building separate and distinct from land, in the provision of law can only mean that a
building is by itself an immovable property. A building by itself may be mortgaged apart from the
land on which it has been built.

NKVS
Tumalad v. Vicencio
41 SCRA 143

DOCTRINE: The view that parties to a deed of chattel mortgage may agree to consider a house
as personal property for the purposes of said contract, "is good only insofar as the contracting
parties are concerned. It is based, partly, upon the principle of estoppel.

FACTS:
On 1 September 1955 defendants executed a chattel mortgage in favor of plaintiffs over their
house located at Quiapo, Manila, which were being rented from Madrigal & Company, Inc. The
mortgage was registered in the Registry of Deeds of Manila on 2 September 1955. The
mortgage was executed to guarantee a loan of P4,800.00 received from plaintiffs. It was also
agreed that default in the payment of any of the amortizations, would cause the remaining
unpaid balance to become immediately due and Payable and the Chattel Mortgage will be
enforceable in accordance with the provisions of Special Act No. 3135, and for this purpose, the
Sheriff of the City of Manila or any of his deputies is hereby empowered and authorized to sell
all the Mortgagor's property after the necessary publication in order to settle the financial debts
of P4,800.00, plus 12% yearly interest, and attorney's fees.

When defendants defaulted in paying, the mortgage was extrajudicially foreclosed, and the
house was sold at public auction pursuant to the said contract. As highest bidder, plaintiffs were
issued the corresponding certificate of sale. Thereafter, plaintiffs commenced Civil Case No.
43073 in the municipal court of Manila, praying, among other things, that the house be vacated
and its possession surrendered to them, and for defendants to pay rent of P200.00 monthly
from 27 March 1956 up to the time the possession is surrendered. MTC granted petition.

Defendants, in their answers in both the municipal court and court a quo impugned the legality
of the chattel mortgage, claiming that they are still the owners of the house. During the
pendency of the appeal to the Court of First Instance, defendants failed to deposit the rent as
ordered in the decision of the municipal court. As a result, the court granted plaintiffs motion for
execution. However, the judgment regarding the surrender of possession to plaintiffs could not
be executed because the subject house had been already demolished pursuant to the order of
the court in a separate civil case for ejectment against the present defendants for non-payment
of rentals on the land on which the house was constructed.

ISSUE:
W/N the house may be a subject of a Chattel Mortgage. YES, it may be the subject of a
chattel mortgage.

HELD:
Defendants predicate their theory of nullity of the chattel mortgage on the ground that the
subject matter of the mortgage is a house of strong materials, and, being an immovable, it can
only be the subject of a real estate mortgage and not a chattel mortgage.

The rule about the status of buildings as immovable property is that it is obvious that the
inclusion of the building, separate and distinct from the land, in the enumeration of what may
constitute real properties could only mean one thing that a building is by itself an immovable
property irrespective of whether or not said structure and the land on which it is adhered to
belong to the same owner.

It is undeniable that the parties to a contract may by agreement treat as personal property that
which by nature would be real property. The view that parties to a deed of chattel mortgage may
agree to consider a house as personal property for the purposes of said contract, "is good only
insofar as the contracting parties are concerned. It is based, partly, upon the principle of
estoppel.

In a case, a mortgaged house built on a rented land was held to be a personal property, not only
because the deed of mortgage considered it as such, but also because it did not form part of the
land for it is now settled that an object placed on land by one who had only a temporary right to
the same, such as the lessee or usufructuary, does not become immobilized by attachment.
Hence, if a house belonging to a person stands on a rented land belonging to another person, it
may be mortgaged as a personal property as so stipulated in the document of mortgage. It
should be noted, however that the principle is predicated on statements by the owner declaring
his house to be a chattel, a conduct that may conceivably estop him from subsequently claiming
otherwise.

Although there is no specific statement referring to the subject house as personal property, yet
by ceding, selling or transferring a property by way of chattel mortgage defendants could only
have meant to convey the house as chattel, or at least, intended to treat the same as such, so
that they should not now be allowed to make an inconsistent stand by claiming otherwise.
Moreover, the subject house stood on a rented lot to which defendants merely had a temporary
right as lessee, and although this can not in itself alone determine the status of the property, it
does so when combined with other factors to sustain the interpretation that the parties,
particularly the mortgagors, intended to treat the house as personalty. Finally, because it is the
defendants themselves, as debtors-mortgagors, who are attacking the validity of the chattel
mortgage in this case, the doctrine of estoppel therefore applies to the defendants, having
treated the subject house as personalty.

AMPS
Sergs Products and Goquiola v. PCI Leasing and Finance
338 SCRA 499

DOCTRINE: After agreeing to a contract stipulating that a real or immovable property be
considered as personal or movable, a party is estopped from subsequently claiming otherwise.
Hence, such property is a proper subject of a writ of replevin obtained by the other contracting
party.

FACTS:


PCI Leasing and Finance, Inc. filed a complaint with the RTC for a sum of money with an
application for a writ of replevin. Upon an ex-parte application of PCI Leasing, respondent judge
issued a writ of replevin directing its sheriff to seize and deliver the machineries and equipment
to PCI Leasing after 5 days and upon the payment of the necessary expenses.

Sergs filed a motion for special protective order. This motion was opposed by PCI Leasing on
the ground that the properties [were] still personal and therefore still subject to seizure and a
writ of replevin.

In their Reply, petitioners asserted that the properties sought to be seized were immovable as
defined in Article 415 of the Civil Code, the parties agreement to the contrary notwithstanding.
They argued that to give effect to the agreement would be prejudicial to innocent third parties.
They further stated that PCI Leasing was estopped from treating these machineries as personal
because the contracts in which the alleged agreement were embodied were totally sham and
farcical.

Citing the Agreement of the parties, the appellate court held that the subject machines were
personal property, and that they had only been leased, not owned, by petitioners. It also ruled
that the words of the contract are clear and leave no doubt upon the true intention of the
contracting parties.

ISSUE:
Whether or not the machineries purchased and imported by SERGS became real property by
virtue of immobilization.

HELD:
The machineries herein are real properties but are considered personal by the parties
agreement.

The Court will resolve whether the said machines are personal, not immovable, property which
may be a proper subject of a writ of replevin. Rule 60 of the Rules of Court provides that writs of
replevin are issued for the recovery of personal property only. Section 3 thereof reads:

SEC. 3. Order. -- Upon the filing of such affidavit and approval of the bond, the
court shall issue an order and the corresponding writ of replevin describing the
personal property alleged to be wrongfully detained and requiring the sheriff
forthwith to take such property into his custody.

On the other hand, Article 415 of the Civil Code enumerates immovable or real property as
follows:

ART. 415. The following are immovable property:
x x x....................................x x x....................................x x x

(5) Machinery, receptacles, instruments or implements intended by the owner of
the tenement for an industry or works which may be carried on in a building or on
a piece of land, and which tend directly to meet the needs of the said industry or
works;

x x x....................................x x x....................................x x x

In the present case, the machines that were the subjects of the Writ of Seizure were placed by
petitioners in the factory built on their own land. Indisputably, they were essential and principal
elements of their chocolate-making industry. Hence, although each of them was movable or
personal property on its own, all of them have become immobilized by destination because
they are essential and principal elements in the industry. In that sense, petitioners are correct
in arguing that the said machines are real, not personal, property pursuant to Article 415 (5) of
the Civil Code.

Be that as it may, we disagree with the submission of the petitioners that the said machines are
not proper subjects of the Writ of Seizure.

The Court has held that contracting parties may validly stipulate that a real property be
considered as personal. After agreeing to such stipulation, they are consequently estopped
from claiming otherwise. Under the principle of estoppel, a party to a contract is ordinarily
precluded from denying the truth of any material fact found therein.

Hence, in Tumalad v. Vicencio, the Court upheld the intention of the parties to treat a house as
a personal property because it had been made the subject of a chattel mortgage.

It should be stressed, however, that our holding -- that the machines should be deemed
personal property pursuant to the Lease Agreement is good only insofar as the contracting
parties are concerned. Hence, while the parties are bound by the Agreement, third persons
acting in good faith are not affected by its stipulation characterizing the subject machinery as
personal. In any event, there is no showing that any specific third party would be adversely
affected.

KGS
Manarang and Manarang v. Ofilada and Esteban
99 SCRA 108

DOCTRINE: House is personal property for purposes of chattel mortgage only; Remains real
property. The mere fact that a house was the subject of a chattel mortgage and was considered
as personal property by the parties does not make said house personal property for purposes of
the notice to be given for its sale at public auction. It is real property within the purview of Rule
39, section 16, of the Rules of Court as it has become a permanent fixture on the land, which is
real property.

FACTS:
Manarang obtained a loan from Esteban, and executed a chattel mortgage over a house of
mixed materials as a security. Upon default, Estaban brought an action to foreclose the property
mortgaged. At Manarangs request, the house mortgaged was to be sold at public auction to
satisfy the debt. However, before the property could be sold, Manarang offered to pay the sum.
But the sheriff refused the tender unless the additional amount is also paid representing the
publication of the notice in two newspapers. Manarang contended that the house in question
should be considered as personal property and the publication of the notice of its sale at public
auction in execution considered unnecessary. The Court of First Instance held that although real
property may sometimes be considered as personal property, the sheriff was in duty bound to
cause the publication of the notice of its sale in order to make the sale valid or to prevent its
being declared void or voidable.

ISSUE:
Can the house of Manarang be classified as personal property since it was considered as such
in a chattel mortgage? -- NO

HELD:
The house of mixed materials levied upon on execution, although subject of a contract of chattel
mortgage between the owner and a third person, is real property within the purview of Rule 39,
section 16, of the Rules of Court as it has become a permanent fixture on the land, which is real
property.

There cannot be any question that a building of mixed materials may be the subject of a chattel
mortgage, in which case it is considered as between the parties as personal property. The
matter depends on the circumstances and the intention of the parties.

The general principle of law is that a building permanently fixed to the freehold becomes a part
of it, that prima facie a house is real estate, belonging to the owner of the land on which it
stands, even though it was erected against the will of the landowner, or without his consent. The
general rule is otherwise, however, where the improvement is made with the consent of the
landowner, and pursuant to an understanding either expressed or implied that it shall remain
personal property. Nor does the general rule apply to a building, which is wrongfully removed
from the land and placed on the land of the person removing it.

Among the principal criteria for determining whether property remains personally or becomes
realty are annexation to the soil, either actual or construction, and the intention of the parties.
Personal property may retain its character as such where it is so agreed by the parties
interested even though annexed to the realty, or where it is affixed in the soil to be used for a
particular purpose for a short period and then removed as soon as it has served its purpose.

These considerations notwithstanding, we hold that the rules on execution do not allow, and we
should not interpret them in such a way as to allow, the special consideration that parties to a
contract may have desired to impart to real estate, for example, as personal property, when they
are not ordinarily so. Sales on execution affect the public and third persons. The regulation
governing sales on execution are for public officials to follow. The form of proceedings
prescribed for each kind of property is suited to its character, not to the character which the
parties have given to it or desire to give it. When the rules speak of personal property, property
which is ordinarily so considered is meant; and when real property is spoken of, it means
property which is generally known as real property. The regulations were never intended to suit
the consideration that parties, may have privately given to the property levied upon.
Enforcement of regulations would be difficult were the convenience or agreement of private
parties to determine or govern the nature of the proceedings. We, therefore, hold that the mere
fact that a house was the subject of a chattel mortgage and was considered as personal
property by the parties does not make said house personal property for purposes of the notice
to be given for its sale at public auction. This ruling is demanded by the need for a definite,
orderly and well-defined regulation for official and public guidance and which would prevent
confusion and misunderstanding
JPOT
Navarro v. Pineda
9 SCRA 631

DOCTRINE: Estoppel, in that "the parties have so expressly agreed" in the mortgage to
consider the house as chattel "for its smallness and mixed materials of sawali and wood".

FACTS:
December 14, 1959, Rufino G. Pineda and his mother Juana Gonzales (married to Gregorio
Pineda), borrowed from plaintiff Conrado P. Navarro, the sum of P2,500.00, payable 6 months
after said date or on June 14, 1959. To secure the indebtedness, Rufino executed a document
captioned "DEED OF REAL ESTATE and CHATTEL MORTGAGES", whereby Juana Gonzales,
by way of Real Estate Mortgage hypothecated a parcel of land, belonging to her, registered with
the Register of Deeds of Tarlac, under Transfer Certificate of Title No. 25776, and Rufino G.
Pineda, by way of Chattel Mortgage, mortgaged his two-story residential house, having a floor
area of 912 square meters, erected on a lot belonging to Atty. Vicente Castro, located at Bo.
San Roque, Tarlac, Tarlac; and one motor truck, registered in his name, under Motor Vehicle
Registration Certificate No. A-171806. Both mortgages were contained in one instrument, which
was registered in both the Office of the Register of Deeds and the Motor Vehicles Office of
Tarlac.

After failing to settle amount due, respondent was then granted an extension on June 30,1960
and consequently July 30th of the same year for still being unable to comply. Rufino Pineda
then issued a document entitled "Promise," stating that defendant would no longer ask for
further extension and there would be no need for any formal demand, and plaintiff could
proceed to take whatever action he might desire to enforce his rights, under the said mortgage
contract.

On August 10, 1960, plaintiff filed a complaint for foreclosure of the mortgage and for damages,
which consisted of liquidated damages in the sum of P500.00 and 12% per annum interest on
the principal, effective on the date of maturity, until fully paid. Defendants admit that the loan is
overdue but deny that portion of paragraph 4 of the First Cause of Action which states that the
defendants unreasonably failed and refuse to pay their obligation to the plaintiff the truth being
the defendants are hard up these days and pleaded to the plaintiff to grant them more time
within which to pay their obligation and the plaintiff refused;

WHEREFORE, in view of the foregoing it is most respectfully prayed that this Honorable Court
render judgment granting the defendants until January 31, 1961, within which to pay their
obligation to the plaintiff.

November 11, 1960, however, the parties submitted a Stipulation of Facts, wherein the
defendants admitted the indebtedness, the authenticity and due execution of the Real Estate
and Chattel Mortgages; that the indebtedness has been due and unpaid since June 14, 1960;
that a liability of 12% per annum as interest was agreed, upon failure to pay the principal when
due and P500.00 as liquidated damages; that the instrument had been registered in the
Registry of Property and Motor Vehicles Office, both of the province of Tarlac.

ISSUE:
W/N the residential house, subject of the mortgage therein, can be considered a Chattel and the
propriety of the attorney's fees.

HELD:
The court ruled "a property may have a character different from that imputed to it in said articles.
It is undeniable that the parties to a contract may by agreement, treat as personal property that
which by nature would be real property" (Standard Oil Co. of N.Y. v. Jaranillo, 44 Phil. 632-
633)."There can not be any question that a building of mixed materials may be the subject of a
chattel mortgage, in which case, it is considered as between the parties as personal property. ...
The matter depends on the circumstances and the intention of the parties". "Personal property
may retain its character as such where it is so agreed by the parties interested even though
annexed to the realty ...". (42 Am. Jur. 209-210, cited in Manarang, et al. v. Ofilada, et al., G.R.
No. L-8133, May 18, 1956; 52 O.G. No. 8, p. 3954.) Moreover, the court continues and makes
plain that it "is good only insofar as the contracting parties are concerned. It is based partly,
upon the principles of estoppel ..." (Evangelista v. Alto Surety, No. L-11139, Apr. 23, 1958).
WHEREBY, previous judgment is hereby AFFIRMED costs against appelant.

MLAV
Manila Electric Co., v. Central Board of Assessment Appeals
114 SCRA 273

DOCTRINE: Oil storage tanks were held to be taxable realty. For purposes of taxation, the term
"real property" may include things which should generally be regarded as personal property.

FACTS:
The case is about the imposition of the realty tax on two oil storage tanks installed in 1969 by
Manila Electric Company in Batangas which it leased in 1968 from Caltex (Phil.), Inc. The tanks
are within the Caltex refinery compound, and are used for storing fuel oil for Meralco's power
plants.

According to Meralco, the storage tanks are made of steel plates welded and assembled on the
spot. Their bottoms rest on a foundation consisting of compacted earth as the outermost layer, a
sand pad as the intermediate layer and a two-inch thick bituminous asphalt stratum as the top
layer. The bottom of each tank is in contact with the asphalt layer. Hence, it is not attached to its
foundation.

On the other hand, according to the hearing commissioners of the Central Board of Assessment
Appeals (CBAA) states that while the tanks rest or sit on their foundation, the foundation itself
and the walls, dikes and steps, which are integral parts of the tanks, are affixed to the land while
the pipelines are attached to the tanks.

The CBAA ruled that the tanks together with the foundation, walls, dikes, steps, pipelines and
other appurtenances constitute taxable improvements.

Meralco filed a motion for reconsideration which the Board denied. They elevated the case to
the SC.

ISSUE:
WON the storage tanks are considered improvements on real property such that it is subject to
real property tax. -- YES

HELD:
Meralco contends that the said oil storage tanks do not fall within any of the kinds of real
property enumerated in article 415 of the Civil Code and, therefore, they cannot be categorized
as realty by nature, by incorporation, by destination nor by analogy. Stress is laid on the fact
that the tanks are not attached to the land and that they were placed on leased land, not on the
land owned by Meralco.

The issue raised by Meralco has to be resolved in the light of the provisions of the Assessment
Law, Commonwealth Act No. 470, and the Real Property Tax Code, Presidential Decree No.
464 which took effect on June 1, 1974.

Section 2 of the Assessment Law provides that the realty tax is due "on real property, including
land, buildings, machinery, and other improvements" not specifically exempted in section 3
thereof. This provision is reproduced with some modification in the Real Property Tax Code
which provides:
Sec. 38. Incidence of Real Property Tax. They shall be levied, assessed and
collected in all provinces, cities and municipalities an annual ad valorem tax on real
property, such as land, buildings, machinery and other improvements affixed or attached
to real property not hereinafter specifically exempted.

The Code contains the following definition in its section 3:
k) Improvements is a valuable addition made to property or an amelioration in its
condition, amounting to more than mere repairs or replacement of waste, costing labor
or capital and intended to enhance its value, beauty or utility or to adapt it for new or
further purposes.

The SC holds that while the two storage tanks are not embedded in the land, they may,
nevertheless, be considered as improvements on the land, enhancing its utility and rendering it
useful to the oil industry. It is undeniable that the two tanks have been installed with some
degree of permanence as receptacles for the considerable quantities of oil needed by Meralco
for its operations.

Oil storage tanks were held to be taxable realty in Standard Oil Co. of New Jersey vs. Atlantic
City, 15 Atl. 2nd 271.

For purposes of taxation, the term "real property" may include things which should generally be
regarded as personal property. It is a familiar phenomenon to see things classed as real
property for purposes of taxation which on general principle might be considered personal
property (Standard Oil Co. of New York vs. Jaramillo, 44 Phil. 630, 633).

DJTV
Caltex Philippines v. Central Board of Assessment Appeals
114 SCRA 296

DOCTRINE: Gasoline station equipment and machineries are permanent fixtures for purposes
of realty taxation.

FACTS:
Machines and equipment are loaned by Caltex to gas station operators under an appropriate
lease agreement or receipt. It is stipulated in the lease contract that the operators, upon
demand, shall return to Caltex the machines and equipment in good condition as when received,
ordinary wear and tear excepted.

The city assessor of Pasay City characterized the said items of gas station equipment and
machinery as taxable realty. The city board of tax appeals ruled that they are personalty. The
assessor appealed to the Central Board of Assessment Appeals.

The Board said machines and equipment are real property within the meaning of sections 3(k) &
(m) and 38 of the Real Property Tax Code, Presidential Decree No. 464, which took effect on
June 1, 1974, and that the definitions of real property and personal property in articles 415 and
416 of the Civil Code are not applicable to this case.

Caltex filed this certiorari petition wherein it prayed for the setting aside of the Board's decision
and for a declaration that t he said machines and equipment are personal property not subject
to realty tax.

ISSUE:
Whether or not Gasoline station equipment and machineries are permanent fixtures for
purposes of realty taxation. -- YES

HELD:
The Supreme Court held that gasoline station equipment and machineries are permanent
fixtures for purposes of realty taxation. Thus, they are subject to the real property tax. The said
equipment and machinery, as appurtenances to the gas station building or shed owned by
Caltex (as to which it is subject to realty tax) and which fixtures are necessary to the operation
of the gas station, for without them the gas station would be useless, and which have been
attached or affixed permanently to the gas station site or embedded therein, are taxable
improvements and machinery within the meaning of the Assessment Law and the Real Property
Tax Code.

The Central Board of Assessment Appeals did not commit a grave abuse of discretion in
upholding the city assessor's is imposition of the realty tax on Caltex's gas station and
equipment.

WHEREFORE, the questioned decision and resolution of the Central Board of Assessment
Appeals are affirmed. The petition for certiorari is dismissed for lack of merit.

JGY
Phil. Refining Co., Inc. v. Jarque
G.R. No. 41506

DOCTRINE: Vessels are considered personal property under the civil law. Similarly under the
common law, vessels are personal property although occasionally referred to as a peculiar kind
of personal property. Since the term "personal property" includes vessels, they are subject to
mortgage agreeably to the provisions of the Chattel Mortgage Law.

FACTS:
Philippine Refining Co., Inc., and Jarque executed three chattel mortgages on the motor vessels
Pandan and Zaragoza. Neither of the first two mortgages had appended an affidavit of good
faith. The third mortgage contained such an affidavit, but this mortgage was not registered in the
customs house within the period of thirty days prior to the commencement of insolvency
proceedings against Jarque. A fourth mortgage was executed and was entered in the chattel
mortgage registry of the register of deeds within the thirty-day period before the institution of
insolvency proceedings. CFI declared Jarque to be an insolvent debtor. As a result, all his
properties were assigned to Corominas. CFI Judge declined to order the foreclosure of the
mortgages, but on the contrary sustained the special defenses of fatal defectiveness of the
mortgages.

ISSUES:
1. WON the vessels are considered as personal property and can be the subject of Chattel
Mortgage. Yes
2. WON the Chattel Mortgages are valid. No.

HELD:
Vessels are considered personal property under the civil law. Similarly under the common law,
vessels are personal property although occasionally referred to as a peculiar kind of personal
property. Since the term "personal property" includes vessels, they are subject to mortgage
agreeably to the provisions of the Chattel Mortgage Law. Indeed, it has heretofore been
accepted without discussion that a mortgage on a vessel is in nature a chattel mortgage. The
only difference between a chattel mortgage of a vessel and a chattel mortgage of other
personalty is that it is not now necessary for a chattel mortgage of a vessel to be noted n the
registry of the register of deeds, but it is essential that a record of documents affecting the title
to a vessel be entered in the record of the Collector of Customs at the port of entry. Otherwise a
mortgage on a vessel is generally like other chattel mortgages as to its requisites and validity.

The Chattel Mortgage Law in its section 5, in describing what shall be deemed sufficient to
constitute a good chattel mortgage, includes the requirement of an affidavit of good faith
appended to the mortgage and recorded therewith. The absence of the affidavit vitiates a
mortgage as against creditors and subsequent encumbrancers. As a consequence a chattel
mortgage of a vessel wherein the affidavit of good faith required by the Chattel Mortgage Law is
lacking, is unenforceable against third persons.

JRPA
Rubiso v. Rivera
37 Phil 72

DOCTRINE: The requisite of registration in the registry, of the purchase of the vessel, is
necessary and indispensable in order that the purchasers rights may be maintained
against a third person.

FACTS:
Rubiso filed a complaint against Rivera for the recovery of a pilot boat. He alleged that he is
the rightful owner of a pilot boat, which was stranded and recovered by Rivera. The latter
refused to return the said boat as he alleged too that he was the owner thereof.

It was known that the original owners of the boat had secretly sold the pilot boat to Rivera on an
earlier date than the sale in a public auction to Rubiso. Nonetheless, material is the fact that the
entry into the customs registry of the sale of the boat was later than the recording of the sale to
Rubiso.

ISSUE:
WON it is Rubiso or Rivera who has a better right to the boat? -- Rubiso has a better right

HELD:
The requisite of registration in the registry, of the purchase of the vessel, is necessary and
indispensable in order that the purchasers rights may be maintained against a third
person. Such registration is required both by the Code of Commerce and Act 1900. It
is undeniable, ergo, that Rivera doesnt have a better right than Rubiso over the pilot boat.

Ships and vessels, whether moved by steam or by sail, partake, to a certain extent of
the nature and conditions of real property, on account of their value and importance in world
commerce; and for this, the provisions of the Code of Commerce are nearly identical with Article
1473 of the Civil Code

ABB
U.S. v. Carlos
21 Phil. 553

DOCTRINE: The true test of what constitutes the proper subject of [theft] is not whether the
subject is corporeal or incorporeal, but whether it is capable of appropriation by another other
than the owner.

FACTS:
Ignacio Carlos has been a consumer of electricity furnished by MERALCO for a building
containing the residence of the accused and 3 other residences. Representatives of the
company believing that more light is consumed than what is shown in the meter installed an
additional meter on the pole outside Carlos house to compare the actual consumption and
found out that a jumper was used to manipulate the readings of the first meter. Further, a
jumper was found in a drawer of a small cabinet in the room of the defendants house where the
meter was installed. In the absence of any explanation for Carlos possession of said device, the
presumption raised was that Carlos was the owner of the device whose only use was to deflect
the current from the meter. Thus, he was charged with the crime of theft amounting to 2,273KW
of electric power worth 909.20 pesos.

Carlos claimed that what he did failed to constitute an offense because the crime of theft applies
only to tangibles, chattels and objects that can be taken into possession.

Deliberation quickly followed at the court which subsequently sentenced him to over a year in
jail. Carlos contested saying that electrical energy cant be stolen because of its nature of being
incorporeal. He filed an appeal on such grounds which the CFI affirmed.

ISSUE:
Whether or not theft can be committed against an intangible such as electricity. -- YES

HELD:
Theft of incorporeal objects is possible. The right of ownership of electrical current was secured
by Art 517 and 518 of the Penal Code which applies to gas.

Analogically, electricity can be considered as gas which can be stolen. However, the true test
of what constitutes the proper subject of larceny is not whether the subject is corporeal or
incorporeal, but whether it is capable of appropriation by another other than the owner. It is a
valuable article of merchandise, a force of nature brought under the control of science (under
Art. 416 of the New Civil Code). Carlos secretly and with intent to deprive the company of its
rightful property, used jumper cables to appropriate the same for his own use. Such acts
constitute theft.

FZC
Piansay v. David
12 SCRA 227

DOCTRINE: regardless of the validity of a contract constituting a chattel mortgage on a house,
the same cannot and does not bind third persons, who are not parties to the contract of their
privies.

FACTS:
David obtained a loan from Uy Kim secured with a chattel mortgage on a house in Tondo. David
failed to pay, thus was Kim foreclosed. Kim then bought the house at the resulting public auction.
Thereafter, Kim sold the same to Marcos Mangubat, who later filed a complaint against David
for the collection of a portrion of the loan. The complaint was later amended to implead Uy Kim
and Piansay praying that the auction sale and deed of absolute sale executed by Uy Kim in
favor of Piansay be annulled. CFI Manila ordered David to pay and annulled the chattel
mortgage. CA affirmed. David was ordered to pay and the house was levied upon. To prevent
the sale at the public auction, Piansay and Uy Kim filed a petition for
certiorari and mandamus with preliminary injunction before the CA; it was denied.

Subsequently, Piansay and Uy Kim instituted an action against David and Mangubat praying
that judgment be rendered declaring Piansay as the true owner and restrain the levy and sale to
public auction. David demanded from Piansay the payment of the rentals for the use and
occupation of the house; the latter claims it is his property. Mangubat, on one hand, moved to
dismiss the complaint on the ground of res adjudicata and lackof personality to sue; it was
granted. CA affirmed explaining that Uy Kim had no right to foreclose the chattel mortgage
because it was in reality a mere contract of an unsecured loan. Piansay assailed Mangubat's
right to levy execution upon the house alleging that the same belongs to him, he having bought
it from Uy Kim who acquired it at the auction sale.

ISSUE:
WON the chattel mortgage and sale are valid - NO

HELD:
Upon the theory that the chattel mortgage and sale in favor of Uy Kim had been annulled in the
original decision, as affirmed by the CA, the fact is that said order became final and executory
upon the denial of the petition for certiorari and mandamus. Hence, Uy Kim and Piansay are
now barred from asserting that the chattel mortgage and sale are valid. At any rate, regardless
of the validity of a contract constituting a chattel mortgage on a house, as between the parties to
said contract, the same cannot and does not bind third persons, who are not parties to the
contract of their privies. As a consequence, the sale of the house in question in the proceedings
for the extrajudicialforeclosure of said chattel mortgage, is null and void insofar as defendant
Mangubat is concerned, anddid not confer upon Uy Kim, as buyer in said sale, any dominical
right in and to said house, so that shecould not have transmitted to her assignee Piansay any
such right as against Mangubat. In short, they donot have a cause of action against Mangubat
and David

LNAC
U.S. v. Tambunting
41 Phil 364

DOCTRINE: There is nothing in the nature of gas used for illuminating purposes which renders
it incapable of being feloniously taken and carried away. It is a valuable article of merchandise,
bought and sold like other personal property, susceptible of being severed from a mass or larger
quantity and of being transported from place to place. Likewise water which is confined in pipes
and electricity which is conveyed by wires are subjects of larceny.

FACTS:
This appeal was instituted for the purpose of reversing a judgment of the CFI, finding the
accused, Manuel Tambunting, guilty of stealing a quantity of gas belonging to the Manila Gas
Corporation, and sentencing him to undergo imprisonment with the accessories prescribed by
law; to indemnify the said corporation, with subsidiary imprisonment in case of insolvency; and
to pay the costs.

The evidence submitted in behalf of the prosecution shows that in January of the year 1918, the
accused and his wife became occupants of the upper floor of the house situated at No. 443,
Calle Evangelista, Manila. In this house the Manila Gas Corporation had previously installed
apparatus for the delivery of gas on both the upper and lower floors, consisting of the necessary
piping and a gas meter, which last mentioned apparatus was installed below. When the
occupants at whose request this installation had been made vacated the premises, the gas
company disconnected the gas pipe and removed the meter, thus cutting off the supply of gas
from said premises.

Upon June 2, 1919, one of the inspectors of the gas company visited the house in question and
found that gas was being used, without the knowledge and consent of the gas company, for
cooking in the quarters occupied by the defendant and his wife: to effect which a short piece of
iron pipe had been inserted in the gap where the gas meter had formerly been placed, and
piece of rubber tubing had been used to connect the gas pipe of rubber tubing had been used to
connect the gas pipe in kitchen with the gas stove, or plate, used for cooking.

At the time this discovery was made, accused Tambunting was not at home. However, he
presently arrived and admitted to the agent to the gas company that he had made the
connection with the rubber tubing between the gas pipe and the stove, though he denied
making the connection below. He also admitted that he knew he was using gas without the
knowledge of the company and that he had been so using it for probably two or three months.

The clandestine use of gas by the accused in the manner stated is thus established in our
opinion beyond a doubt; and inasmuch as the animo lucrandi is obvious.

ISSUE:
Whether gas can be the subject to larceny. -- YES

HELD:.
The right of the ownership of electric current is secured by article 517 and 518 of the Penal
Code; the application of these articles in cases of subtraction of gas, a fluid used for lighting,
and in some respects resembling electricity, is confirmed by the rule laid down in the decisions
of the SC of Spain, construing and enforcing the provisions of articles 530 and 531 of the Penal
Code of that country, articles identical with articles 517 and 518 of the code in force in these
Islands. These expressions were used in a case which involved the subtraction and
appropriation of electrical energy and the court held, in accordance with the analogy of the case
involving the theft of gas, that electrical energy could also be the subject of theft.

In this connection it will suffice to quote the following from the topic "Larceny," at page 34, Vol.
17, of Ruling Case Law: There is nothing in the nature of gas used for illuminating purposes
which renders it incapable of being feloniously taken and carried away. It is a valuable article of
merchandise, bought and sold like other personal property, susceptible of being severed from a
mass or larger quantity and of being transported from place to place. Likewise water which is
confined in pipes and electricity which is conveyed by wires are subjects of larceny.

TKDC
Involuntary Insolvency of Strochecker v. Ramirez
44 Phil. 933

DOCTRINE: All personal properties may be mortgaged. Interest in business is personal
property capable of appropriation and not included in the enumeration of real properties under
Article 335 of the Civil Code. Thus, interest in business may be subject of mortgage.

FACTS:
Three mortgages seek preference in the lower court: one in favor of Fidelity and Surety Co.,
another in favor of Ramirez, and the last one in favor of Ayala. Ayalas claim was rejected by
trial court from which she didnt appeal.

As to the time of the mortgages, the one in favor of Fidelity and Surety Co. is preferred because
it was executed and registered in the registry of property prior to that of Ramirezs. However,
Ramirez claimed that the mortgage in favor of Fidelity and Surety Co. is invalid because the
property, the half interest in the drug business, is incapable of being mortgaged. Trial court ruled
that the mortgage in favor of Fidelity and Surety Co. is entitled to preference.

ISSUE:
Whether or not one-half interest in the business is capable of being mortgaged. -- YES

HELD:
All personal properties may be mortgaged. Interest in business is personal property capable of
appropriation and not included in the enumeration of real properties under Article 335 of the
Civil Code. Thus, interest in business may be subject of mortgage.

In this case, the mortgaged property of one-half interest in the drug business in favor of Fidelity
and Surety Co. is a valid subject of mortgage.

AMD
Laurel v. Abrogar
G.R. No. 155076

DOCTRINE: Telecommunication services and the business of providing said services are not
personal properties and cannot be subject to Article 308 of the Revised Penal Code

Services in business, although properties, are not proper subjects of theft under the Revised
Penal Code because the same cannot be "taken" or "occupied".

FACTS:
PLDT claims that Luis Marcos P. Laurel, board member and corporate secretary of Baynet Co.,
Ltd., stole and used the international long distance calls belonging to PLDT by conducting
International Simple Resale (ISR) a method of routing and completing international long
distance calls using lines, cables, antennae, and/or air wave frequency which connect directly to
the local or domestic exchange facilities of the country where the call is destined. PLDT alleged
that such business was effectively stolen while using their facilities leading to great damage and
prejudice amounting to P20,370,651.92.

Laurel however alleged that the allegations do not constitue the felony of theft under Article 308
of the RPC or any special law. He claimed that, telephone calls with the use of PLDT telephone
lines, whether domestic or international, belong to the persons making the call, not to PLDT. He
argued that the caller merely uses the facilities of PLDT, and what the latter owns are the
telecommunication infrastructures or facilities through which the call is made. He also asserted
that PLDT is compensated for the callers use of its facilities by way of rental; for an outgoing
overseas call, PLDT charges the caller per minute, based on the duration of the call. Thus, no
personal property was stolen from PLDT.

The prosecution asserted that the use of PLDTs intangible telephone services/facilities allows
electronic voice signals to pass through the same, and ultimately to the called partys number. It
averred that such service/facility is akin to electricity which, although an intangible property, may,
nevertheless, be appropriated and be the subject of theft. The prosecution further alleged that
"international business calls and revenues constitute personal property envisaged in Article 308
of the Revised Penal Code." Moreover, the intangible telephone services/facilities belong to
PLDT and not to the movant and the other accused, because they have no telephone services
and facilities of their own duly authorized by the NTC; thus, the taking by the movant and his co-
accused of PLDT services was with intent to gain and without the latters consent.

ISSUE:
W/N telephone calls placed by Bay Super Orient Card holders through the telecommunication
services provided by PLDT are considered as personal property, and thus, proper subjects of
theft under Article 308 of the Revised Penal Code. -- NO

HELD:
The court finds that the international telephone calls placed by Bay Super Orient Card holders,
the telecommunication services provided by PLDT and its business of providing said services
are not personal properties under Article 308 of the Revised Penal Code. The rule is that, penal
laws are to be construed strictly. Penal statutes may not be enlarged by implication or intent
beyond the fair meaning of the language used; and may not be held to include offenses other
than those which are clearly described.

One is apt to conclude that "personal property" standing alone, covers both tangible and
intangible properties and are subject of theft under the Revised Penal Code. But the words
"Personal property" under the Revised Penal Code must be considered in tandem with the word
"take" in the law. The statutory definition of "taking" and movable property indicates that, clearly,
not all personal properties may be the proper subjects of theft. The general rule is that, only
movable properties which have physical or material existence and susceptible of occupation by
another are proper objects of theft.

According to Cuello Callon, in the context of the Penal Code, only those movable properties
which can be taken and carried from the place they are found are proper subjects of theft.
Intangible properties such as rights and ideas are not subject of theft because the same cannot
be "taken" from the place it is found and is occupied or appropriated.

Gas and electrical energy should not be equated with business or services provided by
business entrepreneurs to the public. Business does not have an exact definition. Business is
referred as that which occupies the time, attention and labor of men for the purpose of livelihood
or profit. It embraces everything that which a person can be employed. Business may also
mean employment, occupation or profession. Business is also defined as a commercial activity
for gain benefit or advantage. Business, like services in business, although are properties, are
not proper subjects of theft under the Revised Penal Code because the same cannot be "taken"
or "occupied."

PLDT does not acquire possession, much less, ownership of the voices of the telephone callers
or of the electronic voice signals or current emanating from said calls. The human voice and the
electronic voice signals or current caused thereby are intangible and not susceptible of
possession, occupation or appropriation by PLDT or even the petitioner, for that matter. PLDT
merely transmits the electronic voice signals through its facilities and equipment. Baynet Card
Ltd., through its operator, merely intercepts, reroutes the calls and passes them to its toll center.



PROPERTY IN RELATIONSHIP TO THE PERSON WHOM IT BELONGS (ART. 419-426)

CRF
Villanueva v. Castaeda
154 SCRA 142

DOCTRINE: Article 344 of the Civil Code: "Property for public use in provinces and in towns
comprises the provincial and town roads, the squares, streets, fountains, and public waters, the
promenades, and public works of general service supported by said towns or provinces. Such
is outside the commerce of man and cannot be the object of a valid contract. (Article 1271)

FACTS:
The subject of the herein petition is a talipapa that the petitioners claim they have a right to
remain in and conduct business in this area by virtue of a previous authorization granted to
them by the municipal government. The respondents deny this and justify the demolition of their
stalls as illegal constructions on public property.

This dispute goes back to when the municipal council of San Fernando adopted Resolution No.
218 authorizing some 24 members of the Fernandino United Merchants and Traders
Association to construct permanent stags and sell in the above-mentioned place.
2
The action
was protested where the Court of First Instance of Pampanga, Branch 2, issued a writ of
preliminary injunction that prevented the defendants from constructing the said stalls until final
resolution of the controversy.
3
On January 18, 1964, while this case was pending, the municipal
council of San Fernando adopted Resolution G.R. No. 29, which declared the subject area as
"the parking place and as the public plaza of the municipality. Four years later, the judge
decided that the land occupied by the petitioners, being public in nature, was beyond the
commerce of man and therefore could not be the subject of private occupancy.
5
The writ of
preliminary injunction was made permanent.
6

The decision was apparently not enforced, for the petitioners were not evicted from the place; in
fact, according to then they and the 128 other persons were in 1971 assigned specific areas or
space allotments therein for which they paid daily fees to the municipal government. Then, on
January 12, 1982, the Association of Concerned Citizens and Consumers of San Fernando filed
a petition for the immediate implementation of Resolution No. 29, to restore the subject property
"to its original and customary use as a public plaza thereby the respondent was tasked to
demolish the stalls in the subject place which was favored by the trial court.

The basic contention of the petitioners is that the disputed area is under lease to them by virtue
of contracts they had entered into with the municipal government.

ISSUE:
WON the lease agreement whereby the municipality of Cavite leased to the petitioner valid
given the fact the said area is dedicated for public use? -- NO

HELD:
Applying this well-settled doctrine, we rule that the petitioners had no right in the first place to
occupy the disputed premises and cannot insist in remaining there now on the strength of their
alleged lease contracts. The lease agreement is null and void.

According to article 344 of the Civil Code: "Property for public use in provinces and in towns
comprises the provincial and town roads, the squares, streets, fountains, and public waters, the
promenades, and public works of general service supported by said towns or provinces.
The said Plaza Soledad being a promenade for public use, the municipal council of Cavite could
not in 1907 withdraw or exclude from public use a portion thereof in order to lease it for the sole
benefit of the defendant Hilaria Rojas.

Furthermore, the Civil Code, article 1271, prescribes that everything which is not outside the
commerce of man may be the object of a contract, and plazas and streets are outside of this
commerce.

MPF
Maneclang v. IAC
144 SCRA 553

DOCTRINE: Finding that subject body of water is a creek belonging to the public domain,not
susceptible to private appropriation, a factual determination binding on the Supreme Court.

FACTS:
Maneclang filed a complaint for quieting of title over a certain fishpond located
within the four parcels of land belonging to them.
The trial court dismissed the complaint upon finding that the body of water
traversing the titled properties of petitioners is a creek constituting a tributary of a river;
therefore public in nature and not subject to private appropriation.

ISSUE:
Whether or not a creek can be registered under the Torrens System -- NO

HELD:
A creek is a recess/arm extending from a river and participating in the ebb and flow of the sea. It
is a property belonging to the public domain, It is a property belonging to the public domain. it is
not susceptible to appropriation and acquisitive prescription. As a public water, it cannot be
registered under the Torrens System in the name of any individual.

Its nature as property of the public domain cannot be modified by the construction of irrigation
dikes by the National Irrigation Authority, or by its conversion into a fishpond. Hence, a
compromise agreement adjudicating the ownership of such property in favor of an individual is
null and void. The compromise agreement has no legal effect since it is contrary to law and
public policy.

AMDG
Chavez v. Public Estates Authority
384 SCRA 152

DOCTRINE: Until now, the only way the government can sell to private parties government
reclaimed and marshy disposable lands of the public domain is for the legislature to pass a law
authorizing such sale. However, there exists a constitutional ban wherein private corporations
are prohibited from acquiring alienable lands of the public domain. These corporations may only
lease the lands from a period granted by the law.

FACTS:
The government, through the Commissioner of Public Highways, signed a contract with CDCP
to reclaim certain foreshore and offshore areas of Manila Bay under the MCCRRP. Later on
President Marcos signed PD No. 1084 and 1085 creating PEA and transferring to PEA the
reclaimed lands in the foreshore and offshore of the Manila Bay. In addition, a Memorandum of
Agreement was executed between PEA and CDCP wherein the latter acceded and transferred
its rights and interest in favor of the former as regards CDCPs reclaimed lands under MCCRRP.
During Aquinos administration, special patents as well as 3 TCTs (the lands were known as
Freedom Islands) were issued in favor of PEA.

PEA and AMARI, a private corporation, through negotiation but without conducting any public
bidding entered into a Joint Venture Agreement (JVA for brevity) for the development of the
Freedom Islands. A year later, Senate President Maceda described such JVA during his
privileged speech as the grandmother of all scams. Consequently, a joint investigation was
conducted and the report concluded that the JVA is illegal because what PEA seeks to do is to
transfer ownership of the reclaimed lands which are public lands hence inalienable to AMARI.
However, the Legal Task formed by Pres. Ramos upheld the legality of the JVA.

Phillipine Daily Inquirer and Today published reports that Pres. Ramos ordered that
renegotiations regarding the JVA be again made. Such JVA (now called Amended JVA) was
later on approved by Pres. Estrada. Petitioner Chavez prays that the Amended JVA be declared
null and void for it violating the Constitutional and statutory provisions.

ISSUE:
Whether or not AMARI, a private corporation may acquire the reclaimed lands? NO

HELD:
In this case, the SC traced back the laws governing reclaimed lands as regards its alienability.
The previous Constitutions including the 1987 Constitution has adopted the Regalian Doctrine
wherein it states that all public lands and waters are owned by the State. The court discussed
and emphasized also CA No. 141 which states that the only way the government can sell to
private parties government reclaimed and marshy disposable lands of the public domain is for
the legislature to pass a law authorizing such sale. In addition, the Constitution has established
that private corporations (such as AMARI) cannot acquire the reclaimed lands however; these
corporations are allowed to lease them. This rule is absolute.

Applying these provisions to the case, the reclaimed lands are classified as public property and
in order for PEA to sell these lands; there must be a legislative act granting such right to sell. In
addition, even if there exist an express provision in favor of PEA, such would still subject of the
constitutional ban as regards private corporation acquiring reclaimed alienable lands.

As mentioned and established already, these reclaimed lands are considered inalienable public
property. PD No. 1085 granting PEA the power to oversee the Freedom Islands did not in any
way convert the lands into alienable or disposable lands. The issuance of special patents by
Pres. Aquino as well as the TCTs also did not convert it into private lands. It must be noted that
the registration of public lands under Torrens system cannot convert it into private property.

GCG
Republic v. Court of Appeals
281 SCRA 639

DOCTRINE: When the sea moved towards the estate and the tide invaded it, the invaded
property became foreshore land and passed the realm of the public domain and accordingly
cannot be a subject of a free patent.

FACTS:
Respondent Josefina Morato filed a Free Patent Application on a parcel of land. The patent was
approved at the Register of Deeds of Quezon. Both the free patent and the title specifically
mandate that the land shall not be alienated nor encumbered within five years from the date of
the issuance of the patent (Sections 118 and 124 of CA No. 141, as amended).

Subsequently, the District Land Officer in Lucena City, acting upon reports that respondent
Morato had encumbered the land in violation of the condition of the patent, conducted an
investigation. Thereafter, it was established that the subject land is a portion of the Calauag Bay,
five (5) to six (6) feet deep under water during high tide and two (2) feet deep at low tide, and
not suitable to vegetation. Moreover, a portion of the land was mortgaged by respondent Morato
to respondents Nenita Co and Antonio Quilatan. The spouses Quilatan constructed a house on
the land. Another portion of the land was leased to Perfecto Advincula, where a warehouse was
constructed.
Petitioner filed an amended complaint against respondents Morato, spouses Nenita Co and
Antonio Quilatan, and the Register of Deeds of Quezon for the cancellation of title and reversion
of a parcel of land to the public domain, subject of a free patent in favor of respondent Morato,
on the grounds that the land is a foreshore land and was mortgaged and leased within the five-
year prohibitory period.

The lower court ruled that there was no violation of the 5-year period ban against alienating or
encumbering the land, because the land was merely leased and not alienated. It also found that
the mortgage to Nenita Co and Antonio Quilatan covered only the improvement and not the land
itself.

On appeal, the Court of Appeals affirmed the decision of the trial court. Thereafter, the Republic
of the Philippines filed the present petition.

ISSUE:
Whether the questioned land is a foreshore land and thus must be reverted to the public domain.
-- YES

HELD:
The Supreme Court found that the subject land was foreshore land, it nevertheless sustained
the award thereof to Respondent Morato. It defined a foreshore land as that parcel of land
which is between high and low water and left dry by the flux and reflux of the tides; it is that
strip of land that lies between the high and low water marks and that is alternatively wet and dry
according to the flow of the tide.

From the actual findings of the lower court, it was found out that years before the issuance of
the free patent to private respondent, the questioned land was subjected to several natural
calamities like earthquakes and typhoons that caused severe erosion of the land. Then private
respondent introduced improvements and developments to the land. At the time then of the
issuance of free patent of land to Morato, it was not covered by water but due to the gradual
sinking of the land caused by natural calamities, the sea advances had permanently invaded a
portion of subject land. During high tide, at least half of the land is 6 feet deep under water and
three feet deep during low tide. The Calauag Bay has extended up to a portion of the land.

Thus, uncontestedly, the land has become a foreshore land and is now a part of the public
domain pursuant to Article 420 of the New Civil Code being part of the shores defined
therein. Accordingly, it cannot be disposed of by the government and appropriated by a private
individual, i.e. be a subject of a free patent.

VCL IV
MIAA v. Court of Appeals
G.R. No. 155650

DOCTRINE: The term ports includes seaports and airports. The MIAA Airport Lands and
Buildings constitute a port constructed by the State. Under Article 420 of the Civil Code, the
MIAA Airport Lands and Buildings are properties of public dominion and thus owned by the
State or the Republic of the Philippines.

FACTS:
Manila International Airport Authority (MIAA) operates the Ninoy Aquino International Airport
Complex in Paraaque City. As operator of the international airport, MIAA administers the land,
improvements and equipment within the NAIA Complex.
The MIAA Charter transferred to MIAA approximately 600 hectares of land including the
runways and buildings (Airport Lands and Buildings) then under the Bureau of Air
Transportation. The MIAA Charter further provides that no portion of the land transferred to
MIAA shall be disposed of through sale or any other mode unless specifically approved by the
President of the Philippines.

OGCC (Office of the Government Corporate Counsel) issued Opinion No. 061, in which it said
that the Local Government Code of 1991 withdrew the exemption for real estate tax granted to
MIAA under Section 21 of the MIAA charter.

Therefore, MIAA was held to be delinquent in paying its taxes. The City of Paraaque Levied
upon the properties of MIAA, and posted invitations for public biddings of MIAAs properties.
MIAA filed with CA an action for prohibition / injunction. The City of Paraaque averred that
Section 193 of the Local Government code expressly withdrew tax exemptions from government
owned and controlled corporations (GOCCs).

CA dismissed the petition for filing beyond the 60 day reglementary period

ISSUE:
Whether properties of the MIAA are subject to real estate taxes. -- NO

HELD:
In the first place, MIAA is not a GOCC, it is an instrumentality of the government. MIAA is a
government instrumentality vested with corporate powers to perform efficiently its governmental
functions. MIAA is like any other government instrumentality, the only difference is that MIAA is
vested with corporate powers. As operator of the international airport, MIAA administers the
land, improvements and equipment within the NAIA Complex. The MIAA Charter transferred to
MIAA approximately 600 hectares of land, including the runways and buildings (Airport Lands
and Buildings) then under the Bureau of Air Transportation. The MIAA Charter further provides
that no portion of the land transferred to MIAA shall be disposed of through sale or any other
mode unless specifically approved by the President of the Philippines.

Furthermore, Airport Lands and Buildings of MIAA are property of public dominion and therefore
owned by the State or the Republic of the Philippines. Article 419 of the Civil Code provides,
The Airport Lands and Buildings of MIAA are property of public dominion and therefore owned
by the State or the Republic of the Philippines.

The Civil Code provides:
ARTICLE 419. Property is either of public dominion or of private ownership.

ARTICLE 420. The following things are property of public dominion:
(1) Those intended for public use, such as roads, canals, rivers, torrents, ports
and bridges constructed by the State, banks, shores, roadsteads, and others of
similar character;
(2) Those which belong to the State, without being for public use, and are
intended for some public service or for the development of the national wealth.
(Emphasis supplied)

ARTICLE 421. All other property of the State, which is not of the character stated in the
preceding article, is patrimonial property.

ARTICLE 422. Property of public dominion, when no longer intended for public use or for
public service, shall form part of the patrimonial property of the State.

No one can dispute that properties of public dominion mentioned in Article 420 of the Civil Code,
like roads, canals, rivers, torrents, ports and bridges constructed by the State, are owned by
the State. The term ports includes seaports and airports. The MIAA Airport Lands and
Buildings constitute a port constructed by the State. Under Article 420 of the Civil Code, the
MIAA Airport Lands and Buildings are properties of public dominion and thus owned by the
State or the Republic of the Philippines.

The Airport Lands and Buildings are devoted to public use because they are used by the public
for international and domestic travel and transportation. The fact that the MIAA collects terminal
fees and other charges from the public does not remove the character of the Airport Lands and
Buildings as properties for public use. The operation by the government of a tollway does not
change the character of the road as one for public use. Someone must pay for the maintenance
of the road, either the public indirectly through the taxes they pay the government, or only those
among the public who actually use the road through the toll fees they pay upon using the road.
The tollway system is even a more efficient and equitable manner of taxing the public for the
maintenance of public roads.

The charging of fees to the public does not determine the character of the property whether it is
of public dominion or not. Article 420 of the Civil Code defines property of public dominion as
one intended for public use. Even if the government collects toll fees, the road is still intended
for public use if anyone can use the road under the same terms and conditions as the rest of
the public. The charging of fees, the limitation on the kind of vehicles that can use the road, the
speed restrictions and other conditions for the use of the road do not affect the public character
of the road.

The terminal fees MIAA charges to passengers, as well as the landing fees MIAA charges to
airlines, constitute the bulk of the income that maintains the operations of MIAA. The collection
of such fees does not change the character of MIAA as an airport for public use. Such fees are
often termed users tax. This means taxing those among the public who actually use a public
facility instead of taxing all the public including those who never use the particular public facility.
A users tax is more equitable a principle of taxation mandated in the 1987 Constitution.

The Airport Lands and Buildings of MIAA, which its Charter calls the principal airport of the
Philippines for both international and domestic air traffic, are properties of public dominion
because they are intended for public use. As properties of public dominion, they indisputably
belong to the State or the Republic of the Philippines.
Being a property of public dominion, the properties of MIAA are beyond the commerce of man.

FXRL
Lanzar v. Director of Lands
78 SCRA 130

DOCTRINE: Properties formed by accretion through the action of the sea belong to the public
domain unless the government decrees otherwise.

FACTS:
Petitioner Lanzar filed an application for the registration of
a property in Molo, Iloilo City.
The Director of Lands filed an opposition to such
application on the following grounds:
o the said property is foreshore land
o the City of Iloilo needs the same property as a road right of way for Molo Blvd.
o the petitioner had not possessed the property in such manner as to grant him
ownership
The CFI ruled in favor of the plaintiff on the ground of valid prescription.
The CA reversed, ruling in favor of the respondent on the ground that the property was
formed by accretion from the sea making it public domain.

ISSUE:
W/N the title to property formed by action of the sea as an accretion may be registered to an
applicant on the basis of adverse possession for over 30 years --

HELD: The land is of public domain.

Article 4 of the Law of Waters provides that:
Lands added to the shores by accretions and alluvium deposits caused by the action of the sea,
form part of the public domain. When they are no longer washed by the waters of the sea, and
are not necessary for the purposes of public utility, or for the establishment of special industries,
or for the coastguard service, the Government shall declare them to be the property of the
owners of the estates adjacent thereto and as an increment thereof.

In sum, properties formed by accretion through the action of the sea belong to the public domain
unless the government decree otherwise. As applied to the case on hand, it is undisputed that
the property was formed by accretion through the action of the sea and that there was no such
declaration by the government that the said property is no longer part of public domain. As such,
the petitioner could not have acquired the property by mere adverse possession for the requisite
number of years.

RSDM
Ignacio v. Director of Lands
108 Phil. 335

DOCTRINE: Citing Article 457 of the New Civil Code (Article 366, Old Civil Code), which
provides that: To the owners of lands adjoining the banks of rivers belong the accretion which
they gradually receive from the effects of the current of the waters.
The article cited is clearly inapplicable because it refers to accretion or deposits on the banks of
rivers, while the accretion in the present case was caused by action of the Manila Bay.

FACTS:
Ignacio filed an application alleging that among others that he owned the parcel applied for by
right of accretion. The Director of Lands, Laureano Valeriano and Domingo Gutierrez filed
oppositions. Gutierrez later withdrew his opposition. The Director of Lands claimed the parcel
applied for as a portion of the public domain, for the reason that neither the applicant nor his
predecessor-in-interest possessed sufficient title thereto, not having acquired it either by
composition title from the Spanish government or by possessory information title under the
Royal Decree and that he had not possessed the same openly, continuously and adversely
under a bona fide claim of ownership since July 26, 1894. In his turn, Valeriano alleged he was
holding the land by virtue of a permit granted him by the Bureau of Fisheries and approved by
the President.

It is not disputed that the land applied for adjoins a parcel owned by the applicant which he had
acquired from the Government by virtue of a free patent title in 1936. It has also been
established that the parcel in question was formed by accretion and alluvial deposits caused by
the action of the Manila Bay which boarders it on the southwest.

On the other hand, the Director of Lands sought to prove that the parcel is foreshore land,
covered by the ebb and flow of the tide and, therefore, formed part of the public domain.
After hearing, the trial court dismissed the application, holding that the parcel formed part of the
public domain.

ISSUE:
Whether or not the disputed land is part of public domain -- Yes. It is part of public domain

HELD:
Appellant contends that the parcel belongs to him by the law of accretion, having been formed
by gradual deposit by action of the Manila Bay, and he cites Article 457 of the New Civil Code
(Article 366, Old Civil Code), which provides that:

To the owners of lands adjoining the banks of rivers belong the accretion which they
gradually receive from the effects of the current of the waters.

The article cited is clearly inapplicable because it refers to accretion or deposits on the banks of
rivers, while the accretion in the present case was caused by action of the Manila Bay.
Interpreting Article 4 of the Law of Waters of 1866, in the case of Natividad vs. Director of Lands,
(CA) 37 Off. Gaz., 2905, it was there held that:

Article 4 of the Law of Waters of 1866 provides that when a portion of the shore is no longer
washed by the waters of the sea and is not necessary for purposes of public utility, or for the
establishment of special industries, or for coastguard service, the government shall declare it to
be the property of the owners of the estates adjacent thereto and as an increment thereof. We
believe that only the executive and possibly the legislative departments have the authority and
the power to make the declaration that any land so gained by the sea, is not necessary for
purposes of public utility, or for the establishment of special industries, on for coast-guard
service. If no such declaration has been made by said departments, the lot in question forms
part of the public domain.

MRAM
Villarico v. Court of Appeals
309 SCRA 193

DOCTRINE: Private persons cannot own forest lands. Possession thereof, no matter how long,
does not ripen into a registrable title.

FACTS:
Spouses Villarico filed an application for confirmation of title over a parcel of land in
Meycauayan, Bulacan. The applicants alleged that 1)they are the absolute owners of the
property having bought the same from Segundo Villarico and Mercedes Cardenas, 2) they and
their predecessors-in-interest have been in actual, open, adverse and continuous possession
thereof for more than 30 years, 3) they are not aware of any mortgage or encumbrance thereon
nor of any person having an estate or interest therein, and 4) the land involved is not within the
forest zone or government reservation.

Marcos Camargo opposed the application for the registration of the land claiming that he is the
real owner thereof. The Government interposed its opposition, through the Director of Forestry,
averring that the land in question is part of the public domain, within the unclassified area in
Meycauayan, Bulacan hence, it is not available for private appropriation.

The trial court dismissed the case saying that a certificate of title is void when it covers property
of the public domain classified as forest or timber and mineral lands. Any title thus issued on
non-disposable lots, even in the hands of an innocent purchaser for value, should be cancelled.

ISSUE:
W/N the property in question can be registered by the applicant or by the oppositor or by their
respective predecessors-in-interest?

HELD:
The Supreme Court Held in the negative. Since there is no showing that a declassification has
been made by the Director of Forestry declaring the land in question as disposable or alienable,
the land remains to be part of the public domain as a forest land. Considering this, possession
of the land in question by the applicants and/or their predecessors-in-interest even for more
than 30 years does not convert the land into private property capable of private appropriation.

The SC stated that forest lands cannot be owned by private persons. Its possession, no matter
how long it is, does not ripen into a registrable title. Furthermore, it must be noted that the
adverse possession which may be the basis of a grant of title or confirmation of an imperfect
title refers only to alienable or disposable portions of the public domain.

FMM
Villanueva v. Castaneda
154 SCRA 142

DOCTRINE: A public plaza is beyond the commerce of man and so cannot be the subject of
lease or any other contractual undertaking.

FACTS:
On 7 November 1961, the municipal council of San Fernando adopted Resolution No. 28
authorizing some members of the Fernandino United Merchants and Traders Association
(herein Petitioners) to construct permanent stags and sell on a strip of land measuring 12 by
77 meters along Mercado Street, San Fernando, Pampanga. However, the action was
protested on 10 November 1961 through a civil case. The issuance by the Court of First
Instance of Pampanga of a writ of preliminary injunction prevented the Petitioners from
constructing the stalls until final resolution of the controversy.

On 18 January 1964, while the civil case was pending, the municipal council of San Fernando
adopted Resolution G.R. No. 29 declaring the subject area as the parking place and as the
public plaza of the municipality, thereby impliedly revoking Resolution No. 218, series of 1961.

Four years later, on 2 November 1968, Judge Andres C. Aguilar decided the case and held that
the land occupied by the Petitioners, being public in nature, was beyond the commerce of man
and thus could not be subject of private occupancy. The writ of preliminary injunction was made
permanent. However, the decision was not enforced for the Petitioners were not evicted from
the subject area. The latter claimed that they and 128 other persons were in 1971 assigned
specific areas for which they paid daily fees to the municipal government. This problem
appears to have worsened for some more years under a presumable uneasy truce among the
protagonists, none of whom made any move, for some reason that does not appear on record.

On 12 January 1982, the Association of Concerned Citizens and Consumers of San Fernando
filed a petition for the immediate implementation of Resolution No. 29, to restore the subject
property to its original and customary use as a public plaza.

Acting thereon after an investigation conducted by the municipal attorney, Vicente a. Macalino
(herein Respondent), as officer in charge of the office of the mayor of San Fernando, issued
on 14 June 1982, a resolution requiring the municipal treasurer and the municipal engineer to
demolish the stalls in the subject area beginning 1 July 1982.

The Petitioners reacted by filing a petition for prohibition with the Court of First Instance of
Pampanga on 26 June 1982. However, herein Respondent judge denied the petition as well as
the subsequent motion for reconsideration.

Thus this petition for certiorari to challenge the decision. The basic contention of the Petitioners
is that the disputed area is under lease to them by virtue of contracts they had entered into with
the municipal government, first in 1961 insofar as the original occupants were concerned, and
later with them and the other petitioners by virtue of the space allocations made in their favor in
1971 for which they are paying daily fees. Upon request by the Petitioners, the Supreme Court
(SC) issued a temporary restraining order to preserve the status quo between the parties
pending the decision.

ISSUE:
Whether the lease of the subject area is valid. -- NO

HELD:
The SC held that a public plaza is beyond the commerce of man and cannot be subject of lease
or any other contractual undertaking.

The SC cited the following cases:

1. In Municipality of Cavite vs. Rojas, the SC declared as null and void the lease of a
public plaza of the said municipality in favor of a private person;
2. In Muyot vs. de la Fuente, the SC held that the City of Manila could not lease a
portion of a public sidewalk on Plaza Sta. Cruz, being likewise beyond the commerce
of men; and
3. In Espiritu vs. Municipal Council of Pozorrubio, the SC declared: There is absolutely
no question that the town plaza cannot be used for the construction of market stalls,
specially of residences, and that such structures constitute a nuisance subject to
abatement according to law. Town plazas are properties of public dominion, to be
devoted to public use and to be made available to the public in general They are
outside the common of man and cannot be disposed of or even leased by the
municipality to private parties.

And ruled that applying this well-settled doctrine, the petitioners had no right in the first place to
occupy the disputed premises and cannot insist in remaining there now on the strength of their
alleged lease contracts.

The SC dismissed the petition and affirmed the decision and order of the lower court. It also
lifted the temporary restraining order requested by the petitioners.

RGGM
Dacanay Jr. v. Asistio Jr.
208 SCRA 404

DOCTRINE: A public street is property for public use hence outside the commerce of men. The
right of the public to use the city streets may not be bargained away through a contract.

FACTS:
This is a petition for mandamus to the non-action of the city government of Caloocan in
accordance with the decision of the RTC to evict the occupants of a flea market located in the
streets of Caloocan.

January 5, 1979 Metropolitan Manila Commission enacted an ordinance allowing the use of
streets for the purpose of flea markets subject to several conditions.
1987 Mayor Martinez caused the demolition of the flea markets and the stall owners filed a
case against such action.

RTC dismissed the case on the ground that the streets in questions are of public dominion,
hence outside the commerce of man.
After the decision came out, there was a change in the city administration and current mayor
(Asistio) did not pursue the action of the previous mayor and left the flea markets in the streets
as is.

Dacanay filed a petition for mandamus to remove the stalls in their street.

ISSUE:
May public streets be leased or licensed to market stallholders by virtue of a city ordinance or
resolution of Metropolitan Manila Commission? -- NO

HELD:
A public street is property for public use hence outside the commerce of man. Being outside the
commerce of man, it may not be the subject of lease or other contract. Any executive order or
city resolution cannot change the nature of the public street because it is going to be contrary to
the general law.

MCSS
Cebu Oxygen and Acetylene Co. v. Bercilles
66 SCRA 431

FACTS:
A portion of land, sought to be registered, was declared, through law, an abandoned road. The
lot was awarded to the petitioner for being the highest bidder.

The Assistant Provincial Fiscal of Cebu filed a motion to dismiss the application on the ground
that the property sought to be registered being a public road intended for public use is
considered part of the public domain and therefore outside the commerce of man.

ISSUE:
WON the portion of land is susceptible to registration by a private individual. -- YES

HELD:
Revised Charter of Cebu, under section 31, provides that, the City Council shall have the power
to close any city road, street or alley, etc, withdrawn from public servitude, may be used or
conveyed for any purpose.

It is undoubtedly clear that the City of Cebu is empowered to close a city road or street. Such
power is discretionary and will not ordinarily be controlled or interfered with by the courts,
absent a plain case of abuse or fraud or collusion. It follows that such withdrawn portion
becomes patrimonial property which can be the object of an ordinary contract.

NKVS
Laurel v. Garcia
187 SCRA 797

DOCTRINE: An abandonment of the intention to use the property for public service and to
make it patrimonial property under Article 422 of the Civil Code must be definite Abandonment
and it cannot be inferred from the non-use alone specially if the non-use was attributable not to
the government's own deliberate and indubitable will but to a lack of financial support to repair
and improve the property Abandonment must be a certain and positive act based on correct
legal premises.

FACTS:
These are two petitions for prohibition seeking to enjoin respondents, their representatives and
agents from proceeding with the bidding for the sale of the 3,179 square meters of land at
Tokyo, Japan scheduled on February 21, 1990.

The subject property in this case is 1 of the 4 properties in Japan acquired by the Philippine
government under the Reparations Agreement entered into with Japan on May 9, 1956. The
properties and the capital goods and services procured from the Japanese government for
national development projects are part of the indemnification to the Filipino people for their
losses in life and property and their suffering during World War II.

A proposal was presented to President Corazon C. Aquino by former Philippine Ambassador to
Japan, Carlos J. Valdez, to make the property the subject of a lease agreement with a
Japanese firm. No change of ownership or title shall occur. The Philippine government retains
the title all throughout the lease period and thereafter. However, the government has not acted
favorably.

On July 25, 1987, the President issued Executive Order No. 296 entitling non-Filipino citizens or
entities to avail of separations' capital goods and services in the event of sale, lease or
disposition. The four properties in Japan including the Roppongi were specifically mentioned in
the first "Whereas" clause.

Amidst opposition by various sectors, the Executive branch of the government has been
pushing its decision to sell the reparations properties starting with the Roppongi lot. The
property has twice been set for bidding at a minimum floor price of $225 million.

ISSUE:
W/N the Roppongi property and others of its kind be alienated by the Philippine Government. --
NO

HELD:
NO, the subject property cannot be alienated by the government, even if the property has not
been in use for a long time.

Vice President Laurel asserts that the lands were acquired as part of the reparations for
diplomatic and consular use by the Philippine government. Laurel states that the Roppongi
property is classified as one of public dominion, and not of private ownership under Article 420
of the Civil Code.

The petitioner submits that the Roppongi property comes under "property intended for public
service" in paragraph 2 of the above provision. He states that being one of public dominion, no
ownership by anyone can attach to it, not even by the State. The Roppongi and related
properties were acquired for "sites for chancery, diplomatic, and consular quarters, buildings
and other improvements. The petitioner states that they continue to be intended for a necessary
service. They are held by the State in anticipation of an opportune use. (Citing 3 Manresa 65-
66). Hence, it cannot be appropriated, is outside the commerce of man, or to put it in more
simple terms, it cannot be alienated nor be the subject matter of contracts (Citing Municipality of
Cavite v. Rojas, 30 Phil. 20 [1915]). Noting the non-use of the Roppongi property at the moment,
the petitioner avers that the same remains property of public dominion so long as the
government has not used it for other purposes nor adopted any measure constituting a removal
of its original purpose or use.

As property of public dominion, the Roppongi lot is outside the commerce of man. It cannot be
alienated. Its ownership is a special collective ownership for general use and enjoyment, an
application to the satisfaction of collective needs, and resides in the social group. The purpose
is not to serve the State as a juridical person, but the citizens; it is intended for the common and
public welfare and cannot be the object of appropration

The applicable provisions of the Civil Code are:
ART. 419. Property is either of public dominion or of private ownership.
ART. 420. The following things are property of public dominion
(1) Those intended for public use, such as roads, canals, rivers, torrents,
ports and bridges constructed by the State, banks shores roadsteads,
and others of similar character;
(2) Those which belong to the State, without being for public use, and are
intended for some public service or for the development of the national
wealth.
ART. 421. All other property of the State, which is not of the character stated in the
preceding article, is patrimonial property.

The Roppongi property is correctly classified under paragraph 2 of Article 420 of the Civil Code
as property belonging to the State and intended for some public service.

The fact that the Roppongi site has not been used for a long time for actual Embassy service
does not automatically convert it to patrimonial property. Any such conversion happens only if
the property is withdrawn from public use. A property continues to be part of the public domain,
not available for private appropriation or ownership until there is a formal declaration on the part
of the government to withdraw it from being such.

An abandonment of the intention to use the Roppongi property for public service and to make it
patrimonial property under Article 422 of the Civil Code must be definite Abandonment cannot
be inferred from the non-use alone specially if the non-use was attributable not to the
government's own deliberate and indubitable will but to a lack of financial support to repair and
improve the property Abandonment must be a certain and positive act based on correct legal
premises.

A mere transfer of the Philippine Embassy to Nampeidai in 1976 is not relinquishment of the
Roppongi property's original purpose.

Moreover, President Aquinos approval of the recommendation by the investigating committee
to sell the Roppongi property was premature or, at the very least, conditioned on a valid change
in the public character of the Roppongi property. It does not have the force and effect of law
since the President already lost her legislative powers. The Congress had already convened for
more than a year. Assuming that the Roppongi property is no longer of public dominion, there is
another obstacle to its sale by the respondents. There is no law authorizing its conveyance, and
thus, the Court sees no compelling reason to tackle the constitutional issue raised by petitioner
Ojeda.

AMPS
Palanca v. Commonwealth
G.R. No. 46373

DOCTRINE: A private person may not acquire ownership of a property of public dominion (such
as navigable waters) through prescription or even by virtue of a Torrens title.

FACTS:
On the 17
th
of July 1919 Carlos Palanca obtained a decision from the Court of First Instance of
Bulacan for the registration of 4 parcels of land, each adjacent to the others and separated only
among them by some waterways called esteros. Before the decision is read, the government
through the fiscal general presented a petition for reopening of the case to check on the
existence of esteros and rivers that are navigable in the terrain, with the intention of
setting/including the said circumstance in the map and so that it be excluded from the
registration.

The Court denied this petition because they consider it unnecessary. Despite its presence in
the terrain, the registration of these will not affect the rights of propriety of the Insular
Government or the public use of the said waterways, which will always remain safe with article
39 of the Law Register of Propriety.

Much later, the Government of the Islands of the Philippines presented the current action
against Carlos Palanca claiming that he is illegally occupying portions of the Viray River and
Sapang Sedaria estero, which are navigable and was asked that he be obliged to open it, and
leave it in its primitive state. The court discontinues this action; but elevated it to the Court of
Appeals.

The Court of Appeals declared that the said River Viray and Sapang Sedaria estero are of the
possession and use of the public and that the right acquired by Carlos Palanca over the land in
which the waterways are located does not affect the right of the State over them, as goods
destined for public use.

Carlos Palanca in his turn elevated an appeal through certiorari to this decision of the Court of
Appeals to the Supreme Court.

ISSUE:
What is the status of the river and estero?

HELD:
The river and the estero being navigable, useful for commerce, the navigation and fishing,
have the character of public possession and its legal condition in this sense has not been
affected by the possession of Carlos Palance, whenever has been the time of this possession,
because it does not admit any prescription against the State over goods of public use.

The Court of Appeals declared in view of the proofs presented in the case that the River Viray
and the Sapang Sedaria Estero are found within the land of Carlos Palanca that was the
object of the decision of the Court of First Instance, and that said river and estero are
navigable and of common use, serves as a communication between two courses of water which
flows into the Manila Bay, and that the water current in them are of the possession and use of
the public, useful for commerce, navigation and fishing and that they have these conditions
when the land that surrounds them still pertains to the State.

These being the fact, the request or the recourse opened before this tribunal should be denied,
thus confirming the decision pronounced by the Court of Appeals.

We cannot find basis for the request of the appellant that the decision pronounced by the Court
of First Instance and the Torrens title issued, in its consequence, in favor of Carlos Palanca
establish the non-existence of the river and estero in question as navigable. In that decision, it
is well said that the terrain is crossed by some waterways called esteros. There is no
declaration whatsoever that these esteros are not navigable. On the other hand, when the
government asked for opportunity to prove that some of these esteros are navigable, the court
denied them this opportunity due to the reason that, if they really are navigable, the right of the
Government over them, by reason of such condition of the River Viray and the Sapang Sedaria
estero as navigable, have not been the object of the judicial decision.

As for the rest, the river and the estero being navigable, useful for commerce, the navigation
and fishing, have the character of public possession and its legal condition in this sense has not
been affected by the possession of Carlos Palance, whenever has been the time of this
possession, because it does not admit any prescription against the State over goods of public
use.

For these considerations, the appeal is denied and the decision of the court of appeals is
confirmed.

KGS
Rabuco v. Villegas
G.R. No. L-24661

DOCTRINES:
1. RA 3120 converting communal lands into alienable lands is constitutional.
2. Legislative body has wide discretion to reclassify State property.

FACTS:
The legislatures enacted RA3120 converting certain parcels of land in the City of Manila which
are reserved as communal property into disposable or alienable lands of the state and providing
for their subdivision and sale. It includes a lot located in San Andres, Malate, which was
occupied by petitioners (illegal settlers). The City Mayor of Manila wanted to demolish and eject
said occupants. Subsequently, a large fire gutted the Malate area, which includes said property.
City officials then took over the lot and kept petitioners reconstructing or repairing their burned
dwellings. The petitioners insisted that RA 3120 should be implemented to them as the tenants
and bona fide occupants thereof.

ISSUE:
Can a communal property be converted into disposable or alienable land through legislation? --
YES

HELD:
The Court herein upholds the constitutionality of Republic Act 3120 on the strength of the
established doctrine that the subdivision of communal land of the State (although titled in the
name of the municipal corporation) and conveyance of the resulting subdivision lots by sale on
installment basis to bona fide occupants by Congressional authorization and disposition does
not constitute infringements of the due process clause or the eminent domain provisions of the
Constitution but operates simply as a manifestation of the legislature's right of control and power
to deal with State property.

Respondents city officials' contention that the Act must be stricken down as unconstitutional for
depriving the city of Manila of the lots in question and providing for their sale in subdivided small
lots to bona fide occupants or tenants without payment of just compensation is untenable and
without basis, since the lots in question are manifestly owned by the city in its public and
governmental capacity and are therefore public property over which Congress had absolute
control as distinguished from patrimonial property owned by it in its private or proprietary
capacity of which it could not be deprived without due process and without just compensation.

Here, Republic Act 3120 expressly declared that the properties were "reserved as communal
property" and ordered their conversion into "disposable and alienable lands of the State" for sale
in small lots to the bona fide occupants thereof. It is established doctrine that the act of
classifying State property calls for the exercise of wide discretionary legislative power which will
not be interfered with by the courts.

The case of Salas vs. Jarencio wherein the Court upheld the constitutionality of Republic Act
4118 whereby Congress in identical terms as in Republic Act 3120 likewise converted another
city lot (Lot 1-B-2-B of Block 557 of the cadastral survey of Manila also in Malate) which was
reserved as communal property into disposable land of the State for resale in small lots by the
Land Tenure, Administration to the bona fide occupants is controlling in the case at bar.

The Court therein reaffirmed the established general rule that "regardless of the source or
classification of land in the possession of a municipality, excepting those acquired with its own
funds in its private or corporate capacity, such property is held in trust for the State for the
benefit of its inhabitants, whether it be for governmental or proprietary purposes. It holds such
lands subject to the paramount power of the legislature to dispose of the same, for after all it
owes its creation to it as an agent for the performance of a part of its public work, the
municipality being but a subdivision or instrumentality thereof for purposes of local
administration. Accordingly, the legal situation is the same as if the State itself holds the
property and puts it to a different use"
9
and stressed that "the property, as has been previously
shown, was not acquired by the City of Manila with its own funds in its private or proprietary
capacity. That it has in its name a registered title is not questioned, but this title should be
deemed to be held in trust for the State as the land covered thereby was part of the territory of
the City of Manila granted by the sovereign upon its creation."

There as here, the Court holds that the Acts in question (Republic Acts 4118 in Salas and
Republic Act 3120 in the case at bar) were intended to implement the social justice policy of the
Constitution and the government program of land for the landless and that they were not
"intended to expropriate the property involved but merely to confirm its character as communal
land of the State and to make it available for disposition by the National Government: ... The
subdivision of the land and conveyance of the resulting subdivision lots to the occupants by
Congressional authorization does not operate as an exercise of the power of eminent domain
without just compensation in violation of Section 1, subsection (2), Article III of the Constitution,
but simply as a manifestation of its right and power to deal with state property."

JPOT
Macasiano v. Diokno
G.R. No. 97764

DOCTRINE: "Verily, the powers of a local government unit are not absolute. They are subject to
limitations laid down by the Constitution and the laws such as our Civil Code. Moreover, the
exercise of such powers should be subservient to paramount considerations of health and well-
being of the members of the community."

FACTS:
On June 13, 1990, the respondent municipality passed Ordinance No. 86, Series of 1990 which
authorized the closure of J. Gabriel, G.G. Cruz, Bayanihan, Lt. Garcia Extension and Opena
Streets located at Baclaran, Paranaque, Metro Manila and the establishment of a flea market
thereon. July 20, 1990, the Metropolitan Manila Authority approved Ordinance No. 86, s. 1990
of the municipal council of respondent municipality subject to the following conditions:
1. That the aforenamed streets are not used for vehicular traffic, and that the majority of
the residents do not oppose the establishment of the flea market/vending areas thereon;
2. That the 2-meter middle road to be used as flea market/vending area shall be marked
distinctly, and that the 2 meters on both sides of the road shall be used by pedestrians;
3. That the time during which the vending area is to be used shall be clearly designated;
4. That the use of the vending areas shall be temporary and shall be closed once the
reclaimed areas are developed and donated by the Public Estate Authority.

June 20, 1990, Mayor Walfrido N. Ferrer to enter into contract with any service cooperative for
the establishment, operation, maintenance and management of flea markets and/or vending
areas. On August 8, 1990, respondent municipality and respondent Palanyag, a service
cooperative, entered into an agreement whereby the latter shall operate, maintain and manage
the flea market in the aforementioned streets with the obligation to remit dues to the treasury of
the municipal government of Paranaque. Consequently, market stalls were put up by
respondent Palanyag on the said streets. On September 13, 1990, petitioner Brig. Gen.
Macasiano, PNP Superintendent of the Metropolitan Traffic Command, ordered the destruction
and confiscation of stalls along G.G. Cruz and J. Gabriel St. in Baclaran. These stalls were later
returned to respondent Palanyag. October 16, 1990, petitioner Brig. General Macasiano wrote a
letter to respondent Palanyag giving the latter ten (10) days to discontinue the flea market;
otherwise, the market stalls shall be dismantled. Hence, on October 23, 1990, respondents
municipality and Palanyag filed with the trial court a joint petition for prohibition and mandamus
with damages and prayer for preliminary injunction, to which the petitioner filed his
memorandum/opposition to the issuance of the writ of preliminary injunction. October 24, 1990,
the trial court issued a temporary restraining order to enjoin petitioner from enforcing his letter-
order of October 16, 1990 pending the hearing on the motion for writ of preliminary injunction.
On December 17, 1990, the trial court issued an order upholding the validity of Ordinance No.
86 s. 1990 of the Municipality of Paranaque and enjoining petitioner Brig. Gen. Macasiano from
enforcing his letter-order against respondent Palanyag.

ISSUE:
W/N an ordinance or resolution issued by the municipal council of Paranaque authorizing the
lease and use of public streets or thoroughfares as sites for flea markets is valid. -- NO

HELD:
The Executive Order issued by acting Mayor Robles authorizing the use of Heroes del '96
Street as a vending area for stallholders who were granted licenses by the city government
contravenes the general law that reserves city streets and roads for public use. Mayor Robles'
Executive Order may not infringe upon the vested right of the public to use city streets for the
purpose they were intended to serve: i.e., as arteries of travel for vehicles and pedestrians. The
Solicitor General furthers the matter with his observation, "Verily, the powers of a local
government unit are not absolute. They are subject to limitations laid down by the Constitution
and the laws such as our Civil Code. Moreover, the exercise of such powers should be
subservient to paramount considerations of health and well-being of the members of the
community. Every local government unit has the sworn obligation to enact measures that will
enhance the public health, safety and convenience, maintain peace and order, and promote the
general prosperity of the inhabitants of the local units. Based on this objective, the local
government should refrain from acting towards that which might prejudice or adversely affect
the general welfare." Moreover, the municipality did not even comply with the guidelines set
forth by the Metropolitan Manila Authority. Even if we were to argue for purposes of debate, the
city of Paranaque's claim would still be bereft and lacking in reason.

ACCORDINGLY, the petition is GRANTED and the decision of the respondent Regional Trial
Court dated December 17, 1990 which granted the writ of preliminary injunction enjoining
petitioner as PNP Superintendent, Metropolitan Traffic Command from enforcing the demolition
of market stalls along J. Gabriel, G.G. Cruz, Bayanihan, Lt. Garcia Extension and Opena streets
is hereby RESERVED and SET ASIDE.
SO ORDERED.

MLAV
Province of Zamboanga del Norte v. City of Zamboanga
G.R. No. L-24440

DOCTRINE: Under the law of Municipal Corporations, properties which are devoted to public
service are deemed public and the rest remain patrimonial. Under this norm, to be considered
public, it is enough that the property be held and, devoted for governmental purposes like local
administration, public education, public health, etc.

FACTS:
On October 12, 1936, Commonwealth Act 39 was approved converting the Municipality of
Zamboanga into Zamboanga City. It further provided that buildings and properties which the
province shall abandon upon the transfer of the capital to another place will be acquired and
paid for by the City of Zamboanga at a price to be fixed by the Auditor General.

The properties and buildings referred to consisted of 50 lots and some buildings constructed
thereon, located in the City of Zamboanga and covered individually by Torrens certificates of
title in the name of Zamboanga Province. Pursuant to CA 39, the Auditor General fixed the
value of the properties and buildings in question left by Zamboanga Province in Zamboanga
City at P1,294,244.00.

When RA 711 was approved dividing the province of Zamboanga into Zamboanga del Norte
and Zamboanga del Sur, assets and obligations of the previous Zamboanga province were
divided as follows: 54.39% for Zamboanga del Norte and 45.61% for Zamboanga del Sur.
Zamboanga del Norte therefore became entitled to 54.39% of P1,294,244.00, the total value of
the lots and buildings in question, or P704,220.05 payable by Zamboanga City.

The Executive Secretary issued a ruling

holding that Zamboanga del Norte had a vested right as
owner of the properties mentioned in Sec. 50 of CA 39, and is entitled to the price thereof,
payable by Zamboanga City. This ruling revoked the previous Cabinet Resolution conveying all
the said 50 lots and buildings thereon to Zamboanga City when the provincial capital of the then
Zamboanga Province was transferred to Dipolog.

The Secretary of Finance then authorized the Commissioner of Internal Revenue to deduct an
amount equal to 25% of the regular internal revenue allotment for the City of Zamboanga. The
deductions, all aggregating P57,373.46, was credited to the province of Zamboanga del Norte,
in partial payment of the P704,220.05 due it.

On June 17, 1961, RA 3039 was approved amending Sec. 50 of CA 39 by providing that
buildings, properties and assets belonging to the former province of Zamboanga and located
within the City of Zamboanga are transferred, free of charge, in favor of the said City of
Zamboanga.

Consequently, the Secretary of Finance ordered the CIR to stop from effecting further payments
to Zamboanga del Norte and to return to Zamboanga City the sum taken from it out of the
internal revenue allotment of Zamboanga del Norte.

Zamboanga del Norte filed a complaint for relief with Preliminary Mandatory Injunction against
Zamboanga City, the Secretary of Finance and the Commissioner of Internal Revenue. They
prayed that 1) RA 3039 be declared unconstitutional, 2) plaintiff's rights and obligations under
said law be declared, 3) they be reimbursed the sum paid to defendant City, and 4) the latter be
ordered to continue paying the balance of its internal revenue allotments.

The lower court ruled in favor of the plaintiff.

ISSUE:
WON RA 3039 is valid PARTLY VALID

HELD:
Applying the norm obtaining under the principles constituting the law of Municipal Corporations,
all those of the 50 properties in question which are devoted to public service are deemed public;
the rest remain patrimonial. Under this norm, to be considered public, it is enough that the
property be held and, devoted for governmental purposes like local administration, public
education, public health, etc.

Following this classification, RA 3039 is valid insofar as it affects the lots used as capitol site,
school sites and its grounds, hospital and leprosarium sites and the high school playground
sites a total of 24 lots since these were held by the former Zamboanga province in its
governmental capacity and therefore are subject to the absolute control of Congress.

Regarding the several buildings existing on the lots above-mentioned, the records do not
disclose whether they were constructed at the expense of the former Province of Zamboanga.
Considering however the fact that said buildings must have been erected even before 1936
when CA 39 was enacted and the further fact that provinces then had no power to authorize
construction of buildings at their own expense,

it can be assumed that said buildings were
erected by the National Government, using national funds. Even assuming that provincial funds
were used, still the buildings constitute mere accessories to the lands, which are public in nature,
and so, they follow the nature of said lands, i.e., public.

But RA 3039 cannot be applied to deprive Zamboanga del Norte of its share in the value of the
rest of the 26 remaining lots which are patrimonial properties since they are not being utilized for
distinctly, governmental purposes. Moreover, the fact that these 26 lots are registered
strengthens the proposition that they are truly private in nature. However, the fact that the 24
lots used for governmental purposes are also registered is of no significance since registration
cannot convert public property to private.

It results then that Zamboanga del Norte is still entitled to collect from the City of Zamboanga
the former's 54.39% share in the 26 properties which are patrimonial in nature.

DJTV
Director of Lands v. MERALCO
153 SCRA 686

DOCTRINE: Open, exclusive and undisputed possession of alienable public land for the period
prescribed by law creates the legal fiction whereby the land ceases to be public land and
becomes private property.

FACTS:
Manila Electric Company (MERALCO) filed an amended application for registration of a parcel
of land located in Taguig, Metro Manila. Applicant acquired the land applied for registration by
purchase from Ricardo Natividad who in turn acquired the same from his father Gregorio
Natividad as evidenced by a Deed of Original Absolute Sale. Applicant's predecessors-in-
interest have possessed the property under the concept of an owner for more than 30 years.
The property was declared for taxation purposes under the name of the applicant and the taxes
due thereon have been paid.

The respondent Judge rendered a decision ordering the registration of the property in the name
of the private respondent. The petitioner Director of Lands interposed this petition raising the
issue of whether or not a corporation may apply for registration of title to land. Petitioner
contends that a corporation is not among those that may apply for confirmation of title under
Section 48 of Commonwealth Act No. 141, the Public Land Act.

ISSUES:
1. Whether or not a corporation may apply for registration of titles to public land.
2. Whether or not open, exclusive and undisputed possession of alienable public land for
the period prescribed by law creates the legal fiction whereby the land ceases to be
public land and becomes private property.

HELD:
1. Yes.

The legal issue raised by the petitioner Director of Lands has been squarely dealt with in two
recent cases (The Director of Lands v. Intermediate Appellate Court and Acme Plywood &
Veneer Co., Inc., etc., No. L-73002 (December 29, 1986), 146 SCRA 509. The Director of
Lands v. Hon. Bengzon and Dynamarine Corporation, etc., No. 54045 (July 28, 1987)], and
resolved in the affirmative. There can be no different answer in the case at bar.

Coming to the case at bar, if the land was already private at the time Meralco bought it from
Natividad, then the prohibition in the 1973 Constitution against corporations holding
alienable lands of the public domain except by lease (1973 Const., Art. XIV, See. 11) does
not apply.

The fact that the confirmation proceedings were instituted by a corporation is simply another
accidental circumstance, "productive of a defect hardly more than procedural and in nowise
affecting the substance and merits of the right of ownership sought to be confirmed in said
proceedings." Considering that it is not disputed that the Natividads could have had their title
confirmed, only a rigid subservience to the letter of the law would deny private respondent
the right to register its property which was validly acquired.

2. Yes.
In the Acme decision, Supreme Court upheld the doctrine that open, exclusive and
undisputed possession of alienable public land for the period prescribed by law creates the
legal fiction whereby the land, upon completion of the requisite period ipso jure and without
the need of judicial or other sanction, ceases to be public land and becomes private property.

WHEREFORE, the petition is DENIED. The questioned decision of the respondent Judge is
AFFIRMED.




OWNERSHIP (ART. 427-439)

JGY
Chiao Liong Tan v. Court of Appeals
228 SCRA 75

DOCTRINE: Although a "replevin" action is primarily one for the possession of personality, yet it
is sufficiently flexible to authorize a settlement of all equities between the parties, arising from or
growing out of the main controversy.

FACTS:
Petitioner Chiao Liong Tan claims to be the owner of an Isuzu Elf van. He claims that he has
been in possession, enjoyment and utilization of the said motor vehicle until it was taken from
him by his older brother, Tan Ban Yong, the private respondent herein. Petitioner relies
principally on the fact that the Isuzu Elf van is registered in his name. He claims in his testimony
before the trial court that the said vehicle was purchased from Balintawak Isuzu Motor Center
for a price of over P100K; that he sent his brother to pay for the van and the receipt for payment
was placed in his name because it was his money that was used to pay for the vehicle; that he
allowed his brother to use the van because the latter was working for his company.

Private respondent testified that CLT Industries is a family business. When the family business
needed a vehicle, he asked petitioner to look for a vehicle and gave him the amount of P5K to
be deposited as down payment and he himself paid the whole price out of a loan of P140K.
Since he was still on good terms with the petitioner then, he allowed the registration of the
vehicle in petitioner's name.

TC: Ruled in favor of private respondent. CA: Affirmed.

ISSUE:
WON questions of ownership may be resolved in a replevin proceeding. -- YES

HELD:
It is true that the judgment in a replevin suit must only resolve in whom is the right of possession.
Primarily, the action of replevin is possessory in character and determined nothing more than
the right of possession. However, when the title to the property is distinctly put in issue by the
defendant's plea and by reason of the policy to settle in one action all the conflicting claims of
the parties to the possession of the property in controversy, the question of ownership may be
resolved in the same proceeding.

Although a "replevin" action is primarily one for the possession of personality, yet it is sufficiently
flexible to authorize a settlement of all equities between the parties, arising from or growing out
of the main controversy. Thus, in an action for replevin where the defendant is adjudged entitled
to possession, he need not go to another forum to procure relief for the return of the replevied
property or secure a judgment for the value of the property in case the adjudged return thereof
could not be had. Appropriately, the trial court rendered an alternative judgment.

CRF
Calub v. Court of Appeals
331 SCRA 55

DOCTRINE: Replevin cannot be issued to recover a property lawfully taken by virtue of legal
process and considered in the custody of the law. A replevin case against the State, without its
consent, cannot prosper. (batasnatin)

FACTS:
On January 28, 1992, the Forest Protection and Law Enforcement Team of the Community
Environment and Natural Resources Office (CENRO) of the DENR apprehended two (2) motor
vehicles by whose drivers failed to present proper documents and/or licenses. Thus, the
apprehending team seized and impounded the vehicles and its load of lumber at the DENR-
PENR. Seizure receipts were issued but the drivers refused to accept the receipts. Abuganda,
one of the drivers was charged with for violation of Revised Forestry Code.

On January 31, 1992, the impounded vehicles were forcibly taken by Gabon and Abuganda
from the custody of the DENR. On February 11, 1992, one of the two vehicles was again
apprehended by a composite team of DENR-CENR loaded with forest products. A
corresponding criminal case was again filed against Abuganda because of this incident. In both
criminal cases however, Abegonia and Abuganda were acquitted on the ground of reasonable
doubt.

Subsequently, herein private respondents Manuela Babalcon, the vehicle owner, and
Constancio Abuganda, the driver, filed a complaint for the recovery of possession of the two (2)
impounded vehicles with an application for replevin against herein petitioners before the RTC of
Catbalogan which was granted by the RTC.

The Court issued a TRO, enjoining respondent RTC judge from conducting further proceedings
in the civil case for replevin; and enjoining private respondents from taking or attempting to take
the motor vehicles and forest products seized from the custody of the petitioners. The Court
further instructed the petitioners to see to it that the motor vehicles and other forest products
seized are kept in a secured place and protected from deterioration, said property being in
custodia legis and subject to the direct order of the Supreme Court. On May 27, 1994, the Court
of Appeals denied said petition for lack of merit. It ruled that the mere seizure of a motor vehicle
pursuant to the authority granted by Section 68 [78] of P.D. No. 705 as amended by E.O. No.
277 does not automatically place said conveyance in custodia legis.

ISSUE:
Whether or not the DENR-seized motor vehicle in custodia legis can be a subject to an action
for replevin? - NO

HELD:
Since there was a violation of the Revised Forestry Code and the seizure was in accordance
with law, in our view the subject vehicles were validly deemed in custodia legis. It could not be
subject to an action for replevin. For it is property lawfully taken by virtue of legal process and
considered in the custody of the law, and not otherwise.

JRPA
Sarmiento v. Court of Appeals
250 SCRA 108

DOCTRINE: A boundary dispute is neither a case of forcible entry nor of unlawful detainer, but
an issue of ownership.
FACTS:
Cruz was the owner of a parcel of land. Adjacent to this lot is one wherein Sarmiento had a
house built on. On trying to cause the relocation of her lot, Cruz found out that Sarmiento was
encroaching on her property. When Cruz talked to Sarmiento about constructing a new fence,
which will cover her true property, the latter vehemently refused to do so and threatened Cruz
with legal action. For fear of being sued in court, she sought judicial relief. The trial court
decided in favor of Cruz. Sarmiento tried to assail this decision by saying that the issue was on
ownership of the portion of land and thus, the action should have been an accion
reivindicatoria and not forcible entry.
ISSUE:
WON Cruz filed the proper judicial relief. -- NO
HELD:
A careful reading of the facts averred in said complaint filed by Cruz reveals that the
action is neither of forcible entry nor of unlawful detainer but essentially involves a boundary
dispute, which must be resolved in an accion reivindicatoria on the issue of ownership over the
portion of a lot.

Forcible entry and unlawful detainer cases are distinct actions.

Cruz cannot belatedly claim that petitioners possession of the controverted portion was by
mere tolerance. The complaint didnt characterize Sarmientos alleged entry on the
landwhether legal or illegal. The complaint admitted also of the fact that the fence had
already preexisted on the lot when she acquired the same.

This was definitely not a situation obtained in and gave rise to an ejectment suit for two
reasons. First, forcible entry into the land is an open challenge to the right of the lawful
possessor, the violation of which
right authorizes the speedy redress in the inferior court provided for in the Rules. Second, if a
forcible entry action in the court is allowed after the lapse of a number of years, then the
result may well be no action of forcible entry can really prescribe. No matter how long such
defendant is in physical possession, the plaintiff may just throw in a demand, file a suit in court
and summarily throw him out of the land.

ABB
Bongato v. Malvar
387 SCRA 327

DOCTRINE: The one-year period within which to bring an action for forcible entry is generally
counted from the date of actual entry to the land. However, when entry is made through stealth,
then the one-year period is counted from the time the plaintiff learned about it. After the lapse of
the one-year period, the party deprived of the parcel of land may file either accion publiciana; or
an accion reivindicatoria, which is an action to recover ownership as well as possession.

FACTS:
Spouses Severo and Trinidad Malvar filed a complaint in the MTCC for forcible entry against
Teresita Bongato, alleging that Bongato unlawfully entered a parcel of land belonging to the
spouses and erected thereon a house of light materials.

MTCC decided in favor of Malvar and ordered Bongato to vacate the land. RTC affirmed the
decision. CA also held that MTCC had jurisdiction. On appeal, Bongato raised the issue of
MTCC jurisdiction because the complaint was filed beyond the one-year prescriptive period.

ISSUE:
Whether or not the MTCC had jurisdiction since the Complaint was filed beyond the one-year
period from date of alleged entry? -- NO

HELD:
MTCC had no jurisdiction. Forcible entry is a quieting process, and that the restrictive time bar is
prescribed to complement the summary nature of such process. Indeed, the one-year period
within which to bring an action for forcible entry is generally counted from the date of actual
entry to the land. However, when entry is made through stealth, then the one-year period is
counted from the time the plaintiff learned about it. After the lapse of the one-year period, the
party deprived of the parcel of land may file either accion publiciana; or an accion reivindicatoria,
which is an action to recover ownership as well as possession.

One the basis of the facts, it is clear that the cause of action for forcible entry filed by
respondents had already prescribed when they filed the complaint on July 10, 1992 (the house
was built as early as 1987), thus the MTCC had no more jurisdiction to hear and decide the
case for forcible entry.

FZC
De La Cruz v. CA
286 SCRA 230

DOCTRINE: A positive act of the govt is needed to reclassify land and until such
reclassification, property remains part of the forest reserve incapable of alienation and cannot
be acquired by prescription.

FACTS:
In 1973, a 407 sq. m. residential lot was the subject of an application under the Land
Registration Act by the Ramos bros. Eugenio de la Cruz [petitioner] opposed. After trial, the
application was dismissed on the ground that the land was not yet reclassified and remains part
of the forest reserve. The Ramos bros. pursued the reclassification of the land and were
subsequently awarded ownership of it. Cristina Villanueva, the private respondent, subsequently
purchased the same lot from the brothers. Upon learning of the said sale, petitioner filed a
complaint for reconveyance claiming ownership of the said land having possessed and
occupied it openly, publicly, notoriously and adversely against the whole world and in the
concept of an owner for more than 30 years. His complaint was dismissed. The CA affirmed in
toto the decision of the trial court thus the case at bar.

ISSUES:
Whether or not petitioner is vested with a better right over the residential lot to which he
possessed and devoted time, effort and resources -- NO

HELD:
Petitioner possessed and occupied the land after it was declared by the Govt as part of the
forest zone. Forest lands or forest reserves are not capable of private appropriation, and
possession thereof, however long, cannot convert them into private property.
A positive act by the government is needed to declassify land and to convert it to alienable or
disposable land. And until such declassification, there is no disposable land to speak of.

LNAC
Philippine Economic Zone Authority v. Fernandez
358 SCRA 489

DOCTRINE: Reconveyance is a remedy of those whose property has been wrongfully or
erroneously registered in the name of another. Such recourse, however, cannot be availed of
once the property has passed to an innocent purchaser for value. For an action for
reconveyance to prosper, the property should not have passed into the hands of an innocent
purchaser for value.

FACTS:
The subject of the present controversy is Lot No. 4673 of the Opon Cadastre situated in Lapu-
Lapu City, covered by Original Certificate of Title (OCT) No. RO-2537 (May 19, 1982) and
registered in the names of Florentina Rapaya et al.

On May 15, 1982, Jorgea Igot-Soroo, Frisca Booc and Felix Cuizon executed an Extrajudicial
Partition, in which they declared themselves as the only surviving heirs of the registered owners
of the aforesaid lot. Consequently, they were issued TCT No. 12467 on July 8, 1982.

Considering that the said lot was among the objects of expropriation proceedings, the RTC
rendered a partial Decision on August 11, 1982. In that Decision, the RTC approved the
Compromise Agreement entered into between the Export Processing Zone Authority (EPZA)
and the new registered owners of Lot No. 4673; namely, Jorgea Igot-Soroo, Frisca Booc and
Felix Cuizon. In accordance with the approved Compromise Agreement, EPZA would pay
P68,070 as just compensation for the expropriation of the subject property.

Petitioner acquired title over Lot No. 4673 and the corresponding TCT No. 12788 issued by the
Register of Deeds of Lapu-Lapu City on October 13, 1982.

On July 29, 1996, private respondents filed a Complaint for Nullity of Documents, Redemption
and Damages against petitioner and Jorgea-Igot Soroo et al. The Complaint alleged that
herein private respondents had been excluded from the extrajudicial settlement of the estate. It
likewise sought the nullification of several documents, including TCT No. 12788 dated October
13, 1992, issued in the name of herein petitioner.

On February 17, 1997, petitioner filed a Motion to Dismiss the Complaint on the ground of
prescription. This Motion was denied by respondent judge on January 12, 1998. A Motion for
Reconsideration thereof was likewise denied on March 31, 1998.

On April 30, 1998, petitioner elevated the matter to the CA through a Petition for Certiorari.

CA dismissed the Petition.

Hence, this recourse.

ISSUES:
1. Whether private respondents claim against expropriated property had prescribed. --
YES
2. Whether reconveyance lies against the expropriated property in this case. -- NO

HELD:
1. The pertinent provisions of Section 4, Rule 74 of the Rules of Court, are
reproduced for easy reference, as follows: Section 4. Liability of distributees and estate. - If
it shall appear at any time within two (2) years after the settlement and distribution of an
estate in accordance with the provisions of either of the first two sections of this rule, that an
heir or other person has been unduly deprived of his lawful participation in the estate, such
heir or such other person may compel the settlement of the estate in the courts in the
manner hereinafter provided for the purpose of satisfying such lawful participation. And if
within the same time of two (2) years, it shall appear that there are debts outstanding
against the estate which have not been paid, or that an heir or other person has been
unduly deprived of his lawful participation payable in money, the court having jurisdiction of
the estate may, by order for that purpose, after hearing, settle the amount of such debts or
lawful participation and order how much and in what manner each distributee shall
contribute in the payment thereof, and may issue execution, if circumstances require,
against the bond provided in the preceding section or against the real estate belonging to
the deceased, or both. Such bond and such real estate shall remain charged with a liability
to creditors, heirs, or other persons for the full period of two (2) years after such distribution,
notwithstanding any transfers of real estate that may have been made.

A perusal of the foregoing provision will show that persons unduly deprived of their lawful
participation in a settlement may assert their claim only within the two-year period after the
settlement and distribution of the estate. This prescription period does not apply, however,
to those who had no part in or had no notice of the settlement. Section 4, Rule 74 of the
Rules of Court, is not meant to be a statute of limitations. Moreover, by no reason or logic
can one contend that an extrajudicial partition, being merely an ex parte proceeding, would
affect third persons who had no knowledge thereof. Be that as it may, it cannot be denied,
either, that by its registration in the manner provided by law, a transaction may be known
actually or constructively.

In the present case, private respondents are deemed to have been constructively notified of
the extrajudicial settlement by reason of its registration and annotation in the certificate of
title over the subject lot. From the time of registration, private respondents had two (2) years
or until July 8, 1984, within which to file their objections or to demand the appropriate
settlement of the estate.

The only exception to the above-mentioned prescription is when the title remains in the
hands of the heirs who have fraudulently caused the partition of the subject property or in
those of their transferees who cannot be considered innocent purchasers for value. In this
regard, title to the property in the present case was no longer in the name of the allegedly
fraudulent heirs, but already in that of an innocent purchaser for valuethe government.
Even assuming that there was in fact fraud on the part of the other heirs, private
respondents may proceed only against the defrauding heirs, not against petitioner which
had no participation in or knowledge of the alleged fraud.

2. An action for reconveyance resulting from fraud prescribes four years from the
discovery of the fraud; such discovery is deemed to have taken place upon the issuance of
the certificate of title over the property. Registration of real property is considered a
constructive notice to all persons and, thus, the four-year period shall be counted therefrom.
Clearly then, private respondents action for reconveyance based on fraud has already
prescribed.

Even an action for reconveyance based on an implied or a constructive trust would have
already prescribed just the same, because such action prescribes ten (10) years from the
alleged fraudulent registration or date of issuance of the certificate of title over the property.
The imprescriptibility of an action for reconveyance based on implied or constructive trust
applies only when the plaintiff or the person enforcing the trust is in possession of the
property. Undisputedly, private respondents are not in possession of the disputed property.

Finally, it must be remembered that reconveyance is a remedy of those whose property has
been wrongfully or erroneously registered in the name of another. Such recourse, however,
cannot be availed of once the property has passed to an innocent purchaser for value. For
an action for reconveyance to prosper, the property should not have passed into the hands
of an innocent purchaser for value.

We find that the property has already been conveyed to the government in appropriate
expropriation proceedings, the regularity or validity of which has not been questioned.
Petitioner should, therefore, enjoy the security afforded to innocent third persons under our
registration laws. Equally important, its title to the property must be rightfully preserved.

TKDC
Idolor v. Court of Appeals
351 SCRA 399

DOCTRINE: Injunction is a preservative remedy aimed at protecting substantive rights and
interests. Before an injunction can be issued, the following requisites must be present: (1) a
right in esse or the existence of a right to be protected; and (2) the act, against which the
injunction is to be directed, is a violation of such right. Hence, the existence of a violated right is
a prerequisite to the granting of an injunction.

FACTS:
Idolor executed a deed of real estate mortgage with right of extra-judicial foreclosure upon
failure to redeem over a property in favor of De Guzman to secure a loan. Upon Idolors failure
to pay, De Guzman filed a complaint against Idolor before the Office of the Barangay Captain
that resulted in an agreement wherein Idolor is given another ninety days to settle her loan;
otherwise, she will execute a deed of sale with the agreement to repurchase without interest
within a year.

Idolor again failed to pay back her loan so De Guzman filed an extra judicial foreclosure of the
real estate mortgage resulting in a public auction where the property was sold to Gumersindo.
Idolor filed a complaint with the RTC resulting to the courts issuance of a writ of preliminary
injunction enjoining the issuance of a final deed of sale and consolidation of ownership of the
property in favor of the De Guzmans.

Idolor claims that her proprietary right over the property was not yet lost since her right to
redeem the land for a year had neither lapsed nor run as the sheriffs certificate of sale was null
and void.

ISSUE:
Whether or not the questioned writ of preliminary injunction was issued with grave abuse of
discretion. -- YES

HELD:
Injunction is a preservative remedy aimed at protecting substantive rights and interests. Before
an injunction can be issued, the following requisites must be present: (1) a right in esse or the
existence of a right to be protected; and (2) the act, against which the injunction is to be directed,
is a violation of such right. Hence, the existence of a violated right is a prerequisite to the
granting of an injunction.

In this case, Idolor has no more proprietary right to speak of over the foreclosed property to
entitle her to the issuance of a writ of injunction. She had one year from the registration of the
sheriffs sale to redeem the property but she failed to exercise her right; she failed to show
sufficient interest or title in the property as her right of redemption had already expired two days
before the filing of the complaint. Thus, the de Guzmans are now entitled to a conveyance and
possession of the foreclosed property.

AMD
Cagayan de Oro City Landless Residents v. Court of Appeals
254 SCRA 220

DOCTRINE:
1. Injunction is calculated to preserve or maintain the status quo of things and is generally
availed of to prevent actual or threatened acts, until the merits of the case can be heard.
Before an injunction can be issued, it is essential that the following requisites be present:
a. There must be a right in esse or the existence of a right to be protected; and
b. The act against which the injunction is to be directed is a violation of such right.

2. An Original Certificate of Title serves as a concrete and conclusive evidence of an
indefeasible title to the property. The certificate of title vests not only ownership over the
lot but also the right of possession as a necessary consequence of the right of
ownership.

FACTS:
The subject of the dispute is a 12.82-hectare land located at Cagayan de Oro City. Said parcel
of land was formerly classified as a timberland until1956, when the Bureau of Forestry released
the said land as alienable and disposable public land.

On January 1964, Petitioner Cagayan de Oro City Landless Residents Association, Inc
(COCLAI) was authorized by the Bureau of Lands (BOL) to survey the land for purposes of
subdivision into residential lots. On March 1964, after conducting an ocular survey, the BOL
required COCLAI to file a Miscellaneous Sales Application over the land. The said sales
application was held in abeyance by the BOL pending the final outcome of the civil case filed by
the Republic of the Philippines and the City of CDO against Benedicta Salcedo for the
annulment of an Original Certicate of Title covering the subject land. In this case, COCLAI was
a party-intervenor.

Meanwhile, the NHA initiated an expropriation proceeding to acquire an area which included the
subject land. Petitioner intervened and claimed that instead of being paid through money, it
preferred acquisition of residential lots in any housing area of NHA. Upon learning of the
pending suit (v. Salcedo case) over the same land, the NHA sought the suspension of the
expropriation proceedings. In 1982, the SC finally resolved the pending suit by annulling the
title and declaring the subject lot to be public land. In response to the BOLs order to take
an inventory of the land, the Regional Land Director of Region 10 informed the Director of Lands
that the members of COCLAI were occupying portions of the said lot by virtue of the Survey
Authority.

In 1983, Presidential Proclamation No. 2292 was issued, reserving the entire subject land for
the Slum Improvement and Resettlement (SIR) Project of the NHA. This led to the rejection of
the subdivision survey submitted by COCLAI. Eventually, the NHA caused the demolition of the
settlement structures constructed by COCLAI members. This prompted petitioner to file a case
for forcible entry, which the MTCC decided in petitioners favor. On appeal, the RTC
affirmed the MTCC decision.

However, during the pendency of the case filed by COCLAI, Special Patent No. 3551 was
issued by the President over the entire subject land, and by virtue of which the ROD of CDO
issued an Original certificate of Title in the name of NHA.

Thus, a day after COCLAI moved for the execution of judgment, the NHA filed a complaint for
"Quieting of Title with Application for a Writ of Preliminary Injunction" against COCLAI. The RTC
ruled in favour of COCLAI, denying NHAs Motion to Dismiss and prayer for the issuance of a
preliminary injunction. NHAs MR was also denied. On appeal, the CA reversed the RTC
decision. The CA ordered the issuance of a writ of preliminary injunction to respect the
possession of the NHA over the land in dispute. Hence, this petition.

ISSUES:
1. W/N the NHA is entitled to the injunction prayed for. -- YES
2. W/N the NHA has a better right to the possession of the lot involved. -- YES

HELD:
1. As an extraordinary remedy, injunction is calculated to preserve or
maintain the status quo of things and is generally availed of to prevent actual or
threatened acts, until the merits of the case can be heard. As such, injunction is
accepted as a strong arm of equity or a transcendent remedy to be used cautiously,
as it affects the respective rights of the parties, and only upon full conviction on the part
of the court of its extreme necessity.

NHA was entitled to the writ of injunction because while the case for forcible entry was
pending on appeal, NHA was granted an OCT over the subject land by virtue of the Special
Patent issued by the President. In view of this intervening development, NHA filed a
complaint for quieting of title before the RTC. Thus, it was only proper for the CA to direct
the RTC, where the quieting case was pending, to grant the writ of preliminary injunction to
restrain the enforcement of the decision of the MTCC in the forcible entry case as there was
a material change in the status of the parties with regard to the said land.

2. The Original Certificate of Title issued to respondent NHA serves as a
concrete and conclusive evidence of an indefeasible title to the property. Accordingly, once
a decree of registration is issued under the Torrens systems and the one year period from
the issuance of the decree of registration has lapsed, without said decree being
controverted by any adverse party, the title becomes perfect and cannot later on be
questioned.

Moreover, the said certificate of title was not controverted by petitioner in a proper
proceeding nor did it show that the issuance of the Original Certificate of Title by the register
of deeds to NHA was tainted with bad faith or fraud. Hence, said certificate of title enjoys the
presumption of having been issued by the register of deeds in the regular performance of its
official duty. The certificate of title vested not only ownership over the lot but also the right of
possession as a necessary consequence of the right of ownership.

MPF
Lucero v. Loot
25 SCRA 687

DOCTRINE: When a final decree has been issued in a land registration case, the issuance of a
writ of possession is only a matter of course if nothing in the past has been issued in favor of the
registered owner. There is no period of prescription as to the issuance of writ of possession.

FACTS:
Julio Lucero filed and was granted a writ of possession of property (based on a
final decree in a land registration proceeding).
Although the other party (all surnamed Loot) filed a motion to quash the writ, this was
granted by CFI Iloilos Judge Fernan on September 21, 1959.
The Loots opposed the decision on the ground that there were defects in the
reconstitution of the records and that the motion was not under oath.
The court dismissed these as trivial arguments.
Two motions for reconsideration were also denied.
The writ of possession prayed for was issued in favour of Lucero.
The Loots were very adamant to their opposition.
They then went straight to the Supreme Court for an appeal for certiorari.
The Loots desperately tried to assert

ISSUE:
Whether or not the order granting the writ of possession was in accordance with law. -- YES

HELD:

The order granting the writ of possession was based on a decision promulgated on a land
registration case in 1938, which became a final decree on October 29, 1941. After the final
decree, the issuance of the writ of possession was only a ministerial duty of the court if no writ
has been issued to the registered owner yet. The final decree, in effect, immediately
empowered the court to enforce the order/judgment/decree. This automatic process is to avoid
further delay and inconvenience to a successful land registration litigant if he were compelled to
commence another action to secure possession.

Furthermore, there is no period of prescription as to the issuance of a writ of possession. The
writ may be issued not only against the person who has been defeated in a registration case,
but also against anyone adversely occupying the land or any portion of the land. Even fraud
shall not be a bar to the issuance of the writ of possession, which necessarily implied the
delivery of possession of the land.

As to the questions of fact raised by the Loots, the SC can do nothing. These must be raised at
the CA of appeals; otherwise, the parties contesting the facts are deemed to have waived the
opportunity to question the correctness of the findings.

AMDG
Vencilao v. Vano
182 SCRA 492

DOCTRINE: Writ of Possession may be issued not only against the party defeated in the land
registration case but also against anyone unlawfully and adversely occupying the land, or any
portion thereof during the land registration proceedings up to issuance of final decree

FACTS:
(NOTE: Three consolidated cases)

First case: Juan Reyes applied for the registrations over parcels of land, however, it was argued
that Juan Reyes was not the lawful owner of a portion of the parcels that was registered.

Second case: Bernardina Vda. de Lupso was the administratix over the lot subject in
controversy. When she died, several TCTs were issued to Pedro Lupso and several persons.
As a result, several writs of possession were also issued in their favor. When the writs were
being served by the sheriff, the petitioners refused to vacate and even executed guerilla-like
tactics as their sign of refusal. A petition for contempt was filed by one of the registered owners
against the petitioners to which the court granted. The petitioners were contending that the writs
of possession should not be issued in the first place since there were no complaints of forcible
entry, unlawful detainer or recovery of ownership and possession that were filed against them.
The petitioners also claim that they were not defeated oppositors in the first case.

Third case: As a result of the writ of possessions being issued against the petitioners, the judge
also issued writs of demolition. The petitioners opposed the validity of these writs.

ISSUES:
1. Whether or not the writs of possession were valid? -- YES
2. Whether or not the writ of demolition was valid? -- YES

HELD:
The issuance of the TCTs carried with it the delivery of possession which is an inherent element
of the right of ownership. The issuance of the writ of possession is, therefore, sanctioned by
existing laws in this jurisdiction and by the generally accepted principle upon which the
administration of justice rests. A writ of possession may be issued not only against the person
who has been defeated in a registration case but also against anyone unlawfully and adversely
occupying the land or any portion thereof during the land registration proceedings up to the
issuance of the final decree. Applying this to the case, from the records, though the petitioners
were not the defeated oppositors over the land registration case, they were unlawfully
occupying the lots although writs of possession were already being served to them. Having said
this, the writs of possession were properly issued against them.

The writs of demotion were also issued properly. These writs were described by previous cases
as a complement to the writs of possession. Without it, the writ of possessions would be
ineffective. The writs are issued to prevent the lawful owner from being deprived of his lots.

GCG
German Management and Services v. CA
177 SCRA 495

DOCTRINE: Actual possessors can commence a forcible entry case because ownership is not
in issue. Forcible entry is merely a quieting process and never determines the actual title to an
estate. Title is not involved, only actual possession

FACTS:
Spouses Cynthia Cuyegkeng Jose and Manuel Rene Jose, residents of Pennsylvania,
Philadelphia, USA are the owners of a parcel of land situated in Sitio Inarawan, San Isidro,
Antipolo, Rizal. The land was originally registered in the Office of the Register of Deeds of Rizal,
pursuant to a Homestead Patent granted by the President of the Philippines.

Spouses Jose executed a special power of attorney authorizing petitioner German Management
Services to develop their property into a residential subdivision. Consequently, petitioner
obtained development permit from the Human Settlements Regulatory Commission for said
development. Finding that part of the property was occupied by private respondents and twenty
other persons, petitioner advised the occupants to vacate the premises but the latter refused.
Nevertheless, petitioner proceeded with the development of the subject property which included
the portions occupied and cultivated by private respondents.

Private respondents filed an action for forcible entry against petitioner before the Municipal Trial
Court of Antipolo, Rizal. They alleged that they are mountainside farmers of Sitio Inarawan,
San Isidro, Antipolo, Rizal and members of the Concerned Citizens of Farmer's Association; that
they have occupied and tilled their farmholdings some twelve to fifteen years prior to the
promulgation of P.D. No. 27; that during the first week of August 1983, petitioner, under a permit
from the Office of the Provincial Governor of Rizal, was allowed to improve the Barangay Road
at Sitio Inarawan, San Isidro, Antipolo, Rizal at its expense, subject to the condition that it shall
secure the needed right of way from the owners of the lot to be affected; that, petitioner
deprived private respondents of their property without due process of law by: (1) forcibly
removing and destroying the barbed wire fence enclosing their farmholdings without notice; (2)
bulldozing the rice, corn fruit bearing trees and other crops of private respondents by means of
force, violence and intimidation, in violation of P.D. 1038 and (3) trespassing, coercing and
threatening to harass, remove and eject private respondents from their respective farmholdings
in violation of P.D. Nos. 316, 583, 815, and 1028.

Municipal Trial Court dismissed private respondents' complaint for forcible entry. On appeal, the
Regional Trial Court of Antipolo, Rizal, Branch LXXI sustained the dismissal by the Municipal
Trial Court.

Private respondents then filed a petition for review with the Court of Appeals. The court gave
due course to their petition and reversed the decisions of the Municipal Trial Court and the
Regional Trial Court.

The Appellate Court held that since private respondents were in actual possession of the
property at the time they were forcibly ejected by petitioner, private respondents have a right to
commence an action for forcible entry regardless of the legality or illegality of possession.

Petitioner moved to reconsider but the same was denied by the Appellate Court. Hence, this
recourse.

ISSUE:
Whether or not private respondents are entitled to file a forcible entry case against petitioner? --
YES, they are entitled to file a forcible entry case.

HELD:
Since private respondents were in actual possession of the property at the time they were
forcibly ejected by petitioner, private respondents have a right to commence an action for
forcible entry regardless of the legality or illegality of possession.

Private respondents, as actual possessors, can commence a forcible entry case against
petitioner because ownership is not in issue. Forcible entry is merely a quieting process and
never determines the actual title to an estate. Title is not involved, only actual possession. It is
undisputed that private respondents were in possession of the property and not the petitioners
nor the spouses Jose. Although the petitioners have a valid claim over ownership this does not
in any way justify their act of forcible entry. It must be stated that regardless of the actual
condition of the title to the property the party in peaceable quiet possession shall not be turned
out by a strong hand, violence or terror. Thus, a party who can prove prior possession can
recover such possession even against the owner himself. Whatever may be the character of his
possession, if he has in his favor priority in time, he has the security that entitles him to remain
on the property until he is lawfully ejected by a person having a better right by accion publiciana
or accion reivindicatoria. The doctrine of self help, which the petitioners were using to justify
their actions, are not applicable in the case because it can only be exercised at the time of
actual or threatened dispossession which is absent in the case at bar (in fact they are the ones
who are threatening to remove the respondents with the use of force). Article 536 basically
tells us that the owner or a person who has a better right over the land must resort to judicial
means to recover the property from another person who possesses the land.

When possession has already been lost, the owner must resort to judicial process for the
recovery of property. As clearly stated in Article 536- In no case may possession be acquired
through force or intimidation as long as there is a possessor who objects thereto. He who
believes that he has an action or right to deprive another of the holding of a thing must invoke
the aid of the competent court, if holder should refuse to deliver the thing.

VCL IV
Caisip v. People
36 SCRA 17

Doctrine: Art. 429 cannot be used as a defense of the petitioner to justify their action. The order
to vacate was until June 26 (or 20 days from the execution of the decision.) On June 17, the
spouses REMAINED in possession of the said lot. At the very least the owner of the hacienda is
just a co-possessor of the land, thus the spouses still had rights over it.

FACTS:
Gloria Cabalag is the wife of Marcelino Guevarra who cultivated a parcel of land known as Lot
105-A of Hacienda Palico situated in Sitio Bote-bote, Barrio Tampisao, Nasugbu, Batangas. The
said parcel of land used to be tenanted by the deceased father of the Cabalag. Hacienda Palico
is owned by Roxas y Cia. and administered by Antonio Chuidian. The overseer of the said
hacienda is Felix Caisip, one of the accused herein.

Before the present case happened, Marcelino Guevarra filed an action with the Court of
Agrarian Relations seeking recognition as a lawful tenant of Roxas y Cia. over lot No. 105-A of
Hacienda Palico. In a decision of the Court of Agrarian Relations, it declared that Guevarra is
not a tenant on the said parcel of land. Then after, Roxas y Cia filed a forcible entry case
against Guevarra. The court decided in favour of Roxas y Cia and issued a writ of execution.
The return of the writ showed that possession of Lot 105-A was turned over to the owner thru
Caisip and that spouses Guevarra and Cabalag were given 20 days from June 6, 1959 to
vacate the premises. It also appears in the record that due to the tenacious attitude of Cabalag,
Caisip sought the help of policemen Federico Villadelrey and Ignacio Rojales.

On June 17, 1959, Gloria Cabalag was seen weeding the portion of Lot 105-A which was a
ricefield. Caisip approached her and bade her to leave but Cabalag refused to do so claiming
that she and her husband has a right over the property. She having stuck to this attitude, even
when he threatened to call the police, Caisip went to his co-defendants, Sgt. Rojales and Cpl.
Villadelrey, both of the local police, who were some distance away, and brought them with him.

Gloria Cabalag Version: Rojales told her, who was then in a squatting position, to stop weeding.
As she insisted on her right to stay in said lot, Rojales grabbed her right hand and, twisting the
same, wrested therefrom the trowel she was holding. Thereupon, Villadelrey held her left hand
and, together with Rojales, forcibly dragged her, which resulted for her dress to torn.

Caisip et. al Version: Upon being asked by the policemen to stop weeding and leave the
premises, Cabalag, not only refused to do so, but, also, insulted them, as well as Caisip.
According to the defense, she was arrested because of the crime of slander then committed by
her. Rojales and Villadelrey, moreover, testified that, as they were heading towards the barrio of
Camachilihan, Gloria proceeded to tear her clothes.

Due to the aforementioned incidents, A case filed against Caisip et al. for Grave Coercion,
(Petitioners Caisip et al. also filed grave coercion and unjust vexation against Gloria Cabalag
after 8 days) One of their defences was ART. 429 (including the doctrine of self help.)

TC Ruled in favor of Gloria Cabalag and that Caisip et al. are guilty of Grave Coercion. CA
Upheld the TC Ruling.

*Caisip et al. argued in the SC that the Court of Appeals erred in not finding that their acts are
justified under Article 429 of the Civil Code; The owner or lawful possessor of a thing has the
right to exclude any person from the enjoyment and disposal thereof. For this purpose, he may
use such force as may be reasonably necessary to repel or prevent an actual or threatened
unlawful physical invasion or usurpation of his property.

ISSUE:
Whether Article 429 of the Civil Code applies in the present case. -- NO

HELD:
No, Article 429 is inapplicable, Cabalag was given 20 days from June 6, 1959 within which to
vacate the premises. Cabalag did not, on June 17, 1959 or within said period invade or
usurp said lot. She had merely remained in possession thereof, even though the hacienda
owner may have become its co-possessor. Appellants did not repel or prevent in actual or
threatened . . . physical invasion or usurpation. They expelled Gloria from a property of which
she and her husband were in possession.

It is urged, that, by weeding and refusing to leave Lot 105-A, Gloria had committed a crime in
the presence of the policemen, despite the aforementioned 20-day period, which, appellants
claim, the sheriff had no authority to grant. This contention is manifestly untenable, because: (1)
said period was granted in the presence of the hacienda owners representative, appellant
Caisip, who, by not objecting thereto, had impliedly consented to or ratified the act performed by
the sheriff; 2) Gloria and her husband were thereby allowed to remain, and had, in fact,
remained, in possession of the premises, perhaps together with the owner of the hacienda or
his representative, Caisip; (3) the act of removing weeds from the ricefield was beneficial to its
owner and to whomsoever the crops belonged, and, even if they had not authorized it, does not
constitute a criminal offense; and (4) although Gloria and her husband had been sentenced to
vacate the land, the judgment against them did not necessarily imply that they, as the parties
who had tilled it and planted thereon, had no rights, of any kind whatsoever, in or to the standing
crops, inasmuch as necessary expenses shall be refunded to every possessor, and the cost of
cultivation, production and upkeep has been held to partake of the nature of necessary
expenses.

It is, accordingly, clear that appellants herein had, by means of violence, and without legal
authority therefor, prevented the complainant from doing something not prohibited by law,
(weeding and being in Lot 105-A), and compelled her to do something against her will
(stopping the weeding and leaving said lot), whether it be right or wrong, thereby taking the
law into their hands, in violation of Art. 286 of the Revised Penal Code, Grave Coercion.

FXRL
Heirs of Vencilao, Sr. v. CA
288 SCRA 574

DOCTRINE: Tax declarations and receipts do not by themselves conclusively prove title to the
land.

They only constitute prima facie evidence of ownership or possession.

Where the certificate of title is in the name of the vendor when the land is sold, the vendee for
value has the right to rely on what appears on the face of the title. He is under no obligation to
look beyond the certificate and investigate the title of the vendor appearing on the face of the
certificate. However, the vendee is required to make the necessary inquiries if there is anything
in the certificate of title which indicates any cloud or vice in the ownership of the property.
Otherwise, his mere refusal to believe that such defect exists, he will not be deemed a
purchaser in good faith should such title indeed be defective.

FACTS:
Both petitioner and respondent claim ownership over a parcel of land located at Bohol.

Petitioner claims ownership by virtue of inheritance from their father who exercised
uninterrupted possession over the land, declared the property for taxation purpose and
religiously paid the real estate tax.

The respondent claims ownership as registered owner of said lot as a portion of the lot owned
by Pedro Luspo who mortgaged his land to PNB and subsequently was foreclosed. The
respondents won as the highest bidder and they became the owner of that portion of land.

The RTC ruled in favor to the petitioners holding that they had validly acquired the land by
prescription. Although a Torrens Title is indefeasible and not subject for prescription it is not so
when the respondents purchased the land from PNB with prior knowledge that the land was in
possession of the petitioners father.

The CA reversed the court ruling and declared the petitioners as the true owners of the property.

ISSUE:
W/N land registered under the Torrens system can be acquired by prescription -- NO

HELD:
No. The previously registered land may not be acquired by prescription.

The Property Registration Decree states that no land under the Torrens System may be
acquired through prescription. Such title is indefeasible.

The tax declarations and tax receipts presented by petitioners as evidence of ownership prevail
over respondents certificate of title which is an incontrovertible proof of ownership. Tax
declarations and receipts do not by themselves conclusively prove title to the land.

They only
constitute prima facie evidence of ownership or possession.

Petitioners are wrong to contend that the prior knowledge of the respondents on the possession
of the petitioners father defies the Torrens titles imprescriptibility because there is no flaw on
the title when they purchased it from PNB that was the registered owner of the land.

The general rule is that where the certificate of title is in the name of the vendor when the land is
sold, the vendee for value has the right to rely on what appears on the face of the title. He is
under no obligation to look beyond the certificate and investigate the title of the vendor
appearing on the face of the certificate.

However, the vendee is required to make the necessary inquiries if there is anything in the
certificate of title which indicates any cloud or vice in the ownership of the property. Otherwise,
his mere refusal to believe that such defect exists, or his willful closing of his eyes to the
possibility of the existence of a defect in his vendors title, will not make him an innocent
purchaser for value if it afterwards develops that the title was in fact defective, and it appears
that he had such notice of the defect as would have led to its discovery had he acted with that
measure of precaution which may reasonably be required of a prudent man in a like situation.

In the instant case, there is nothing from the records showing that the title of PNB, the vendor,
was flawed.

Petitioners not only failed to substantiate their claim of acquisitive prescription as basis of
ownership but they also failed to allege, and much less adduce, any evidence that there was a
defect in the title of PNB. In the absence of such evidence, the presumption leans towards the
validity of the vendors title.

RSDM
Oclarit v. CA
233 SCRA 239

DOCTRINE: Although it is true that what defines a piece of land is not the area mentioned in its
description but the boundaries therein laid down, in controversial cases as in this case where
there appears to be an overlapping of boundaries, the actual sizeof the property gains
importance.

FACTS:
The late Juan Oclarit, petitioners predecessor-in-interest, allegedly purchased from Martin
Macalos a parcel of unregistered land located in Antipolo, Garcia-Hernandez, Bohol, with no
permanent landmarks . This particular land did not have specified boundaries, as it was only
indicated that the borders were a brook, lands of Gales, and another of Baja.Then he bought
five more parcels of land located in Antipolo and Ulbujan, also in Garcia-Hernandez, Bohol, from
Dalmacio Gales.

The heirs of Oclarit filed an action for the quieting of title and damages against respondent
Balasabas. The complaint alleged that private respondent entered the properties subject of the
action. Failing to work on the area planted to palay, private respondent climbed the coconut
trees, replaced the "J.O." markings on the trees with "F.G.", representing Felipa Gales, his
mother, and caused to be recorded in the cadastral survey of the land the name of Felipa Gales
as claimant against Juan Oclarit. The heirs of Oclarit considered the acts of private respondent
as having cast a cloud of doubt over their title to the property and therefore deprived them of the
enjoyment of the fruits of the coconut trees. Petitioners further alleged that the late Juan Oclarit,
from the time of the acquisition of said properties, had exercised dominion and ownership
thereon openly, peacefully, adversely and uninterruptedly. It was also claimed even after the
death of Oclarit they still enjoyed the property.

Respondent Balasabas claims to have actually and lawfully possessed the disputed parcels of
land "since time immemorial". The first parcel of land was owned by his mother, Felipa Gales,
by virtue of inheritance, and declared in her name under Tax Declaration; while the second
parcel of land was acquired by him from his own mother as evidenced by a deed of absolute
sale and which he declared in his name under Tax Declaration. In addition, respondent likewise
alleged possession of the parcels of land openly, peacefully, adversely and continuously without
disturbance from any party until he was molested by the heirs of Oclarit.
Trial court appointed an Assessor (commissioner) for the purpose of determining whether the
lands described in the complaint overlapped with any of those lands claimed by respondents.

Trial court dismissed the complaint. CA affirmed.

ISSUE:
Whether or not the heirs of Oclarit are lawfully entitled to the subject properties. -- NO

HELD:
Had the petitioners been in possession of solid evidence that the parcels of land they are
claiming are "alien" or "foreign" to those declared by private respondent as his, they should
have questioned the commissioners report which was based on the relocation survey and
ocular inspection which were conducted in their presence. Moreover, petitioners claim that their
property is different from those of private respondents is indeed antithetical to their filing of the
complaint for quieting of title there would not have been any basis for claiming that private
respondent cast a cloud of doubt to their title over their two parcels of land.

The deed of sale wherein Martin Macalos conveyed to Oclarit a parcel of land did not even
indicate with particularity the area of the land covered thereby. This explains why they
indiscriminately pointed at boundaries which are even beyond what could have been bought by
Oclarit. Although it is true that what defines a piece of land is not the area mentioned in its
description but the boundaries therein laid down, in controversial cases as in this case where
there appears to be an overlapping of boundaries, the actual size of the property gains
importance. Thus, the lower court correctly stressed that it would have done petitioners some
good had they correctly specified even in their tax declarations the areas of the land they were
claiming. It is well settled that anyone who claims that he has a better right to the property, must
prove both ownership and identity of the said property. An area delimited by boundaries
properly identifies a parcel of land.

With regard to tax declarations as bases for claim of ownership, petitioners capitalize on what
was obviously an obiter that no one in his right mind would be continuously paying taxes for
property that is not in his actual possession. On the contrary, any person who claims ownership
by virtue of tax declarations must also prove he is in actual possession of the property. Thus,
proof that the property involved had been declared for taxation purposes did not constitute proof
of possession, nor is it proof of ownership in the absence of the claimants actual possession of
said property.

MRAM
Cutanda v. Heirs of Roberto Cutanda
335 SCRA 418

DOCTRINE: Under the Code of Civil Procedure, therefore, ten years of actual adverse
possession was required, regardless of how such occupancy may have commenced or
continued, before possession ripened into full and complete title over the land.

FACTS:
Private respondents brought an action for recovery of possession, accounting and damages
against petitioners in the RTC of Tagbilaran City. They alleged that in the 1900s, their
grandfather, Roberto Cutanda, owned two parcels of land in Bohol. Tax declarations were used
by them as evidence. Upon Robertos death, these lands were inherited by his children, namely:
Doque, Diego, Pedro, Andres, and Anastacia. Except for Doque who stayed in Bohol and
administered the lands, all of Robertos children established residence in Leyte. In 1987, they
returned to Bohol to personally work the inherited lands. Their plan, however, was frustrated as
petitioners, who were occupying the lands, refused to leave. Private respondent thus prayed
that each be declared owner of 1/5 of the subject real properties and that petitioners be ordered
to return to them said properties.

The petitioners, on the other hand averred that the lands were owned by their uncle Anastacio
who died without children. As such, petitioners, who were nieces and nephews of Anastacio
occupied and cultivated the land. The petitioners were claiming rightful ownership of the land for
having openly, contiguously, adversely, and continuously possessing the land for about 55
years.

ISSUE:
Whether or not private respondents right over the land has prescribed?

HELD:
The Court held in the affirmative. The Civil Code provides that title to land by prescription is
acquired if the land has been in open, continuous, adverse possession and occupancy of the
land over 10 years. Under the Code of Civil Procedure, therefore, ten years of actual adverse
possession was required, regardless of how such occupancy may have commenced or
continued, before possession ripened into full and complete title over the land. Applying this to
the present case, by 1943, ten years after his possession of the subject parcel of land had
begun, Anastacio Cutanda became owner of the land in question through acquisitive
prescription.

Hence, the rightful owner of the land is the petitioners.

FMM
Sps. Luis Cruz v. Sps. Alejando Fernando, Sr.
G.R. No. 145470

DOCTRINE: Occupation of a property merely through the tolerance of the owners could
neither ripen into ownership nor operate to bar any action by the subsequent owners to recover
absolute possession thereof.

FACTS:
Spouses Luis V. Cruz and Aida Cruz (the Spouses Cruz) are occupants of the front portion of
a 710-square meter property located in Sto. Cristo, Baliuag, Bulacan.

On 21 October 1994, Spouses Alejandro Fernando, Sr. and Rita Fernando (the Spouses
Fernando) filed before the Regional Trial Court (the RTC) a complaint for accion publiciana
against the Spouses Cruz, demanding the latter to vacate the premises and to pay the amount
of P500.00 a month as reasonable rental for the use thereof. The Spouses Fernando alleged
that they are owners of the property having bought the same from Spouses Clodualdo and
Teresita Glorioso (the Gloriosos) pursuant to a Deed of Sale dated 9 March 1987. They also
alleged that the Gloriosos offered to sell to the Spouses Cruz the rear portion of the property but
the transaction did not materialize due to the latters failure to exercise their option and for this
reason, the Spouses Fernando were able to buy the whole property.

For their part, the Spouses Cruz argued that the Kasunduan dated 6 August 1983 they made
with the Gloriosos is a perfected contract of sale and that the agreement has already been
partially consummated as they already relocated their house from the rear portion of the lot to
the front portion that was sold to them.

After due proceedings, the RTC rendered a decision on 3 April 1998 in favor of the Spouses
Fernando and ordered the Spouses Cruz to vacate the property and pay the rent beginning from
2 October 1994 when the case was filed before the RTC.

The Spouses Cruz appealed the RTC decision but the Court of Appeals affirmed it on 3 October
2000.

Thus, the Spouses Cruz herein petition before the Supreme Court. The Spouses Cruz argued
that the Kasunduan is a perfected contract of sale and thus they have ownership over the
property.

ISSUE:
Whether the Spouses Cruz have a right of ownership over the property. -- NO

HELD:
The SC held that the terms and conditions of the Kasunduan show that it is a contract to sell
and that a contract of sale is yet to be consummated and thus ownership of the property
remained in the Gloriosos.

The SC also held that the Spouses Cruz have no superior right of ownership or possession to
speak of. Their occupation of the property was merely through the tolerance of the owners.
Evidence on record shows that the Spouses Cruz and their predecessors were able to live and
build their house on the property through the permission and kindness of the previous owner,
Pedro Hipolito, who was their relative, and subsequently, Teresita Glorioso, who is also their
relative. They have no title or, at the very least, a contract of lease over the property. Based as
it was on mere tolerance, the Spouses Cruz possession could neither ripen into ownership nor
operate to bar any action by the Spouses Fernando to recover absolute possession thereof.



RIGHTS OF ACCESSION (ART. 440-475)

Accession Industrial

RGGM
Depra v. Dumlao
136 SCRA 475

DOCTRINE: The owner of land on which improvement was built by another in good faith is
entitled to removal of improvement only after land owner has opted to sell the land and the
builder refused to pay for the same. where the lands value is considerably more than the
improvement, the landowner cannot compel the builder to buy the land. In such event, a forced
lease is created and the court shall fix the terms thereof in case the parties disagree thereon.

FACTS:
Francisco Depra, is the owner of a parcel of land registered, situated in the municipality of
Dumangas, Iloilo. Agustin Dumlao, defendant-appellant, owns an adjoining lot. When Dumlao
constructed his house on his lot, the kitchen thereof had encroached on an area of thirty four
(34) square meters of Depras property, After the encroachment was discovered in a relocation
survey of Depras lot made on November 2, 1972, his mother, Beatriz Depra after writing a
demand letter asking Dumlao to move back from his encroachment, filed an action for Unlawful
Detainer. Said complaint was later amended to include Depra as a party plaintiff. After trial, the
Municipal Court found that Dumlao was a builder in good faith, and applying Article 448 of the
Civil Code. Depra did not accept payment of rentals so that Dumlao deposited such rentals with
the Municipal Court. In this case, the Municipal Court, acted without jurisdiction, its Decision
was null and void and cannot operate as res judicata to the subject complaint for Quieting of
Title. The court conceded in the MCs decision that Dumlao is a builder in good faith.

ISSUE:
Whether or not the factual situations of Dumlao and Depra conform to the juridical positions
respectively defined by law, for a "builder in good faith" under Article 448, a "possessor in good
faith" under Article 526 and a "landowner in good faith' under Article 448?

HELD:
Owner of the land on which improvement was built by another in good faith is entitled to removal
of improvement only after landowner has opted to sell the land and the builder refused to pay for
the same. Res judicata doesnt apply wherein the first case was for ejectment and the other was
for quieting of title.

ART. 448. The owner of the land on which anything has been built sown or planted in good faith,
shall have the right to appropriate as his own the works, sowing or planting, after payment of the
indemnity provided for in articles 546 and 548, or to oblige the one who built or planted to pay
the price of the land, and the one who sowed, the proper rent.

However, the builder or planter cannot be obliged to buy the land if its value is considerably
more than that of the building or trees. In such case, he shall pay reasonable rent, if the owner
of the land does not choose to appropriate the building or trees after proper indemnity. The
parties shall agree upon the terms of the lease and in case of disagreement, the court shall fix
the terms thereof.

MCSS
Sarmiento v. Agana
129 SCRA 122

FACTS:
While one Ernesto was still courting his wife, the latter's mother had told him the couple could
build a residential house on a certain lot. They constructed a residential house.

Subsequently, the land was sold to petitioner Sarmiento, who asked Ernesto and his wife to
vacate. Sarmiento filed an Ejectment suit against them.

In the evidentiary hearing, Sarmiento submitted the deed of sale of the land which showed the
price to be PHP15,000. On the otherhand, Ernesto testified that the residential house then cost
PHP30,000-40,000, which was not questioned.

The MTC found that Ernesto was a builder in good faith and the house had a value of
PHP20,000. When the case was elevated, the CFI of Pasay ordered Sarmiento to exercise his
option, to reimburse Ernesto for the sum of the house or allow them to purchase the land, within
sixty days. Upon expiration of the period, Ernesto was allowed to deposit the sum of PHP25,000
with the Court as the purchase price for the land.

ISSUE:
WON private respondents are builders in good faith. -- YES

HELD:
Ernesto and his wife were builders in good faith in view of the peculiar circumstance under
which they had constructed the residential house. As far as they knew, the land was owned by
Ernesto's mother-in-law, and could reasonably be expected to later on give them the land.

The owner of the building erected in good faith on a land owned by another, is entitled to retain
the possession of the land until he is paid the value of his building. The owner of the land, has
the option either to pay for the building or to sell his land to the owner of the building. But he
cannot, as Sarmiento did, refuse both to pay for the building and to sell the land and compel the
owner of the building to remove it from the land where it is erected.

NKVS
Nuguid v. CA
452 SCRA 243

FACTS:
Pedro P. Pecson owned a commercial lot located at Quezon City, on which he built a four-door
two-storey apartment building. For failure to pay realty taxes, the lot was sold at public auction
by the City Treasurer of Quezon City to Nepomuceno, who in turn sold it for P103,000 to the
spouses Juan and Erlinda Nuguid.

Pecson challenged the validity of the auction sale before the RTC of Quezon City. In its
Decision, the RTC upheld the spouses title but declared that the four-door two-storey apartment
building was not included in the auction sale. This was affirmed in toto by the Court of Appeals
and thereafter by this Court, in its Decision dated May 25, 1993, in G.R. No. 105360 entitled
Pecson v. Court of Appeals.

Nuguid spouses moved for delivery of possession of the lot and the apartment building.

In its Order

of November 15, 1993, the trial court, relying upon Article 546 of the Civil Code,
ruled that the Spouses Nuguid were to reimburse Pecson for his construction cost of P53,000,
following which, the spouses Nuguid were entitled to immediate issuance of a writ of possession
over the lot and improvements. In the same order the RTC also directed Pecson to pay the
same amount of monthly rentals to the Nuguids as paid by the tenants occupying the apartment
units or P21,000 per month from June 23, 1993, and allowed the offset of the amount of
P53,000 due from the Nuguids against the amount of rents collected by Pecson from June 23,
1993 to September 23, 1993 from the tenants of the apartment.

After conducting a hearing, the lower court issued an Order dated, directing the spouses to pay
the sum of P1,344,000 as reimbursement of the unrealized income of Pecson for the period
beginning November 22, 1993 up to December 1997. The sum was based on the computation
of P28,000/month rentals of the four-door apartment

Pecson filed a petition for review before this Court. The Court handed down the decision
remanding to the trial court for it to determine the current market value of the apartment building
on the lot. The value so determined shall be forthwith paid by Spouses Juan and Erlinda Nuguid
to Pedro Pecson otherwise the petitioner shall be restored to the possession of the apartment
building until payment of the required indemnity.

On the basis of this Courts decision, Pecson filed a Motion to Restore Possession and a Motion
to Render Accounting, praying respectively for restoration of his possession over the subject
256-square meter commercial lot and for the spouses Nuguid to be directed to render an
accounting under oath, of the income derived from the subject four-door apartment from
November 22, 1993 until possession of the same was restored to him.

ISSUE:
Whether or not the petitioners should reimburse the respondent for the improvements of the
building -- YES

HELD:
It is not disputed that the construction of the four-door two-storey apartment, subject of this
dispute, was undertaken at the time when Pecson was still the owner of the lot. When the
Nuguids became the uncontested owner of the lot on June 23, 1993, by virtue of entry of
judgment of the Courts decision, dated May 25, 1993, in G.R. No. 105360, the apartment
building was already in existence and occupied by tenants. In its decision dated May 26, 1995
in G.R. No. 115814, the Court declared the rights and obligations of the litigants in accordance
with Articles 448 and 546 of the Civil Code. These provisions of the Code are directly
applicable to the instant case.

Under Article 448, the landowner is given the option, either to appropriate the improvement as
his own upon payment of the proper amount of indemnity or to sell the land to the possessor in
good faith. Relatedly, Article 546 provides that a builder in good faith is entitled to full
reimbursement for all the necessary and useful expenses incurred; it also gives him right of
retention until full reimbursement is made.

While the law aims to concentrate in one person the ownership of the land and the
improvements thereon in view of the impracticability of creating a state of forced co-ownership,it
guards against unjust enrichment insofar as the good-faith builders improvements are
concerned. The right of retention is considered as one of the measures devised by the law for
the protection of builders in good faith. Its object is to guarantee full and prompt reimbursement
as it permits the actual possessor to remain in possession while he has not been reimbursed
(by the person who defeated him in the case for possession of the property) for those necessary
expenses and useful improvements made by him on the thing possessed. Accordingly, a builder
in good faith cannot be compelled to pay rentals during the period of retention nor be disturbed
in his possession by ordering him to vacate. In addition, as in this case, the owner of the land is
prohibited from offsetting or compensating the necessary and useful expenses with the fruits
received by the builder-possessor in good faith. Otherwise, the security provided by law would
be impaired. This is so because the right to the expenses and the right to the fruits both pertain
to the possessor, making compensation juridically impossible; and one cannot be used to
reduce the other.

The text of the decision in G.R. No. 115814 expressly exempted Pecson from liability to pay
rentals, for we found that the Court of Appeals erred not only in upholding the trial courts
determination of the indemnity, but also in ordering him to account for the rentals of the
apartment building from June 23, 1993 to September 23, 1993, the period from entry of
judgment until Pecsons dispossession. As pointed out by Pecson, the dispositive portion of our
decision in G.R. No. 115814 need not specifically include the income derived from the
improvement in order to entitle him, as a builder in good faith, to such income. The right of
retention, which entitles the builder in good faith to the possession as well as the income
derived therefrom, is already provided for under Article 546 of the Civil Code.

AMPS
Sulo ng Nayon v. Nayong Filipino
G.R. No. 170923

DOCTRINE: Article 448 does not apply to a lessee-BPS. The basis of the application of Article
448 is the BPSs (good faith) belief of a claim of title.

FACTS:
Respondent Nayong Pilipino Foundation, a government-owned and controlled corporation, is
the owner of a parcel of land in Pasay City, known as the Nayong Pilipino Complex. Petitioner
Philippine Village Hotel, Inc. (PVHI), formerly called Sulo sa Nayon, Inc., is a domestic
corporation duly organized and existing under Philippine laws.

Contract of lease: respondent leased a portion of the Nayong Pilipino Complex, consisting of
36,289 square meters, to petitioner Sulo sa Nayon, Inc. for the construction and operation of a
hotel building, to be known as the Philippine Village Hotel. The lease was for an initial period of
21 years, or until May 1996. The contract was renewed for another 25 years or until 2021.

Beginning January 2001, petitioners defaulted in the payment of their monthly rental.
Respondent repeatedly demanded petitioners to pay the arrears and vacate the premises.
Hence, respondent filed a complaint for unlawful detainer before the MeTC of Pasay City.

ISSUE:
Should the rules on accession, as found in Articles 448 and 546 of the Civil Code, apply to the
improvements made by the lessee? -- NO. Article 1678 applies.

HELD:
The late Senator Arturo M. Tolentino, a leading expert in Civil Law, explains: This article [Article
448] is manifestly intended to apply only to a case where one builds, plants, or sows on land in
which he believes himself to have a claim of title, and not to lands where the only interest of the
builder, planter or sower is that of a holder, such as a tenant.

In the case at bar, petitioners have no adverse claim or title to the land. In fact, as lessees, they
recognize that the respondent is the owner of the land. What petitioners insist is that because of
the improvements, which are of substantial value, that they have introduced on the leased
premises with the permission of respondent, they should be considered builders in good faith
who have the right to retain possession of the property until reimbursement by respondent.

We affirm the ruling of the CA that introduction of valuable improvements on the leased
premises does not give the petitioners the right of retention and reimbursement which rightfully
belongs to a builder in good faith. Otherwise, such a situation would allow the lessee to easily
"improve" the lessor out of its property. We reiterate the doctrine that a lessee is neither a
builder in good faith nor in bad faith that would call for the application of Articles 448 and 546 of
the Civil Code. His rights are governed by Article 1678 of the Civil Code, which reads:

Art. 1678. If the lessee makes, in good faith, useful improvements which are
suitable to the use for which the lease is intended, without altering the form or
substance of the property leased, the lessor upon the termination of the lease
shall pay the lessee one-half of the value of the improvements at that time.
Should the lessor refuse to reimburse said amount, the lessee may remove the
improvements, even though the principal thing may suffer damage thereby. He
shall not, however, cause any more impairment upon the property leased than is
necessary.

With regard to ornamental expenses, the lessee shall not be entitled to any
reimbursement, but he may remove the ornamental objects, provided no damage
is caused to the principal thing, and the lessor does not choose to retain them by
paying their value at the time the lease is extinguished.

Under Article 1678, the lessor has the option of paying one-half of the value of the
improvements which the lessee made in good faith, which are suitable for the use for which the
lease is intended, and which have not altered the form and substance of the land. On the other
hand, the lessee may remove the improvements should the lessor refuse to reimburse.

Petitioners argue that to apply Article 1678 to their case would result to sheer injustice, as it
would amount to giving away the hotel and its other structures at virtually bargain prices. They
allege that the value of the hotel and its appurtenant facilities amounts to more than two billion
pesos, while the monetary claim of respondent against them only amounts to a little more than
twenty six-million pesos. Thus, they contend that it is the lease contract that governs the
relationship of the parties, and consequently, the parties may be considered to have impliedly
waived the application of Article 1678.

We cannot sustain this line of argument by petitioners. Basic is the doctrine that laws are
deemed incorporated in each and every contract. Existing laws always form part of any contract.
Further, the lease contract in the case at bar shows no special kind of agreement between the
parties as to how to proceed in cases of default or breach of the contract.

KGS
Pecson v. CA
244 SCRA 407

DOCTRINES:
1. Article 448 of the Civil Code does not apply to a case where the owner of the
land is the builder, sower, or planter who then later loses ownership of the land by sale or
donation.

2. The provision of Art. 448 on indemnity may be applied by analogy to a case
where one loses the ownership of the land on which he earlier built an apartment.

FACTS:
Pecson was the owner of a commercial lot on which he built a 4-door, 2-storey apartment. For
his failure to pay realty tax on said property, the lot was sold at public auction by the City
Treasurer of Quezon City to Nepomuceno, who in turn the property to Sps. Nuguid. Pecson
questioned the validity of the auction sale. RTC then, dismissed the complaint and ruled that the
apartment building was included in the sale. It reached the SC, but the same was denied.
Thereafter, Sps. Nuguid filed a Motion for delivery of possession, which was granted by the trial
court. Pecson contested. The Court of Appeals affirmed in part the order of the trial court citing
Article 448 of the Civil Code. CA ordered that Pecson should be indemnified with the
construction cost of the apartment.

ISSUE:
Is Article 448 applicable in a case wherein the owner of the land is also the builder who then
later loses ownership of the land by sale? -- NO

HELD:
By its clear language, Article 448 refers to a land whose ownership is claimed by two or more
parties, one of whom has built some works, or sown or planted something. The building, sowing
or planting may have been made in good faith or in bad faith. The rule on good faith laid down in
Article 526 of the Civil Code shall be applied in determining whether a builder, sower or planter
had acted in good faith. Article 448 does not apply to a case where the owner of the land is the
builder, sower, or planter who then later loses ownership of the land by sale or donation.
Elsewise stated, where the true owner himself is the builder of works on his own land, the issue
of good faith or bad faith is entirely irrelevant. Thus in strict point of law, Article 448 is not
apposite to the case at bar. Nevertheless, we believe that the provision therein on indemnity
may be applied by analogy considering that the primary intent of Article 448 is to avoid a state of
forced co-ownership and that the parties, including the two courts below, in the main agree that
Articles 448 and 546 of the Civil Code are applicable and indemnity for the improvements may
be paid although they differ as to the basis of the indemnity.

JPOT
Spouses Benitez v. CA
266 SCRA 242

DOCTRINE: "His right is older, and because, by the principle of accession, he is entitled to the
ownership of the accessory thing."

FACTS:
Spouses Macapagal bought a 303 sq. m. lot and found that Spouses Benitez had encroached
on their land. Both parties were amenable to a compromise and had sold the land in excess for
a P1000 per sq.m. The former spouses again acquired a lot adjacent to the Benitezes and after
a survey had discovered that their lot was encroached on yet again by the latter. They went to
court and filed legal action in the MeTC (San Juan), RTC (Pasig) as well as with the CA. It was
held by each respectively below:

On January 18, 1990, private respondents filed with the Metropolitan Trial Court of San Juan,
Branch 58, Civil Case No. 61004 for ejectment against petitioners. The MeTC of San Juan
decided in favor of the former, with the following disposition:

"WHEREFORE, in view of all the foregoing, judgment is hereby rendered for the
plaintiffs and against the defendants ordering them and all persons claiming rights under
them to vacate and surrender possession of the subject premises to the plaintiffs as well
as to pay the following:
1. The amount of P930.00 a month starting July 17, 1989 until they finally
vacate the subject premises;
2. The amount of P5,000.00 for and as attorney's fees; and
3. Cost of suit."

On appeal, the Regional Trial Court of Pasig, Branch 167, affirmed said decision. The RTC said:
"The controversy in this case is not an encroachment or overlapping of two (2) adjacent
properties owned by the parties. It is a case where a part of the house of the defendants is
constructed on a portion of the property of the plaintiffs. So that as new owner of the real
property, who has a right to the full enjoyment and possession of the entire parcel covered by
Transfer Certificate of Title No. 41961, plaintiffs have the right to demand that defendants
remove the portion of the house standing on plaintiff's realty. . . ."
The dispositive portion thereof reads:

"WHEREFORE, finding no reversible error in the decision appealed from, it being more
consistent with the facts and the law applicable, the same is hereby AFFIRMED in toto. Costs
against the defendant-appellants. SO ORDERED."

On further appeal, the respondent Court found no merit in petitioners' plea. In a Resolution
dated March 24, 1992, the Sixth Division of said Court found the petition to be a mere rehash of
the issues and arguments presented before the lower courts. It ruled in part that:
"3) Petitioners were fully aware that part of their house encroached on their neighbor's
property, while respondents became aware of it only after purchasing said property.
Petitioners cannot claim good faith as against the respondents.
"4) Since petitioners are not builders in good faith, they cannot demand that
respondents sell the disputed portion; what the law provides is that the builders in bad
faith can be ordered to dismantle said structure at their own expense. In the interim
period that petitioners' structure remains, they should pay reasonable rent until they
remove the structure."

The dispositive portion thereof reads:

"For reasons indicated, We find the appeal without merit and deny it due course, with costs
against the petitioners. SO ORDERED."

Spouses Benitez contest and have firm belief to the contrary, for which they assailed tweaked
claims to be reviewed by the SC.

ISSUE:

W/N possession of a lot encroached upon by a part of another's house be recovered in an
action for ejectment? -- YES

HELD:
The jurisdictional requirements for ejectment, as borne out by the facts, are: after conducting a
relocation survey, private respondents discovered that a portion of their land was encroached by
petitioners' house; notices to vacate were sent to petitioners, the last one being dated October
26, 1989; and private respondents filed the ejectment suit against petitioners on January 18,
1990 or within one (1) year from the last demand. Prior possession is not always a condition
sinequa non in ejectment. This is one of the distinctions between forcible entry and unlawful
detainer. Actual or physical occupation is not always necessary. Hence, ejectment is the proper
remedy for the case at bar.

In addition, the court also ruled that the MeTC had jurisdiction over the matter and were appaled
by the audacity of petitioners persistently arguing otherwise after receiving a decision which was
sound from the beginning. Furthermore, the "rental" issue is technically damages for which the
land owner is entitled to for the non-enjoyment and deprivation of his property. Moreover, the
option to sell rests solely with the land owner and only his. Article 448 is accorded the
landowner because "his right is older, and because, by the principle of accession, he is entitled
to the ownership of the accessory thing."

In sum, the SC affirms the decision of all lower courts and said decision is unanimous.

MLAV
Technogas Phil. V. CA
268 SCRA 5

DOCTRINE: Possession acquired in good faith continues to be enjoyed in the same character
in which it was acquired, until the contrary is proved. Good faith consists in the belief of the
builder that the land he is building on is his, and his ignorance of any defect or flaw in his title.
The good faith ceases from the moment defects in the title are made known to the possessor,
by extraneous evidence or by suit for recovery of the property by the true owner.

FACTS:
Technogas Philippines Manufacturing Corp is the registered owner of Lot 4531-A of Lot 4531
with all buildings, walls, and improvements therein, which they bought from Pariz Industries Inc.
The lot that it adjoined, Lot 4531-B, is owned and registered under the name of Eduardo Uy. It
was later found out that a portion of the walled lot encroached on the land owned by Uy.
Technogas offered to by the encroachment but Uy refused.

They entered into a private agreement to demolish the wall on the encroached land.
Consequently, Uy filed a complaint for encroachment before the office of the Municipal Engineer
and the Provincial Fiscal. However, the complaint of did not prosper. This prompted Uy to dig a
canal along the wall, causing it to collapse. Due to the damage, petitioner filed a complaint
against Uy for malicious mischief.

The RTC ruled in favor of Technogas and ordered Uy to sell the encroached land and pay for
damages to the wall.

The CA reversed the decision of the RTC and ordered Technogas to pay rent for the
encroachment.

ISSUE:
WON Technogas is a builder in good faith -- YES

HELD:
Article 527 of the Civil Code presumes good faith, and since no proof exists to show that the
encroachment over a narrow, needle-shaped portion of private respondents land was done in
bad faith by the builder of the encroaching structures, the latter should be presumed to have
built them in good faith. It is presumed that possession continues to be enjoyed in the same
character in which it was acquired, until the contrary is proved.

Good faith consists in the belief of the builder that the land he is building on is his, and his
ignorance of any defect or flaw in his title. Hence, such good faith, by law, passed on to Parizs
successor, petitioner in this case. The good faith ceases from the moment defects in the title
are made known to the possessor, by extraneous evidence or by suit for recovery of the
property by the true owner.

Consequently, the builder, if sued by the aggrieved landowner for recovery of possession, could
have invoked the provisions of Art. 448 of the Civil Code. The benefit to the builder under this
article is that, instead of being outrightly ejected from the land, he can compel the landowner to
make a choice between the two options: (1) to appropriate the building by paying the indemnity
required by law, or (2) sell the land to the builder. The landowner cannot refuse to exercise
either option and compel instead the owner of the building to remove it from the land.

In view of the good faith of both petitioner and private respondent, their rights and obligations
are to be governed by Art. 448. Hence, his options are limited to: (1) appropriating the
encroaching portion of petitioners building after payment of proper indemnity, or (2) obliging the
latter to buy the lot occupied by the structure. He cannot exercise a remedy of his own liking.

Petitioner, however, must also pay the rent for the property occupied by its building only up to
the date private respondent serves notice of its option upon petitioner and the trial court; that is,
if such option is for private respondent to appropriate the encroaching structure. In such event,
petitioner would have a right of retention which negates the obligation to pay rent. The rent
should however continue if the option chosen is compulsory sale, but only up to the actual
transfer of ownership.

DJTV
Manotok Reality v. Tecson
164 SCRA 587

DOCTRINES:
Issuance of writ of execution is proper even if private respondent was adjudged a builder
in good faith or peculiar circumstances supervened; Option to retain the premises and
pay for improvements or to sell the premises to the builder in good faith belongs to the
owner of the property.
Where the improvements have been gutted by fire, the basis for private respondents
right to retain the premises has already been extinguished without petitioners fault.

FACTS:
In a complaint filed by the petitioner for recovery of possession and damages against the private
respondent, the then Court of First Instance of Manila rendered judgment declaring the
defendant Nilo Madlangawa as a builder or possessor in good faith; ordering the plaintiff to
recognize the right of said defendant to remain in Lot No. 345, Block 1, of the Clara Tambunting
Subdivision until after he shall have been reimbursed by the plaintiff the sum of P7,500.00,
without pronouncement as to costs.

Petitioner filed with the trial court, presided over by respondent Judge Jose H. Tecson, a motion
for the approval of petitioner's exercise of option and for satisfaction of judgment, praying that
the court issue an order: a) approving the exercise of petitioner's option to appropriate the
improvements introduced by the private respondent on the property; b) thereafter, private
respondent be ordered to deliver possession of the property in question to the petitioner.

The respondent judge denied the motion by issuing the disputed order that under the peculiar
circumstances which supervened after the institution of this case, like, for instance, the
introduction of certain major repairs of and other substantial improvements on the controverted
property, the instant motion of the plaintiff is not well-taken and therefore not legally proper and
tenable.

After a denial of its motion for reconsideration, the petitioner filed the present petition for
mandamus alleging that the respondent judge committed grave abuse of discretion in denying
his motion to exercise option and for execution of judgment on the grounds that under Articles
448 and 546 of the Civil Code, the exercise of option belongs to the owner of the property, who
is the petitioner herein, and that upon finality of judgment, the prevailing party is entitled, as a
matter of right, to its execution which is only a ministerial act on the part of the respondent judge.

The private respondent filed his comment on the petition alleging that the same has already
become moot and academic because fire gutted not only the house of the private respondent
but the majority of the houses in Tambunting Estate.

Petitioner argues that since the judgment of the trial court has already become final, it is entitled
to the execution of the same and that moreover, since the house of the private respondent was
gutted by fire, the execution of the decision would now involve the delivery of possession of the
disputed area by the private respondent to the petitioner.

ISSUES:
1. Whether or not the private respondent is a builder in good faith. --
NO
2. Whether or not the issuance of writ of execution is proper even if
private respondent was adjudged a builder in good faith or peculiar circumstances
supervened. -- YES
3. Whether or not the basis for private respondents right to retain the
premises has already been extinguished without petitioners fault when the improvements
have been gutted by fire. -- YES

HELD:

1. The private respondent's good faith ceased after the filing of the complaint by the
petitioner. A possessor in good faith is entitled to the fruits only so long as his possession is
not legally interrupted, and such interruption takes place upon service of judicial summons
(Arts. 544 and 1123, Civil Code). Thus, the repairs and improvements introduced by the said
respondents after the complaint was filed cannot be considered to have been built in good
faith, much less, justify the denial of the petitioner's option.

2. When the decision of the trial court became final and executory, it became
incumbent upon the respondent judge to issue the necessary writ for the execution of the
same. There is, therefore, no basis for the respondent judge to deny the petitioner's motion
to avail of its option to approriate the improvements made on its property.

Neither can the respondent judge deny the issuance of a writ of execution because the
private respondent was adjudged a builder in good faith or on the ground of "peculiar
circumstances which supervened after the institution of this case, like, for instance, the
introduction of certain major repairs of and other substantial improvements..." because the
option given by law either to retain the premises and pay for the improvements thereon or to
sell the said premises to the builder in good faith belongs to the owner of the property.

3. Since the improvements have been gutted by fire, and therefore, the basis for
private respondent's right to retain the premises has already been extinguished without the
fault of the petitioner, there is no other recourse for the private respondent but to vacate the
premises and deliver the same to herein petitioner.


WHEREFORE, IN VIEW OF THE FOREGOING, the petition is GRANTED and the respondent
judge is hereby ordered to immediately issue a writ of execution ordering the private respondent
to vacate the disputed premises and deliver possession of the same to the petitioner.

JGY
Ballatan v. CA
304 SCRA 34

DOCTRINE: In the event that the owner elects to sell to the builder, planter or sower the land
on which the improvement stands, the price must be fixed at the prevailing market value at the
time of payment.

FACTS:
The parties herein are owners of adjacent lots. Lot 24 is registered in the name of petitioners
Ballatan. Lots 25 & 26 are registered in the name of respondent Go Sr. His son, Winston Go
constructed a house on Lot 25. Adjacent to Lot 26 is Lot 27, 4 registered in the name of
respondent Li Ching Yao.

Petitioner Ballatan constructed her house on Lot 24. During the construction, she noticed that
the concrete fence and side pathway of respondents house encroached her property. Ballatan
informed Go of this discrepancy and his encroachment on her property. Surveys were made
and it was found out that the lot area of petitioner Ballatan was less by a few meters and that of
respondent Li Ching Yao, which was three lots away, increased by two 2 meters.

Petitioner Ballatan made a written demand on respondents Go to remove and dismantle their
improvements on Lot 24. Respondents Go refused. Hence, Ballatan filed for recovery of
possession of real property. TC decided in favor of petitioners. It ordered the Go's to vacate the
subject portion, demolish their improvements. CA modified: ordered Go to pay Ballatan, and
respondent Li Ching Yao to pay Go and the value to be fixed at the time of the taking.

ISSUE:
WON the value should be fixed at the time of the taking. -- NO

HELD:

All the parties are presumed to have acted in good faith. Their rights must, therefore, be
determined in accordance with the appropriate provisions of the Civil Code on property
specifically Art. 448. Petitioners, as owners of Lot 24, may choose to purchase the improvement
made by respondents Go on their land, or sell to respondents Go the subject portion. If buying
the improvement is impractical as it may render the Go's house useless, then petitioners may
sell to respondents Go that portion of Lot 24 on which their improvement stands. If the Go's are
unwilling or unable to buy the lot, then they must vacate the land and, until they vacate, they
must pay rent to petitioners. Petitioners, however, cannot compel respondents Go to buy the
land if its value is considerably more than the portion of their house constructed thereon. If the
value of the land is much more than the Go's improvement, then respondents Go must pay
reasonable rent. If they do not agree on the terms of the lease, then they may go to court to fix
the same.

In the event that petitioners elect to sell to respondents Go the subject portion of their lot, the
price must be fixed at the prevailing market value at the time of payment.

JRPA
Geminiano v. CA
259 SCRA 344

DOCTRINE: Article 448 of the Civil Code, in relation to Article 546 of the same Code, which
allows full reimbursement of useful improvements and retention of the premises until
reimbursement is made, applies only to a possessor in good faith, i.e., one who builds on land
with the belief that he is the owner thereof. It does not apply where one's only interest is that of
a lessee under a rental contract; otherwise, it would always be in the power of the tenant to
"improve" his landlord out of his property.

FACTS:
Lot No. 3765-B-1 containing an area of 314sq was originally owned by the petitioners' mother,
Paulina Amado vda. de Geminiano. On a 12-square-meter portion of that lot stood the
petitioners' unfinished bungalow, which the petitioners sold to the private respondents with an
alleged promise to sell to the latter that portion of the lot occupied by the house. Subsequently,
the petitioners' mother executed a 7-year contract of lease over a 126 square-meter portion of
the lot, including that portion on which the house stood, in favor of the private respondents. The
private respondents then introduced additional improvements and registered the house in their
names. After the expiration of the lease contract however, the petitioners' mother refused to
accept the monthly rentals.

It turned out that the lot in question was the subject of a suit, which resulted in its acquisition by
one Maria Lee. Lee sold the lot to Lily Salcedo, who in turn sold it to the spouses Agustin and
Ester Dionisio. Dionisio spouses executed a Deed of Quitclaim over the said property in favor of
the petitioners.

As such, the lot was registered in the latter's name.

The petitioners sent, via
registered mail, a letters addressed to private respondent Mary Nicolas demanding that she
vacate the premises and pay the rentals in arrears. Upon failure of the private respondents to
heed the demand, the petitioners filed with the MTCC of Dagupan City a complaint for unlawful
detainer and damages.

ISSUES:
1. Whether or not Art 448 applies to this case? -- NO
2. Whether or not the private respondents are builder in good faith or mere
lessees? -- LESSEES

HELD:
The private respondents claim they are builders in good faith, hence, Article 448 of the Civil
Code should apply. They rely on the lack of title of the petitioners' mother at the time of the
execution of the contract of lease, as well as the alleged assurance made by the petitioners that
the lot on which the house stood would be sold to them.

It has been said that while the right to let property is an incident of title and possession, a
person may be lessor and occupy the position of a landlord to the tenant although he is not the
owner of the premises let.

After all, ownership of the property is not being transferred, only the
temporary use and enjoyment thereof.

In this case, both parties admit that the land in question was originally owned by the petitioners'
mother. The land was allegedly acquired later by one Maria Lee by virtue of an extrajudicial
foreclosure of mortage. Lee, however, never sought a writ of possession in order that she gain
possession of the property in question.

The petitioners' mother therefore remained in
possession of the lot.

It is undisputed that the private respondents came into possession of 126 square-meter portion
of the said lot by virtue of contract of lease executed by the petitioners' mother as lessor, and
the private respondents as lessees, is therefore well-established, and carries with it a
recognition of the lessor's title.

The private respondents, as lessees who had undisturbed
possession for the entire term under the lease, are then estopped to deny their landlord's title,
or to assert a better title not only in themselves, but also in some third person while they remain
in possession of the leased premises and until they surrender possession to the landlord. This
estoppel applies even though the lessor had no title at the time the relation of lessor and lessee
was created,

and may be asserted not only by the original lessor, but also by those who
succeed to his title.

Being mere lessees, the private respondents knew that their occupation of the premises would
continue only for the life of the lease. Plainly, they cannot be considered as possessors nor
builders in good faith.

Article 448 of the Civil Code, in relation to Article 546 of the same Code, which allows full
reimbursement of useful improvements and retention of the premises until reimbursement is
made, applies only to a possessor in good faith, i.e., one who builds on land with the belief that
he is the owner thereof. It does not apply where one's only interest is that of a lessee under a
rental contract; otherwise, it would always be in the power of the tenant to "improve" his landlord
out of his property.

Anent the alleged promise of the petitioners to sell the lot occupied by the private respondents'
house, the same was not substantiated by convincing evidence. Neither the deed of sale over
the house nor the contract of lease contained an option in favor of the respondent spouses to
purchase the said lot. And even if the petitioners indeed promised to sell, it would not make the
private respondents possessors or builders in good faith so as to covered by the provision of
Article 448 of the Civil Code. The latter cannot raise the mere expectancy or ownership of the
aforementioned lot because the alleged promise to sell was not fulfilled nor its existence even
proven.

ABB
Spouses del Campo v. Obesia
160 SCRA

DOCTRINE: When the co-ownership is terminated by the partition and it appears that the house
of the defendants occupies a portion of 5 square meters of the land pertaining to plaintiffs which
the defendant obviously built in good faith, the provisions of Article 448 of the new Civil Code
should apply.

FACTS:
An action for partition of a parcel of land was filed by the spouses Del Campo in the CFI of Cebu.
Plaintiffs and defendants are co-owners pro indiviso of this lot in the proportion of and 1/3 share
each, respectively. The trial court appointed a commissioner in accordance with the agreement
of the parties. The commissioner conducted a survey, prepared a sketch plan and submitted a
report to the trial court on May 29, 1976, recommending that the property be divided into two
lots: Lot 1161-A with an area of 30 square meters for the spouses Del Campo and Lot No.
1161-B with an area of 15 square meters for Obesia. Upon surveying, it was shown that the
house of Obesia occupied the portion with an area of 5 square meters of Lot 1161-A of the
spouses Del Campo. The parties manifested their conformity to the report and asked the trial
court to finally settle and adjudicate who among the parties should take possession of the 5
square meters of the land in question.

ISSUE:
Whether or Not Article 448 of the Civil Code is applicable to a builder in good faith when the
property involved is owned in common.

HELD:
When the co-ownership is terminated by the partition and it appears that the house of Obesia
occupies a portion of 5 square meters of the land pertaining to spouses Del Campo which
Obesia obviously built in good faith, the provisions of Article 448 of the new Civil Code should
apply.

In applying Article 448 of the New Civil Code, the plaintiffs have the right to appropriate said
portion of the house of defendants upon payment of indemnity to defendants as provided for in
Article 546 of the Civil Code. Otherwise, the plaintiffs may oblige the defendants to pay the price
of the land occupied by their house. But if the price asked for is considerably much more than
the value of the portion of the house of defendants built thereon, then the latter cannot be
obliged to buy the land. The defendants shall then pay the reasonable rent to the plaintiff upon
such terms and conditions that they may agree. In case of disagreement, the trial court shall fix
the terms thereof. The defendants may opt to demolish or remove the said portion of their house,
at their own expense, if they so decide.

FZC
Rosales, et al. v. Castellfort, et al.
G.R. No. 157044

DOCTRINE: Where the builder, planter or sower has acted in good faith, a conflict of rights
arises between the owners, and it becomes necessary to protect the owner of the improvements
without causing injustice to the owner of the land. In view of the impracticability of creating a
state of forced co-ownership, the law has provided a just solution by giving the owner of the land
the option to acquire the improvements after payment of the proper indemnity, or to oblige the
builder or planter to pay for the land and the sower the proper rent. He cannot refuse to exercise
either option. It is the owner of the land who is authorized to exercise the option, because his
right is older, and because, by the principle of accession, he is entitled to the ownership of the
accessory thing.

FACTS:
Spouses-petitioners Rodolfo V. Rosales and Lily Rosqueta-Rosales (petitioners) are the
registered owners of a parcel of land designated as Lot 17, Block 1 of Subdivision Plan situated
in Los Baos, Laguna.

Petitioners discovered that a house was being constructed on their lot, without their knowledge
and consent, by respondent Miguel Castelltort (Castelltort). It turned out that respondents
Castelltort and his wife Judith had purchased a lot, Lot 16 of the same Subdivision Plan, from
respondent Lina Lopez-Villegas (Lina) through her son-attorney-in-fact Rene Villegas (Villegas)
but that after a survey thereof by geodetic engineer Augusto Rivera, he pointed to Lot 17 as the
Lot 16 the Castelltorts purchased.

Negotiations for the settlement of the case thus began, with Villegas offering a larger lot near
petitioners lot in the same subdivision as a replacement thereof. In the alternative, Villegas
proposed to pay the purchase price of petitioners lot with legal interest. Both proposals were,
however, rejected by petitioners whose counsel directed Castelltort to stop the construction of
and demolish his house and any other structure he may have built thereon, and desist from
entering the lot. Petitioners subsequently filed a complaint for recovery of possession and
damages with prayer for the issuance of a restraining order and preliminary injunction against
spouses-respondents Miguel and Judith Castelltort before the RTC of Calamba, Laguna

ISSUE:
Under Art 448, who has the right of option?

HELD:
Under Article 448, the landowner can choose between appropriating the building by paying the
proper indemnity or obliging the builder to pay the price of the land, unless its value is
considerably more than that of the structures, in which case the builder in good faith shall pay
reasonable rent. If the parties cannot come to terms over the conditions of the lease, the court
must fix the terms thereof.The choice belongs to the owner of the land, a rule that accords with
the principle of accession, i.e.,that the accessory follows the principal and not the other way
around. Even as the option lies with the landowner, the grant to him, nevertheless, is preclusive.
The landowner cannot refuse to exercise either option and compel instead the owner of the
building to remove it from the land.The raison detre for this provision has been enunciated thus:
Where the builder, planter or sower has acted in good faith, a conflict of rights arises between
the owners, and it becomes necessary to protect the owner of the improvements without
causing injustice to the owner of the land. In view of the impracticability of creating a state of
forced co-ownership, the law has provided a just solution by giving the owner of the land the
option to acquire the improvements after payment of the proper indemnity, or to oblige the
builder or planter to pay for the land and the sower the proper rent. He cannot refuse to exercise
either option. It is the owner of the land who is authorized to exercise the option, because his
right is older, and because, by the principle of accession, he is entitled to the ownership of the
accessory thing.

LNAC
Ignao v. IAC, G.R. No. 72876
193 SCRA 17

DOCTRINE: When the co-ownership is terminated by a partition and it appears that the house
of an erstwhile co-owner has encroached upon a portion pertaining to another co-owner which
was however made in good faith, then the provisions of Article 448 should apply to determine
the respective rights of the parties.

FACTS:
Petitioner Florencio Ignao and his uncles private respondents Juan Ignao and Isidro Ignao were
co-owners of a parcel of land with an area of 534 square meters situated in Barrio Tabon,
Municipality of Kawit, Cavite. Pursuant to an action for partition filed by petitioner, the then CFI
directed the partition of the aforesaid land, allotting 133.5 square meters or 2/8 thereof to private
respondents Juan and Isidro, and giving the remaining portion with a total area of 266.5 square
meters to petitioner Florencio. However, no actual partition was ever effected.

Petitioner instituted a complaint for recovery of possession of real property against private
respondents Juan and Isidro before the CFI. In his complaint petitioner alleged that the area
occupied by the two (2) houses built by private respondents exceeded the 133.5 square meters
previously allotted to them by the trial court.

It was found that the houses of Juan and Isidro actually encroached upon a portion of the land
belonging to Florencio. Upon agreement of the parties, the trial court ordered a licensed
geodetic engineer to conduct a survey to determine the exact area occupied by the houses of
private respondents. The survey subsequently disclosed that the house of Juan occupied 42
square meters while that of Isidro occupied 59 square meters of Florencio's land or a total of
101 square meters.

Trial court ruled that although private respondents occupied a portion of Florencio's property,
they should be considered builders in good faith.

Petitioner appealed to IAC which subsequently affirmed the decision of the trial court.

Thus, herein petition.

ISSUE:
Whether the provisions of Article 448 should apply to a builder in good faith on a property held
in common. -- YES

HELD:
It should be noted that prior to partition, all the co-owners hold the property in common dominion
but at the same time each is an owner of a share which is abstract and undetermined until
partition is effected. As co-owners, the parties may have unequal shares in the common
property, quantitatively speaking. But in a qualitative sense, each co-owner has the same right
as any one of the other co-owners. Every co-owner is therefore the owner of the whole, and
over the whole he exercises the right of dominion, but he is at the same time the owner of a
portion which is truly abstract, because until division is effected such portion is not concretely
determined.

Article 448 provides: The owner of the land on which anything has been built, sown or planted in
good faith, shall have the right to appropriate as his own the works, sowing or planting, after
payment of the indemnity provided for in articles 546 and 548, or to oblige the one who built or
planted to pay the price of the land, and the one who sowed, the proper rent. However, the
builder or planter cannot be obliged to buy the land if its value is considerably more than that of
the building or trees. In such case, he shall pay reasonable rent, if the owner of the land does
not choose to appropriate the building or trees after proper indemnity. The parties shall agree
upon the terms of the lease and in case of disagreement, the court shall fix the terms thereof.

Whether or not the provisions of Article 448 should apply to a builder in good faith on a property
held in common has been resolved in the case of Spouses del Campo vs. Abesia, wherein the
Court ruled that:

The court a quo correctly held that Article 448 of the Civil Code cannot apply where a co-owner
builds, plants or sows on the land owned in common for then he did not build, plant or sow upon
land that exclusively belongs to another but of which he is a co-owner. The co-owner is not a
third person under the circumstances, and the situation is governed by the rules of co-
ownership.
However, when, as in this case, the ownership is terminated by the partition and it appears that
the home of defendants overlaps or occupies a portion of 5 square meters of the land pertaining
to plaintiffs which the defendants obviously built in good faith, then the provisions of Article 448
of the new Civil Code should apply.
Both the trial court and the Appellate Court erred when they peremptorily adopted the "workable
solution" in the case of Grana vs. CA, and ordered the owner of the land, petitioner Florencio, to
sell to private respondents, Juan and Isidro, the part of the land they intruded upon, thereby
depriving petitioner of his right to choose. Such ruling contravened the explicit provisions of
Article 448 to the effect that "(t)he owner of the land xxx shall have the right to appropriate xxx
or to oblige the one who built xxx to pay the price of the land xxx." The law is clear and
unambiguous when it confers the right of choice upon the landowner and not upon the builder
and the courts
TKDC
MWSS v. C.A.
143 SCRA 623

DOCTRINE: The right of a possessor in bad faith to remove improvements applies only to
improvements for pure luxury or mere pleasure, provided the thing does not suffer any injury
and the lawful possessor does not prefer to retain them by paying their value at the time of his
possession.

FACTS:
Dagupan City filed a complaint against MWSS for recovery of ownership and possession of the
Dagupan Waterworks System. MWSS interposed R.A. 1383 as its defense; it vested to MWSS
the ownership, possession, and control of all waterworks system throughout the Philippines.
MWSS also filed a counterclaim for reimbursement of expenses it incurred for necessary and
useful improvements.

Trial court ruled that MWSS is a possessor in bad faith so it is not entitled to claim
reimbursement. MWSS appealed to the Court of Appeals arguing that Dagupan City should be
liable for payment of the balance of the loan secured by MWSS for the improvement of the
Dagupan Waterworks System; however the Court of Appeals affirmed trial courts judgment.

MWSS appealed to the Supreme Court for the removal of useful improvements. Dagupan City
argues that MWSS is a possessor in bad faith so it has absolutely no right to the useful
improvements.

ISSUE:
Whether or not a possessor in bad faith has the right to remove useful improvements. -- NO

HELD:
Under Article 499 of the Civil Code, he who builds, plants, or sows in bad faith on the land of
another, loses what is built, planted, or sown without right to indemnity. Additionally, under
Article 546 of the Civil Code, only a possessor in good faith shall be refunded for useful
expenses with the right of retention until reimbursed. Finally, under Article 547 of the Civil Code,
only a possessor in good faith may remove useful improvements if this can be done without
damage to the principal thing and if the person who recovers the possession does not exercise
the option of reimbursing the useful expenses.

The right of a possessor in bad faith to remove improvements applies only to improvements for
pure luxury or mere pleasure, provided the thing does not suffer any injury and the lawful
possessor does not prefer to retain them by paying their value at the time of his possession.

In this case, MWSS is a builder in bad faith so it loses whatever useful improvements it made
without right to indemnity.

AMD
Alviola v. CA
289 SCRA 537

DOCTRINE: For Article 448 to apply, the construction must be of permanent character,
attached to the soil with an idea of perpetuity; but if it is of a transitory character or is
transferable, there is no accession, and the builder must remove the construction. The proper
remedy of the landowner is an action to eject the builder from the land.

FACTS:
In 1950, Victoria Tinagan purchased from Mauro Tinagan 2 parcels of land situated in Valencia,
Negros Occidental. Victoria and her son, Agustin Tinagan took possession of said parcels of
land. In 1960, the Alviola spouses occupied portions thereof whereat they built a copra dryer
and put up a store engaged in the buying and selling of copra. Both Victoria and Augustin died
in 1975. The latter was survived by private respondents, his wife Florencia and their children.

Editha Aviola, assisted by her husband, filed a complaint for partition and damages before the
CFI, claiming to be an acknowledged natural child of Augustin TInagan and demanding the
delivery of her share in the estate. The case was dismissed on the ground that recognition of
natural children should be brought only during the lifetime of the presumed parent.

In 1988, private respondents filed a complaint for recovery of possession against the Alviolas.
The RTC ruled in favor of private respondents and ordered the Alviolas to peacefully vacate and
to surrender the possession of the premises. The latter may remove their store and dryer
without injury and prejudice to the plaintiffs. Petitioners appealed to the CA, said court upheld
the RTC decision. Hence, this petition.

ISSUE:
W/N the copra dryer and the store are susceptible of accession (as accession industrial). -- NO

HELD:
The SC ruled that private respondents adduced overwhelming evidence to prove their
ownership and possession of the 2 parcels of land on portions of which petitioners built the
copra dryer and a store. In addition, the Alviolas stay thereon, since 1961, was merely by
tolerance on the part of the private respondents and their predecessor-in-interest. The evidence
shows that the petitioners were permitted by Victoria Tinagan to build a copra dryer on the land
when they got married.

As correctly ruled by the respondent court, there was bad faith on the part of the petitioners
when they constructed the copra dryer and store on the disputed portions since they were fully
aware that the parcels of land belonged to Victoria Tinagan. And, there was likewise bad faith
on the part of the private respondents, having knowledge of the arrangement between
petitioners and Victoria Tinagan relative to the construction of the copra dryer and store. Thus,
for purposes of indemnity, Article 448 of the New Civil Code should be applied. However, the
copra dryer and the store, as determined by the trial court and respondent court, are
transferable in nature. Thus, it would not fall within the coverage of Article 448. As the noted civil
law authority, Senator Arturo Tolentino, aptly explains: "To fall within the provision of this Article,
the construction must be of permanent character, attached to the soil with an idea of perpetuity;
but if it is of a transitory character or is transferable, there is no accession, and the builder must
remove the construction. The proper remedy of the landowner is an action to eject the builder
from the land." The private respondents action for recovery of possession was the suitable
solution to eject petitioners from the premises.

MPF
Arangote v. Maglunob
GR 178906

DOCTRINE: The rights under Article 448 and 546 of the Civil Code are applicable only to
builders in good faith and not to possessors in good faith.

Possession in Good Faith; Every possessor in good faith becomes a possessor in bad faith from
the moment he becomes aware that what he believed to be true is not so.

FACTS:
All the respondents in the case are siblings of Esperanza.
1985. Esperanza executed a Last Will and Testament bequeathing the
subject property she inherited from her father based on the Partition Agreement to Arangote.
1986. Esperanza executed an affidavit waiving and quitclaiming all her
rights, interest, and participation in the subject property in favor of Arangote.
Tax Declaration of the subject property was cancelled from Esperanza and was
issued in the name of Arangote
1989. Arangote constructed a house on the subject property.
1993. OCT was issued in the name of Arangote on the subject property.
However, respondents (Maglunobs) entered the subject property and built a concrete
wall behind and front of Arangotes house effectively blocking the entrance.
Arangote file a complaint against respondent for quieting title, declaration
of ownership and possession at MTC.

ISSUES:
Whether or not Arangote were builders in good faith. -- NO
Whether or not Arangote were entitled rights under Article 448 and 526. -- NO

HELD:
Possessors in good faith are those who are not aware that there exists in his title or mode of
acquisition and flaw which invalidates it. Good faith of the possessor consists in the reasonable
belief that the person from whom he received the thing was the owner thereof, and could
transmit his ownership. Every possessor in good faith becomes possessor in bad faith from the
moment he becomes aware that what believed to be true is not so. In the case, the subject
property waived and quitclaimed by Esperanza to the petitioners in affidavit was only covered by
a tax declaration in the name of Esperanza.

Arangote did not look into the origin if the subject property and to probe if Esperanza has the
right to relinquish the subject property. Thus, when Arangote built their house, they cannot be
considered to have acted in good faith because they only relied on tax declaration.

Settles is the rule that a tax declaration does not prove ownership and the Supreme Court also
added that payment of taxes is not a proof of ownership. Hence, Arangote is not a builder and
possessor in good faith.

Accession Natural

AMDG
Reynante v. C.A.
207 SCRA 794

DOCTRINE: Granting that the lots were created by alluvial formation and while it is true that
accretions which the bank of rivers may gradually receive from the effect of the current become
the property of the owner of the banks, such accretion to registered land does not preclude
acquisition of the additional are by another person through prescription

FACTS:
Petitioner, Jose Reynante was a tenant over the two lots of Don Cosme Carlos for more than 50
years. A fishpond is situated between the two lots. During his tenancy, Reynante constructed a
nipa hut, and planted and harvested nipa plams, to which he appropriated it as his own. When
Don Cosme Carlos died, his heirs, private respondents, executed a written agreement with
Reynante wherein the latter would turn over the fishpond to the former. After executing the
agreement, Reynante surrendered the fishponds as well as the lots. The fishpond was leased to
one Carlos de la Cruz. However, Reynante did not vacate the lots and continued to live there
and harvest the nipa palms he planted. The private respondents formally demanded that
Reynante vacate the lots to which the latter refused to do. A forcible entry with preliminary
injunction was filed against Reynante however the trial court dismissed the complaint basing
from the fact that Reynante was the prior possessor of the lots. The trial courts decision was
affirmed by the Court of Appeals.

ISSUES:
1. Who between the petitioner and private respondents has prior physical
possession the two lots? -- PETITIONER REYNANTE
2. Whether or not the disputed lots belong to private respondents as a result of
accretion? -- NO

HELD:
It has been held that party who can prove prior possession can recover such possession even
against the owner himself. As long as the party is able to prove prior possession he is entitled to
remain on the property until he is lawfully ejected. It is clear from the records that Reynante has
been in possession of the lots for more than 50 years. Consequently, the court cannot legally
grant the possession over the two lots to the private respondents.

The Court of Appeals ruled that the two lots were created by alluvial formation hence under
Article 457 of the New Civil code the ownership of such is granted to the private respondents.
However, although the lands to which the additional areas (which are the two lots subject in this
case) are attached to are registered; the failure to register the additional lands subjected it to
acquisition through prescription. Applying this to the case, the private respondents never
registered the two lots and since Reynante has been in possession of such for more than 50
years already, its possession over it must be respected unless the private respondents are able
to show that they have the better title over it.

GCG
Vda. de Nazareno v. C.A.
257 SCRA 589

DOCTRINE: Accretion, as a mode of acquiring property under Article 457 of the Civil Code,
requires the concurrence of these requisites: (1) that the deposition of soil or sediment be
gradual and imperceptible; (2) that it be the result of the action of the waters of the river (or sea);
and (3) that the land where accretion takes place is adjacent to the banks or rivers (or the sea
coast). These are called the rules on alluvion which if present in a case, give to the owners of
lands adjoining the banks of rivers or streams any accretion gradually received from the effects
of the current of waters.

FACTS:
A parcel of land situated in Telegrapo, Puntod, Cagayan de Oro City is said to have been
formed as a result of sawdust dumped into the dried-up Balacanas Creek and along the banks
of the Cagayan river. Jose Salasalan and Leo Rabaya leased the subject lots on which their
houses stood from one Antonio Nazareno, petitioners predecessor-in-interest. Salasalan and
Rabaya allegedly stopped paying rentals.

As a result, Antonio Nazareno and petitioners filed a case for ejectment with the MTC. A
decision was rendered against Salasalan and Rabaya, which decision was affirmed by the RTC.
The case was remanded to the Municipal trial court for execution of judgment after the same
became final and executory. Private respondents filed a case for annulment of judgment before
the RTC Misamis Oriental which was dismissed. The decision of the lower court was finally
enforced with the private respondents being ejected from portions of the subject lots they
occupied.

Before he died, Antonio Nazareno caused the approval by the Bureau of Lands of the survey
plan designated with a view to perfecting his title over the accretion area being claimed by him.
Before the approved survey plan could be released to the applicant, however, it was protested
by private respondents before the Bureau of Lands. The report of the Land Investigator, made in
compliance with the order of the District Land Officer, recommended the Survey Plan MSI-10-
06-000571-D (Lot 36302, Cad. 237) in the name of Antonio Nazareno be cancelled and that
private respondents be directed to file appropriate public application. Based on the report, the
Regional Director of the Bureau of Lands rendered a decision ordering an amendment to the
survey plain of Nazareno by segregating therefrom the areas occupied by the private
respondents. Antonio Nazareno filed a motion for reconsideration with the Undersecretary of
Natural Resources and OIC of the Bureau of Lands; which was denied.

The petitioners Desamparada vda. De Nazareno and Leticia Tapia Nazero filed a case before
the RTC for the annulment of the decision and order of the Bureau of Lands regarding the
parcel of land.

The RTC dismissed the complaint for failure to exhaust administrative remedies, resulting to the
finality of the administrative decision of the Bureau of Lands. On appeal, the Court of Appeals
affirmed the decision of the RTC dismissing the complaint. Hence, the petition.

ISSUE:
Whether or not accretion belongs to the riparian owners -- NO


HELD:
No. Supreme Court dismissed the petition for lack of merit.

Article 457 of the Civil Code provides that to the owners of land adjoining the banks of rivers
belong the accretion which they gradually receive from the effects of the current of the waters.
In the case of Meneses v. CA, it was held that accretion, as a mode of acquiring property under
Article 457 of the Civil Code, requires the concurrence of these requisites: (1) that the
deposition of soil or sediment be gradual and imperceptible; (2) that it be the result of the action
of the waters of the river (or sea); and (3) that the land where accretion takes place is adjacent
to the banks or rivers (or the sea coast). These are called the rules on alluvion which if present
in a case, give to the owners of lands adjoining the banks of rivers or streams any accretion
gradually received from the effects of the current of waters.

The application of the rules on alluvion cannot be made in the present case as the first and
second requirements of the rules were not met. Thus, the Nazarenos cannot claim the rights of
a riparian owner. By their own admission, the accretion was formed by the dumping of boulders,
soil and other filling materials on portions of the Balacanas Creek and the Cagayan River
bounding their land. It cannot be claimed, therefore, that the accumulation of such boulders, soil
and other filling materials was gradual and imperceptible, resulting from the action of the waters
or the current of the Balacanas Creek and the Cagayan River.

In Hilario v. City of Manila, it was held that the word current indicate the participation of the
body of water in the ebb and flow of waters due to high and low tide.

Petitioners are estopped from denying the public character of the subject land, as well as the
jurisdiction of the Bureau of Lands when the late Antonio Nazareno filed his Miscellaneous
Sales Application MSA (G-6) 571. The mere filing of said Application constituted an admission
that the land being applied for was public land, having been the subject of Survey Plan MSI-10-
06-000571-D which was conducted as a consequence of Antonio Nazarenos Miscellaneous
Sales Application wherein said land was described as an orchard. Said description by Antonio
Nazareno was controverted by the findings of the ocular inspection that said land actually
covers a dry portion of Balacanas Creek and a swampy portion of Cagayan River.

The Bureau of Lands classified the subject land as an accretion area which was formed by
deposits of sawdust in the Balacanas Creek and the Cagayan river, in accordance with the
ocular inspection conducted by the Bureau of Lands. It has often enough held that findings of
administrative agencies which have acquired expertise because their jurisdiction is confined to
specific matters are generally accorded not only respect but even finality. Again, when said
factual findings are affirmed by the Court of Appeals, the same are conclusive on the parties
and not reviewable by the Supreme Court.

In Republic v. CA, it was ruled that the requirement that the deposit should due to the effect of
the current of the river is indispensable. This excludes from Article 457 of the Civil Code all
deposits caused by human intervention. Putting it differently, alluvion must be the exclusive
work of nature. Thus, in Tiongco v. Director of Lands, et al., where the land was not formed
solely by the natural effect of the water current of the river bordering said land but is also the
consequence of the direct and deliberate intervention of man, it was deemed a man-made
accretion and, as such, part of the public domain. In the present case, the subject land was the
direct result of the dumping of sawdust by the Sun Valley Lumber Co. consequent to its sawmill
operations. As the accretion site was the result of the late Antonio Nazarenos labor consisting
in the dumping of boulders, soil and other filling materials into the Balacanas Creek and
Cagayan River bounding his land, the same would still be part of the public domain.

VCL IV
Bagaipo v. C.A.
347 SCRA 443

DOCTRINE: In the absence of evidence that the change in the course of the river was sudden
or that it occurred through avulsion, the presumption is that the change was gradual and was
caused by alluvium and erosion.

FACTS:
Petitioner Dionisia P. Bagaipo is the registered owner of Lot which located southeast of Davao
river. While respondent Leonor Lozano is the owner of a registered parcel of land located
across and opposite the southeast portion of petitioners lot facing the Davao River.

On May 26, 1989, Bagaipo filed a complaint for Recovery of Possession with Mandatory Writ of
Preliminary Injunction and Damages against Lozano for: (1) the surrender of possession by
Lozano of a certain portion of land measuring 29,162 square meters which is supposedly
included in the area belonging to Bagaipo under TCT No. T-15757; and (2) the recovery of a
land area measuring 37,901 square meters which Bagaipo allegedly lost when the Davao River
traversed her property. Bagaipo contended that as a result of a change in course of the said
river, her property became divided into three lots, namely: Lots 415-A, 415-B and 415-C.In
January 1988, Bagaipo commissioned a resurvey of Lot 415 and presented before the trial court
a survey plan prepared by Geodetic Engineer Gersacio A. Magno which concluded that the land
presently located across the river and parallel to Bagaipos property still belonged to the latter
and not to Lozano who planted some 350fruit-bearing trees on Lot 415-C and the old
abandoned river bed.

For his part, Lozano insisted that the land claimed by Bagaipo is actually an accretion to their
titled property. He asserted that the Davao River did not change its course and that the
reduction in Bagaipos domain was caused by gradual erosion due to the current of the Davao
River. Lozano added that it is also because of the rivers natural action that silt slowly deposited
and added to his land over a long period of time. He further averred that this accretion continues
up to the present and that registration proceedings instituted by him over the alluvial formation
could not be concluded precisely because it continued to increase in size.

The Trial Court, upon inspection, found that the decrease in land area was brought about by
erosion and not a change in the rivers course. CA affirmed the TC ruling.

ISSUE:
Whether or not there was a change in the rivers course which resulted to avulsion? -- NO

HELD:
The trial court and the appellate court both found that the decrease in land area was brought
about by erosion and not a change in the rivers course. This conclusion was reached after the
trial judge observed during ocular inspection that the banks located on petitioners land are
sharp, craggy and very much higher than the land on the other side of the river. Additionally, the
riverbank on respondents side is lower and gently sloping. The lower land therefore naturally
received the alluvial soil carried by the river current.

These findings are factual, thus conclusive on this Court, unless there are strong and
exceptional reasons, or they are unsupported by the evidence on record, or the judgment itself
is based on a misapprehension of facts.

The decrease in petitioners land area and the corresponding expansion of respondents
property were the combined effect of erosion and accretion respectively. Art. 461 of the Civil
Code is inapplicable. Petitioner cannot claim ownership over the old abandoned riverbed
because the same is inexistent. The riverbeds former location cannot even be pinpointed with
particularity since the movement of the Davao River took place gradually over an unspecified
period of time, up to the present.

The rule is well-settled that accretion benefits a riparian owner when the following requisites are
present: 1) That the deposit be gradual and imperceptible; 2) That it resulted from the effects of
the current of the water; and 3) That the land where accretion takes place is adjacent to the
bank of the river.

These requisites were sufficiently proven in favor of respondents. In the absence of evidence
that the change in the course of the river was sudden or that it occurred through avulsion, the
presumption is that the change was gradual and was caused by alluvium and erosion.

FXRL
Agustin v. IAC
187 SCRA 218

DOCTRINE: Accretion benefits a riparian owner when the following requisites are present: (1)
that the deposit be gradual and imperceptible; (2) that it resulted from the effects of the current
of the water; and (3) that the land where accretion takes place is adjacent to the bank of a river .

FACTS:
Petitioner Agustin is the owner of a parcel of land on the eastern side of the
Cagayan river.
Private respondents Binuyag, et al. own parcels of land on the western side of
the Cagayan river.
From 1919 to 1968, the river transferred land (by alluvium) from the property of
Agustin to that of the respondents.
In 1968, a flood made the river changed its course and now cut through the land
of the respondents.
This change in course forced the respondents to cross the river whenever they
have to cultivate their land
Seeing this, the petitioner sought the assistance of the local government in
displacing the respondents while they were planting crops on the land now separated by the
river
RTC ruled in favor of the respondents and ordered the petitioners to vacate the
land.
IAC affirmed the ruling.

ISSUE:
W/N the respondents ownership of the land was affected by the sudden change in the course of
the river. -- NO

HELD:
No. The land separated by the course of the river remains to be that of the respondents.

Art. 457 of the civil code provides that:
To the owners of lands adjoining the banks of rivers belong the accretion which they gradually
receive from the effects of the current of the waters. (366)

Accretion benefits a riparian owner when the following requisites are present: (1) that the
deposit be gradual and imperceptible; (2) that it resulted from the effects of the current of the
water; and (3) that the land where accretion takes place is adjacent to the bank of a river .

All these requisites of accretion are present in this case as the river had indeed, over a period of
time (49 years), deposited the land gradually and imperceptibly from the current if its water and
that it is adjacent to the riverbank.

Further, the private respondents' ownership of the accretion to their lands was not lost upon the
sudden and abrupt change of the course of the Cagayan River in 1968 or 1969 when it reverted
to its old 1919 bed, and separated or transferred said accretions to the eastern bank of the river.
Articles 459 and 463 of the Civil Code apply to this situation.

Art. 459. Whenever the current of a river, creek or torrent segregates from an estate on its bank
a known portion of land and transfers it to another estate, the owner of the land to which the
segregated portion belonged retains the ownership of it, provided that he removes the same
within two years.

Art. 463. Whenever the current of a river divides itself into branches, leaving a piece of land or
part thereof isolated, the owner of the land retains his ownership. He also retains it if a portion of
land is separated from the estate by the current.

In the case at bar, the sudden change of course of the Cagayan River as a result of a strong
typhoon in 1968 caused a portion of the lands of the private respondents to be "separated from
the estate by the current." The private respondents have retained the ownership of the portion
that was transferred by avulsion to the other side of the river.

RSDM
Viajar v. CA
168 SCRA 405

DOCTRINE: Registration does not protect the riparian owner against the diminution of the area
of his land through gradual changes in the course of the adjoining stream. Accretions which the
banks of rivers may gradually receive from the effect of the current become the property of the
owners of the banks (Art. 366 of the Old Civil Code; Art. 457 of the New). Such accretions are
natural incidents to land bordering on running streams and the provisions of the Civil Code in
that respect are not affected by the Registration Act.

FACTS:
The spouses Ricardo Y. Ladrido and Leonor P. Ladrido were the owners of Lot No. 7511 of the
Cadastral Survey of Pototan situated in barangay Cawayan, Pototan, Iloilo. This lot contained
an area of 154,267 square meters and was registered in the names of the spouses under
Transfer Certificate of Title No. T-21940 of the Register of Deeds of Iloilo.

Spouses Rosendo H. Te and Ana Te were also the registered owners of a parcel of land
described in their title as Lot No. 7340 of the Cadastral Survey of Pototan.

Rosendo H. Te, with the conformity of Ana Te, sold this lot to Angelica F. Viajar and Celso F.
Viajar. A Torrens title was later issued in the names of Angelica F. Viajar and Celso F. Viajar.

Later, Angelica F. Viajar had Lot No. 7340 relocated and found out that the property was in the
possession of Ricardo Y. Ladrido. Consequently, she demanded its return but Ladrido refused.

Angelica F. Viajar and Celso F. Viajar instituted a civil action for recovery of possession and
damages against Ricardo Y. Ladrido. Summoned to plead, defendant Ladrido filed his answer
with a counterclaim. Plaintiffs filed their reply to the answer.

CFI ruled in favor of the defendants which the CA confirmed. There was a mention in the case
that the issue from which the decision of the CFI was not the issue appealed in the CA so the
affirmation made by the CA should be void.

ISSUES:
1. Whether or not the change in the course of the Suague River was sudden
2. Whether or not the plaintiffs are protected by the Torrens Title.

HELD:
No it was gradual. The trial court found that the change in the course of the Suague River was
gradual and this finding was affirmed by the respondent Court of Appeals. We do not find any
valid reason to disturb this finding of fact.

Article 457 of the New Civil Code must be construed to limit the accretion mentioned therein as
accretion of unregistered land to the riparian owner, and should not extend to registered land.
Thus, the lot in question having remained the registered land of the petitioners, then the private
respondents cannot acquire title there in derogation to that of the petitioners, by accretion, for
that will defeat the indefeasibility of a Torrens Title.

The rule that registration under the Torrens System does not protect the riparian owner against
the diminution of the area of his registered land through gradual changes in the course of an
adjoining stream is well settled.

MRAM
Binalay v. Manalo
195 SCRA 374

DOCTRINE: For accretion to take place as a mode of acquiring ownership over the land, the
land formed should be directly adjacent to the land owned.

FACTS:
Guillermo Manalo bought parcels of land in Isabela; 8.65 hectares from was acquired from
Faustino Taccad and 1.80 hectares was bought from Gregorio Taguba. The parcels of land
were described as having the Cagayan River on their west.

During a cadastral survey conducted, the two parcels of land was consolidated into one lot and
was designated as Lot No. 307. But since the survey was conducted on a rainy day, a portion of
the land bought from Taccad was covered with water and was not included in Lot No. 37.

The Sketch Plan shows that the Cagayan River running from south to north, forks at a certain
point to form 2 branches (eastern and western branches) and then unites at the other end,
further north, to form a narrow strip of land. It appears that eastern branch of the river cuts
through the land of Manalo and is inundated with water during rainy season. The bed of the
eastern branch is the unsurveyed portion of the land belonging to Manalo, and is, for most part
of the year (about 8 months), dry and susceptible to cultivation.

Lot 821 is located directly opposite Lot 307 and is separated from it during the rainy season.
Being a portion of the land bought from Taccad, Manalo claims that Lot 821 also belongs to him
by way of accretion to the submerged portion of the property to which it is adjacent. Petitioners
(Binalay et al) however claims ownership over the land as they have possessed it, occupied it
and have cultivated it.

ISSUES:
Whether or not Manalo owns Lot 821?

HELD:
The Court held in the negative saying that 1.) it is part of public dominion and 2.) it fails to meet
all the requisites needed for accretion to take place.

First, it must be noted that Art. 70 of the Law of Waters defines the natural bed or channel of a
creek or river as the ground covered by its waters during the highest floods. The periodic
swelling of the water was taken into consideration and it was concluded that the submerged
portion of the land during rainy days forms part of the natural bed of the river. In connection with
this conclusion, Art. 420 of the Civil Code provides that rivers form part of public dominion.

Second, accretion as a mode of acquiring property under Art. 457 of the Civil Code requires the
concurrence of 3 requisites: 1) that the deposition of soil or sediment be gradual and
imperceptible; 2) that it be the result of the action of the waters of the river (or sea); and 3) that
the land where accretion takes place is adjacent to the banks of rivers (or the sea coast). In this
case, the claimed accretion lies on the bank of the river not adjacent to Lot 307 but directly
opposite Lot 307 across the river.

Lastly, SC held that it is difficult to suppose that a land with an area of 22.72 hectares resulted
from slow accretion to another lot of almost equal size. If Manalos contention is accepted, then
his land would have doubled in a span of 10 years.

All these considered, the Court held that the land in question is part of public dominion and
neither Manalo nor the petitioners were held owners of the land.

FMM
Republic v. CA
132 SCRA 514

DOCTRINE: Article 457 of the New Civil Code states that, To the owners of lands adjoining the
banks of rivers belong the accretion which they gradually receive from the effects of the current
of the waters.

Article 457 requires the concurrence of three requisites before an accretion is said to have taken
place: (1) That the deposit be gradual and imperceptible; (2) that it be made through the effects
of the current of the water; and (3) that the land where accretion takes place is adjacent to the
banks of the rivers.

FACTS:
Tancinco (the Tancincos) are registered owners of a parcel of land situated at Barrio Ubihan,
Meycauayan, Bulacan bordering on the Meycauayan and Bocaue Rivers.

On 24 June 1973, the Tancincos filed the application for the registration of three lots adjacent to
their fishpond property (Lots 1, 2 and 3 of plan Psu-131892).

However, on 5 April 1974, Assistant Provincial Fiscal Amando C. Vicente, representing the
Bureau of Lands (the Republic) filed a written opposition to the said application for registration.

On 6 March 1975, in line with the recommendation of the Commissioner appointed by the Court,
the Tancincos filed a partial withdrawal of the application for registration of Lot 3.

On 7 March 1975, Lot 3 was ordered withdrawn from the application and trial proceeded only
with respect to Lots 1 and 2.

On 26 June 1976, the Court of First Instance of Bulacan rendered a decision granting the
application on the finding that the lots (1 and 2) are accretions to the Tancincos fishpond
property.

On 30 July 1976, the Republic appealed to the Court of Appeals (the CA)

However, the CA, on 19 August 1982, rendered a decision affirming in toto the decision of the
lower court.

Hence, the Republic filed this petition with the Supreme Court (the SC). The Republic
contended that there is no accretion to speak of under Article 457 of the New Civil Code
because the Tancincos simply transferred their dikes further down the riverbed of the
Meycauayan River, and thus, if there is any accretion to speak of, it is man-made and artificial
and not the result of the gradual and imperceptible sedimentation by the waters of the river.

ISSUE:
Whether there was accretion to the fishpond property of the Tancincos. -- NO

HELD:
The SC agreed with the Republics contention that there is no accretion to speak of under
Article 457 of the New Civil Code.

The SC held that Article 457 of the New Civil Code which provides that To the owners of lands
adjoining the banks of rivers belong the accretion which they gradually receive from the effects
of the current of the waters, requires the concurrence of three requisites before an accretion is
said to have taken place, namely: (1) that the deposit be gradual and imperceptible; (2) that it be
made through the effects of the current of the water; and (3) that the land where accretion takes
place is adjacent to the banks of rivers.

The SC emphasized that the requirement that the deposit should be due to the effect of the
current of the river is indispensable. It held that this excludes from Article 457 of the New Civil
Code all deposits caused by human intervention.

It further held that alluvion must be the exclusive work of nature. On the contrary, in the instant
case, there is no evidence to prove that the addition to the fishpond property was made
gradually through the effects of the current of the Meycauayan and Bocaue rivers. The alleged
alluvial deposits came into being not because of the sole effect of the current of the rivers but as
a result of the transfer of the dike towards the river and encroaching upon it by reclamation.

Thus, the SC granted the Republics petition, reversed and set aside the decisions of the lower
courts and ordered the Tancincos to move back the dikes of their fishpond to their original
location and return the disputed property to the river to which it belongs.

RGGM
Republic v. CA
131 SCRA 532

DOCTRINE: Laguna de bay is a lake and that part around it which becomes covered with water
four to five months a year, not due to tidal actions, but due to rains cannot be considered as part
of the bed or basin of Laguna de Bar nor as a foreshore land; Property being so, the land is
registerable under the LRA.

FACTS:
The subject land in this case is situated 20 meters away from the shores of Laguna de Bay.
Said land was owned by Benedicto del Rio. After the death of Benedicto, the land was acquired
by his son Santos Del Rio. The private oppositors in this case sought and obtained permission
from Santos Del Rio to construct duck houses on said land. The private oppositors, however,
violated their agreement and instead constructed residential houses thereon. Santos then filed
an ejectment suit against the private oppositors and later on sought to register the land.
Meanwhile, private oppositors simultaneously filed their respective sales applications with
Bureau of Lands, and they opposed Santos del Rios application for registration.

The CFI of Laguna dismissed the application for registration. Applicant appealed and obtained a
favourable judgment from the Court of Appeals. The Director of Lands and the private
oppositors filed their respective petitions for review on said decision to the Supreme Court.

The Director of Lands contends that since a portion of the land is covered with water four to five
months a year, the same is part of the lake bed of Laguna de Bay and therefore it cannot be the
subject of registration.

ISSUE:
1. Whether or not the parcel of land in question is public land; and
2. Whether or not applicant private respondent has registerable title to the land.

HELD:
The inundation of a portion of the land is not due to "flux and reflux of tides." It cannot be
considered a foreshore land, hence it is not a public land and therefore capable of registration
as private property provided that the applicant proves that he has a registerable title. The
purpose of land registration under the Torrens System is not the acquisition of lands but only the
registration of title which applicant already possesses over the land.

While it is true that by themselves tax receipts and declarations of ownership for taxation
purposes are not incontrovertible evidence of ownership, they become strong evidence of
ownership acquired by prescription when accompanied by proof of actual possession of the
property. Applicant by himself and through his father before him, has been in open, continuous,
public, peaceful, exclusive and adverse possession of the disputed land for more than thirty (30)
years and has presented tax declarations and tax receipts.

Applicant has more than satisfied the legal requirements. Thus, he is clearly entitled to the
registration in his favor of said land.

MCSS
Heirs of Navarro v. IAC
268 SCRA 74

FACTS:
On October 3, 1946, Sinforoso Pascual filed an application for foreshore lease covering a tract
of foreshore land in Sibocon, Balanga, Bataan, having an area of approximately seventeen (17)
hectares.

This application was denied on January 15, 1953. So was his motion for reconsideration.
Subsequently, petitioners' predecessor-in-interest, also now deceased, Emiliano Navarro, filed a
fishpond application with the Bureau of Fisheries covering twenty five (25) hectares of foreshore
land also in Sibocon, Balanga, Bataan. Initially the application was denied, eventually however
the grant was given. Pascual claimed that this land is an accretion to his property, The Talisay
River as well as the Bulacan River flow downstream and meet at the Manila Bay thereby
depositing sand and silt on Pascual's property resulting in an accretion thereon. Sinforoso
Pascual claimed the accretion as the riparian owner.

On March 25, 1960, the Director of Lands, represented by the Assistant Solicitor General, filed
an opposition thereto stating that neither Pascual nor his predecessors-in-interest possessed
sufficient title to the subject property, the same being a portion of the public domain and,
therefore, it belongs to the Republic of the Philippines. On November 10, 1975, the courta
quorendered judgment finding the subject property to be foreshore land and, being a part of the
public domain, it cannot be the subject of land registration proceedings.

On appeal, the respondent court reversed the findings of the court a quo and granted the
petition for registration of the subject property but excluding certain areas. A motion for
reconsideration was filed by in the CA but the same was denied. Anchoring their claim of
ownership on Article 457 of the Civil Code, petitioners vigorously argue that the disputed 14-
hectare land is an accretion caused by the joint action of the Talisay and Bulacan Rivers which
run their course on the eastern and western boundaries, respectively, of petitioners' own tract of
land.

ISSUE:
WON the petitioners can rightfully claim the land under the principle of accretion.

HELD:
The petitioners claim is misplaced. The principle of accretion is only applicable to owners
whose estates are adjacent to rivers as stated in Article 457 of the Civil Code. The disputed land
is an accretion not on a river bank but on a sea bank, or on what used to be the foreshore of
Manila Bay which adjoined petitioners' own tract of land on the northern side. As such, the
applicable law is not Article 457 of to Civil Code but Article 4 of the Spanish Law of Waters of
1866.

The disputed property is an accretion on a sea bank, Manila Bay being an inlet or an arm of the
sea; as such, the disputed property is, under Article 4 of the Spanish Law of Waters of 1866,
part of the public domain. As part of the public domain, the herein disputed land is intended for
public uses, and "so long as the land in litigation belongs to the national domain and is reserved
for public uses, it is not capable of being appropriated by any private person, except through
express authorization granted in due form by a competent authority."Only the executive and
possibly the legislative departments have the right and the power to make the declaration that
the lands so gained by action of the sea is no longer necessary for purposes of public utility or
for the cause of establishment of special industries or for coast guard services.

Petitioners utterly fail to show that either the executive or legislative department has already
declared the disputed land as qualified, under Article 4 of the Spanish Law of Waters of 1866, to
be the property of petitioners as owners of the estates adjacent thereto.

NKVS
Grande v. CA
5 SCRA 524

FACTS:
Petitioners are the owners of a parcel of land, with an area of 3.5032 hectares, located at the
province of Isabela, by inheritance from their deceased mother Patricia. Said property is
identified as Lot No. 1. When it was surveyed for purposes of registration sometime in 1930, its
northeastern boundary was the Cagayan River. Since then, and for many years thereafter, a
gradual accretion on the northeastern side took place, by action of the current of the Cagayan
River, so much so, that by 1958, the bank thereof had receded to a distance of about 105
meters from its original site, and an alluvial deposit of 19,964 square meters, more or less, had
been added to the registered area.

On January 25, 1958, petitioners instituted the present action in the CFI of Isabela against
respondents, to quiet title to said portion formed by accretion, alleging in their complaint that
they and their predecessors-in-interest, were formerly in peaceful and continuous possession
thereof, until September, 1948, when respondents entered upon the land under claim of
ownership. Petitioners also asked for damages corresponding to the value of the fruits of the
land as well as attorney's fees and costs. In their answer, respondents claim ownership in
themselves, asserting that they have been in continuous, open, and undisturbed possession of
said portion, since prior to the year 1933 to the present.

ISSUE:
1. W/N the land formed through accretion becomes automatically registered land of the
riparian owner. -- NO
2. W/N defendants acquired the property through prescription. -- YES

HELD:
1
st
Issue: NO, land formed through accretion does not become automatically registered.

There can be no dispute that both under Article 457 of the New Civil Code and Article 366 of the
old, petitioners are the lawful owners of said alluvial property, as they are the registered owners
of the land which it adjoins. The question is whether the accretion becomes automatically
registered land just because the lot which receives it is covered by a Torrens title thereby
making the alluvial property imprescriptible. We agree with the Court of Appeals that it does not,
just as an unregistered land purchased by the registered owner of the adjoining land does not,
by extension, become ipso facto registered land. Ownership of a piece of land is one thing, and
registration under the Torrens system of that ownership is quite another. Ownership over the
accretion received by the land adjoining a river is governed by the Civil Code. Imprescriptibility
of registered land is provided in the registration law. Registration under the Land Registration
and Cadastral Acts does not vest or give title to the land, but merely confirms and thereafter
protects the title already possessed by the owner, making it imprescriptible by occupation of
third parties. But to obtain this protection, the land must be placed under the operation of the
registration laws wherein certain judicial procedures have been provided. The fact remain,
however, that petitioners never sought registration of said alluvial property (which was formed
sometime after petitioners' property covered by Original Certificate of Title No. 2982 was
registered on June 9, 1934) up to the time they instituted the present action in the Court of First
Instance of Isabela in 1958. The increment, therefore, never became registered property, and
hence is not entitled or subject to the protection of imprescriptibility enjoyed by registered
property under the Torrens system. Consequently, it was subject to acquisition through
prescription by third persons.

2
nd
Issue: YES, the defendants acquired the property through prescription.

This is a question which requires determination of facts: physical possession and dates or
duration of such possession. The Court of Appeals, after analyzing the evidence, found that
respondents-appellees were in possession of the alluvial lot since 1933 or 1934, openly,
continuously and adversely, under a claim of ownership up to the filing of the action in 1958.
This finding of the existence of these facts, arrived at by the Court of Appeals after an
examination of the evidence presented by the parties, is conclusive as to them and can not be
reviewed by us.
The law on prescription applicable to the case is that provided in Act 190 and not the provisions
of the Civil Code, since the possession started in 1933 or 1934 when the pertinent articles of the
old Civil Code were not in force and before the effectivity of the new Civil Code in 1950. Hence,
the conclusion of the Court of Appeals that the respondents acquired alluvial lot in question by
acquisitive prescription is in accordance with law.

AMPS
Ronquillo v. CA
195 SCRA 433

DOCTRINE: The rules of accretion do not apply where the drying-up of river is not due to a
natural change in the course of the waters. Herein, the change was man-made (pollution).
The dried-up portion belongs to the State as land of public domain.

FACTS:

Plaintiff Rosendo del Rosario was a registered owner of a certain Lot No. 34. Adjoining said lot
is a dried-up portion of the old Estero Calubcub occupied by the defendant since 1945.

Plaintiffs claim that long before the year 1930, when title was issued over the lot to his name,
Rosendo del Rosario had been in possession of this including the adjoining dried-up portion of
the old Estero Calubcub having bought the same from Arsenio Arzaga.

In the early part of 1945 defendant (Ronquillo) occupied the eastern portion of said titled lot as
well as the dried-up portion of the old Estero Calubcub which abuts plaintiffs' titled lot. After a
relocation survey of the land in question sometime in 1960, plaintiffs learned that defendant was
occupying a portion of their land and thus demanded defendant to vacate said land when the
latter refused to pay the reasonable rent for its occupancy. However, despite said demand
defendant refused to vacate.

Rosendo, Amparo and Florencia, all surnamed del Rosario (Del Rosarios), lodged a complaint
with the Court of First Instance of Manila praying, among others, that they be declared the
rightful owners of the dried-up portion of Estero Calubcub.

The trial court upheld the complaint. On appeal, respondent court, in affirming the decision of
the trial court, declared that since Estero Calubcub had already dried-up way back in 1930 due
to the natural change in the course of the waters, under Article 370 of the old Civil Code, the
abandoned river bed belongs to the Del Rosarios as riparian owners.

Hence, the petition before the SC.

Before the SC, the Director of Lands in his Comment stated:

We do not see our way clear to subscribe to the ruling of the Honorable Court of
Appeals on this point for Article 370 of the Old Civil Code, insofar as ownership
of abandoned river beds by the owners of riparian lands are concerned, speaks
only of a situation where such river beds were abandoned because of a natural
change in the course of the waters. Conversely, we submit that if the
abandonment was for some cause other than the natural change in the course of
the waters, Article 370 is not applicable and the abandoned bed does not lose its
character as a property of public dominion not susceptible to private ownership in
accordance with Article 502 (No. 1) of the New Civil Code. In the present case,
the drying up of the bed, as contended by the petitioner, is clearly caused by
human activity and undeniably not because of the natural change of the course
of the waters.

ISSUE:
Whether the dried-up portion of Estero Calubcub being claimed by herein petitioner was caused
by a natural change in the course of the waters; and, corollary thereto, is the issue of the
applicability of Article 370 of the old Civil Code.

HELD:
Caused by man. Article 370, thus, does not apply.

A careful perusal of the evidence presented by both parties in the case at bar will reveal that the
change in the course of Estero Calubcub was caused, not by natural forces, but due to the
dumping of garbage therein by the people of the surrounding neighborhood.

Private respondent Florencia del Rosario, in her testimony, made a categorical statement which
in effect admitted that Estero Calubcub changed its course because of the garbage dumped
therein, by the inhabitants of the locality.

In addition, the relocation plan which also formed the basis of respondent court's ruling, merely
reflects the change in the course of Estero Calubcub but it is not clear therefrom as to what
actually brought about such change. There is nothing in the testimony of lone witness Florencia
del Rosario nor in said relocation plan which would indicate that the change in the course of the
estero was due to the ebb and flow of the waters. On the contrary, the aforequoted testimony of
the witness belies such fact, while the relocation plan is absolutely silent on the matter. The
inescapable conclusion is that the dried-up portion of Estero Calubcub was occasioned, not by a
natural change in the course of the waters, but through the active intervention of man.

The foregoing facts and circumstances remove the instant case from the applicability of Article
370 of the old Civil Code which provides:

Art. 370. The beds of rivers, which are abandoned because of a natural change
in the course of the waters, belong to the owners of the riparian lands throughout
the respective length of each. If the abandoned bed divided tenements belonging
to different owners the new dividing line shall be equidistant from one and the
other.

Hence, the Del Rosarios cannot be entitled thereto supposedly as riparian owners.

The dried-up portion of Estero Calubcub should thus be considered as forming part of the land
of the public domain which cannot be subject to acquisition by private ownership.

KGS
Baes v. CA
224 SCRA 562

DOCTRINE: If the riparian owner is entitled to compensation for the damage to or loss of his
property due to natural causes, there is all the more reason to compensate him when the
change in the course of the river is effected through artificial means.

FACTS:
The government dug a canal on a private parcel of land to streamline the Tripa de Gallina creek.
This lot was later acquired by Baes and registered it under his name. Said lot was subdivided
into 3 parts: A, B, and C. However, Lot B was totally occupied by the canal. To resolve this, the
government gave Baes a lot exactly the same area in exchange for Lot B. It was near but not
contiguous to Lot C. The soil displaced by the canal was used to fill up the old bed of the creek.
Thereafter, the lots were resurveyed and subdivided wherein Lot A was designated as Lot 1-A
and B.

After years, the Republic of the Philippines discovered that Lot 1-B on which Baes erected an
apartment building, was a filled-up portion of the Tripa de Gallina creek. Baes claimed that they
own said lot as they became the owners of the old bed by virtue of Article 461. The government
rejects this claim and avers that sps. Baes had already been fully compensated for it when they
agreed to exchange their Lot B to another lot belonging to the government.

ISSUE:
Is Baes entitled for compensation for his lot which was totally occupied by the man-made canal
by virtue of Article 461? -- YES

HELD:
If the riparian owner is entitled to compensation for the damage to or loss of his property due to
natural causes, there is all the more reason to compensate him when the change in the course
of the river is effected through artificial means. The loss to the petitioners of the land covered by
the canal was the result of a deliberate act on the part of the government when it sought to
improve the flow of the Tripa de Gallina creek. It was therefore obligated to compensate the
Baeses for their loss. We find, however, that the petitioners have already been so compensated.
Felix Baes was given Lot 3271-A in exchange for the affected Lot 2958-B through the Deed of
Exchange of Real Property dated June 20, 1970. This was a fair exchange because the two lots
were of the same area and value and the agreement was freely entered into by the parties. The
petitioners cannot now claim additional compensation because, as correctly observed by the
Solicitor General,. . . to allow petitioners to acquire ownership of the dried-up portion of the
creek would be a clear case of double compensation and unjust enrichment at the expense of
the state. The exchange of lots between the petitioners and the Republic was the result of
voluntary negotiations. If these had failed, the government could still have taken Lot 2958-B
under the power of eminent domain, upon payment of just compensation, as the land was
needed for a public purpose.

JPOT
Jagualing v. CA
194 SCRA 607

DOCTRINE: Article 465 of the Civil Code -an island belongs to the owner of the land along the
nearer margin as sole owner thereof; or more accurately, because the island is longer than the
property of private respondents, they are deemed ipso jure to be the owners of that portion
which corresponds to the length of their property along the margin of the river.

FACTS:
Janita Eduave inherited a parcel of land in Mindoro which was eroded and became
surmountably underwater because of typhoon Ineng. She has actual possesion of land and
tends to its needs. She pays taxes, even though, declared land fails in comparison to its present
size. Eduave also made a loan with Luzon Surety in consideration of said land for P6000.
Eventually, because of the sudden increase in size of said lot from 4,937 sq.m. to16,452 sq.m.
and the formation of an island, Janita permitted petitioners to occupy land in support of her
endeavors warding away informal settlers as well as planting corn and bananas in duration of
their stay.

Afterwhich, petitioners assail that they would be the rightful owners of the said land by
admission stating that they acquired it through prescription.

ISSUE:
Between the one who has actual possession of an island that forms in a non-navigable and non-
floatable river and the owner of the land along the margin nearest the island, who has the better
right thereto?

HELD:
Although there is much controversy regarding the matter, the meat of it is stated on the issue
aforementioned. In this regard the Court of Appeals did not err in applying Article 465 of the
Civil Code.
12
Under this provision, the island belongs to the owner of the land along the nearer
margin as sole owner thereof; or more accurately, because the island is longer than the property
of private respondents, they are deemed ipso jure to be the owners of that portion which
corresponds to the length of their property along the margin of the river.

What then, about the adverse possession established by petitioners? Are their rights as such
not going to be recognized? It is well-settled that lands formed by accretion belong to the
riparian owner.
13
This preferential right is, under Article 465, also granted the owners of the land
located in the margin nearest the formed island for the reason that they are in the best position
to cultivate and attend to the exploitation of the same.
14
In fact, no specific act of possession
over the accretion is required.
15
If, however, the riparian owner fails to assert his claim thereof,
the same may yield to the adverse possession of third parties, as indeed even accretion to land
titled under the torrens system must itself still be registered.

WHEREFORE, We find no error committed by respondent court and DENY the petition for lack
of sufficient merit. The decision of respondent Court of Appeals is hereby AFFIRMED, without
pronouncement as to costs. SO ORDERED.

MLAV
Payatas-Estate Improvement Co. v. Tuason
G.R. No. L-30067

DOCTRINE: Accretions, as contemplated in Article 366 of the Civil Code, are natural incidents
to land bordering on running streams and are not affected by the registration laws. It follows that
registration does not protect the riparian owner against diminution of the area of his land
through gradual changes in the course of the adjoining stream.

FACTS:
Maria de la Concepcion Martinez Canas was originally the owner of Payatas estate, principal
part was bounded by the east side Mariquina river. An estate belonging to the Tuasons adjoined
the river on the other side.

After the initiation of the Torrens system of land registration, Maria de la Concepcion Canas had
the property surveyed and obtained the certificate of title. Later on, the land, consisting of 3
parcels, A, B and C, was sold to the Payatas Estate Improvement Company.

In 1920, another survey was made for subdivision purposes, and on October 15, 1924, the
subdivision plans were submitted to the Court of First Instance of Rizal for approval.

On October 25, 1924, the Payatas Estate Improvement Company filed another motion in which
it asked that transfer certificate of title no. 8691 be cancelled as to parcels A and C but not in
regard to parcel B, the latter not being included in the subdivision. The CFI approved, and
certificate of titles were issued accordingly.

On March 18, 1925, the Payatas Estate Improvement Co., filed another motion alleging that the
area of the subdivided land parcels A and C together with parcel B did not include all the land to
which the company was entitled. The motion was accompanied by a plan of two strips of land
situated along the eastern side of the Mariquina River, and the company asked that a certificate
of title be issued in its favor, to which the Tuasons opposed.

The CFI Denied the motion of Payatas Estate.

ISSUE:
WON Payatas Estate has right of ownership over the disputed area -- NO

HELD:
Article 366 of the Civil Code states: "any accretions which the banks of rivers may gradually
receive from the effect of the current belong to the owners of the estates bordering thereon."
Accretions of that character are natural incidents to land bordering on running streams and are
not affected by the registration laws. It follows that registration does not protect the riparian
owner against diminution of the area of his land through gradual changes in the course of the
adjoining stream.

Mariquina river separates the Payatas estate from the Mariquina estate and constitutes the
boundary between the two estates. The river has changed its course to the prejudice of the
Payatas estate on the western side of the stream and to the benefit of Mariquina estate by
increasing the latter's area, and the 22 hectares now in controversy which formerly were on the
Payatas side of the river are now on the Mariquina estate side.

Assuming this to be true, was the change in the course of the direction of the river was caused
by erosion and accretion or has it occurred through avulsion? The presumption is that the
change is gradual and caused by the erosion of the Payatas bank of the river and consequent
accretion to the Mariquina estate. (Martinez Canas vs. Tuason (5 Phil., 688)) It follows that the
land in question is now a part of the estate and no longer pertains to the Payatas estate.

Accession with Respect to Movable Property

DJTV
Santos v. Bernabe
54 Phil 19

DOCTRINE: Article 381 of the Civil Code states: If, by the will of their owners, two things of
identical or dissimilar nature are mixed, or if the mixture occurs accidentally, if in the latter case
the things cannot be separated without injury, each owner shall acquire a right in the mixture
proportionate to the part belonging to him, according to the value of the things mixed or
commingled.

FACTS:
Plaintiff Urbano Santos deposited 778 cavans and 38 kilos of palay and appellant Pablo
Tiongson deposited 1,026 cavans and 9 kilos of the same grain in defendant Jose C. Bernabe's
warehouse. It does not appear that the sacks of palay deposited in Jose C. Bernabe's
warehouse bore any marks or signs nor were they separated one from the other.

Pablo Tiongson filed with the Court of First Instance of Bulacan a complaint against Jose C.
Bernabe, to recover the cavans and kilos palay he deposited in the defendant's warehouse. At
the same time, the application of Pablo Tiongson for a writ of attachment was granted, and the
attachable property of Jose C. Bernabe, including 924 cavans and 31 1/2 kilos of palay found by
the sheriff in his warehouse, were attached, sold at public auction, and the proceeds thereof
delivered to said defendant Pablo Tiongson, who obtained judgment in said case.

Plaintiff, Urbano Santos, intervened in the attachment of the palay, but upon Pablo Tiongson's
filing the proper bond, the sheriff proceeded with the attachment, giving rise to the present
complaint.

ISSUE:
Whether or not plaintiff acquired right over the mixture of cavans and kilos of palay. -- YES

HELD:
The sheriff having found only 924 cavans and 31 1/2 kilos of palay in said warehouse at the
time of the attachment thereof and there being no means of separating form said 924 cavans
and 31 1/2 of palay belonging to Urbano Santos and those belonging to Pablo Tiongson, the
following rule prescribed in article 381 of the Civil Code for cases of this nature, is applicable:

Art. 381. If, by the will of their owners, two things of identical or dissimilar nature are mixed, or if
the mixture occurs accidentally, if in the latter case the things cannot be separated without injury,
each owner shall acquire a right in the mixture proportionate to the part belonging to him,
according to the value of the things mixed or commingled.

The number of kilos in a cavan not having been determined, we will take the proportion only of
the 924 cavans of palay which were attached and sold, thereby giving Urbano Santos, who
deposited 778 cavans, 398.49 thereof, and Pablo Tiongson, who deposited 1,026 cavans,
525.51, or the value thereof at the rate of P3 per cavan.

Wherefore, the judgment appealed from is hereby modified, and Pablo Tiongson is hereby
ordered to pay the plaintiff Urbano Santos the value of 398.49 cavans of palay at at the rate of
P3 a cavan, without special pronouncement as to costs. So ordered.

JGY
Siari Valley Estate v. Lucasan
97 Phil. 987

DOCTRINE: One who has stolen a part of the stolen money must have taken the larger sum
lost by the offended party. If the commingling of two things is made in bad faith, the one
responsible for it will lose his share.

FACTS:
Siara Valley Estate filed an action to recover 200 head of cattle that were driven to the adjoining
ranch of Lucasan, which the latter denied having appropriated or retained any cattle belonging
to the former. Lucasan alleging that theres no actual evidence on the number of missing bulls
and that plaintiffs cattle comingle with his. Trial Court: Ruled in favor of Siara Valley.

ISSUE:
WON Lucasan can recover his share of the cattle. -- NO

HELD:
Defendants cowboys and even his sons Rafael and Vicente- rounded up and drove plaintiff's
cattle into his pasture; he knew he had plaintiff's cattle, but refused toreturn them despite
demands by plaintiff; he even threatened plaintiff's men when the latter tried to retrieve its
animals; he harassed them with false prosecutions for their attempts to get back the company's
animals; he wouldn't allow plaintiff' s cowboys to get into his pasture to identify its flock; he
rebranded several Siari Valley cattle with his own brand; he sold cattle without registering the
sales; after some cattle impounded were entrusted to his custody as trustee, he disposed of not
less than 5 head of cattle among those he received as such trustee; lastly, he disposed of much
more cattle than he had a right to.

One who has stolen a part of the stolen money must have taken the larger sum lost by the
offended party. If the commingling of two things is made in bad faith, the one responsible for it
will lose his share.

JRPA
Aguirre v. Pheng
18 SCRA 18

DOCTRINE: Although ordinarily, the owner would be entitled to any accession thereto, the rule
is different where the works or improvements or the accession was made on the property by
one who acted in good faith.

FACTS:
On June 28, 1954, Vicente Aldaba and Teresa V. Aldaba sold to Jesus Aguirre a circular bolted
steel tank with a capacity of 5,000 gallons, for the sum of P900.00, for which the latter delivered
to the sellers duly endorsed, Security Bank & Trust Company check No. 281912, in the amount
of P900.00. Aguirre, however, failed to, take physical possession of the tank, having been
prevented from doing so by the municipal authorities of Los Baos, Laguna (where the tank was
located), in view of the claim of ownership being made by the Bureau of Public Highways. It
appears, however, that Vicente and Teresa Aldaba again sold the same tank on December 2,
1954 to Zosimo Gabriel, for P900.000. Gabriel, in turn, sold it to the Leonora & Company on
December 5, 1954, for P2,500.00. After some alterations and improvements made on the tank,
Leonora & Company was able to sell the tank to National Shipyards & Steel Corporation
(Nassco), for P14,500.00.



ISSUE:
1. WON Aguirre can take ownership of the property -- YES
2. WON Aguirre should reimburse Leonora for the improvements -- YES

HELD:
It is clear that we have here a case of accession by specification: Leonora and Company, as
purchaser acting in good faith, spending P11,299.00 for the reconditioning of the tank which is
later adjudged to belong to petitioner Aguirre. There is no showing that without the works made
by Leonora & Company, the tank in its original condition when Aguirre paid P900.00 therefor,
would command the price of P14,500 which Nassco was willing to pay. Although ordinarily,
therefore, Aguirre, as owner of the tank, would be entitled to any accession thereto, the rule is
different where the works or improvements or the accession was made on the property by one
who acted in good faith.
2
And, it is not contended that the making of the improvements and
incurring of expenses amounting to P11,299.00 by Leonora & Company was done in bad faith.
Furthermore, to uphold petitioner's contention that he is entitled to the sum of P14,500.00 the
price of the tank in its present condition, would be to allow him to enrich himself at the expense
of another. The lower courts, therefore, acted correctly in ordering the reimbursement to
Leonora & Company of the expenses it made on the tank.



QUIETING OF TITLE AND CO-OWNERSHIP (ART. 476-501)

Quieting of Title

ABB
Realty Sales Enterprises, Inc. v. IAC
G.R. No. L-67451

DOCTRINE: One is considered an innocent purchaser for value only if, relying on the title, he
bought the property from the registered owner without notice that some other person has a right
to or interest in such property and pays a full and fair price for the same, at the time such
purchase or before he receives notice of the claim or interest of some other persons in the
property.

FACTS:
Two adjacent lands situated in Las Pinas were covered by three (3) distinct sets of Torrents
titles. Morris Carpo, filed a complaint with the CFI (Vera Court) for declaration of nullity of
Decree no. N-66934 and TCT 20408 against Realty Sales Enterprise, Inc. Macondray Farms
and the Commission of Land Registration.

Prior to all these, Estanislao Mayuga, father of Dominador Mayuga (Realtys predecessor-in-
interest) originally filed on June 24, 1927 a registration proceeding to confirm his title over
parcels of land (Lots 1, 2, 3; lots 2 and 3 currently the subject of the litigation). The LRC case
was jointly tried with the cases filed by Eduardo Guico and Florentino Baltazar involving the
same pieces of land.

The Baltazars herein impleaded in this case are the heirs of Florentino Baltazar of the original
application filed by Estanislao Mayuga. Carpo bought the disputed property from them. The CFI
confirmed title of Estanislao Mayuga to Lots 1, 2, and 3 and held that the succesors of
Florentino Baltazar cannot feign ignorance to the land registration proceedings initiated by the
elder Baltazar, Guico and Mayuga.

The title in the name of Dominador Mayuga from whom Realty Sales Enterprise, Inc. derived its
title was issued in 1958, 12 years before the issuance of the title in the name of Baltazar in 1970.

ISSUE:
Whether or not Carpo was an innocent purchaser of value and in good faith. -- NO

HELD:
One is considered an innocent purchaser for value only if, relying on the title, he bought the
property from the registered owner without notice that some other person has a right to or
interest in such property and pays a full and fair price for the same, at the time such purchase or
before he receives notice of the claim or interest of some other persons in the property.

In the case at bar, at the time of the sale, there was as yet no Torrens title which Carpo could
have relied upon so that he may qualify as an innocent purchaser for value. Therefore, Carpos
claim should be discredited by the court because Realty Sales Enterprise, Inc. is the one with
the valid title to these properties.

Also, the general rule is that in case of two (2) certificates of title purporting to include the same
land, the one earlier in date prevails. Since the title in the name of Dominado Mayuga
(Predecessor-in-interest of Realty Sales Enterprise, Inc) was issued in 1958, 12 years earlier
from the issuance of title in the name of Baltazar in 1970, it is held that Mayuga hold the better
right to the property in question.

FZC
Lucasan v. PDIC
G.R. No. 176926

DOCTRINE: To avail of the remedy of quieting of title, two (2) indispensable requisites must
concur, namely: (1) the plaintiff or complainant has a legal or an equitable title to or interest in
the real property subject of the action; and (2) the deed, claim, encumbrance or proceeding
claimed to be casting a cloud on his title must be shown to be in fact invalid or inoperative
despite its prima facie appearance of validity or legal efficacy.

For levied properties sold on public auctions, the right to redeem becomes functus officio on the
date of its expiry, and its exercise after the period is not really one of redemption but a
repurchase. The judgment debtor therefore, no longer has legal or equitable title to or interest in
the real property subject of the action. Thus, he may not avail of the remedy of quieting of title.

FACTS:
Petitioner Inocencio Y. Lucasan (Lucasan) and his wife Julianita Sorbito (now deceased) were
the owners of Lot situated in Bacolod City. Pacific Banking Corporation (PBC) extended a loan
to Lucasan, with Carlos Benares as his co-maker. Lucasan and Benares failed to pay the loan
when it became due and demandable. Consequently, PBC filed a collection case with the RTC
of Bacolod City. The RTC rendered a decision ordering Lucasan and Benares to jointly and
severally pay PBC until the full payment of the obligation. Lucasan failed to pay the monetary
award; thus, to satisfy the judgment, the RTC issued a writ of execution directing the sheriff to
effect a levy on the properties owned by Lucasan and sell the same at public auction. In
compliance with the writ, the City Sheriff of Bacolod issued a Notice of Embargo. Annotated
therefore on Lucasans TCTs as prior encumbrances on the same titles were the mortgages in
favor of Philippine National Bank (PNB) and Republic Planters Bank (RPB) executed to secure
Lucasans loans with the banks.

The lots were later sold at a public auction where PBC was awarded the highest bidder. A
certificate of sale was executed in its favor and was registered. Neither PNB nor RPB, the
mortgagees, assailed the auction sale. Lucasan, as well as the mortgagee banks, PNB and
RPB, did not redeem the properties within the redemption period. Nevertheless, PBC did not file
a petition for consolidation of ownership.

Lucasan, through counsel, wrote a letter to the Philippine Deposit Insurance Corporation (PDIC),
PBCs receiver and liquidator seeking the cancellation of the certificate of sale and offering to
pay PBCs claim against Lucasan. Not long thereafter, Lucasan paid his loans with the PNB and
RPB. Consequently, the mortgagee banks executed their respective releases of mortgage,
resulting in the cancellation of the prior encumbrances in favor of PNB and RPB.

Later, PDIC denied Lucasans request for the cancellation of the certificate of sale stating that
the subject lots have already become part of the acquired assets of Pacific Banking Corporation
by virtue of a Certificate of Sale executed by the City Sheriff of Bacolod. That reacquisition of
the subject properties have to be through PDICs public bidding disposal policy.

Lucasan then filed a petition denominated as declaratory relief with the RTC of Bacolod City. He
sought confirmation of his rights provided in the second paragraph of Section 1, Rule 63 of the
Rules of Court in relation to Section 75 of Presidential Decree (P.D.) No. 1529 and pleaded for
the lifting and/or cancellation of the notice of embargo and the certificate of sale, offering to pay
as consideration for the cancellation.

PDIC moved to dismiss the complaint for lack of cause of action. It averred that an action to
quiet title under Section 1 of Rule 63 may only be brought when there is a cloud on, or to
prevent a cloud from being cast upon, the title to real property. It asseverated that a cloud on
the title is an outstanding instrument record, claim, encumbrance or proceeding which is actually
invalid or inoperative, but which may nevertheless impair or affect injuriously the title to property.
PDIC claimed that the notice of embargo was issued pursuant to a writ of execution while the
certificate of sale was executed as a result of a public bidding. Thus, their annotations on the
titles were valid, operative or effective. PDIC asserted that Lucasans petition is nothing but a
disguised attempt to compel PDIC to resell the properties at a reduced price. Accordingly, it
prayed for the dismissal of the petition.

Lucasan opposed the motion. He countered that the subject properties were still in his
possession, and neither PBC nor PDIC instituted an action for consolidation of ownership. Since
the certificate of title was still in his name, he contended that he could pursue all legal and
equitable remedies, including those provided for in Section 1, Rule 63 of the Rules of Court to
reacquire the properties. He also claimed that PDICs policy of disposing the subject properties
through public bidding was unjust, capricious and arbitrary, considering that the judgment debt
was lower in amount.

The RTC granted PDICs motion to dismiss, finding the claim of any cloud over the titles of
[Lucasan] to be bereft of basis in fact and in law.

Lucasan filed a motion for reconsideration, but the RTC denied. On appeal, the CA affirmed in
toto the RTC ruling. It declared that Lucasan already lost his right to redeem the properties
when he failed to exercise it within the prescribed period. The effect of such failure was to vest
in PBC absolute ownership over the subject properties. Lucasan sought a reconsideration of the
CA Decision, but the same was denied.

ISSUE:
WON Lucasan has sufficient cause of action for quieting of title. -- NO

HELD:
The requisites of quieting of title are wanting in this case. Quieting of title is a common law
remedy for the removal of any cloud of doubt or uncertainty with respect to real property. To
avail of the remedy of quieting of title, two (2) indispensable requisites must concur, namely: (1)
the plaintiff or complainant has a legal or an equitable title to or interest in the real property
subject of the action; and (2) the deed, claim, encumbrance or proceeding claimed to be casting
a cloud on his title must be shown to be in fact invalid or inoperative despite its prima facie
appearance of validity or legal efficacy. Stated differently, the plaintiff must show that he has a
legal or at least an equitable title over the real property in dispute, and that some deed or
proceeding beclouds its validity or efficacy.

Admittedly, the subject parcels of land were levied upon by virtue of a writ of execution. They
were later sold on a public auction, awarded to PBC as the highest bidder. A certificate of sale
in favor of PBC was issued. Under the Rules of Court, the judgment debtor or redemptioner had
the right to redeem the property from PBC within twelve (12) months from the registration of the
certificate of sale. With the expiration of the twelve-month period of redemption and no
redemption having been made, the judgment debtor or the redemptioner lost whatever right he
had over the land in question.

Lucasan admitted that he failed to redeem the properties within the redemption period, on
account of his then limited financial situation.It was only fifteen (15) years later that he
manifested his desire to reacquire the properties. Clearly thus, he had lost whatever right he
had over the subject lots.

The payment of loans made by Lucasan to PNB and RPB cannot, in any way, operate to restore
whatever rights he had over the subject properties. Such payment only extinguished his loan
obligations to the mortgagee banks and the liens which Lucasan claimed were subsisting at the
time of the registration of the notice of embargo and certificate of sale.

Neither can Lucasan capitalize on PBCs failure to file a petition for consolidation of ownership
after the expiration of the redemption period.

Certainly, Lucasan no longer possess any legal or equitable title to or interest over the subject
parcels of land; hence, he cannot validly maintain an action for quieting of title.

LNAC
Coronel v. IAC
G.R. No. 70191

DOCTRINE: Laches has been defined as the failure or neglect, for an unreasonable and
unexplained length of time, to do that which by exercising due diligence could or should have
been done earlier; it is negligence or omission to assert a right within a reasonable time,
warranting a presumption that the party entitled to assert it either has abandoned it or declined
to assert it. Private respondents have always been in peaceful possession of the 1/3 portion of
the subject lot, exercising ownership thereto for more than 25 years disrupted only in 1975 when
the petitioner tried to remove them by virtue of his torrens title covering the entire Lot 1950-A of
the Naic Estate. It was only at this point that private respondents knew about the supposed sale
of their 1/3 portion of Lot 1950-A of the Naic Estate.

FACTS:
The complaint was filed against the private respondents Elias Merlan, Brigido Merlan, Jose
Merlan, Teodorico Nostrates, Severo Jeciel Santiago Fernan and Fortunato Ocampo before the
then CFI.

Coronel alleged in his complaint that at the time he purchased the subject parcel of land, the
defendants (private respondents herein) were already occupying a portion thereof as "tenants at
will" and that despite demands to vacate the premises, the defendants failed and refused to
move out from the land.

In their Answer with Counterclaim and With Third-Party Complaint, the defendants denied that
Coronel was the owner of the whole parcel of land and alleged that the lots occupied by them
form part of a 1/3 undivided share of brothers Brigido Merlan and Jose Merlan which they
inherited from their deceased father Gabriel Merlan, one of the three heirs of Bernabela Lontoc,
the original owner of Lot No. 1950-A of the Naic Estate; that the Merlan brothers together with
their two brothers and a sister never sold their undivided 1/3 share of the lot to anybody; that it
was actually their other co-heirs who sold their undivided portions and that the plaintiff's claim of
ownership of the whole parcel of land, if ever it has basis, is fraudulent, void, and without effect;
that the Merlans have always been in open and peaceful possession of their undivided share of
the lot throughout the years from the first sale by their co-heirs of Lot No. 1950-A in 1950; and
that the other defendants were legitimate tenants. They prayed that the plaintiff respect their
rights over 1/3 (4,063 square meters) of Lot No. 1950-A of the Naic Estate.

In their Third-Party Complaint, the defendants charged that the third-party defendants, owners
of the remaining portion of Lot No. 1950-A, defrauded them when they sold the entire parcel.

Third-Party Defendants Marcelo Novelo, Paz Anuat Daniel Anuat and Rosario Cailao, the
defendants' co-owners of Lot No. 1950-A, denied that they had something to do with the
fraudulent acts or illegal machinations which deprived the defendants of their share in the
subject parcel of land, and that what they sold was only their 2/3 undivided shares in said parcel.
They also filed a cross-claim against their co-defendant Mariano Manalo whom they charged
might have connived with others, including the plaintiff to deprive the defendants and their co-
heirs of their share in the subject parcel of land.

Lower court ruled in favor of the defendants and on appeal, the lower court's decision was
affirmed with modification by the then IAC.

Thus, herein petition.

ISSUE:
Whether the claim of private respondents to the land in question is barred by the statute of
limitation or by estoppel by laches. -- NO

HELD:
No. In dispute is the 2/8 share of Bernabela Lontoc which is equivalent to 12,189 square meters.

When Lontoc died in 1945, she was survived by three sets of heirs: 1) Bernardino Merlan, a
grandson by her son Enrique Merlan who died in 1918; 2) Jose Merlan and Brigido Merlan,
defendants in the case below and private respondents herein, Graciano Merlan, Agapito Merlan
and Corazon Merlan, children of her son Gabriel who died in 1937; and 3) Daniel Anuat and Paz
Anuat children of her daughter Francisca Merlan.

In 1950, Bernardino Merlan, Daniel Anuat and Paz Anuat sold their 2/3 undivided portion of the
lot to spouses Ignacio Manalo and Marcela Nobelo.

In 1960, TCT No. (T-3116) RT-5010 was cancelled by TCT No. T-1444 but carried the same
afore-specified registered co-owners with an annotation carried from the former TCT.

In 1968, Lot No. 1950 of the Naic Estate was subdivided according to a Sketch Plan. The sketch
plan was approved by the Commission on Land Registration. Bernabela Lontoc's 2/8 portion of
Lot No. 1950 became Lot No. 1950-A with an area of 12,189 square meters.

Sometime in 1970, Ignacio Manalo sold his interest in Lot 1950-A to Mariano Manalo.

Thereafter, TCT No. T-1444 was cancelled and TCT No. T-41175 was issued for Lot No. 1950-
A of the Naic Estate in the name of Mariano Manalo married to Jorga Lagos. The certificate of
title issued in the name of spouses Mariano Manalo and Jorga Lagos covered the whole Lot No.
1950-A without any mention of the 1/3 share of the private respondents in the parcel of land
which was not sold to them.

Relying on the TCT of the spouses Mariano Manalo and Jorga Lagos and the Sketch Plan,
petitioner Coronel then bought Lot No. 1950-A of the Naic Estate from the former for the
consideration of P27,000.00. The deed of sale was registered on December 19, 1974 causing
the cancellation of TCT No. T-41175 and the issuance of TCT No. T-75543 in the name of
petitioner Coronel.

It is evident that the private respondents never sold their 1/3 share over Lot No. 1950-A of the
Naic Estate; that what their co-owners sold to Ignacio Manalo was their 2/3 share of the same
lot; and that Ignacio Manalo sold only the 2/3 share to third-party defendant Mariano Manalo,
the predecessor-in-interest of petitioner Coronel. There was a mistake when TCT No. 41175
was issued to Mariano Manalo covering the whole area of Lot No. 1950-A. Unfortunately,
Mariano Manalo who was included as third-party defendant as well as the subject of a cross-
claim filed by the other third-party defendants, and who could have shed light on this
controversy was at the time residing abroad and was not served with the third-party complaint.

There is no bar based on laches to assert their right over 1/3 of the disputed property. Laches
has been defined as the failure or neglect, for an unreasonable and unexplained length of time,
to do that which by exercising due diligence could or should have been done earlier; it is
negligence or omission to assert a right within a reasonable time, warranting a presumption that
the party entitled to assert it either has abandoned it or declined to assert it. Private respondents
have always been in peaceful possession of the 1/3 portion of the subject lot, exercising
ownership thereto for more than 25 years disrupted only in 1975 when the petitioner tried to
remove them by virtue of his torrens title covering the entire Lot 1950-A of the Naic Estate. It
was only at this point that private respondents knew about the supposed sale of their 1/3 portion
of Lot 1950-A of the Naic Estate and they immediately resisted.

Neither the private respondents nor their co-owners of the subject parcel of land sold the
former's share of the lot. Furthermore, even Ignacio Manalo to whom the third-party defendants
sold their share resold only the 2/3 shares to Mariano Manalo, the successor-in-interest of the
petitioner. Whether or not there was fraud or just a mistake or oversight of an employee of the
Register of Deeds of Cavite is not clear from the records. The point is that the 1/3 undivided
portion of the private respondents over Lot No. 1950-A was mistakenly included in the TCT of
Mariano Manalo.

TKDC
Sapto v. Fabiana
103 Phil 683

DOCTRINE: The statute of limitations is not a defense to an action to remove a cloud from title
when complainant is in possession of the property. However, the statute of limitations is a
defense when the property is in anothers possession; the complainant must invoke his remedy
within the statutory period.

FACTS:
Upon his death, Moro was left a parcel of land registered under his name to his children,
Samuel, Constancio, and Ramon.

After Ramons death, Samuel and Constancio executed a deed of sale of a portion of the
property in favor of Fabiana; the sale was approved by the Provincial Governor but was
unregistered. Since 1931 when possession of conveyed land was transferred, Fabiana has
possessed the property.

After Constancios death, Samuel married Dora and had two children. After Samuels death, his
widow and children filed an action with the Court of First Instance to recover the parcel of land
sold to Fabiana. Trial court held that, although the sale was unregistered, it was valid and
binding upon the parties and also the vendors heirs; trial court ordered the Samuels heirs to
execute the necessary deed of conveyance in Fabianas favor.

Samuels heirs appealed to the Supreme Court claiming that Fabianas action had long
prescribed; twenty years had lapsed since the original sale.

ISSUE:
Whether or not Fabianas action had already prescribed since twenty years had passed since
the original sale. -- NO

HELD:
The statute of limitations is not a defense to an action to remove a cloud from title when
complainant is in possession of the property. However, the statute of limitations is a defense
when the property is in anothers possession; the complainant must invoke his remedy within
the statutory period. In this case, prescription does not run again Fabiana because she already
possesses the property. Thus, her action may prosper.

AMD
Titong v. CA
287 SCRA 102

DOCTRINE: The ground or reason for filing a complaint for quieting of title must therefore be
"an instrument, record, claim, encumbrance or proceeding" which constitutes or casts a cloud,
doubt, question or shadow upon the owner's title to or interest in real property.

FACTS:
Mario Titong filed an action for quieting of title against Victorico and Angeles Laurio. Petitioner
alleged that he was the owner of an unregistered parcel of land. He claimed that private
respondents (Laurios), with heir hired laborers, forcibly entered a portion of the land containing
an area of approximately 2 hectares; and began plowing the same under pretext of ownership.
The Laurios denied this allegation, and averred that the disputed property formed part of the
5.5-hectare agricultural land which they had purchased from their predecessor-in-interest, Pablo
Espinosa, an adjoining owner of Titongs land. The RTC ruled in favor of private respondents,
declaring Laurio as the true and absolute owner of the property. Petitioner appealed to the CA,
the RTC decision was affirmed and the MR was denied. Hence, this petition.

ISSUE:
W/N the action for quieting of title should have prospered. -- NO

HELD:
The SC held that the instant petition must be denied for the reason that the lower court should
have outrightly dismissed the complaint for quieting of title. The remedy of quieting of title may
be availed of under the circumstances enumerated in the Civil Code:

Art. 476. Whenever there is a cloud on title to real property or any interest therein, by reason of
any instrument, record, claim, encumbrance or proceeding which is apparently valid or effective
but is in truth and in fact invalid, ineffective, voidable, or unenforceable, and may be prejudicial
to said title, an action may be brought to remove such cloud or to quiet the title.

An action may also be brought to prevent a cloud from being cast upon title to real property or
any interest therein.

Under this provision, a claimant must show that there is an instrument, record, claim,
encumbrance or proceeding which constitutes or casts a cloud, doubt, question or shadow upon
the owner's title to or interest in real property. The ground or reason for filing a complaint for
quieting of title must therefore be "an instrument, record, claim, encumbrance or proceeding."
Under the maxim expressio unius est exclusio alterius, these grounds are exclusive so that
other reasons outside of the purview of these reasons may not be considered valid for the same
action.

Had the lower court thoroughly considered the complaint filed, it would have had no other
course of action under the law but to dismiss it. The complaint failed to allege that an
"instrument, record, claim, encumbrance or proceeding" beclouded the plaintiff's title over the
property involved. Petitioner merely alleged that the respondents, together with their hired
laborers and without legal justification, forcibly entered the southern portion of the land of the
plaintiff and plowed the same. He then proceeded to claim damages and attorney's fees.

Hence, through his allegations, what petitioner imagined as clouds cast on his title to the
property were private respondents' alleged acts of physical intrusion into his purported property.
Clearly, the acts alleged may be considered grounds for an action for forcible entry but definitely
not one for quieting of title. In addition, when the issues were joined by the filing of the answer to
the complaint, it would have become apparent to the court that the case was a boundary dispute.

MPF
Pingol v. CA
226 SCRA 118

DOCTRINE: A vendee (buyer) in an oral contract to convey land who had made part payment
thereof, entered upon the land and had made valuable improvements thereon is entitled to bring
suit to clear his title against the vendor who had refused to transfer the title to him. It is not
necessary that the vendee should have an absolute title, an equitable title being sufficient to
clothe him with personality to bring an action to quiet title.

FACTS:
1969 - Pingol, the owner of a lot in Caloocan City, executed a DEED OF
ABSOLUTE SALE OF ONE-HALF OF AN UNDIVIDED PORTION OF [his] PARCEL OF
LAND (274.5 sqm) in favor of Donasco (private respondent), payable in 6 years. And
Donasco agreed to pay in monthly basis (for 72 months).
Both agreed that in case of default in the payment due the same should earn a legal
interest.
Donasco took possession immediately and constructed a house thereon.
1984 - Donasco died and was only able to pay P8,369 plus P2,000
downpayment, leaving a balance of P10,161.
The heirs of Donasco remained in possession of such lot and offered to settle the
balance with Pingol.
However, Pingol refused to accept the offer and demanded a larger amount.
Thus, the heirs of Donasco filed an action for specific performance (with
Prayer for Writ of Prelim. Injunction, because Pingol were encroaching upon
Donascos lot).
Pingol averred that the sale and transfer of title was conditional upon the full
payment of Donasco (contract to sell, not contract of sale).
With Donascos breach of the contract in 1976 and death in 1984, the sale
was deemed cancelled, and the continuous occupancy of the heirs was only
being tolerated by Pingol.

RTC: ordering heirs of Donasco to pay monthly rental to Pingol
CA: ordering Pingol to accept the sum of P10,161 plus legal interest from heirs of Donasco as
payment to the land in question

ISSUES:
1. Whether or not Pingol can refuse to transfer title to Donasco. -- NO
2. Whether or not Donasco has the right to quiet title.

HELD:
1. The contract between Pingol and Donasco is a contract of sale and not a contract to sell.
The acts of the parties, contemporaneous and subsequent to the contract, clearly show that
the parties intended an absolute deed of sale; the ownership of the lot was transferred to
the Donasco upon its actual (upon Donascos possession and construction of the house)
and constructive delivery (upon execution of the contract). The delivery of the lot divested
Pingol of his ownership and he cannot recover the title unless the contract is resolved or
rescinded under Art. 1592 of NCC. It states that the vendee may pay even after the
expiration of the period stipulated as long as no demand for rescission has been made
upon him either judicially or by notarial act. Pingol neither did so. Hence, Donasco has
equitable title over the property.

2. Although the complaint filed by the Donascos was an action for specific performance, it was
actually an action to quiet title. A cloud has been cast on the title, since despite the fact that
the title had been transferred to them by the execution of the deed of sale and the delivery
of the object of the contract, Pingol adamantly refused to accept the payment by Donascos
and insisted that they no longer had the obligation to transfer the title.

Donasco, who had made partial payments and improvements upon the property, is entitled
to bring suit to clear his title against Pingol who refused to transfer title to him. It is not
necessary that Donasco should have an absolute title, an equitable title being sufficient to
clothe him with personality to bring an action to quiet title.

Prescription cannot also be invoked against the Donascos because an action to quiet title
to property in ONEs POSSESSION is imprescriptible.

SC: appealed decision affirmed

AMDG
Gallar v. Hussain
20 SCRA 186

DOCTRINE: An action to quiet title, brought by a person who is in possession of the property, is
imprescriptible. However, if the plaintiff is not in possession, the action would prescribe within
the proper period.

FACTS:
Teodoro Husain is the owner of the land to which he sold to Serapio Chichirita for P30 with the
right to repurchase of such land within six years. Teodoro did not avail his right to repurchase
however, his sister, Graciana Husain, bought the said land shortly after the sale between
Teodoro and Chichirita. It is alleged that that the sale was described to be a resale of the land.
Afterwhich, Graciana sold it to appellee, Elias Gallar in exchange for one cow. Owners
duplicate of the TCT was given to Gallar and since then has been the possessor of the subject
land.

Gallar was unsuccessful when he went to the Cadastral Court to ask that the TCT be issued
under his name. It is noted however that the Cadastral Court granted his request as regards
amending the certificate and changing the registered owner of the land to Husain. Gallar then
filed the suit in the CFI (now RTC) to compel the heirs of Teodoro Husain (Heremenegilda and
Bonifacio) to execute the deed of conveyance in his favor to which the latter refused to execute.

The heirs were denied and alleged that when Graciana bought the land from Serapio, she was
exercising the right to repurchase granted to Teodoro. Also, they invoked that prescription has
set in which bars the appellees action.

ISSUES:
1. WON the land was acquired by Graciana Husain on behalf of his brother Teodoro? -- NO
2. WON this action this is action is for specific performance? -- NO
3. WON appellees action is imprescriptible? -- YES

HELD:
The right to repurchase may only be availed by the person to whom such right was given to or
to the person to whom such right was transferred to. There was no evidence whatsoever to
prove that Graciana acquired the land on behalf of his brother. Graciana bought the land subject
only to his brothers right of remdeption. Therefore, being the owner, she has every right to sell
the land which she did to Elias.

When Teodoro failed to exercise his right of redemption, its ownership became consolidated in
the appellee. Though the sale was in a private document, such was still considered valid. The
delivery of the land and title to Gallar indicates that a sale was consummated.

This action is not for a specific performance but a mere quieting of title. The object of the action
was to remove the cloud cast on Gallars ownership because of the heirs refusal to recognize
the sale. Lastly being the Gallar is in possession when this action was constituted, the action is
imprescriptible. The heirs argument as regards prescription would have been granted by the
court in their favor if the land was not possessed by Gallar.

GCG
Vda de Aviles v. CA
264 SCRA 473

DOCTRINE: Quieting of Title Not Proper Remedy For Settling Boundary Dispute. Quieting of
title is a common law remedy for the removal of any cloud upon or doubt or uncertainty with
respect to title to real property.

FACTS:
Petitioners aver that they are the actual possessors of a parcel of land situated in Malawa,
Lingayen, Pangasinan, more particularly described as fishpond, cogonal, unirrigated rice and
residential land, bounded on the N by Camilo Aviles; on the E by Malawa River, on the S by
Anastacio Aviles and on the W by Juana and Apolonio Joaquin, with an area of 18,900 square
meters and declared under Tax Declaration No. 31446. This property is the share of their father,
Eduardo Aviles and brother of the defendant, in the estate of their deceased parents.
Eduardo Aviles was in actual possession of the afore-described property since 1957. In fact, the
latter mortgaged the same with the Rural Bank and Philippine National Bank branch in Lingayen.
When the property was inspected by a bank representative, Eduardo Aviles, in the presence of
the boundary owners, namely, defendant Camilo Aviles, Anastacio Aviles and Juana and
Apolonio Joaquin pointed to the inspector the existing earthen dikes as the boundary limits of
the property and nobody objected. When the real estate mortgage was foreclosed, the property
was sold at public auction but this was redeemed by plaintiffs mother and the land was
subsequently transferred and declared in her name.
Defendant Camilo Aviles asserted a color of title over the northern portion of the property with
an area of approximately 1,200 square meters by constructing a bamboo fence (thereon) and
moving the earthen dikes, thereby molesting and disturbing the peaceful possession of the
plaintiffs over said portion.
Defendant Camilo Aviles admitted the agreement of partition executed by him and his brothers,
Anastacio and Eduardo. The respective area(s) alloted to them was agreed and measured
before the execution of the agreement but he was not present when the measurement was
made. Defendant agreed to have a smaller area because his brother Eduardo asked him that he
wanted a bigger share because he has several children to support. The portion in litigation
however is part of the share given to him in the agreement of partition.
At present, he is only occupying a smaller than his actual share. Tax Declarations Nos. 23575,
481 and 379 covering his property from 1958 show that the area of his property is 14,470
square meters. The riceland portion of his land is 13,290 square meters, the fishpond portion is
500 square meters and the residential portion is 680 square meters, or a total of 14,470 square
meters. That the topography of his land is not the same, hence, the height of his pilapils are
likewise not the same.
The trial court disposed of the case thus ordering the parties to employ the services of a Land
Surveyor of the Bureau of Lands, Region I, San Fernando, La Union, to relocate and determine
the extent and the boundary limit of the land of the defendant on its southern side in order that
the fourteen thousand four hundred seventy (14,470) square meters which is the actual area
given to the defendant be determined. It ordered the complaint dismissed for lack of basis and
merits
Dissatisfied with the trial courts decision, petitioners appealed to the respondent appellate
Court. In its now-assailed Decision, the Court of Appeals affirmed in part the decision of the trial
court, reasoning that a special civil action for quieting of title is not the proper remedy for settling
a boundary dispute, and that petitioners should have instituted an ejectment suit instead. It
affirmed the decision of the trial court in dismissing the complaint.
ISSUE:
Whether or not the Hon. Court of Appeals is correct when it opined that the complaint for
quieting of title instituted by the petitioners against private respondent before the court a quo is
not the proper remedy but rather, it should be a case for ejectment.

HELD:
The Supreme Court ruled that Quieting of Title Not Proper Remedy For Settling Boundary
Dispute. Quieting of title is a common law remedy for the removal of any cloud upon or doubt or
uncertainty with respect to title to real property.
The Civil Code authorizes the said remedy in the following language:

Art. 476. Whenever there is a cloud on title to real property or any interest therein, by reason of
any instrument, record, claim, encumbrance or proceeding which is apparently valid or effective
but is, in truth and in fact, invalid, ineffective, voidable, or unenforceable, and may be prejudicial
to said title, an action may be brought to remove such cloud or to quiet the title.

An action may also be brought to prevent a cloud from being cast upon a title to real property of
any interest therein."

In fine, to avail the remedy of quieting of title, a plaintiff must show that there is an instrument,
record, claim, encumbrance or proceeding which constitutes or casts a cloud, doubt, question or
shadow upon the owners title to or interest in real property. Thus, petitioners have wholly
misapprehended the import of the foregoing rule by claiming that respondent Court erred in
holding that there was no xxx evidence of any muniment of title, proceeding, written contract,
xxx, and that there were, as a matter of fact, two such contracts, (i) the Agreement of Partition
executed by private respondent and his brothers (including the petitioners father and
predecessor-in-interest), in which their respective shares in the inherited property were agreed
upon, and (ii) the Deed of Sale evidencing the redemption by petitioner Anastacia Vda. de
Aviles of the subject property in a foreclosure sale. However, these documents in no way
constitute a cloud or cast a doubt upon the title of petitioners. Rather, the uncertainty arises
from the parties failure to situate and fix the boundary between their respective properties.

As correctly held by the respondent Court, both plaintiffs and defendant admitted the existence
of the agreement of partition and in accordance therewith, a fixed area was alloted (sic) to them
and that the only controversy is whether these lands were properly measured. There is no
adverse claim by the defendant which is apparently valid, but is, in truth and in fact, invalid,
ineffective, voidable, or unenforceable and which constitutes a cloud thereon.

Corollarily, and equally as clear, the construction of the bamboo fence enclosing the disputed
property and the moving of earthen dikes are not the clouds or doubts which can be removed
in an action for quieting of title.

An action to quiet title or to remove cloud may not be brought for the purpose of settling a
boundary dispute. The precedent on this matter cited by the respondent Court in its Decision is
herewith reproduced in full.

VCL IV
Oblea v. CA
244 SCRA 101

DOCTRINE: The pendency of an action for quieting of title before the RTC does not divest the
MTC of its jurisdiction to proceed with the ejectment case over the same property. The
subsequent acquisition of ownership by petitioners is not a supervening event that will bar the
execution of the judgment in said unlawful detainer case.

FACTS:
The lot in issue was originally registered in the names of Manuel Melencio, Pura Melencio,
Wilfredo Wico and Mariabelle Wico. But was subsequently re-registered in the name of Ramon
Melencio (son of deceased Manuel Melencio), Pura Melencio and the Wicos via a deed of sale.

On 6 June 1958 subject lot was bought by private respondent Juan S. Esteban from Mauricio
Ramos who claimed to have acquired the property from Ursula Melencio, the alleged
administratrix of the estate of Manuel and Pura Melencio.

Meanwhile, petitioner Romeo V. Oblea leased a building located on the subject lot from a
certain Marius Esteban, an alleged son of private respondent Juan S. Esteban. Oblea
eventually bought from Marius the lot on which the building stood. As a consequence, on 4 July
1991 Juan Esteban filed an ejectment suit against petitioner Oblea.

MTC decided for Juan Esteban and ordered Oblea to vacate and pay arrears. RTC affirmed
MTCs decision.

On 3 June 1993, the registered owners (Ramon Melencio, Pura Melencio and Wilfredo Wico
and Mariabelle Wico) sold the disputed lot to petitioner Oblea. Afterwards, Oblea together with
the registered owners filed before the RTC an action for quieting of title against Juan Esteban.
They contended that the deeds of sale executed by Mauricio Ramos in favor of Juan Esteban
and by Ursula Melencio in favor of Mauricio Ramos were a nullity.

Meanwhile, the ejectment case was appealed thrice in the CA, but all were denied.

In the appeal to the SC, Oblea asserts that the subsequent sale to him by the registered owners
is a supervening event that gave him a better right of possession and ownership. Hence the
judgment of eviction can no longer be enforced.

ISSUE:
Whether or not a subsequent action to quiet title in the RTC divests the MTC of its jurisdiction
over an ejectment case -- NO

HELD:
The sole issue in an action for unlawful detainer is physical or material possession, i.e.,
possession de facto and not possession de jure. The pendency of an action for quieting of title
before the RTC does not divest the MTC of its jurisdiction to proceed with the ejectment case
over the same property. The subsequent acquisition of ownership by petitioners is not a
supervening event that will bar the execution of the judgment in said unlawful detainer case, the
fact remaining that when judgment was rendered by the MTC in the ejectment case, petitioner
Oblea was a mere possessor of the subject lot.

Similarly, the fact that petitioners instituted a separate action for quieting of title is not a valid
reason for defeating the execution of the summary remedy of ejectment. On the contrary., it
bolsters the conclusion that the eviction case did not deal with the issue of ownership which was
precisely the subject matter of the action for quieting of title before the RTC. With the finality of
the decision in the ejectment case, execution in favor of the prevailing party has become a
matter of right; its implementation mandatory. It cannot be avoided.

FXRL
Gapacan v. Omipet
387 SCRA 383

DOCTRINE: An action to Quiet Title is a valid remedy to ascertain property rights.

FACTS:
Gapacan is a native Igorot possessor of a parcel of land in Bauko, Mr. Province. The
property is divided into three and declared by him for taxation purposes.
He has two children. Maria and Antonio.
Antonio left to work in mining while Maria remained and eventually took over
management and cultivation of the property.
Antonio returned to their home and thereafter executed an Affidavit of Transfer of Real
Property which now makes him the legal owner of the property in question as it transfers
ownership from his sister to him.
Since then, the family of Antonio (Gapacan) had been occupying and cultivating the
property.
The family of Maria (Omipet)

ISSUE:
W/N property rights may be determined by an action to Quiet Title. -- YES

HELD:

Art. 476 of the Civil Code provides that an action to quiet title may be brought when there exists
a cloud on the title to a real property or any interest therein.

The property owner whose property rights were being disturbed may ask a competent court for
a proper determination of the respective rights of the party-claimants, places things in their
proper place by:

1. Requiring the party with no right over the property to refrain from acts injurious to
the peaceful enjoyment of the property by rightful owner and
2. Mutually benefitting both parties with the view of dissipating any cloud of doubt
over the property.

The appellate court in resolving the present controversy is well within its authority to adjudicate
on the respective rights of the parties, that is, to pass upon the ownership of the property; hence
to declare the same as common property.

The court also noted that Omipet did not present sufficient evidence to overcome Gapacans
better right to possession. The Supreme Court ruled that CA was correct in its determination
that the land in dispute is common property and must be partitioned.

RSDM
Robles v. CA
328 SCRA 97

DOCTRINE: An action may also be brought to prevent a cloud from being cast upon title to real
property or any interest therein.

FACTS:
Leon Robles primitively owned the land situated in Kay Taga, Lagundi, Morong, Riza. He also
declared the same in his name for taxation purposes as early as 1916 and paid the
corresponding taxes thereon. When Leon Robles died, his son Silvino Robles inherited the land,
who took possession of the land, declared it in his name for taxation purposes and paid the
taxes thereon.

Upon the death of Silvino Robles, his widow Maria de la Cruz and his children inherited the
property. They took adverse possession of said property and paid taxes thereon. The task of
cultivating the land was assigned to plaintiff Lucio Robles who planted trees and other crops.
The plaintiffs entrusted the payment of the land taxes to their co-heir and half-brother, Hilario
Robles.

In 1962, for unknown reasons, the tax declaration of the parcel of land in the name of Silvino
Robles was canceled and transferred to one Exequiel Ballena, father of Andrea Robles who is
the wife of defendant Hilario Robles. Thereafter, Exequiel Ballena secured a loan from the
Antipolo Rural Bank, using the tax declaration as security. Somehow, the tax declaration was
transferred to the name of Antipolo Rural Bank and later on, was transferred to the name of
defendant Hilario Robles and his wife.

In 1996, Andrea Robles secured a loan from the Cardona Rural Bank, Inc., using the tax
declaration as security. Andrea Robles testified without contradiction that somebody else, not
her husband Hilario Robles, signed the loan papers because Hilario Robles was working in
Marinduque at that time as a carpenter.

For failure to pay the mortgage debt, foreclosure proceedings were had and defendant Rural
Bank emerged as the highest bidder during the auction sale in October 1968.
"The spouses Hilario Robles failed to redeem the property and so the tax declaration was
transferred in the name of defendant Rural Bank. Rural Bank sold the same to the Spouses
Vergel Santos and Ruth Santos.
Plaintiff discovered the mortgage and attempted to redeem the property, but was unsuccessful.
Defendant spouses Santos took possession of the property in question and was able to secure
Free Patent in their names.

ISSUE:
Whether or not the petitioners have the appropriate title that will entitle them to avail themselves
of the remedy of quieting of title. -- YES

HELD:
Undisputed is the fact that the land had previously been occupied by Leon and later by Silvino
Robles, petitioners' predecessor-in-interest, as evidenced by the different tax declarations
issued in their names. Also undisputed is the fact that the petitioners continued occupying and
possessing the land from the death of Silvino in 1942 until they were allegedly ousted therefrom
in 1988.

The failure to show the indubitable title of Exequiel to the property in question is vital to the
resolution of the present Petition. It was from him that Hilario had allegedly derived his title
thereto as owner, an allegation which thereby enabled him to mortgage it to the Rural Bank of
Cardona. The occupation and the possession thereof by the petitioners and their predecessors-
in-interest until 1962 was not disputed, and Exequiel's acquisition of the said property by
prescription was not alleged. Thus, the deed of conveyance purportedly evidencing the transfer
of ownership and possession from the heirs of Silvino to Exequiel should have been presented
as the best proof of that transfer. No such document was presented, however.
Therefore, there is merit to the contention of the petitioners that Hilario mortgaged the disputed
property to the Rural Bank of Cardona in his capacity as a mere co-owner thereof. Clearly, the
said transaction did not divest them of title to the property at the time of the institution of the
Complaint for quieting of title.

Contrary to the disquisition of the Court of Appeals, Hilario effected no clear and evident
repudiation of the co-ownership. It is a fundamental principle that a co-owner cannot acquire by
prescription the share of the other co-owners, absent any clear repudiation of the co-ownership.
In order that the title may prescribe in favor of a co-owner, the following requisites must concur:
(1) the co-owner has performed unequivocal acts of repudiation amounting to an ouster of the
other co-owners; (2) such positive acts of repudiation have been made known to the other co-
owner; and (3) the evidence thereof is clear and convincing.

In the present case, Hilario did not have possession of the subject property; neither did he
exclude the petitioners from the use and the enjoyment thereof, as they had indisputably shared
in its fruits. Likewise, his act of entering into a mortgage contract with the bank cannot be
construed to be a repudiation of the co-ownership. As absolute owner of his undivided interest
in the land, he had the right to alienate his share, as he in fact did. Neither should his payment
of land taxes in his name, as agreed upon by the co-owners, be construed as a repudiation of
the co-ownership. The assertion that the declaration of ownership was tantamount to
repudiation was belied by the continued occupation and possession of the disputed property by
the petitioners as owners.

MRAM
Metropolitan Bank v. Alejo
364 SCRA 812

DOCTRINE: In a suit to nullify an existing TCT in which a real estate mortgage is annotated, the
mortgagee is an indispensable party. In such suit, a decision canceling the TCT and the
mortgage annotation is subject to a petition for annulment of judgment, because the non-joinder
of the mortgagee deprived the court of jurisdiction to pass upon the controversy.

FACTS:
Spouses Acampado obtained a loan from petitioner and as security for their payment of the
credit accommodations, they executed a real estate mortgage in Valenzuela City. The land was
also located in Valenzuela and the TCT covering it is registered in the Registry of deeds in the
same city.

On June 1996, a Complaint for Declaration of Nullity of TCT was filed by Respondent Sy Tan
Se against Spouses Acampado.

Despite being the registered mortgagee of the real property covered by the title sought to be
annulled, petitioner was not made a party to the case nor was it notified of its existence.

The spouses defaulted in the payment of their loan. Thereafter,extrajudicial foreclosure
proceedings over the mortgaged property were initiated and then the sheriff of Valenzuela
conducted an auction sale of the property, during which petitioner submitted the highest and
winning bid. A Certificate of Sale was issued in its favor and the sale was entered in the
Registry of Deeds of Valenzuela.

When the redemption period lapsed exactly a year after, petitioner executed an Affidavit of
Consolidation of Ownership to enable the Registry of Deeds of Valenzuela to issue a new TCT
in its name however petitioner was informed that an RTC decision declared the TCT as null and
void for having proceeded from an illegitimate source.

ISSUE:
Whether or not the judgment of the RTC that declared the TCT as null and void should be
annulled?

HELD:
The Court held in the affirmative.

The Rules of Court provide that no final determination can be had of an action wherein the
indispensable party is not joined in the complaint either as the defendant or the complainant.

It is undisputed that the property covered by the TCT was mortgaged to petitioner, and that the
mortgage was annotated on it before the institution of Civil Cas. It is also undisputed that all
subsequent proceedings pertaining to the foreclosure of the mortgage were entered in the
Registry of Deeds. The nullification and cancellation of TCT carried with it the nullification and
cancellation of the mortgage annotation.

Although a mortgage affects the land itself and not merely the TCT covering it, the cancellation
of the TCT and the mortgage annotation exposed petitioner to real prejudice. Evidently, the
nullification of the TCT adversely affected its property rights, considering that a real mortgage is
a real right and a real property by itself.

Because petitioner falls under the definition of an indispensable party, it should have been
impleaded as defendant in the civil case questioning the title.

FMM
Sps. Benito v. Saquitan Ruiz
394 SCRA 250

DOCTRINE: A petition for the quieting of title, although essentially an action for reconveyance,
should not be dismissed on the ground of prescription, if it is alleged that the plaintiff is in
possession of the property.

FACTS:
On 1 April 1999, Agapita Saquitan-Ruiz (Saquitan-Ruiz) filed against Spouses Horacio and
Felisa Benito (Spouses Benito) a civil suit for specific performance with declaration of nullity
of titles and damages. The Complaint alleged that the couple had sold in her favor a property
located in Pasig City and despite repeated demands, they failed to deliver or cause the
issuance of a new certificate of title in her name.

However, it turned out that instead of issuing or delivering to Saquitan-Ruiz her certificate of title
over the said property, Spouses Benito re-subdivided the property into five lots.

On 28 June 1999, the Regional Trial Court (the RTC) dismissed Saquitan-Ruizs Complaint on
the grounds of prescription and/or laches. It held that from the moment the contract was
perfected, the parties could reciprocally demand performance of their obligations. There was a
breach of obligation when, despite repeated demands, Spouses Benito failed to deliver to
respondent the corresponding certificate of title to the lot. She, however, failed to file any action
to compel performance until 16 April 1999, or 20 years from the time of the execution of the
Deed of Absolute Sale on 17 April 1979. Moreover, the assailed Certificates of Title had been
issued 25 March 1996, or more than one year before the Complaint was filed. An action to
invalidate title certificates on the ground of fraud prescribes upon the expiration of one year from
the entry of the decree of registration.

Upon appeal with the Court of Appeals (the CA), the latter reversed the RTCs decision. It
held that the Saquitan-Ruizs second cause of action was for reconveyance, not for the
invalidation of certificates of title. As long as the property was still in the name of the person
who had caused the wrongful registration, and as long as it had not yet passed to an innocent
purchaser for value, an action for reconveyance was still available. Such cause of action
prescribes in ten (10) years, counted from the date of the issuance of the assailed certificate of
title. Since the Complaint alleged that the questioned titles had been issued on 25 March 1996,
the cause of action for reconveyance has not prescribed.

Thus this petition filed by Spouses Benito. They contended that the action for reconveyance
has been rendered moot and academic, because the disputed lot was already sold to Basilia
dela Cruz at a public auction. They maintain that although the action for reconveyance may not
have expired, the exercise of the right is no longer feasible, because the property was already
transferred in good faith and for value to a third party.

ISSUE:
Whether the cause of action for reconveyance has already prescribed -- NO

HELD:
The SC noted that the Spouses Benito is in possession of the disputed property. Thus the SC
held that if a person claiming to be the owner of a wrongfully registered parcel of land is in
actual possession, the right to seek reconveyance does not prescribe.

A petition for the quieting of title, although essentially an action for reconveyance, should not be
dismissed on the ground of prescription, if it is alleged that the plaintiff is in possession of the
property.

Ruinous Buildings

CRF
Nakpil and Sons v. CA
144 SCRA 596

DOCTRINE: To be exempt from liability due to an act of God, the engineer/architect/contractor
must not have been negligent in the construction of the building. (batasnatin)

FACTS:
The plaintiff, Philippine Bar Association decided to construct an office building on its 840 square
meters lot located at the comer of Aduana and Arzobispo Streets, Intramuros, Manila. The
construction was undertaken by the United Construction, Inc. on an "administration" basis, on
the suggestion of Juan J. Carlos, the president and general manager of said corporation. The
plans and specifications for the building were prepared by the other third-party defendants Juan
F. Nakpil & Sons. The building was completed in June, 1966. In the early morning of August 2,
1968 an unusually strong earthquake hit Manila and its environs and the building in question
sustained major damage. The tenants vacated the building in view of its precarious condition.
As a temporary remedial measure, the building was shored up by United Construction, Inc. at
the cost of P13,661.28. On November 29, 1968, the plaintiff commenced this action for the
recovery of damages arising from the partial collapse of the building against United Construction,
Inc. and its President and General Manager Juan J. Carlos as defendants. Plaintiff alleges that
the collapse of the building was accused by defects in the construction, the failure of the
contractors to follow plans and specifications and violations by the defendants of the terms of
the contract.

Defendants in turn filed a third-party complaint against the architects who prepared the plans
and specifications, alleging in essence that the collapse of the building was due to the defects in
the said plans and specifications.

Finally, on April 30, 1979 the building was authorized to be demolished at the expense of the
plaintiff, but not another earthquake of high intensity on April 7, 1970 followed by other strong
earthquakes on April 9, and 12, 1970, caused further damage to the property. The actual
demolition was undertaken by the buyer of the damaged building.

ISSUE:
Whether or not an act of God-an unusually strong earthquake-which caused the failure of the
building, exempts from liability, parties who are otherwise liable because of their negligence
(NAKPILS and UNITED) -- NO

HELD:
There should be no question that the NAKPILS and UNITED are liable for the damage resulting
from the partial and eventual collapse of the PBA building as a result of the earthquakes. The
applicable law governing the rights and liabilities of the parties herein is Article 1723 of the New
Civil Code, which provides:

Art. 1723. The engineer or architect who drew up the plans and specifications for a building is
liable for damages if within fifteen years from the completion of the structure the same should
collapse by reason of a defect in those plans and specifications, or due to the defects in the
ground. The contractor is likewise responsible for the damage if the edifice fags within the same
period on account of defects in the construction or the use of materials of inferior quality
furnished by him, or due to any violation of the terms of the contract. If the engineer or architect
supervises the construction, he shall be solidarily liable with the contractor.

Thus it has been held that when the negligence of a person concurs with an act of God in
producing a loss, such person is not exempt from liability by showing that the immediate cause
of the damage was the act of God. To be exempt from liability for loss because of an act of God,
he must be free from any previous negligence or misconduct by which that loss or damage may
have been occasioned.

Co-ownership

RGGM
Robles v. C.A.
328 SCRA 97

DOCTRINE: It is a fundamental principle that a co-owner cannot acquire by prescription the
share of the other co-owners, absent any clear repudiation of the co-ownership. in order that the
title may prescribe in favor of a co-owner, the following requisites must concur: 1.) the co-owner
has performed unequivocal acts of repudiation amounting to an ouster of the other co-owners;
2.) such positive acts of repudiation have been made known to the other co-owners; and 3.) the
evidence thereof is clear and convincing.

FACTS:
Leon Robles owned a land in Morong Rizal. When Leon died, his son Silvino Robles inherited
the land. Both of them declared the property under their name for taxation purposes. Upon the
death of Silvino, his widow Maria dela Cruz and his children inherited the property. They took
adverse possession of it and paid the taxes thereon. The task of cultivating the land was
assigned to one of Silvinos son, Lucio Roles while the payment of the taxes was entrusted to
their half-brother, Hilario Robles.

In 1962, for unknown reasons, the tax declaration of the parcel of land in the name of Silvino
Robles was canceled and transferred to one Exequiel Ballena, father of Andrea Robles who is
the wife of defendant Hilario Robles. Thereafter, Exequiel Ballena secured a loan from the
Antipolo Rural Bank, using the tax declaration as security. Somehow, the tax declaration was
transferred to the name of Antipolo Rural Bank and later on, was transferred to the name of
defendant Hilario Robles and his wife. In 1996, Andrea Robles secured a loan from the Cadona
Rural Bank, Inc., using the tax declaration as security. For failure to pay the mortgage debt,
foreclosure proceedings were had and defendant Rural Bank emerged as the highest bidder
during the auction sale in October 1968.

The spouses Hilario Robles failed to redeem the property and so the tax declaration was
transferred in the name of defendant Rural Bank. On September 25, 1987, defendant Rural
Bank sold the same to the Spouses Vergel Santos and Ruth Santos who took possession of the
property and was able to secure Free Patent No. IV-1-010021 in their names.

ISSUES:
1. Whether petitioners have the appropriate title essential to an action for quieting of title
(relevant issue) and whether title claimed by respondents is valid
2. Whether real estate mortgage between Hilario and RBC is valid
3. Whether issuance of free patent is valid

HELD:
1. Petitioners have valid title by virtue of their continued and open
occupation and possession as owners of the subject property.

In this case, the cloud on petitioners title emanate from the apparent validity of the free
patent issued and the tax declarations and other evidence in favour of respondents
ultimately leading to the transfer of the property to spouses Santos. WRT title of the spouses
Santos, such is deemed invalid/inoperative insofar as it is rooted in the title and
appropriation of Hilario. Hilario could not have prejudiced the rights of his co-heirs as co-
owners of the real estate. He must have first repudiated the ownership clearly and evidently.
CA failed to consider the irregularities in the transactions involving the property. No
instrument/deed of conveyance was presented to show any transaction between petitioners
and Ballane or even Hilario.

2. Mortgage was only valid insofar as Hilarios undivided interest is
concerned there being co-ownership between the heirs. Court also delved into gross
negligence which amounted to bad faith on part of bank by not exercising due diligence in
verifying the ownership of the land considering such was unregistered.

Free patent was also not valid, the land in question having been converted ipso jure to
private land by virtue of the adverse possession in the concept of owners since.

3. 1916 by the petitioners. Issuance of patents covering private lands is out
of the jurisdiction of the Director of Lands or Bureau of Lands.

MCSS
Acevedo v. Abesamis
217 SCRA 186

FACTS:
Herodotus and 7 others were left an estate consisting of real properties in Quezon City and
Caloocan City. Herodotus became the administrator pending partition. For the meantime, the
property is owned in common by the heirs.

The case pended for 16 years with the court. Miguel et al (respondents) then filed a Motion for
Approval of Sale for them to sell their shares of the estate. The court approved the motion.
Respondents were able to find a buyer in the person of Yu Hwa Ping who agreed to buy the
properties for P12 Million. He paid P6 million as earnest money.

Herodotus assailed the approval of the sale claiming that the price is quite low. The court
ordered Miguel et al to find a higher bidder within a specified time frame which was later
extended to 7 months but still no other buyer could provide better terms.

Finally, it was agreed by the parties that respondents sell their share to the price already agreed
upon with Ping and that the Herodotus can negotiate his price with Ping. But Herodotus still filed
a Supplemental Opposition against the approval of the conditional sale.

The court affirmed the approval of the sale and ordered Herodotus to sell his share at the same
rate that the other heirs sold their share to Ping.

ISSUE:
WON the other heirs can sell their shares of the estate prior to adjudication. -- YES

HELD:
An heir can sell his share without final adjudication. An heir is a co-owner of the property
(estate) before adjudication.

Although the Rules of Court do not specifically state that the sale of an immovable property
belonging to an estate of a decedent, in a special proceeding, should be made with the approval
of the court, this authority is necessarily included in its capacity as a probate court. Therefore, it
is clear that the probate court in the case at bar, acted within its jurisdiction in issuing the Order
approving the Deed of Conditional Sale.

The right of an heir to dispose of the decedents property, even if the same is under
administration, is based on the Civil Code provision stating that the possession of hereditary
property is deemed transmitted to the heir without interruption and from the moment of the
death of the decedent, in case the inheritance is accepted. Where there are however, two or
more heirs, the whole estate of the decedent is, before its partition, owned in common by such
heirs.

NKVS
Paulmitan v. C.A.
215 SCRA 866

DOCTRINE: Even if a co-owner sells the whole property as his, the sale will affect only his own
share but not those of the other co-owners who did not consent to the sale. This is because
under the aforementioned codal provision, the sale or other disposition affects only his
undivided share and the transferee gets only what would correspond to his grantor in the
partition of the thing owned in common.

FACTS:
Agatona Sagario Paulmitan, who died sometime in 1953, left the two parcels of land located in
the Province of Negros Occidental. From her marriage with Ciriaco Paulmitan, who is also now
deceased, Agatona begot two legitimate children, namely: Pascual Paulmitan, who also died in
1953, apparently shortly after his mother passed away, and Donato Paulmitan, who is one of
the petitioners. Petitioner Juliana P. Fanesa is Donato's daughter while the third petitioner,
Rodolfo Fanes, is Juliana's husband. Pascual Paulmitan, the other son of Agatona Sagario, is
survived by the respondents, who are his children, name: Alicio, Elena, Abelino, Adelina, Anita,
Baking and Anito, all surnamed Paulmitan.

Donato Paulmitan executed an Affidavit of Declaration of Heirship, extrajudicially adjudicating
unto himself Lot No. 757 based on the claim that he is the only surviving heir of Agatona
Sagario. As regards Lot No. 1091, Donato executed on May 28, 1974 a Deed of Sale over the
same in favor of petitioner Juliana P. Fanesa, his daughter.

For non-payment of taxes, Lot No. 1091 was forfeited and sold at a public auction, with the
Provincial Government of Negros Occidental being the buyer. Juliana P. Fanesa redeemed the
property. On learning of these transactions, respondents children of the late Pascual Paulmitan
filed on with the Court of First Instance of Negros Occidental a Complaint against petitioners to
partition the properties plus damages.

Petitioners set up the defense of prescription with respect to Lot No. 757 contending that the
Complaint was filed more than eleven years after the issuance of a transfer certificate of title to
Donato Paulmitan over the land as consequence of the registration with the Register of Deeds,
of Donato's affidavit extrajudicially adjudicating unto himself Lot No. 757. As regards Lot No.
1091, petitioner Juliana P. Fanesa claimed in her Answer to the Complaint that she acquired
exclusive ownership thereof not only by means of a deed of sale executed in her favor by her
father, petitioner Donato Paulmitan, but also by way of redemption from the Provincial
Government of Negros Occidental.

ISSUE:
1. W/N Donato and Pascual (through his heirs) are co-owners of the land.
2. W/N Juliana has acquired ownership of the property due to the same being
forfeited and subsequently redeemed.

HELD:
1
st
Issue: YES, they are co-owners.

When Agatona Sagario Paulmitan died intestate in 1952, her two (2) sons Donato and Pascual
were still alive. From the time of the death of Agatona Sagario Paulmitan to the subsequent
passing away of her son Pascual in 1953, the estate remained unpartitioned. Article 1078 of the
Civil Code provides: "Where there are two or more heirs, the whole estate of the decedent is,
before its partition, owned in common by such heirs, subject to the payment of debts of the
deceased." Donato and Pascual Paulmitan were, therefore, co-owners of the estate left by their
mother as no partition was ever made.

When Pascual Paulmitan died intestate in 1953, his children, the respondents, succeeded him
in the co-ownership of the disputed property. Pascual Paulmitan's right of ownership over an
undivided portion of the property passed on to his children, who, from the time of Pascual's
death, became co-owners with their uncle Donato over the disputed decedent estate.
When Donato Paulmitan sold on May 28, 1974 Lot No. 1091 to his daughter Juliana P. Fanesa,
he was only a co-owner with respondents and as such, he could only sell that portion which may
be allotted to him upon termination of the co-ownership. The sale did not prejudice the rights of
respondents to one half (1/2) undivided share of the land which they inherited from their father.
It did not vest ownership in the entire land with the buyer but transferred only the seller's pro-
indiviso share in the property 14 and consequently made the buyer a co-owner of the land until
it is partitioned.

Art. 493. Each co-owner shall have the full ownership of his part and of the fruits and
benefits pertaining thereto, and he may therefore alienate, assign or mortgage it and even
substitute another person its enjoyment, except when personal rights are involved. But the
effect of the alienation or mortgage, with respect to the co-owners, shall be limited to the portion
which may be allotted to him in the division upon the termination of the co-ownership.
Even if a co-owner sells the whole property as his, the sale will affect only his own share but not
those of the other co-owners who did not consent to the sale. This is because under the
aforementioned codal provision, the sale or other disposition affects only his undivided share
and the transferee gets only what would correspond to his grantor in the partition of the thing
owned in common.

From the foregoing, it may be deduced that since a co-owner is entitled to sell his undivided
share, a sale of the entire property by one co-owner without the consent of the other co-owners
is not null and void. However, only the rights of the co-owner-seller are transferred, thereby
making the buyer a co-owner of the property.

2
nd
Issue: NO, she is only entitle to the spiritual share of Donato.

The redemption of the land made by Fanesa did not terminate the co-ownership nor give her
title to the entire land subject of the co-ownership.

The right of repurchase may be exercised by co-owner with respect to his share alone. While
the records show that petitioner redeemed the property in its entirety, shouldering the expenses
therefor, that did not make him the owner of all of it. In other words, it did not put to end the
existing state of co-ownership. There is no doubt that redemption of property entails a
necessary expense. Under the Civil Code:

Art. 488. Each co-owner shall have a right to compel the other co-owners to contribute to
the expenses of preservation of the thing or right owned in common and to the taxes. Any one
of the latter may exempt himself from this obligation by renouncing so much of his undivided
interest as may be equivalent to his share of the expenses and taxes. No such waiver shall be
made if it is prejudicial to the co-ownership.

Although petitioner Fanesa did not acquire ownership over the entire lot by virtue of the
redemption she made, nevertheless, she did acquire the right to reimbursed for half of the
redemption price she paid to the Provincial Government of Negros Occidental on behalf of her
co-owners. Until reimbursed, Fanesa hold a lien upon the subject property for the amount due
her.

AMPS
Del Campo v. C.A.
351 SCRA 1

DOCTRINE:
1. The co-owners undivided interest could properly be the object of the contract of sale.

2. Also, undisturbed possession by a co-owner has the effect of a partial partition of the co-
owner property which entitles the possessor to the definite portion which he occupies.

FACTS:
Salome, Consorcia, Alfredo, Maria, Rosalia, Jose, Quirico and Julita, all surnamed Bornales,
were the original co-owners of Lot 162 (27,179 sq.m.). It was divided in aliquot shares among
the eight (8) co-owners as follows:

Salome Bornales 4/16
Consorcia Bornales 4/16
Alfredo Bornales 2/16
Maria Bornales 2/16
Jose Bornales 1/16
Quirico Bornales 1/16
Rosalia Bornales 1/16
Julita Bornales 1/16

Sale by co-owner of her portion: Salome sold part of her 4/16 share to Soledad Daynolo. The
land was specified in the deed of sale. Thereafter, Soledad Daynolo immediately took
possession of the land and built a house thereon. Mortgage: A few years later, Soledad and her
husband, Simplicio Distajo, mortgaged this portion to Jose Regalado. Soledad eventually died.
The husband was able to redeem the mortgage (portion of) land. The heirs subsequently sold
this to herein petitioners, the spouses Manuel Del Campo and Salvacion Quiachon.

Another sale by the co-owners of the entire lot: Later on, three of the eight co-owners (Salome,
Consorcia and Alfredo) sold 24,993 sq. m. of said lot to Jose Regalado. Meanwhile, Jose
Regalado was able to obtain a title in his name of the whole lot previously co-owned. The whole
was subdivided and covered in further titles in his name.

Hence, complaint by previous buyers: Thus, petitioners Manuel and Salvacion del Campo broug
ht this complaint for "repartition, resurvey and reconveyance" against the heirs of the now
deceased Jose Regalado. They claim that they owned an area of 1,544 square meters located
within Lot 162-C-6 which was erroneously included in TCT No. 14566 in the name of Regalado.
Petitioners alleged that they occupied the disputed area as residential dwelling ever since they
purchased the property from the Distajos way back in 1951. They also declared the land for
taxation purposes and paid the corresponding taxes.

ISSUES:
1. Could Salome have validly sold her pro-indiviso share in Lot 162 by metes and bounds
to Soledad, from whom petitioners derived their title?

2. May the ownership of the previous buyers be upheld against that of the second buyer
who has now a registered title?

HELD:
1. YES. Sale valid up to portion of entitlement.

Even if a co-owner sells the whole property as his, the sale will affect only his own share but
not those of the other co-owners who did not consent to the sale.

Soledad became a co-owner of Lot 162 as of the year 1940 when the sale was made in her
favor. It follows that Salome, Consorcia and Alfredo could not have sold the entire Lot 162 to
Jose Regalado. Regalado merely became a new co-owner of Lot 162 to the extent of the
shares which Salome, Consorcia and Alfredo could validly convey. Soledad retained her
rights as co-owner and could validly transfer her share to petitioners in 1951.

2. Previous buyers who only had uninterrupted possession prevails over the second buyer
despite the latters registered title. His purchase constitutes fraud which defeats a
purportedly indefeasible title.

The area subject matter of this petition had already been effectively segregated from the
mother lot even before title was issued in favor of Regalado. It must be noted that 26 years
had lapsed from the time petitioners bought and took possession of the property in 1951
until Regalado procured the issuance of TCT No. 14566. X x x. In the case of Vda. De
Cabrera vs. Court of Appeals, we had occasion to hold that where the transferees of an
undivided portion of the land allowed a co-owner of the property to occupy a definite portion
thereof and had not disturbed the same for a period too long to be ignored, the possessor is
in a better condition or right than said transferees. (Potior est condition possidentis). Such
undisturbed possession had the effect of a partial partition of the co-owner property which
entitles the possessor to the definite portion which he occupies. Conformably, petitioners are
entitled to the disputed land, having enjoyed uninterrupted possession thereof for a total of
49 years up to the present.

Although Regalados certificate of title became indefeasible after the lapse of one year from
the date of the decree of registration, the attendance of fraud in its issuance created an
implied trust in favor of petitioners and gave them the right to seek reconveyance of the
parcel wrongfully obtained by the former. Regalado was aware of petitioners possession of
the subject portion as well as the sale between Salome and Soledad.


KGS
Villanueva v. Florendo
139 SCRA 329

DOCTRINE: Where a surviving spouse sold his undivided portion of the conjugal property to the
wife of one of his sons, the vendors other children are entitled to exercise the right of
redemption as co-owners of the right portion sold.

FACTS:
Macario and Basilia are husband and wife. They have five children. After Basilia died intestate,
the subject parcel of land (165sqm) was left undivided. Thereafter, Macario sold his share to
said property (one-half) to his daughter-in-law, Erlinda. Having been informed of the sale, the
petitioners (co-heirs) signified their intention to redeem the lot in question but Erlinda refused to
allow such redemption contending that she is the wife of one of the legal heirs and therefore
redemption will not lie against her because she is not the "third party" or "stranger"
contemplated in the law.

ISSUE:
Is Erlinda considered as a co-heir and hence, the legitimate heirs are disallowed redemption
contemplated in Article 1620 of the Civil Code? NO

HELD:
NO. Erlinda is a third person. The co-owners have the right to redeem.

Art. 1620 of the New Civil Code provides: A co-owner of a thing may exercise the right of
redemption in case the shares of all the other co-owners or of any of them, are sold to a third
person. If the price of the alienation is grossly excessive, the redemptioner shall pay only a
reasonable one. Should two or more co-owners desire to exercise the right of redemption, they
may only do so in proportion to the share they may respectively have in the thing owned in
common.

It is not disputed that co-ownership exists but the lower court disallowed redemption because it
considered the vendee, Erlinda Vallangca, a co-heir, being married to Concepcion Villanueva,
and the conveyance was held valid since it was in favor of the conjugal partnership of the
spouses in the absence of any statement that it is paraphernal in character.

Within the meaning of Art. 1620, the term "third person" or "stranger" refers to all persons who
are not heirs in succession, and by heirs are meant only those who are called either by will or
the law to succeed the deceased and who actually succeeds. In short, a third person is any one
who is not a co-owner. The vendee is related by affinity to the deceased by reason of her
marriage to one of the heirs and being married to Concepcion does not entitle the vendee to
inherit or succeed in her own right. She is not an heir of Basilia Garcia nor included in the
"family relations" of spouses Macario and Basilia as envisioned in Art. 217 of the Civil Code. Art.
217. Family relations shall include those:
(1) Between Husband and wife;
(2) Between parent and child;
(3) Among other ascendants and their descendants;
(4) Among brothers and sisters.

The co-owners should therefore be allowed to exercise their right to redeem the property sold to
Erlinda Vallangca. To deny petitioners the right of redemption recognized in Art. 1620 of the
Civil Code is to defeat the purpose of minimizing co-ownership and to contravene the public
policy in this regard. Moreover, it would result in disallowing the petitioners a way out of what, in
the words of Manresa, "might be a disagreeable or inconvenient association into which they
have been thrust." Respondent seller Macario, as co-owner and before partition, has the right to
freely sell and dispose of his undivided interest or his Ideal share but not a divided part and one
with boundaries as what was done in the case at bar. It is an inherent and peculiar feature of co-
ownership that although the co-owners may have unequal shares in the common property
quantitatively speaking, each co-owner has the same right in a qualitative sense as any one of
the other co-owners. In other words, every co-owner is the owner of the whole and over the
whole, he exercises the right of dominion, but he is at the same time the owner of a portion
which is truly abstract because until division is effected, such portion is not correctly determined.

Separate Opinions

AQUINO, J ., dissenting:
An undivided portion of a lot sold to a daughter-in-law is a sale to the conjugal partnership of
gains; hence, her sisters and brothers-in-law cannot exercise the co-owners right of redemption.
I am of the opinion that no right of redemption exists in favor of Erlinda's sisters-in-law and
brothers-in-law because the sale was made to the (conjugal partnership of Erlinda and
Concepcion), a co-owner. The sale made Concepcion and Erlinda the co-owners of 6/10 or 3/5
of the 165-square- meter lot. Erlinda is not a third person with respect to the co-ownership.

JPOT
Adille v. C.A.
157 SCRA 455

DOCTRINE: Prescription, as a mode of terminating a relation of co-ownership, must have been
preceded by repudiation (of the co-ownership). The act of repudiation, in turn is subject to
certain conditions: (1) a co-owner repudiates the co-ownership; (2) such an act of repudiation is
clearly made known to the other co-owners; (3) the evidence thereon is clear and conclusive,
and (4) he has been in possession through open, continuous, exclusive, and notorious
possession of the property for the period required by law.

FACTS:
Felisa Alzul owns a parcel of land in Legazpi City, Albay. In her lifetime, she married twice.
First,with Bernabe Adille which was Rustico Adille's father and second, with Prospero Asejo
from whom she had other children. In 1939, she entered in a pacto de retro sale with a period of
three years for repurchase. Unfortunately, she died in 1942 and was unable to settle her affairs
accordingly. Rustico, acted on his own and acquired the land along with a OCT, which he was
able to procure stating that he was the only child of Felisa and Bernabe. He has then been in
charge of the land in question and has kept his actions from his siblings. Although there is one,
namely, Emeteria who happens to live in the same land he resides in. He was then deemed
absolute owner of the land and through his actuations seek to keep the land for himself not
taking into account his other brothers nor his sister. It is because of this that plaintiffs have come
here and contend that trial court erred in:
I. ... declaring the defendant absolute owner of the property;
II. ... not ordering the partition of the property; and
III. ... ordering one of the plaintiffs who is in possession of the portion of the property to
vacate the land, p. 1 Appellant's brief.

ISSUE:
May a co-owner acquire exclusive ownership over the property held in common? -- NO

HELD:
Given that Rustico acquired the property individually, tends to it and is in possession together
with the corresponding OCT, it does not discount the fact that the perfection of such was done
through fraudulent means. His function of reacquisition only makes him a trustee in place of his
other siblings. In addition, a torrens title does not make for a mode of extinguishment with
regard to co-ownership. The court also refuses to recognize his claim of ownership by way of
prescription given that he registered said land in 1955 and has been in his possession until
1974.

Prescription, as a mode of terminating a relation of co-ownership, must have been preceded by
repudiation (of the co-ownership). The act of repudiation, in turn is subject to certain conditions:
(1) a co-owner repudiates the co-ownership; (2) such an act of repudiation is clearly made
known to the other co-owners; (3) the evidence thereon is clear and conclusive, and (4) he has
been in possession through open, continuous, exclusive, and notorious possession of the
property for the period required by law.

Rustico's claim falls short because he did not repudiate. In fact, he had been keeping
substantive information for himself leaving all other co-owners oblivious with concern to his acts.
So, the court finds it fit that although the span of time is indeed what prescription necessitates, it
is still not conclusive nor meritorious to its effect.

WHEREFORE, there being no reversible error committed by the respondent Court of Appeals,
the petition is DENIED. The Decision sought to be reviewed is hereby AFFIRMED in toto. No
pronouncement as to costs.

MLAV
Delima v. C.A.
201 SCRA 641

DOCTRINE: The issuance of the new title constituted an open and clear repudiation of the trust
or co-ownership.

FACTS:
Lino Delima acquired a lot from the Friar Lands Estate in Cebu. He later died leaving as his only
heirs three brothers and a sister namely: Eulalio Delima, Juanita Delima, Galileo Delima and
Vicente Delima. After his death, a title was issued in the name of his legal heirs, as represented
by Galileo Delima.

Galileo Delima declared the lot in his name for taxation purposes and paid the taxes thereon
from 1954 to 1965. Petitioners, who are the surviving heirs of Eulalio and Juanita Delima, filed
with the Court of First Instance of Cebu an action for reconveyance and/or partition of property
and for the annulment of the land title with damages against their uncles Galileo Delima and
Vicente Delima.

The trial court rendered a decision in favor of petitioners.

The decision was appealed, and the CA reversed the lower courts decision. It held that all other
heirs (Eulalio, Juanita and Vicente Delima) had already relinquished and waived their rights to
the property in favor of Galileo, considering that he alone paid the remaining balance of the
purchase price and the realty taxes.

ISSUE:
WON Galileo Delima already acquired the property by prescription -- YES

HELD:
When a co-owner of the property in question executed a deed of partition and on the strength
thereof obtained the cancellation of the title in the name of their predecessor and the issuance
of a new one in his name, in effect denying or repudiating the ownership of the other co-owners
over their shares, the statute of limitations started to run for the purposes of the action instituted
by the latter seeking a declaration of the existence of the co-ownership and of their rights
thereunder.

Since an action for reconveyance of land based on implied or constructive trust prescribes after
ten (10) years, it is from the date of the issuance of such title that the effective assertion of
adverse title for purposes of the statute of limitations is counted.

Evidence shows that TCT No. 2744 in the name of the legal heirs of Lino Delima, represented
by Galileo Delima, was cancelled by virtue of an affidavit executed by Galileo Delima and that
on February 4, 1954, Galileo Delima obtained the issuance of a new title in his name numbered
TCT No. 3009 to the exclusion of his co-heirs. The issuance of this new title constituted an open
and clear repudiation of the trust or co-ownership, and the lapse of ten (10) years of adverse
possession by Galileo Delima from February 4, 1954 was sufficient to vest title in him by
prescription.

As the certificate of title was notice to the whole world of his exclusive title to the land, such
rejection was binding on the other heirs and started as against them the period of prescription.
Hence, when petitioners filed their action for reconveyance and/or to compel partition on
February 29, 1968, such action was already barred by prescription. Whatever claims the other
co-heirs could have validly asserted before can no longer be invoked by them at this time.

DJTV
Mariategui v. C.A.
205 SCRA 337

DOCTRINE: Prescription, as a mode of terminating a relation of co-ownership, must have been
preceded by repudiation (of the co-ownership). The act of repudiation, in turn, is subject to
certain conditions: (1) a co-owner repudiates the co-ownership; (2) such an act of repudiation is
clearly made known to the other co-owners; (3) the evidence thereon is clear and conclusive;
and (4) he has been in possession through open, continuous, exclusive, and notorious
possession of the property for the period required by law.

FACTS:
Lupo Mariategui died without a will. During his lifetime, Lupo Mariategui contracted three (3)
marriages. With his first wife he begot four (4) children. With his second wife, he begot a
daughter. And with his third wife, he begot three children.

At the time of his death, Lupo Mariategui left certain properties which he acquired when he was
still unmarried. Lupo's descendants by his first and second marriages executed a deed of
extrajudicial partition whereby they adjudicated unto themselves lots in the Muntinglupa Estate.
Thereafter, an Original Certiicate of Title (OCT) was issued in the name of the said heirs.
Subsequently, the registered owners caused the subdivision of the said lot into for which
separate transfer certificates of title were issued to the respective parties.

Lupo's children by his third marriage filed with the lower court an amended complaint claiming
that the lots were owned by their common father, Lupo Mariategui, and that, with the
adjudication of the lots to their co-heirs, they (children of the third marriage) were deprived of
their respective shares in the lots. Plaintiffs pray for partition of the estate of their deceased
father and annulment of the deed of extrajudicial partition.

The defendants filed a motion to dismiss on the grounds of lack of cause of action and
prescription. The motion to dismiss was denied by the trial court. The plaintiffs elevated the case
to the Court of Appeals (CA) but the CA upheld the trial courts decision.

ISSUE:
Whether or not prescription barred private respondents' right to demand the partition of the
estate of Lupo Mariategui. -- NO

HELD:
Prescription does not run against private respondents with respect to the filing of the action for
partition so long as the heirs for whose benefit prescription is invoked, have not expressly or
impliedly repudiated the co-ownership. In other words, prescription of an action for partition
does not lie except when the co-ownership is properly repudiated by the co-owner.

Otherwise stated, a co-owner cannot acquire by prescription the share of the other co-owners
absent a clear repudiation of co-ownership duly communicated to the other co-owners.
Furthermore, an action to demand partition is imprescriptible and cannot be barred by laches.
On the other hand, an action for partition may be seen to be at once an action for declaration of
co-ownership and for segregation and conveyance of a determinate portion of the property
involved.

Petitioners contend that they have repudiated the co-ownership when they executed the
extrajudicial partition excluding the private respondents and registered the properties in their
own names. However, no valid repudiation was made by petitioners to the prejudice of private
respondents. Assuming petitioners' registration of the subject lot was an act of repudiation of the
co-ownership, prescription had not yet set in when private respondents filed the present action
for partition.

Petitioners' registration of the properties in their names did not operate as a valid repudiation of
the co-ownership.

In Adille vs. Court of Appeals (157 SCRA 455, 461-462 [1988]), the Court held:
Prescription, as a mode of terminating a relation of co-ownership, must have been
preceded by repudiation (of the co-ownership). The act of repudiation, in turn, is subject
to certain conditions: (1) a co-owner repudiates the co-ownership; (2) such an act of
repudiation is clearly made known to the other co-owners; (3) the evidence thereon is
clear and conclusive; and (4) he has been in possession through open, continuous,
exclusive, and notorious possession of the property for the period required by law.

WHEREFORE, the petition is DENIED and the assailed decision of the Court of Appeals dated
December 24, 1980 is Affirmed.

JRPA
Estreller v. Ysmael
G.R. No. 170264

DOCTRINE: Section 2 of Presidential Decree (P.D.) No. 2016, reinforced by P.D. No. 1517,
which prohibits the eviction of qualified tenants/occupants, extends only to landless urban
families who are rightful occupants of the land and its structures, and does not include those
whose presence on the land is merely tolerated and without the benefit of contract, those who
enter the land by force or deceit, or those whose possession is under litigation.

Petitioners claim that they are lawful lessees of the property. However, they failed to prove any
lease relationship or, at the very least, show with whom they entered the lease contract.
Respondents, on the other hand, were able to prove their right to enjoy possession of the
property. Thus, petitioners, whose occupation of the subject property by mere tolerance has
been terminated by respondents, clearly do not qualify as "tenants" covered by these social
legislations.

FACTS:
Petition for Review under Rule 45.Ysmael and Santos-Alvarez (respondents) filed with the RTC
of Quezon City, a case for Recovery of Possession against Estreller et al.(petitioners), claiming
ownership of the property subject of dispute, by virtue of a TCT issued by the Register of Deeds.
Ysmaels allegations:

On various dates in 1973, Estreller et al. entered the property through stealth and strategy and
had since occupied the same; and despite demands refused to vacate the premises.

Estreller et al.s counterclaims:

Ysmael had no personality to file the suit since he only owned a small portion of the property,
while Santos-Alvarez did not appear to be a registered owner thereof.

That their occupation of the property was lawful, having leased the same from the Magdalena
Estate, and later on from Alvarez. That the property has already been proclaimed by the
Quezon City Government as an Area for Priority Development under P. D. Nos. 1517 and 2016,
which prohibits the eviction of lawful tenants and demolition of their homes.RTC decision: in
favour of Ysmael and Santos-Alvarez.CA decision: affirmed in toto the RTC.

ISSUE:
WON petitioners are rightful occupants of the property. -- NO

HELD:
Finally, petitioners' claim that they are entitled to the protection against eviction and demolition
afforded by P.D. Nos. 2016, 1517, and Republic Act (R.A.) No. 7279, is not plausible. Section 6
of P.D. No. 1517 grants preferential rights to landless tenants/occupants to acquire land within
urban land reform areas, while Section 2 of P.D. No. 2016 prohibits the eviction of qualified
tenants/occupants.

In Dimaculangan v. Casalla, the Court was emphatic in ruling that the protective mantle of P.D.
No. 1517and P.D. No. 2016 extends only to landless urban families who meet these
qualifications: a) they are tenants as defined under Section 3(f) of P.D. No. 1517; b) they built a
home on the land they are leasing or occupying; c) the land they are leasing or occupying is
within an Area for Priority Development and Urban Land Reform Zone; and d) they have resided
on the land continuously for the last 10 years or more.

Section 3(f) of P.D. No. No. 1517 defines the term
"tenant" covered by the said decree as the "rightful occupant of land and its structures, but does
not include those whose presence on the land is merely tolerated and without the benefit of
contract, those who enter the land by force or deceit, or those whose possession is under
litigation."

It has already been ruled that occupants of the land whose presence therein is devoid of any
legal authority, or those whose contracts of lease were already terminated or had already
expired, or whose possession is under litigation, are not considered "tenants" under the Section
3(f).

Petitioners claim that they are lawful lessees of the property. However, they failed to prove any
lease relationship or, at the very least, show with whom they entered the lease contract.
Respondents, on the other hand, were able to prove their right to enjoy possession of the
property. Thus, petitioners, whose occupation of the subject property by mere tolerance has
been terminated by respondents, clearly do not qualify as "tenants" covered by these social
legislations.

ABB
Pada-Kilario v. C.A.
322 SCRA 481


DOCTRINE: It is not required that partition among heirs be in writing and registered in order to
be valid. The object of registration is to serve as constructive notice to third persons who might
have interest on the said property. Thus, it follows that the intrinsic validity of partition not
executed with the prescribed formalities is not undermined when no creditors are involved.

FACTS:
Silverio Pada filed an ejectment case against the spouses Ricardo and Verona Kilario. The
Kilarios occupy a portion of the intestate estate of Jacinto Pada, grandfather of Silverio and
have been living there since 1960 by sheer tolerance. Upon the death of Jacinto Pada, his heirs
entered into extrajudicial partition of his estate in 1951. As a result thereof, Lot 5581 was
allocated to Ananias and Marciano who became co-owners of the said lot.

Ananias died and his daughter succeeded in his right as co-owner. Eventually, Juanita sold her
right in the co-ownership to Engr. Paderes. Maria, Marcianos heir, on the other hand, sold her
share to her cousin respondent Silverio Pada. The latter demanded spouses Kilario to vacate
the lot for their use but the spouses Kilario refused. On June 1995, a complaint for ejectment
was filed against the spouses Kilario. On July 1995 a deed of donation in their favor was
executed by heirs of Amador Pada.

ISSUE:
Whether or not the partition was valid -- YES

HELD:
The extrajudicial partition of the estate of Jacinto Pada among his heirs made in 1951 is VALID,
even if executed in an unregistered private document. It is not required that partition among
heirs be in writing and registered in order to be valid. The object of registration is to serve as
constructive notice to third persons who might have interest on the said property. Thus, it
follows that the intrinsic validity of partition not executed with the prescribed formalities is not
undermined when no creditors are involved. Without creditors to take into consideration, it is
competent for the heirs of an estate to enter into an agreement for distribution thereof in a
manner and upon a plan different from those provided by the rules from which, in the first place,
nothing can be inferred that a writing or other formality is essential for the partition to be valid.
The partition of inherited property need not be embodied in a public document so as to be
effective as regards the heirs that participated therein. The extrajudicial partition which the heirs
of Jacinto Pada executed voluntarily and spontaneously in 1951 has produced a legal status.
When they discussed and agreed on the division of the estate of Jacinto Pada, it is presumed
that they did so in furtherance of their mutual interests. As such, their division is conclusive,
unless and until it is shown that there were debts existing against the estate which had not been
paid. No showing, however, has been made of any unpaid charges against the estate of Jacinto
Pada. Thus, there is no reason why the heirs should not be bound by their voluntary acts.

The belated act of Concordia, Esperanza and Angelito, who are the heirs of Amador Pada, of
donating the subject property to petitioners after forty four (44) years of never having disputed
the validity of the 1951 extrajudicial partition that allocated the subject property to Marciano and
Ananias, produced no legal effect. The donation made by his heirs to petitioners of the subject
property, thus, is void for they were not the owners thereof. In any case, it is too late for the
heirs of Amador Pada to repudiate the legal effects of the 1951 extrajudicial partition as
prescription and laches have equally set in. Having said this, the petitioners are then estopped
from impugning the extrajudicial partition executed by the heirs of Jacinto Pada after explicitly
admitting in their Answer that they had been occupying the subject property since 1960 without
ever paying any rental as they only relied on the liberality and tolerance of the Pada family.
Such admissions are binding to them insofar as the character of their possession of the subject
property is concerned.

FZC
Maestrado v. C.A.
327 SCRA 678

DOCTRINE: A possessor or real estate property is presumed to have title thereto unless the
adverse claimant establishes a better right. partition is the separation, division and assignment
of a thing held in common among those to whom it may belong. If may be effected extra-
judicially by the heirs themselves through a public instrument filed before the register of deeds.
However, as between the parties, a public instrument is neither constitutive nor an inherent
element of a contract of partition. Since registration serves as constructive notice to third
persons, an oral partition by the heirs is valid if no creditors are affected.

FACTS:
These consolidated cases involve Lot No. 5872 and the rights of the contending parties thereto.
The lot has an area of 57.601 sq.m. and is registered in the name of the deceased spouses
Ramon and Rosario Chaves. The spouses died intestate in 1943 and 1944, respectively. They
were urvived by six heirs. To settle the estate of said spouse, Angel Chaves, one of the heirs,
initiated intestate proceedings and was appointed dministrator of said estates in the process. An
inventory of the estates was made and thereafter, the heirs agreed on a project partition. The
court approved the partition but a copy of said decision was missing. Nonetheless, the estate
was divided among the heirs. Subsequently, in 1956, the partition case effected and the
respective shares of the heirs were delivered to them.

Significantly, Lot No.5872 was not included in a number of documents. Parties offered different
explanations as to the omission of said lot in the documents. Petitioners maintain the existence
of an oral partition agreement entered into by all heirs after the death of their parents. To set
things right, petitioners then prepared a quitclaim to confirm the alleged oral agreement.
Respondents dispute voluntariness of their consent
to the quitclaims.

Six years after the execution of the quitclaims, respondents discovered that indeed subject lot
was still a common property in the name of the deceased spouses. Eventually, an action for
Quieting of Title was filed by petitioners on December 22, 1983.

The trial court considered Lot No. 5872 as still a common property and therefore must be
divided into six parts, there being six heirs. Petitioners appealed to the Court of Appeals which
sustained the decision of the trial court.

ISSUE:
Whether or not the action for quieting of title had already prescribed.

HELD:
Lot No. 5872 is no longer common property of the heirs of the deceased spouses Ramon and
Rosario Chaves. Petitioners ownership over said lot was acquired by reason of the oral partition
agreed upon by the deceased spouses heirs sometime before 1956. That oral agreement was
confirmed by the notarized quitclaims executed by the said heirs on August 16, 1977 and
September 8, 1977. There was indeed an oral agreement of partition entered into by the
heirs/parties. A possessor of real estate property is presumed to have title thereto unless the
adverse claimant establishes a better right. In the instant case it is the petitioners, being the
possessors of Lot No. 5872, who have established a superior right thereto by virtue of the oral
partition which was also confirmed by the notarized quitclaims of the heirs. Partition is the
separation,division and assignment of a thing held in common among those to whom it may
belong. It may be effected extra-judicially by the heirs themselves through a public instrument
filed before the register of deeds. However, as between the parties, a public instrument is
neither constitutive nor an inherent element of a contract of partition. Since registration serves
as constructive notice to third persons, an oral partition by the heirs is valid if no creditors are
affected. Moreover, even the requirement of a written memorandum under the statute of frauds
does not apply to partitions effected by the heirs where no creditors are involved considering
that such transaction is not a conveyance of property resulting in change of ownership but
merely a designation and segregation of that part which belongs to each heir.

LNAC
Pilapil v. Briones
GR 150175

DOCTRINE: Since an implied trust is an obligation created by law, then respondents had 10
years within which to bring an action for reconveyance of their shares in Maximinos properties.
The general rule is that an action for reconveyance of real property based on implied trust
prescribes ten years from registration and/or issuance of the title to the property, not only
because registration under the Torrens system is a constructive notice of title, but also because
by registering the disputed properties exclusively in her name, Donata had already
unequivocally repudiated any other claim to the same.

FACTS:
Petitioners are the heirs of the late Donata Ortiz-Briones, consisting of her surviving sister,
Rizalina Ortiz- Aguila; Rizalinas daughter, Erlinda Pilapil; and the other nephews and nieces of
Donata, in representation of her two other sisters who had also passed away. Respondents, on
the other hand, are the heirs of the late Maximino Briones, composed of his nephews and
nieces, and grandnephews and grandnieces, in representation of the deceased siblings of
Maximino.

Maximino was married to Donata but their union did not produce any children. When Maximino
died, Donata instituted intestate proceedings to settle her husbands estate. The CFI issued
Letters of Administration appointing Donata as the administratrix of Maximinos estate. She
submitted an Inventory of Maximinos properties, which included, among other things, parcels of
land.

The CFI subsequently issued an Order, awarding ownership of the real properties to Donata.
Donata had the said CFI Order recorded in the Primary Entry Book of the Register of Deeds,
and by virtue thereof, received new TCTs, covering the said properties, now in her name.

Donata died. Erlinda, one of Donatas nieces, instituted a petition for the administration of the
intestate estate of Donata. Erlinda and her husband, Gregorio, were appointed by the RTC as
administrators of Donatas intestate estate.

Silverio Briones, a nephew of Maximino, filed a Petition for Letters of Administration for the
intestate estate of Maximino, which was initially granted by the RTC. The RTC also issued an
Order, allowing Silverio to collect rentals from Maximinos properties. But then, Gregorio filed a
Motion to Set Aside the Order, claiming that the said properties were already under his and his
wifes administration as part of the intestate estate of Donata. Silverios Letters of Administration
for the intestate estate of Maximino was subsequently set aside by the RTC.

The heirs of Maximino filed a Complaint against the heirs of Donata for the partition, annulment,
and recovery of possession of real property. They alleged that Donata, as administratrix of the
estate of Maximino, through fraud and misrepresentation, in breach of trust, and without the
knowledge of the other heirs, succeeded in registering in her name the real properties belonging
to the intestate estate of Maximino.

RTC rendered its Decision in favor of the heirs of Maximino.

Thus, the heirs of Donata filed the present Petition.

ISSUE:
Whether Donata succeeded in registering in her name the real properties belonging to the
intestate estate of Maximino. -- YES

HELD:
In summary, the heirs of Maximino failed to prove by clear and convincing evidence that Donata
managed, through fraud, to have the real properties, belonging to the intestate estate of
Maximino, registered in her name. In the absence of fraud, no implied trust was established
between Donata and the heirs of Maximino under Article 1456 of the New Civil Code. Donata
was able to register the real properties in her name, not through fraud or mistake, but pursuant
to an Order issued by the CFI. The CFI Order, presumed to be fairly and regularly issued,
declared Donata as the sole, absolute, and exclusive heir of Maximino; hence, making Donata
the singular owner of the entire estate of Maximino, including the real properties, and not merely
a co-owner with the other heirs of her deceased husband.

Prescription of the action for reconveyance of the disputed properties based on implied trust is
governed by Article 1144 of the New Civil Code. The following actions must be brought within
ten years from the time the right of action accrues: (1) Upon a written contract; (2) Upon an
obligation created by law; (3) Upon a judgment.

Since an implied trust is an obligation created by, then respondents had 10 years within which
to bring an action for reconveyance of their shares in Maximinos properties. The general rule is
that an action for reconveyance of real property based on implied trust prescribes ten years
from registration and/or issuance of the title to the property, not only because registration under
the Torrens system is a constructive notice of title, but also because by registering the disputed
properties exclusively in her name, Donata had already unequivocally repudiated any other
claim to the same.

By virtue of the CFI Order, dated 15 January 1960, Donata was able to register and secure
certificates of title over the disputed properties in her name on 27 June 1960. The respondents
filed their Complaint for partition, annulment, and recovery of possession of the disputed real
properties only on 3 March 1987, almost 27 years after the registration of the said properties in
the name of Donata. Therefore, respondents action for recovery of possession of the disputed
properties had clearly prescribed.

Other than prescription of action, respondents right to recover possession of the disputed
properties, based on implied trust, is also barred by laches. The defense of laches, which is a
question of inequity in permitting a claim to be enforced, applies independently of prescription,
which is a question of time. Prescription is statutory; laches is equitable. Laches is defined as
the failure to assert a right for an unreasonable and unexplained length of time, warranting a
presumption that the party entitled to assert it has either abandoned or declined to assert it. This
equitable defense is based upon grounds of public policy, which requires the discouragement of
stale claims for the peace of society.

It is uncontested that from the time of Maximinos death on 1 May 1952, Donata had possession
of the real properties. She managed the real properties and even collected rental fees on some
of them until her own death on 1 November 1977. After Donatas death, Erlinda took possession
of the real properties, and continued to manage the same and collect the rental fees thereon.
Donata and, subsequently, Erlinda, were so obviously exercising rights of ownership over the
real properties, in exclusion of all others, which must have already put the heirs of Maximino on
guard if they truly believed that they still had rights thereto.

It is well established that the law serves those who are vigilant and diligent and not those who
sleep when the law requires them to act. The law does not encourage laches, indifference,
negligence or ignorance. On the contrary, for a party to deserve the considerations of the courts,
he must show that he is not guilty of any of the aforesaid failings.

TKDC
Oliveros v. Lopez
168 SCRA 431

DOCTRINE: Under Article 494 and 1083 of the Civil Code, co-ownership of an estate should not
exceed twenty years, while the agreement to keep a thing undivided should not exceed ten
years. When parties stipulated a definite period to keep a thing undivided which exceeds the
maximum allowed by law, said stipulation shall be void only as to the period beyond such
maximum. Thus, co-ownership of an estate cannot exceed twenty years so property should be
divided after twenty years. Each co-owner may demand at any time the partition of the thing
owned in common insofar as his share is concerned.

Article 494 specifically mandates that each co-owner may demand at any time the partition of
the thing owned in common insofar as his share is concerned.

FACTS:
Upon his death, Lopez left the lot he owned to his widow and six children; the heirs did not
initiate any move to legally partition the property. After many years, the widow and the eldest
son, Candido, executed a deed of absolute sale of the undivided eastern portion of their
interests in favor of spouses Oliveras & Minor; another deed of absolute sale of the undivided
eastern part in favor of spouses Oliveras & Gaspar. The two Oliveras spouses had since
possessed the properties.

After many years, the counsel of the two Oliveras spouses wrote to the remaining heirs of Lopez
reminding them of the Oliveras spouses demand to partition the property so they could acquire
their titles without court action. The heirs didnt answer so the Oliveras spouses filed a complaint
for partition and damages.

According to the Oliveras spouses, possession of the disputed properties was delivered to them
with the knowledge and consent of the heirs; however, according to the heirs, no sale transpired
as the vendors, the widow and Candido, could not sold specific portions of the property making
Oliveras spouses possession and occupation of specific portions of the properties illegal. Trial
court ruled that the deeds of absolute sale are valid and ordered the segregation of the lot.

ISSUES:
1. Whether or not the two deeds of absolute sale were null and void since the lot had not
yet been partitioned. -- NO
2. Whether or not the action for partition has prescribed. -- NO

HELD:
1. Under Article 494 and 1083 of the Civil Code, co-ownership of an estate should not
exceed twenty years, while the agreement to keep a thing undivided should not exceed ten
years. When parties stipulated a definite period to keep a thing undivided which exceeds the
maximum allowed by law, said stipulation shall be void only as to the period beyond such
maximum. Thus, co-ownership of an estate cannot exceed twenty years so property should
be divided after twenty years. Each co-owner may demand at any time the partition of the
thing owned in common insofar as his share is concerned.

In this case, the heirs maintained the co-ownership beyond 20 years so when the widow and
Candido sold definite portions of the lot, they validly exercised dominion over them because,
by operation of law, the co-ownership had ceased. The filing of the complaint for partition by
the Oliveras spouses who are legally considered as subrogated to the rights over the
partitions of lot in their possession merely served as formality on the widow and Candidos
act of terminating co-ownership.

2. Prescription may have barred the filing of complaint under Article 1144 (a) of the Civil
Code; however, Article 494 specifically mandates that each co-owner may demand at any
time the partition of the thing owned in common insofar as his share is concerned. In this
case, although the complaint was filed after thirteen years from the execution of the deeds of
sale, the Oliveras spouses action for partition was timely and properly filed.

AMD
Sering v. Plazo
166 SCRA 84

DOCTRINE:
1. Anyone of the co-owners of an immovable may bring an action in ejectment. A
co-owner may bring an ejectment action without joining the other co-owners, the suit being
deemed instituted for the benefit of all.
2. The term, "action in ejectment," includes a suit of forcible entry (detentacion) or
unlawful detainer (desahucio).

FACTS:
Alfredo Sering filed a forcible entry suit against respondent Spouses Restituto Plazo and
Gertrudes Suan with the Municipal Court of del Carmen, Surigao del Norte. The case was
adjudged against the Plazos, who then appealed to the CFI. Therein, the Plazos learned that
the property subject of the suit was not owned solely by Sering but was owned in common by
him and others. The Plazos moved for the impleading of the other co-owners as plaintiff,
claiming that they were indispensable parties. The CFI agreed and ordered Sering to amend his
complaint so as to include his co-owners as co-plaintiffs. Sering demurred, claiming that under
the law anyone of the co-owners could bring suit for ejectment without joining the others. The
Plazos contended that the invoked provision had no application to forcible entry actions, but
only to suits of unlawful detainer. Because Sering failed to comply with the order for amendment
of the complaint, the CFI dismissed his complaint, it also denied his MR. Hence, this petition.

ISSUE:
W/N Sering, as co-owner, can file an action for ejectment in behalf of and without joining the
other co-owners. -- YES

HELD:
Article 487 of the Civil Code provides that anyone of the co-owners of an immovable may bring
an action in ejectment. A co-owner may thus bring an ejectment action without joining the other
co-owners, the suit being deemed instituted for the benefit of all. And the term, "action in
ejectment," includes a suit of forcible entry (detentacion) or unlawful detainer (desahucio).

As early as 1969, in the case Vencilao v. Camarento, the SC pertinently ruled as follows:
Anent the question of whether an action of forcible entry and detainer should be brought in the
name of all co-owners, We hold that under Article 487 of the new Civil Code, any of the co-
owners may bring the action... In forcible entry and detainer action(s) the matter to be
determined is simply the question of prior physical possession. It having been alleged in the
complaint that the plaintiff was in actual possession of the properties, certainly the plaintiff alone,
who was in actual possession, could file the complaint.

CRF
Adlawan v. Adlawan
G.R. No. 161916

DOCTRINE: A co-owner by virtue of Art. 487 is allowed to bring an action without necessity of
including all the co-owners as plaintiffs for it is presumed to be for the benefit of all BUT if the
action of the plaintiff alone, the action should be dismissed. (batasnatin)

FACTS:
The instant ejectment suit stemmed from the parties dispute over Lot 7226 and the house built
thereon, covered by Transfer Certificate of Title No. 8842, registered in the name of the late
Dominador Adlawan and located in Cebu. Petitioner averred that he is an acknowledged
illegitimate son and the sole heir of Dominador. In ruling for the petitioner, the RTC held that the
questioned January 31, 1962 deed of sale validly transferred title to Dominador and that
petitioner is his acknowledged illegitimate son who inherited ownership of the questioned lot.
The petitioner further claims that he allowed the respondents to occupy the property out of
respect and generosity to respondents who are the siblings of his father who stayed on the
questioned property since birth, provided they would vacate the same should his need for the
property arise. When the petitioner verbally requested respondents to vacate the house and lot,
but they refused and filed instead an action for quieting of title with the RTC.

The respondents argued that even if petitioner is indeed Dominadors acknowledged illegitimate
son, his right to succeed is doubtful because Dominador was survived by his wife, Graciana.
This means that the petitioner is not the sole owner of Lot 7226. This is so because Dominador
was survived not only by petitioner but also by his legal wife, Graciana, who died 10 years after
the demise of Dominador on May 28, 1987. By intestate succession, Graciana and petitioner
became co-owners of Lot 7226.

ISSUE:
Whether or not petitioner can validly maintain the instant case for ejectment against his co-
owners of lot 7226. -- NO

HELD:
No. The ejectment suit will not prosper as the petitioner filed it for his benefit alone and not for
the benefits of the rest of co owners.

ART. 487. Any one of the co-owners may bring an action in ejectment.

This article covers all kinds of actions for the recovery of possession. Article 487 includes
forcible entry and unlawful detainer (accion interdictal), recovery of possession (accion
publiciana), and recovery of ownership (accion de reivindicacion). A co-owner may bring such
an action without the necessity of joining all the other co-owners as co-plaintiffs because the suit
is presumed to have been filed to benefit his co-owners. It should be stressed, however, that
where the suit is for the benefit of the plaintiff alone who claims to be the sole owner and entitled
to the possession of the litigated property, the action should be dismissed.

MPF
Cruz v. Leis
327 SCRA 570

DOCTRINE: Redemption by co-owner does not terminate the co-ownership nor give her title to
the entire land subject of the co-ownership.

FACTS:
Leis and Isidro married each other in 1923. Isidro subsequently acquired from the
Department of Agriculture and Natural Resources a parcel of land, which was titled in her
name, with the description that she was a widow. Leis only passed away in 1973 without
executing a will.
Isidro then secured a loan from Cruz (PhP 15,000, with 5% interest) secured by a
mortgage on the land from DANR, but failed to pay on due date. Isidro executed 2 contracts
in favor of Cruz: a Deed of Absolute Sale and a Contract indicating a pacto de retro sale.
Isidro still failed to repurchase the property within 1 year, so she consolidated the ownership
of the land in favor of Cruz.
When Isidro died, Cruz demanded her heirs to vacate the premises. The heirs
then filed a complaint with the RTC averring that the land was conjugal property having been
purchased during their marriage. The RTC found in favor of the heirs. The case was
appealed to the CA, but the CA merely affirmed the ruling because Cruz failed to get a
judicial order to have the land consolidated in his name after failure of Isidro to comply with
the requirements of the right to repurchase (Art. 1607).

ISSUE:
Whether or not the land in question is conjugal property, and therefore subject to the rules on
co-ownership?

HELD:
Although the land was purchased during the marriage, upon Leis death, the conjugal property
regime ceased, and gave Isidro an equal portion of Leis half of the property to be divided
among his legitimes. Co-ownership of the land then began.

However, upon failure of Isidro the heirs to exercise the right to repurchase, the ownership of
the land transferred to Cruz. Despite the TCT being void for non-compliance with 1607, the
ownership did not transfer back to the heirs, for compliance with 1607 is merely for purposes of
registering the title in the Torrens System.

AMDG
Mariano v. CA
222 SCRA 736

DOCTRINE: Redemption of the whole property by a co-owner does not vest in him sole
ownership over said property but will inure to the benefit of all co-owners. In other words, it will
not put an end to the existing state of co-ownership. Redemption is not a mode of termination of
a co-ownership

FACTS:
Decendent Francisco Gosienfiao, mortgage a residential lot to Rural Bank. Failing to pay the
loan, the lot was foreclosed and was awarded to be bank for being the highest bidder. Francisco
died intestate and was survived by his heirs, the third-party defendants and plaintiff-appellants.
Within the lots one year redemption period, one of the third-party defendants, Amparo, bought
the land from the bank using her personal funds. It is noted that the rest of the third-party
defendants executed a Deed of Assignment of the Right of Repurchase to Amparo.

Amparo sold the lot to then defendant spouses Leonardo and Avelina Mariano (now petitioners
in this case) who built a house on the said lot. When one of the plaintiff-appellants, Grace,
discovered such sale, she requested for a confrontation between her and the defendants in the
presence of the Barangay Captain. Grace asked that the defendants to present her claim of the
said property which never happened. It is also noted that the spouses sold the questioned lot to
their children, Lazaro and Dionicia.

The plaintiff-appellants filed a complaint for recovery of possession and legal redemption with
damages against the spouses. The plaintiffs alleged that they are co-owners of the lot hence
they have the right to it since they did not sell their shares. Also they alleged that they have the
right of redemption with regard to the shares of other co-owners sold to the spuses. In their
answer, the spouses raised that such lot was purchased by Amparo alone hence she became
the sole owner of the land. As to the right of redemption, though such right indeed exists, the
spouses alleged that it was already barred by Statute of Limitations or of by laches.

The RTC ruled in favor of the spouses and held that the plaintiffs had no right of ownership or
possession over the lot. The lower court further explained that when the lot was bought by the
bank, their rights were reduced to a mere right of redemption. Having failed to redeem the land
and Amparo solely repurchasing the land, the plaintiffs lost all their rights over the lot. However,
the Court of Appeals reversed the lower courts decision and ruled the plaintiffs were the co-
owners of the lot. It explained that although Amparo used her own money when she bought the
land, it did not make her the sole owner. Such redemption inured to the benefit of all the co-
owners.

ISSUES:
1. WON a notice was given to the petitioners? -- NO
2. WON the petitioners lost their right to redeem although no notice was received by them?
-- NO

HELD:
The right of legal redemption is granted within one month after the written notice was delivered
to the party granted of such right. It has been previously held by this Court that such written
notice is an indispensible requirement and the right of legal redemption will only run after such
notice was presented. Also, a copy of a deed of sale was considered as a notice.

Looking at the pieces of evidence, no written notice of the sale was ever presented to the
petitioners even during the confrontation. The private respondents (then plaintiff-apellants)
exercised their right of redemption at the first opportunity by tendering the repurchase price to
the spouses which refused to accept it.

A written notice must be given to remove all uncertainty as to the sale, its terms and its validity,
and to quiet and doubt that the alienation is not definitive. Establishing that no notice was ever
presented, the 30-day period has not yet started.

Lastly, the Court of Appeals was correct in ruling that the redemption of Amparo alone did not
terminate the co-ownership rather such action inured to the benefit of all co-owners.
Redemption is not a recognized mode of extinguishing co-ownership.

GCG
Tan v. CA
172 SCRA 660

DOCTRINE: Consolidation of ownership by mortgagee after expiration of redemption period
terminates co-ownership. A co-owner who redeems a property with her own funds after such
consolidation becomes the sole owner thereof.

FACTS:
Tan Tiong Tick, married to Tan Ong Hun was the registered owner of a parcel of land and its
improvements in Binondo Manila. During his lifetime, he obtained a loan secured by a real
estate mortgage over the land. The land was mortgaged to China Bank to secure payment for
several obligations.

Tan Tiong Tick and Tan Ong Hun died without paying their obligations. They had six children
George Laurel, Teodora, Rosa, Rosita, Mauro Umali, and D. Annie Tan.

China Bank foreclosed on the mortgage. Two weeks before the redemption period expired,
China Bank and the Heirs of Tan Tiong Tick entered into a settlement. It provided that the heirs
were given right to repurchase even after the redemption period but before August 3, 1973.

The heirs failed to redeem before the legal redemption period so China Bank consolidated its
ownership and was issued a new TCT. However, D. Annie Tan exercised the right to
repurchase pursuant to the settlement using her personal funds. But the title to the land was
registered in the name of all the heirs.

D. Annie Tan filed an action to reconvey the property to her and damages. RTC ruled that the
property was co-owned by the heirs. CA affirmed.

ISSUE:
Whether or not co-ownership among the heirs was dissolved by the foreclosure and
consolidation of title by the bank after the redemption period has expired? -- YES

RULING:
Since the lot and its improvement were mortgaged by the deceased parents, there can be no
question that a co-ownership existed among the heirs during the period given by law to redeem
the foreclosed property. Redemption by one during this period would have inured to the benefit
of all.

The records show, however, that when the petitioner purchased the disputed property on
August 30, 1974, any co-ownership among the brothers and sisters no longer existed. The
period to redeem had expired more than one year earlier, on July 6, 1973. The respondent
China Bank consolidated its ownership and a new title was issued in the bank's name. When
the heirs allowed the one year redemption period to expire without redeeming their parents'
former property and permitted the consolidation of ownership and the issuance of a new title,
the co-ownership was extinguished.

Since D. Annie Tan used her personal fund to repurchase the property, she is the lawful sole
owner. The respondent China Banking Corporation is ordered to execute the deed of sale over
the disputed property in favor of the petitioner alone.

VCL IV
Heirs of Maninding v. CA
276 SCRA 601

DOCTRINE: Prescription, as a rule, does not run in favor of a co-heir or co-owner as long as he
expressly or impliedly recognizes the co-ownership.

Co-owners cannot acquire by prescription
the share of the other co-owners, absent a clear repudiation of the co-ownership. It must be
clearly shown that he has repudiated the claims of the others, and that they were apprised of his
claim of adverse and exclusive ownership, before the prescriptive period would begin to run.

FACTS:
On 31 July 1979 Segunda Maningding died. Her heirs allegedly discovered the transfers made
by Roque Bauzon in favor of his children only in 1986. Consequently, the heirs sought the
partition of the properties as well as the accounting of the produce but were unsuccessful.

With regard to the sugarland, Roque Bauzon denied having executed the Affidavit of Self-
Adjudication presented by petitioners. He claimed that he acquired ownership over both the
sugarland and the riceland by donation propter nuptias from his parents Ramon Bauzon and
Sotera Zulueta on 21 April 1926 in consideration of his marriage to Petra Loresco. Since the
death of Ramon Bauzon in 1948, Roque had been in open, continuous, notorious, adverse and
actual possession of the subject properties.

RTC ruled that the parcels of land formed part of the estate of Ramon Bauzon and his wife
Sotera Zulueta which, upon their death, devolved by right of succession to their children
Segunda Maningding, Maria Maningding, Juan Maningding and Roque Bauzon in equal pro-
indiviso shares. The court a quo however awarded both parcels to Segunda Maningding and
Roque Bauzon as co-owners in equal shares after finding that Juan Maningding and Maria
Maningding had already executed an Affidavit of Quitclaim and Renunciation. It rejected the
deed of donation for failure to prove its due execution and authenticity and ruled that the same
was negated by the Affidavit of Quitclaim and Renunciation of Juan Maningding and Maria
Maningding in favor of Roque Bauzon and nullified the deed of sale by Roque Bauzon in favor
of Luis Bauzon as regards the riceland and to Eriberta Bauzon with respect to the sugarland. It
concluded that Roque Bauzon could not have validly conveyed both parcels as one-half (1/2) of
each parcel rightfully belonged to Segunda Maningding and her heirs.

CA ruled that the properties validly pertained to Roque Bauzon by virtue of the donation propter
nuptias. Consequently, the transfers made by Roque Bauzon must be given effect. However,
upon motion for reconsideration, the same deed of donation was declared null and void by the
appellate court for failure to comply with Art. 633 of the old Civil Code, the law then applicable,
which required for the validity of the deed of donation to be in a public instrument. Nevertheless,
the same court maintained that the properties belonged to Roque Bauzon by virtue of
acquisitive prescription.

ISSUE:
Whether or not Bauzon acquired the ownership of the land by acquisitive prescription -- YES

HELD:
Bauzon acquired ownership over the subject properties by acquisitive prescription. Prescription,
in general, is a mode of acquiring (or losing) ownership and other real rights through the lapse
of time in the manner and under conditions laid down by law, namely, that the possession
should be in the concept of an owner, public, peaceful, uninterrupted and adverse. Acquisitive
prescription is either ordinary or extraordinary. Ordinary acquisitive prescription requires
possession in good faith and with just title for ten (10) years. In extraordinary prescription
ownership and other real rights over immovable property are acquired through uninterrupted
adverse possession thereof for thirty (30) years, without need of title or of good faith.

The disputed lots are unregistered lands. While tax declarations and receipts are not conclusive
evidence of ownership, yet, when coupled with proof of actual possession, as in the instant case,
tax declarations and receipts are strong evidence of ownership.

Even assuming that the donation proper nuptias is void for failure to comply with formal
requisites,

it could still constitute a legal basis for adverse possession. Sixty (60) years have
already elapsed.

Prescription, as a rule, does not run in favor of a co-heir or co-owner as long as he expressly or
impliedly recognizes the co-ownership.

Co-owners cannot acquire by prescription the share of
the other co-owners, absent a clear repudiation of the co-ownership. It must be clearly shown
that he has repudiated the claims of the others, and that they were apprised of his claim of
adverse and exclusive ownership, before the prescriptive period would begin to run.

The evidence relative to the possession, as a fact upon which the alleged prescription is based,
must be clear, complete and conclusive in order to establish said prescription without any
shadow of doubt.

Therefore while prescription among co-owners cannot take place when the acts of ownership
exercised are vague and uncertain, such prescription arises and produces all its effects when
the acts of ownership do not evince any doubt as to the ouster of the rights of the other co-
owners.

As disclosed by the records, Roque Bauzon and his heirs possessed the property from
1948 to 1986 to the exclusion of petitioners who were never given their shares of the fruits of
the properties, for which reason they demanded an accounting of the produce and the
conveyance to them of their shares.

CRF
Aguilar v. CA
227 SCRA 472

DOCTRINE: Any of the Co-owners may demand the sale of the house and lot at any time and
the other cannot object to such demand; thereafter the proceeds of the sale shall be divided
equally according to their respective interests. (batasnatin)

FACTS:
Petitioner Virgilio and respondent Senen are brothers and purchased a house and lot in
Paraaque where their father could spend and enjoy his remaining years in a peaceful
neighborhood. Initially, the brothers agreed that Virgilio's share in the co-ownership was two-
thirds while that of Senen was one-third. By virtue of a written memorandum dated 23 February
1970, Virgilio and Senen agreed that henceforth their interests in the house and lot should be
equal, with Senen assuming the remaining mortgage obligation of the original owners with the
Social Security System (SSS) in exchange for his possession and enjoyment of the house
together with their father. It was further agreed that Senen would take care of their father and
his needs since Virgilio and his family were staying in Cebu. After Maximiano Aguilar died in
1974, petitioner demanded from private respondent that the latter vacate the house and that the
property be sold and proceeds thereof divided among them but the latter refused to do so
thereby causing the petitioner to file a complaint. In his complaint, petitioner prayed that the
proceeds of the sale, be divided on the basis of two-thirds (2/3) in his favor and one-third (1/3)
to respondent. Petitioner also prayed for monthly rentals for the use of the house by respondent
after their father died. The respondent alleged that he had no objection to the sale as long as
the best selling price could be obtained; that if the sale would be effected, the proceeds thereof
should be divided equally; and, that being a co-owner, he was entitled to the use and enjoyment
of the property.

On 26 July 1979, rendering judgment by default against defendant, the trial court found him and
plaintiff to be co-owners of the house and lot, in equal shares on the basis of their written
agreement. However, it ruled that plaintiff has been deprived of his participation in the property
by defendant's continued enjoyment of the house and lot, free of rent, despite demands for
rentals and continued maneuvers of defendants, to delay partition. The trial court also upheld
the right of plaintiff as co-owner to demand partition. Since plaintiff could not agree to the
amount offered by defendant for the former's share, the trial court held that this property should
be sold to a third person and the proceeds divided equally between the parties.The trial court
likewise ordered defendant to vacate the property and pay plaintiff P1,200.00 as rentals
2
from
January 1975 up to the date of decision plus interest from the time the action was filed.

ISSUE:
Whether or not the plaintiff as a co-owner demand a partition of the said property? -- YES

HELD:
The Court holds that on the basis of the pleadings of the parties and the evidence presented ex
parte, petitioner and respondents are co-owners of subject house and lot in equal shares; either
one of them may demand the sale of the house and lot at any time and the other cannot object
to such demand; thereafter the proceeds of the sale shall be divided equally according to their
respective interests.

Article 494 of the Civil Code provides that no co-owner shall be obliged to remain in the co-
ownership, and that each co-owner may demand at any time partition of the thing owned in
common insofar as his share is concerned. Corollary to this rule, Art. 498 of the Code states
that whenever the thing is essentially, indivisible and the co-owners cannot agree that it be,
allotted to one of them who shall indemnify the others, it shall be sold and its proceeds
accordingly distributed. This is resorted to (1) when the right to partition the property is invoked
by any of the co-owners but because of the nature of the property it cannot be subdivided or its
subdivision would prejudice the interests of the co-owners, and (b) the co-owners are not in
agreement as to who among them shall be allotted or assigned the entire property upon proper
reimbursement of the co-owners.

JGY
Coronel v. Constantino
397 SCRA 128

DOCTRINE: The sale of the subject property made by a co-owner is limited to the portion which
may be allotted to her upon the termination of co-ownership over the subject property with her
children.

FACTS:
The subject property consists of two parcels of land is originally owned by Aguinaldo. One-half
was inherited by petitioners Emilia together with her sons Benjamin, Catalino and Ceferino. The
other half was inherited by respondents Constantino and Buensuceso.

Respondents filed a complaint for declaration of ownership, quieting of title against the
Petitioners. Respondents allege that a certain Santos and Bernardo purchased the property
belonging to Emilia and her sons by virtue of a deed of sale signed by Emilia. Santos and
Bernardo in turn sold the same to Constantino and Buensuceso.TC ruled in favor of the
respondents.

ISSUE:
WON the contract of sale executed by a parent co-owner is enforceable with respect to the
share of her children. -- NO

HELD:
It is clear that petitioner Benjamin did not sign the document and that the shares of Catalino and
Ceferino in the subject property were not sold by them.

The deed of sale is not a competent proof that petitioner Benjamin had sold his own share of the
subject property. It cannot be disputed that Benjamin did not sign the document and therefore,
it is unenforceable against him.

Emilia executed the instrument in her own behalf and not in representation of her three children.

Article 493 of the Civil Code states:

Each co-owner shall have the full ownership of his part and of the fruits and benefits pertaining
thereto, and he may therefore alienate, assign or mortgage it, and even substitute another
person in its enjoyment, except when personal rights are involved. But the effect of the
alienation or the mortgage, with respect to the co-owners, shall be limited to the portion which
may be allotted to him in the division upon the termination of the co-ownership.

Consequently, the sale of the subject property made by Emilia in favor of Santos and Bernardo
is limited to the portion which may be allotted to her upon the termination of her co-ownership
over the subject property with her children.




POSSESSION (ART. 523-561)

FXRL
Yu v. Pacleb
G.R. No. 130316

DOCTRINE: Should a question arise regarding the fact of possession, the present possessor
shall be preferred; if there are two possessors, the one longer in possession; if the dates of the
possession are the same, the one who presents a title; and if all these conditions are equal, the
thing shall be placed in judicial deposit pending determination of its possession or ownership
through proper proceedings.

FACTS:
Respondents sold a parcel of land divided into three partitions to Rebecca del Rosario
Del Rosario sold the land to Ruperto Javier.
The title to the property remained in the name of the respondents. None of the sales
earlier mentioned were annotated on the title.
Javier offered to sell a parcel of land to the spouses Yu (petitioners).
Petitioners accepted the offer and formally took over the property after execution of a
contract to sell.
At the time of such turnover, a portion of the property was occupied by the son of the
respondent and the wife thereof as tenants.
The son allegedly surrendered possession of his portion of the property to the petitioners.
He was made trustee by the petitioners.
Petitioners also caused the annotation on the title of the property
Petitioners allege that they exercised open, public, and peaceful possession of the
property for three years. During this span of time, the respondent was in the US.
Respondent allegedly entered the property using Force, Intimidation, Strategy, Threat,
and Stealth (FISTS) and ousted the trustee of the property, replacing him with another one
of his sons.
Petitioners filed an action for forcible entry.
MTC and RTC ruled in favor of the petitioners.
CA reverses the decision.

ISSUE:
W/N the petitioners had prior physical possession of the subject property. -- NO

HELD:
The Civil Code states that possession is the holding of a thing or the enjoyment of a right. In the
grammatical sense, to possess means to have, to actually and physically occupy a thing, with or
without right. "Possession always includes the idea of occupation x x x. It is not necessary that
the person in possession should himself be the occupant. The occupancy can be held by
another in his name." Without occupancy, there is no possession.

Two things are paramount in possession. First, there must be occupancy, apprehension or
taking. Second, there must be intent to possess (animus possidendi).

In the current case, the petitioners contentions fail on three grounds. First, they never truly
occupied the land as a lower court ruling categorically mentioned that they were never in
possession of the property and that the tax declarations for the two years prior the return of the
respondent were made in the name of the latter. Further, the land was in the possession of the
son of the respondent. Second, the petitioners acknowledged the fact that the son of the
respondent and the wife thereof were tenants of the respondent. As a mere tenant, the son had
no right at all to surrender the property in question.

Most importantly, the title of the land in question remained in the name of respondent. "As the
registered owner, petitioner had a right to the possession of the property, which is one of the
attributes of ownership." The Civil Code states that possession as a fact cannot be recognized
at the same time in two different personalities except in the cases of co-possession. Should a
question arise regarding the fact of possession, the present possessor shall be preferred; if
there are two possessors, the one longer in possession; if the dates of the possession are the
same, the one who presents a title; and if all these conditions are equal, the thing shall be
placed in judicial deposit pending determination of its possession or ownership through proper
proceedings.

RSDM
De Jesus v. CA
217 SCRA 307

DOCTRINE: Under the present Civil Code, the prescriptive period required for acquisition of
immovable property is ten years if the possession is in good faith, and thirty years if in bad faith

FACTS:
The property in dispute is a parcel of residential land situated in Dampol 2nd, Pulilan, Bulacan,
bounded on the North by a Vereda: on the South, by the Provincial Road; on the East, by
Catalino Tayag (Tayao); on the West, by Macario de Leon, containing an area of 2565 square
meters and covered by Tax Declaration in the name of Victoriano Felipe

Private respondent executed a sworn statement declaring herself the only heir of the deceased
Victoriano Felipe and adjudicating to herself the ownership of the land in question.

More than twelve years later, petitioners herein filed in the Court of First Instance, an action for
recovery of ownership and possession and quieting of title to the abovementioned piece of land
covered by Tax Declaration, alleging among others: "that their grandfather, Santiago de Jesus
during his lifetime owned the residential lot; that Santiago de Jesus died before the outbreak of
World War II, leaving three (3) sons, namely: Mariano, Exequiel, and Jose, all surnamed de
Jesus; that Mariano de Jesus died on September 3, 1956 leaving eight (8) surviving children,
namely: Edgardo, Remedios, Juanita, Juliano, Jose, Flordeliza, Reynaldo, and Ernesto, all
surnamed de Jesus and all of them plaintiffs; that Exequiel de Jesus died on April 3, 1948,
survived by two (2) children Priscilo and Corazon, both surnamed de Jesus, also plaintiffs in
this case; while Jose de Jesus died before the outbreak of World War II without any issue.
CFI found for the plaintiffs. The Court of Appeals set aside the judgment of the trial court in a
decision.

ISSUE:
Whether or not the petitioner has the right to the ownership and possession of the residential lot.
-- YES

HELD:
Yes. The petitioner has the rigth to the ownership and possesion of the residential lot.

Private respondent's pretensions to acquisitive prescription may not succeed even under Act No.
190, the Code of Civil Procedure. Under Section 41 thereof, good faith and just title are not
required for purposes of acquisitive prescription; adverse possession in either character ripens
into ownership after the lapse of ten years. The just title required for acquisitive prescription to
set in is not "titulo verdadero y valido" such title which by itself is sufficient to transfer
ownership without the necessity of letting the prescriptive period elapse, but only "titulo
colorado" or such title where, although there was a mode of transferring ownership, still
something is wrong because the grantor is not the owner, and incidentally, it may perhaps be
mentioned that prescription running even after the effectivity of the New Civil Code on August
30, 1950, continued to be governed by Section 41 of the Old Civil Code.

Under the present Civil Code, the prescriptive period required for acquisition of immovable
property is ten years if the possession is in good faith, and thirty years if in bad faith. Such open,
continuous, exclusive and notorious occupation of the disputed property for thirty years must be
conclusively established.

Reckoned from the time she executed the affidavit of adjudication in 1961, eleven years after
the New Civil Code had taken effect, private respondent's possession of the contested lot is far
too short of the prescriptive period of thirty years considering that her possession is in bad faith.
The filing of the petition for recovery of ownership and possession and quieting of title by
petitioners on April 27, 1973 was well below the acquisitive prescriptive period for private
respondent, which is thirty years under Article 1141 of the present Civil Code. In this case, the
statutory period of prescription is deemed to have commenced when petitioners were made
aware of a claim adverse to them, that is, when the affidavit of adjudication was duly registered
with the Registry of Deeds which, at the earliest may be considered to be in 1974, when private
respondent was able to secure a tax declaration in her name.

MRAM
SMPSM v. BCDA
G.R. No. 142255

DOCTRINE: Prescription does not apply if the subject land is covered by a Torrens Title, as in
the case at bar. Moreover, prescription does not run against the State.

FACTS:
As a background, in 1992, RA 7227 created the BCDA to "accelerate the sound and balanced
conversion into alternative productive uses of the Clark and Subic military reservations and their
extensions," and "to raise funds by the sale of portions of Metro Manila military camps."
Pursuant to this Act, Pres. Ramos issued EO No. 40, series of 1992, specifying, among others,
the portions of Metro Manila military camps to be utilized to generate capital for the BCDA.
Among these Metro Manila military camps is Fort Bonifacio, located in the City of Makati and the
Municipality of Taguig. Under EO No. 40, 214 hectares in Fort Bonifacio were earmarked for
development and disposition to raise funds for BCDA projects and to use such funds to
accelerate the sound and balanced conversion into alternative productive uses of the Clark and
Subic military reservations and their extensions.

The members of SMPMI, allegedly comprising over 20,000 families, are residents of Fort
Bonifacio occupying a portion of it specifically Lot 4, Lot 3, and Lot 1 with an aggregate area of
97.58 hectares allegedly covered by SWO-00-001265 in the name of BCDA. Petitioner
maintains that its members have been occupying peacefully and continuously these lots in Fort
Bonifacio. It alleges that Fort Bonifacio is covered by the TCT in the name of the United States
of America, hence the Philippine Government.It further alleges that BCDA, pursuant to RA 7227,
the Municipality of Taguig, through its Mayor, sent 30-day notices of eviction to its members. It
asserts the illegality of the imminent eviction, for which the present action was filed, as the land
which petitioners members are occupying is still owned by the USA and not by the Philippine
Government.

It further asserts that Section 8

of RA 7227, which stipulates the area of Fort Bonifacio
specifically covering 2,276 hectares, did not provide any technical description on what is indeed
covered. Besides, it strongly argues that because of the lack of "tie line" locating the exact
position claimed by BCDA, the latter cannot illegally stake its claim on the whole of Fort
Bonifacio to the prejudice not only of its members but also of all persons or entities occupying
said area. Petitioner also contends that what complicates the controversy is the approval of the
BCDA plan by the Bureau of Land without due certification from the Land Registration Authority.

ISSUE:
Who between petitioner SMPMI and BCDA has the right of possession over the particular
parcels of land which are subject of this petition.

HELD:
The Court ruled in favor of BCDA for the ff. reasons:

1. The Philippine Government, and now the BCDA, has title and ownership
over Fort Bonifacio.
2. BCDA has convincingly shown that the in the name of the USA covering
Fort Bonifacio was cancelled by TCT No. 61524 issued in 1958 in the name of the Republic
of the Philippines. In 1995, the TCT was transferred in the name of BCDA. Thus, BCDA has
valid titles over Fort Bonifacio which have become indefeasible and beyond question. On the
other hand, SMPMI has not presented any title or deed to demonstrate ownership or any
interest in the subject lots.
3. Third, it is clear from the records that BCDA has been granted a clear
mandate by RA 7227, specifically by its Sections 7 and 8, and re-enforced by EO No. 40,
series of 1992, to take over and administer Fort Bonifacio for its development and
disposition to raise funds for BCDA projects, among others, the conversion of Clark and
Subic military reservations and their extensions to alternative productive uses.
4. It is basic that ownership or dominion includes the right of possession. In
traditional Roman law, jus possidendi or the right to possess is fundamentally not only an
attribute of ownership but also a direct consequence of ownership. Thus, from BCDAs
ownership of the subject lots originates the rights of possession, use, and disposition
5. Prescription does not apply if the subject land is covered by a Torrens
Title, as in the case at bar.
6. Neither prescription nor laches runs against the State. Thus, even
granting arguendo that the subject lands had been erroneously issued titles in favor of third
parties, which is definitely not the case; neither prescription nor estoppel by laches applies
against the State. In a catena of cases, we have consistently reiterated this hornbook
doctrine. In Reyes v. Court of Appeals, it was held that:
In so far as the timeliness of the action of the Government is concerned, it is basic
that prescription does not run against the State. x x x The case law has also been:
When the government is the real party in interest, and is proceeding mainly
to assert its own rights and recover its own property, there can be no defense
on the ground of laches or limitation. x x x

Public land fraudulently included in patents or certificates of title may be
recovered or reverted to the State in accordance with Section 101 of the
Public Land Act. Prescription does not lie against the State in such cases for
the Statute of Limitations does not run against the State. The right of
reversion or reconveyance to the State is not barred by prescription.

FMM
Heirs of Soriano v. CA
363 SCRA 87

DOCTRINE: Possession and ownership are distinct legal concepts. There is ownership when
a thing pertaining to one person is complete does not own it and cannot evict themly subjected
to his will in a manner not prohibited by law and consistent with the rights of others. Ownership
confers certain rights to the owner, among which are the right to enjoy the thing owned and the
right to exclude other persons from possession thereof. On the other hand, possession is
defined as the holding of a thing or the enjoyment of a right. Literally, to possess means to
actually and physically occupy a thing with or without right. Possession may be had in one of
two ways: possession in the concept of an owner and possession of a holder.

FACTS:
On 30 June 1967, the heirs of Adrian Soriano, who died intestate in 1947, leased the property to
spouses David de Vera and Consuelo Villasista for a period of fifteen (15) years beginning 1
July 1967.

The contract of lease provided that Roman Soriano (Soriano), one of the children of the late
Adriano, will be the caretaker of the property during the period of the lease.

During the effectivity of the lease contract, the heirs of Adriano Soriano entered into an
extrajudicial settlement of his estate. The property subject of this case was adjudicated to seven
(7) of his rune (9) children pro-indiviso.

On 11 January 1968, the property was divided into two (2) lots, Lot No. 60052 and Lot No. 8459.
The former lot was assigned to Lourdes, Candido and the heirs of Dionisia while the latter lot
was assigned to Francisco, Librada, Elcocadio and Soriano. The new owners of Lot No. 60052
sold the portions assigned to them to spouses Braulio and Aquilina Abalos. Likewise, the new
owners of Lot 8459, except Soriano, sold their shares to the Spouses Abalos.

On 14 March 1968, On March 14, 1968, the Spouses de Vera ousted Soriano as caretaker and
appointed Isidro Versoza and Vidal Versoza as his substitutes. Thereafter, Soriano filed a case
for reinstatement and reliquidation against the Spouses de Vera.

On 30 September 1969, the Agrarian Court rendered a decision authorizing the ejectment of
Roman. On appeal, the decision was reversed by the Court of Appeals (CA). However, before
it was executed, the parties entered into a post-decisional agreement wherein the spouses de
Vera allowed Soriano to sub-lease the property until the termination of the original lease on 30
June 1982. This agreement was approved by the court in an order dated 22 December 1972.

On 16 August 1976, the Spouses Abalos filed with the then Court of First Instance (CFI) of
Pangasinan at Lingayen an application for registration of title. The application claimed
ownership of the entire lot No. 60052 and 3/4 pro-indiviso of Lot No. 8459. The Director of
Lands (Republic) and Soriano filed separate oppositions to the application. The latters
opposition alleged that the two (2) lots subject of the application have not yet been subdivided
and remained as one parcel; that he is the co-owner pro-indiviso of the combined area of the
two (2) lots and not just to one-fourth (1/4) of Lot No. 8459 as alleged in the application; and that
the applicants source of ownership is voidable. The Republic subsequently conceded that the
land applied for was private and disposable. The RTC, acting as a Land Registration Court,
granted the application for Registration.

On 13 April 1983, after the expiration of the original lease and the sub-lease in favor of Soriano,
the Spouses Abalos filed a case for unlawful detainer against Soriano. This case, however, was
dismissed on motion of the complainants, Spouses Abalos.

For their part, Elcocadio, Librada, Soriano, Francisco, Lourdes, Candido and the heirs of
Dionisia, filed a complaint to annul the deeds of sale they executed in favor of the Spouses
Abalos or should the deeds be not annulled, to allow Soriano, Elcocadio and Librada to redeem
those shares sold by Candido, Lourdes, Francisca and the heirs of Dionisia and to uphold
Sorianos possession of the fishpond portion of the property as a tenant-caretaker. After the
dismissal of the case for unlawful detainer, the Abalos spouses a motion for execution of the
post-decisional order embodying the agreement of Soriano and the de Vera spouses allowing
the former to sublease the property.

In the mean time, Soriano died on 11 December 1985. Thus, the complaint in the civil case for
annulment of document and/or redemption, ownership and damages, was amended to
substitute Sorianos heirs.

Several decisions by the Supreme Court (SC) were rendered but the issue on possession
remains challenged.

ISSUE:
Whether an owner is also entitled to possession. -- NO

HELD:
The Supreme Court held that possession and ownership are distinct legal concepts. There is
ownership when a thing pertaining to one person is completely subjected to his will in a manner
not prohibited by law and consistent with the rights of others. Ownership confers certain rights
to the owner, among which are the right to enjoy the thing owned and the right to exclude other
persons from possession thereof. On the other hand, possession is defined as the holding of a
thing or the enjoyment of a right. Literally, to possess means to actually and physically occupy
a thing with or without right. Possession may be had in one of two ways: possession in the
concept of an owner and possession of a holder. A person may be declared owner but he may
not be entitled to possession. The possession may be in the hands of another either as a
lessee or a tenant. The SC also held that the exercise of their rights of ownership are subject to
limitations that may be imposed by law.

RGGM
State Investment House v. CA
254 SCRA 368

DOCTRINE: A mortgagee who has foreclosed property is not considered in good faith when
such mortgagee has or is expected to have knowledge of any defect in the title; a prior buyer in
good faith, although merely under a contract to sell, is preferred over a mortgagee since if the
original owner delivered title, he would not anymore be able to mortgage the thing.

FACTS:
A contract to sell was executed between spouses Canuto and Oreta, and Solid Homes. The
sale involved a parcel of land (511 sq. m.). Upon signing of the contract, Oreta made payment
with the agreement that the balance shall be paid in installments. Meanwhile. Solid Homes
executed several mortgages in favor of State Investment over its subdivided parcels of land,
including the subject of land of the mentioned contract to sell. Such mortgage was foreclosed
upon failure of Solid to comply with its obligations. Thereafter, Solid through a MOA, negotiated
for the deferment of consolidation of ownership over the foreclosed properties. It further
committed itself to redeem the properties.

Spouses Canuto after a few years filed a complaint before the HLURB against Solid and State
for failure on the part of Solid to execute the necessary absolute deed of sale as well as to
deliver title to property subject of the contract to sell despite full payment. Solid alleged that its
obligations under the contract have become so difficult for performance. Solid , in effect, asked
to be partially released from its obligations by delivering another parcel of land in substitution to
the subject of the sale. State, on the other hand averred that unless Solid pays the redemption
price, it has a right to hold on to the foreclosed properties. However, HLURB ordered State to
execute a deed of conveyance in favor of complainants and deliver the title to the land. Solid
was then ordered to pay State the portion of the loan which corresponds to the value of the lot.
this judgment was sustained by the Board of Commissioners, Office of the President, and Court
of Appeals.

ISSUES:
1. W/N spouses Oreta's unregistered rights are superior over State's registered mortgage
over the property
2. W/N State has the right to rely on the face of the Torrens title

HELD:
1. State's registered mortgage right over the property is inferior to that of respondents'
unregistered right. The unrecorded sale is preferred for the reason that if the original
owner (Solid) had parted with the ownership of the thing sold, he would no longer have the
free disposal of it and would not be able to mortgage it. Registration of the mortgage is not
important since it is understood to be without prejudice to the rights of third persons.

2. As a general rule, where there is nothing in the title to indicate any cloud or vice in the
ownership thereof, the purchaser is not required to explore further. An exception to this is
when the mortgagee or purchaser has knowledge of a defect or lack of title on the part of
the vendor or that he was aware of sufficient facts to induce a reasonably prudent man to
inquire further. In this case, petitioner knows full well that Solid is engaged in selling
subdivision lots. Therefore, as founded on jurisprudence, it should have taken necessary
precautions to ascertain any flaw. Moreover, the uniform practice of financing institutions is
to investigate, examine, and assess real property offered as security. State is therefore not
a mortgagee in good faith.

MCSS
DBP v. CA
316 SCRA 650

FACTS:
DBP bought 91,188.30 square meters of land, consisting of 159 lots, in the proposed Diliman
Estate Subdivision of the PHHC. However, the sale of the lots to DBP, Lots 2 and 4, which form
part of said 159 lots, were still sold by PHHC to the spouses Nicandro, for which 2 deeds of sale
were issued to them by PHHC. Upon learning of PHHCs previous transaction with DBP, the
spouses filed a complaint against DBP and the PHHC to rescind the sale of Lots 2 and 4 by
PHHC in favor of DBP. The CFI held that the sale of Lots 2 and 4, to DBP is null and void, for
being in violation of Section 13 of the DBP Charter.

ISSUE:
WON spouses possess the legal personality to question the legality of the sale? -- YES

HELD:
The spouses stand to be prejudiced by reason of their payment in full of the purchase price for
the same lots which had been sold to DBP by virtue of the transaction in question.The general
rule is that the action for the annulment of contracts can only be maintained by those who are
bound either principally or subsidiarily by virtue thereof. However, a person who is not obliged
principally or subsidiarily in a contract may exercise an action for nullity of the contract if he is
prejudiced in his rights with respect to one of the contracting parties, and can show the
detriment which could positively result to him from the contract in which he had no intervention.

NKVS
San Miguel Corporation v. CA
185 SCRA 727

DOCTRINE: Tax declarations and receipts are not conclusive evidence of ownership or right of
possession over a piece of land. They are merely indicia of a claim of ownership.

FACTS:
San Miguel Corporation purchased from Silverio Perez a parcel of land located in Sto. Tomas,
Batangas. Claiming ownership in fee simple of the land, SMC filed before the then CFI, now
RTC of Batangas an application for its registration under the Land Registration Act.

The Solicitor General, opposed the application for registration contending that SMC's claim of
ownership, that the parcel of land in question is part of the public domain, and that SMC, being
a private corporation, is disqualified under Section 11, Article XIV of the Constitution from
holding alienable lands of the public domain. The Solicitor General thereafter authorized the
Provincial Fiscal of Batangas to appear in said case, subject to his supervision and control.

Petitioner's claim that its predecessor-in-interest had open, exclusive and undisputed
possession of Lot 684 for more than thirty years is anchored on certain documentary and
testimonial evidence. Its documentary evidence consist of tax declaration No. 923 wherein it
appears that in 1974, Silverio Perez declared as his own for taxation purposes, a certain
riceland with an area of 1.5657 hectares located in Sta. Anastacia, Sto. Tomas, Batangas, and
a certification of the Office of the Treasurer of Sto. Tomas to the effect that in 1977, Silverio
Perez paid realty taxes for the land subject of tax declaration no. 923.

ISSUE: W/N Tax declarations are evidence of ownership. -- NO

HELD:
NO, it is not a conclusive evidence of ownership or right of possession of a piece of land.

Tax declarations and receipts are not conclusive evidence of ownership or right of possession
over a piece of land. They are merely indicia of a claim of ownership. Tax declarations only
become strong evidence of ownership of land acquired by prescription, a mode of acquisition of
ownership relied upon by petitioner in this case, when accompanied by proof of actual
possession.

Such proof of actual possession was sought to be provided by the testimony of vendor Silverio
Perez that he had been in possession of the property since 1933 until he sold it to SMC in 1975;
that the property was given to him by his parents when he got married; that no document
evidenced that transfer; that it had been in the possession of his parents since 1925; that he
had declared the property in his name for taxation purposes; that he had paid taxes therefor,
and that he was in peaceful, continuous and exclusive possession of the property until its sale to
SMC.

Petitioner did not present other witnesses to corroborate Perez' testimony. Its other witness,
Antonio M. de las Alas, Jr., a lawyer of the petitioner, simply testified that he handled the
negotiations for the purchase of the property; that SMC was authorized to own and acquire
property as shown by its articles of incorporation and by-laws; that since its acquisition in 1975,
the property had been used as a hatchery farm of SMC; that SMC's possession in the concept
of an owner had been continuous, adverse and against the whole world, and that the land was
declared for taxation purposes still in the name of Silverio Perez .

We hold that there is paucity of evidence of actual, notorious and exclusive possession of the
property on the part of vendor Silverio Perez so as to attach to it the character of an express
grant from the govemment. Indeed, as correctly held by the Court of Appeals, Silverio Perez's
testimony, being uncorroborated, is simply self-serving and hence, undeserving of any weight.

AMPS
Equatorial Realty Development v. Mayfair Theatre
370 SCRA 56

DOCTRINE: Under Article 1380 to 1381(3) of the Civil Code, a contract otherwise valid may be
rescinded by reason of injury to third persons, like creditors. Nonetheless, acquisition by a
third person of the property subject of the contract is an obstacle to the action for its rescission
where it is shown that such third person is in lawful possession of the subject of the contract and
that he is in good faith.

FACTS:
Carmelo Inc. (Lessor) and Mayfair entered into two contracts of lease over certain portions of a
two-storey building. Both contracts contained the following stipulation:

That if the LESSOR should desire to sell the leased premises, the LESSEE shall
be given 30-days exclusive option to purchase the same.

In the event, however, that the leased premises is sold to someone other than
the LESSEE, the LESSOR is bound and obligated, as it hereby binds and
obligates itself, to stipulate in the Deed of Sale hereof that the purchaser shall
recognize this lease and be bound by all the terms and conditions thereof.

Later on, Carmelo informed Mayfair that a certain Jose Araneta was offering to buy the whole
property for US Dollars 1,200,000. Carmelo asked Mayfair if it was willing to buy the property
for Six to Seven Million Pesos.

Mayfair sent letter to Carmelo purporting to express interest in acquiring not only the leased
premises but "the entire building and other improvements if the price is reasonable. However,
Carmelo proceeded with the sale of the whole property which included the leased premises to
Equatorial. Hence, Mayfair instituted the action a quo for specific performance and annulment
of the sale of the leased premises to Equatorial.

ISSUE:
Is the contract of sale between Carmelo (lessor) and Equatorial still rescissible in spite of the
possession of Equatorial?

HELD:
Still rescissible as the exception does not apply to Equatorial not third party to the rescissible
contract and possession was in bad faith.

What Carmelo and Mayfair agreed to, by executing the two lease contracts, was that Mayfair
will have the right of first refusal in the event Carmelo sells the leased premises. It is undisputed
that Carmelo did recognize this right of Mayfair, for it informed the latter of its intention to sell the
said property in 1974. There was an exchange of letters evidencing the offer and counter-offers
made by both parties. Carmelo, however, did not pursue the exercise to its logical end. While it
initially recognized Mayfair's right of first refusal, Carmelo violated such right when without
affording its negotiations with Mayfair the full process to ripen to at least an interface of a
definite offer and a possible corresponding acceptance within the "30-day exclusive option" time
granted Mayfair, Carmelo abandoned negotiations, kept a low profile for some time, and then
sold, without prior notice to Mayfair, the entire Claro M Recto property to Equatorial.

Since Equatorial is a buyer in bad faith, this finding renders the sale to it of the property in
question rescissible. We agree with respondent Appellate Court that the records bear out the
fact that Equatorial was aware of the lease contracts because its lawyers had, prior to the sale,
studied the said contracts. As such, Equatorial cannot tenably claim to be a purchaser in good
faith, and, therefore, rescission lies.

Contract of Sale was not voidable but rescissible. Under Article 1380 to 1381(3) of the Civil
Code, a contract otherwise valid may nonetheless be subsequently rescinded by reason of
injury to third persons, like creditors.

According to Tolentino, rescission is a remedy granted by law to the contracting parties and
even to third persons, to secure reparation for damages caused to them by a contract, even if
this should be valid, by means of the restoration of things to their condition at the moment prior
to the celebration of said contract. It is a relief allowed for the protection of one of the
contracting parties and even third persons from all injury and damage the contract may cause,
or to protect some incompatible and preferent right created by the contract. Rescission implies a
contract which, even if initially valid, produces a lesion or pecuniary damage to someone that
justifies its invalidation for reasons of equity.

It is true that the acquisition by a third person of the property subject of the contract is an
obstacle to the action for its rescission where it is shown that such third person is in lawful
possession of the subject of the contract and that he did not act in bad faith. However, this rule
is not applicable in the case before us because the petitioner is not considered a third party in
relation to the Contract of Sale nor may its possession of the subject property be regarded as
acquired lawfully and in good faith

KGS
Maglucot v. Maglucot
329 SCRA 78

DOCTRINES:
1. Partition; The first phase of a partition and/or accounting suit is taken up with the
determination of whether or not a co-ownership in fact exists, (i.e. not otherwise legally
proscribed) and may be made by voluntary agreement of all the parties interested in the
property.

2. Partition; The second phase commences when it appears that the parties are unable to
agree upon the partition directed by the court. In that event, partition shall be done for
the parties by the court with the assistance of not more than three (3) commissioners.

3. Under the present rule, the proceeding of the commissioners without being confirmed by
the court are not binding upon the parties.

4. Parties to a partition proceeding, who elected to take under partition, and who took
possession of the portion allotted to them, are estopped to question title to portion
allotted to another party.

5. In cases involving oral partition under which the parties went into possession, exercised
acts of ownership, or otherwise partly performed the partition agreement, equity will
confirm such partition and in a proper case decree title in accordance with the
possession in severalty.

FACTS:
Lot 1639 was registered under the names of six persons. Thereafter, one of the co-owners and
his predecessor-in-interest (Tomas) filed a petition to subdivide the lot, which was granted. As
such, said lot was physically subdivided into A-F. Lot D (subject lot) was given to Roberto
Maglucot. Portion of it was rented to Guillermo Maglucot; and subsequently, to Leopoldo and
Severo Maglucot (respondents). The respondents built houses on their corresponding leased
lots. After years, they stopped paying rentals as they are claiming ownership over the subject lot.
Then, Tomas and other petitioners filed a case in the RTC for recovery of the possession for Lot
D. They claimed that no partition took place, as it was not annotated in the title. The RTC ruled
that there was partition; and ordered respondents to demolish the houses in Lot D and vacate
the premises thereof. The CA reversed the decision and ruled that the sketch plan and tax
declarations relied upon by petitioners are not conclusive evidence of partition.

ISSUE:
Was the oral and physical partition of Lot 1639 valid considering it was not annotated in the
title? -- YES

HELD:
An action for partition is comprised of two phases: first, an order for partition which determines
whether a co-ownership in fact exists, and whether partition is proper; and, second, a decision
confirming the sketch or subdivision submitted by the parties or the commissioners appointed by
the court, as the case may be. The first phase of a partition and/or accounting suit is taken up
with the determination of whether or not a co-ownership in fact exists, (i.e., not otherwise legally
proscribed) and may be made by voluntary agreement of all the parties interested in the
property. This phase may end with a declaration that plaintiff is not entitled to have a partition
either because a co-ownership does not exist, or partition is legally prohibited. It may end, upon
the other hand, with an adjudgment that a co-ownership does in truth exist, partition is proper in
the premises and an accounting of rents and profits received by the defendant from the real
estate in question is in order. In the latter case, the parties may, if they are able to agree, make
partition among themselves by proper instruments of conveyance, and the court shall confirm
the partition so agreed upon. In either case i.e., either the action is dismissed or partition
and/or accounting is decreed the order is a final one, and may be appealed by any party
aggrieved thereby. The second phase commences when it appears that "the parties are unable
to agree upon the partition" directed by the court. In that event, partition shall be done for the
parties by the court with the assistance of not more than three (3) commissioners. This second
stage may well also deal with the rendition of the accounting itself and its approval by the court
after the parties have been accorded opportunity to be heard thereon, and an award for the
recovery by the party or parties thereto entitled of their just share in the rents and profits of the
real estate in question. Such an order is, to be sure, final and appealable.

However, this Court notes that the order of partition was issued when the ruling in Fuentebella
vs. Carrascoso, which held that the order of partition is interlocutory, was controlling. In addition,
the reports of the commissioners not having been confirmed by the trial court are not binding. In
this case, both the order of partition and the unconfirmed sketch plan are, thus, interlocutory.
Nevertheless, where parties do not object to the interlocutory decree, but show by their conduct
that they have assented thereto, they cannot thereafter question the decree, especially, where,
by reason of their conduct, considerable expense has been incurred in the execution of the
commission. Respondents in this case have occupied their respective lots in accordance with
the sketch/subdivision plan. They cannot after acquiescing to the order for more than forty (40)
years be allowed to question the binding effect thereof.

This case is to be distinguished from the order in the action for partition in Arcenas vs. Cinco. In
that case, the order was clearly interlocutory since it required the parties " to submit the
corresponding deed of partition to the Court for its approval." Here, the order appointed two
commissioners and directed them merely to approve the sketch plan already existing and
tentatively followed by the parties.

Under the present rule, the proceedings of the commissioners without being confirmed by the
court are not binding upon the parties. However, this rule does not apply in case where the
parties themselves actualized the supposedly unconfirmed sketch/subdivision plan. The
purpose of court approval is to give effect to the sketch/subdivision plan. In this case, the parties
themselves or through their predecessors-in-interest implemented the sketch plan made
pursuant to a court order for partition by actually occupying specific portions of Lot No. 1639 in
1952 and continue to do so until the present until this case was filed, clearly, the purpose of the
court approval has been met. This statement is not to be taken to mean that confirmation of the
commissioners may be dispensed with but only that the parties herein are estopped from raising
this question by their own acts of ratification of the supposedly non-binding sketch/subdivision
plan.

The records of the case show that sometime in 1946 there was a prior oral agreement to
tentatively partition Lot No. 1639. By virtue of this agreement, the original co-owners occupied
specific portions of Lot No. 1639. It was only in 1952 when the petition to subdivide Lot No.
1639 was filed because two of the co-owners, namely Hermogenes Olis and heirs of Pascual
Olis, refused to have said lot subdivided and have separate certificates of title. Significantly,
after the 1952 proceedings, the parties in this case by themselves and/or through their
predecessors-in-interest occupied specific portions of Lot No. 1639 in accordance with the
sketch plan. Such possession remained so until this case arose, or about forty (40) years later.

From its order in 1952, it can be gleaned that the CFI took notice of the tentative subdivision
plan by oral partition of the parties therein. Further, it appears that said court was aware that the
parties therein actually took possession of the portions in accordance with the
sketch/subdivision plan. With this factual backdrop, said court ordered the partition and
appointed two (2) commissioners to approve the tentative sketch/subdivision plan. It would not
be unreasonable to presume that the parties therein, having occupied specific portions of Lot No.
1639 in accordance with the sketch/subdivision plan, were aware that it was that same
sketch/subdivision plan which would be considered by the commissioners for approval. There is
no showing that respondents by themselves or through their predecessors-in-interest raised any
objections. On the contrary, the records show that the parties continued their possession of the
specific portions of Lot No. 1639 pursuant to the sketch/subdivision plan.

It has been previously held that a co-owner, who, though not a party to a partition accepts the
partition allotted to him, and holds and conveys the same in severalty, will not be subsequently
permitted to avoid partition. It follows that a party to a partition is also barred from avoiding
partition when he has received and held a portion of the subdivided land especially in this case
where respondents have enjoyed ownership rights over their share for a long time.

Parties to a partition proceeding, who elected to take under partition, and who took possession
of the portion allotted to them, are estopped to question title to portion allotted to another party.
A person cannot claim both under and against the same instrument. In other words, they
accepted the lands awarded them by its provisions, and they cannot accept the decree in part,
and repudiate it in part. They must accept all or none. Parties who had received the property
assigned to them are precluded from subsequently attacking its validity of any part of it.[38]
Here, respondents, by themselves and/or through their predecessors-in-interest, already
occupied of the lots in accordance with the sketch plan. This occupation continued until this
action was filed. They cannot now be heard to question the possession and ownership of the
other co-owners who took exclusive possession of Lot 1639-D also in accordance with the
sketch plan.

Partition may be inferred from circumstances sufficiently strong to support the presumption.
Thus, after a long possession in severalty, a deed of partition may be presumed. It has been
held that recitals in deeds, possession and occupation of land, improvements made thereon for
a long series of years, and acquiescence for 60 years, furnish sufficient evidence that there was
an actual partition of land either by deed or by proceedings in the probate court, which had been
lost and were not recorded. And where a tract of land held in common has been subdivided into
lots, and one of the lots has long been known and called by the name of one of the tenants in
common, and there is no evidence of any subsequent claim of a tenancy in common, it may
fairly be inferred that there has been a partition and that such lot was set off to him whose name
it bears.

It must be noted that there was a prior oral partition in 1946. Although the oral agreement was
merely tentative, the facts subsequent thereto all point to the confirmation of said oral partition.
By virtue of that agreement, the parties took possession of specific portions of the subject lot.
The action for partition was instituted because some of the co-owners refused to have separate
titles issued in lieu of the original title. In 1952, an order for partition was issued by the cadastral
court. There is no evidence that there has been any change in the possession of the parties.
The only significant fact subsequent to the issuance of the order of partition in 1952 is that
respondents rented portions of Lot No. 1639-D. It would be safe to conclude, therefore, that the
oral partition as well as the order of partition in 1952 were the bases for the finding of actual
partition among the parties. The legal consequences of the order of partition in 1952 having
been discussed separately, we now deal with oral partition in 1946. Given that the oral partition
was initially tentative, the actual possession of specific portions of Lot No. 1639 in accordance
with the oral partition and the continuation of such possession for a very long period indicate the
permanency and ratification of such oral partition. The validity of an oral partition is already well-
settled. In Espina vs. Abaya, we declared that an oral partition is valid. In Hernandez vs. Andal,
reiterated in Tan vs. Lim, this Court has ruled, thus: on general principle, independent and in
spite of the statute of frauds, courts of equity have enforce oral partition when it has been
completely or partly performed. Regardless of whether a parol partition or agreement to partition
is valid and enforceable at law, equity will proper cases where the parol partition has actually
been consummated by the taking of possession in severalty and the exercise of ownership by
the parties of the respective portions set off to each, recognize and enforce such parol partition
and the rights of the parties thereunder. Thus, it has been held or stated in a number of cases
involving an oral partition under which the parties went into possession, exercised acts of
ownership, or otherwise partly performed the partition agreement, that equity will confirm such
partition and in a proper case decree title in accordance with the possession in severalty.

In numerous cases it has been held or stated that parol partition may be sustained on the
ground of estoppel of the parties to assert the rights of a tenant in common as to parts of land
divided by parol partition as to which possession in severalty was taken and acts of individual
ownership were exercised. And a court of equity will recognize the agreement and decree it to
be valid and effectual for the purpose of concluding the right of the parties as between each
other to hold their respective parts in severalty.

A parol partition may also be sustained on the ground that the parties thereto have acquiesced
in and ratified the partition by taking possession in severalty, exercising acts of ownership with
respect thereto, or otherwise recognizing the existence of the partition. A number of cases have
specifically applied the doctrine of part performance, or have stated that a part performance is
necessary, to take a parol partition out of the operation of the statute of frauds. It has been held
that where there was a partition in fact between tenants in common, and a part performance, a
court of equity would have regard to enforce such partition agreed to by the parties.

JPOT
Gabrito v. CA
167 SCRA 771

DOCTRINE: Pending final adjudication of ownership by the Bureau of Lands, the Court has
jurisdiction to determine in the meantime the right of possession over the land.

FACTS:
The spouses Roberto Tan and Benita Ching-Tan filed a complaint in the Municipal Trial Court
against defendants Maximo Gabrito, et al., alleging that they are the possessors and legal
owners of the property situated at No. 107 Gordon Ave., New Kalalake, Olongapo City as
evidenced by Tax Declaration No. 4-2046. The defendants are leasing portions of this parcel of
land, each paying the corresponding monthly rentals due thereon. The Tans found it fit for them
to make said lot a residential house for them instead. They furnished requests to Gabrito and
others stating their reasons and three months later they (Gabrito, et al.) were still being stubborn
in keeping with the operations of their commercial spaces. On November 22, 1985, the
Municipal Trial Court ruled and the dispositive portion of which reads:

WHEREFORE, judgment is hereby rendered for all the defendants to vacate the parcel
of land described in par. 3 of the complaint, removing therefrom the buildings and any
other improvements respectively owned by them; and to pay plaintiffs the following as
reasonable compensation for the use of the premises:
Maximo Gabrito-aT P250.00 per month from April 1984 until he vacates the premises;
Roger Libut-at P150.00 per month from May 1984 until he vacates the premises;
Liza de Vera-at P150.00 per month from April 1984, until she vacates the premises;
Carmelita Uy-at Pl 70.00 per month from April 1984, until she vacates the premises. for
all defendants to pay, in equal shares, damages by way of attorney's fees in the amount
of ONE THOUSAND PESOS ( P1,000.00 ) as well as costs SO ORDERED. (Rollo, p.
35).

Gabrito, et al., found the findings contrary and elevated the matter to the Regional Trial Court as
well as the Court of Appeals, leading up to the Supreme Court.

ISSUES:
1. W/N an action for unlawful detainer is the proper action to oust petitioners from their
occupation of the land in dispute?
2. Who has a better right to possess the land which definitely falls under the jurisdiction of
the Municipal Trial Court and which of the summary procedures may properly be
applied?

HELD:
In unlawful detainer, the defendant unlawfully withholds possession after the expiration or
termination of his right thereto under any contract, express or implied. In such a case, prior
physical possession is not required. Possession can also be acquired, not only by material
occupation, but also by the fact that a thing is subject to the action of one's will or by the proper
acts and legal formalities established for acquiring such right. Possession of land can be
acquired upon the execution of the deed of sale thereof by its vendor. Actual or physical
occupation is not always necessary. Although, there is still a dispute between the Bureau of
Lands and the courts, unlawful detainer is still applicable and is indeed the correct action.
Promulgated on March 28, 1988, the Court clearly stated that "pending final adjudication of
ownership by the Bureau of Lands, the Court has jurisdiction to determine in the meantime the
right of possession over the land." Corollary thereto, the power to order the sheriff to remove
improvements and turn over the possession of the land to the party adjudged entitled thereto,
belongs only to the courts of justice and not to the Bureau of Lands. In fact, the Bureau of Lands
in its decision of June 7, 1987, admitted the jurisdiction of the courts to decide the case on the
question of physical possession, although not on the question of ownership (Rollo, p. 179).

Under the circumstances, a careful study of the records failed to show any cogent
reason to disturb the findings of the Municipal Trial Court in Cities and of the Regional Trial
Court, both of Olongapo City, and finally of the Court of Appeals.
WHEREFORE, the decision of respondent Court of Appeals is AFFIRMED and the temporary
restraining order is lifted. Costs against petitioners.
SO ORDERED.


MLAV
Vda. De Borromeo v. Pogoy
126 SCRA 217

DOCTRINE: Prescription for actions involving forcible entry and unlawful detainer is one year.

The Barangay Lupon is required only where the parties thereto are "individuals" or a single
human being as contrasted with a social group or institution. It applies only to cases involving
natural persons, and not where any of the parties is a juridical person such as a corporation,
partnership, corporation sole, testate or intestate, estate, etc.

FACTS:
The intestate estate of the late Vito Borromeo is the owner of a building in Cebu City. Said
building has been leased and occupied by petitioner Petra Vda. de Borromeo at a monthly
rental of P500.00.

Private respondent Atty. Ricardo Reyes, administrator of the estate and a resident of Cebu City,
served upon petitioner a letter demanding that she pay the overdue rentals, and thereafter to
vacate the premises. When petitioner failed to comply, Atty. Reyes instituted an ejectment case
against petitioner.

Petitioner moves for the dismissal of the casce for want of jurisdiction as conciliation
proceedings with the Lupon Barangay should have first been instituted since they both live in
the same city. Respondent justified such omission through PD 1508, which allows the direct
filing of an action in court where there was danger of prescribing under the Statute of Limitations.

RTC dismissed the petition. Petitioner elevated the case to the Supreme Court via a petition for
certiorari.

ISSUES:
1. WON there was prescription -- NO
2. WON there was a need to go to the Lupon Barangay -- YES

HELD
Article 1147 of the Civil Code states that the period for filing actions for forcible entry and
detainer is one year, and this period is counted from demand to vacate the premises.

In the case at bar, the letter-demand was dated August 28, 1982, while the complaint for
ejectment was filed in court on September 16, 1982. There is at least eleven (11) full months of
the prescriptive period.

Under the procedure outlined in PD 1508, the time needed for the conciliation proceeding
before the Barangay Chairman and the Pangkat should take no more than 60 days. Thus, it
cannot be truthfully asserted that his case would be barred by the Statute of Limitations if he
had to course his action to the Barangay Lupon.

Be that as it may, the instant petition should be dismissed. Referral of a dispute to the Barangay
Lupon is required only where the parties thereto are "individuals" or a single human being as
contrasted with a social group or institution. It applies only to cases involving natural persons,
and not where any of the parties is a juridical person such as a corporation, partnership,
corporation sole, testate or intestate, estate, etc.

Atty. Ricardo Reyes is a mere nominal party who is suing in behalf of the Intestate Estate of Vito
Borromeo. The real party in interest is the intestate estate under administration and thus, it does
not fall within the individual description.

DJTV
Wilmon Auto Supply v. CA
208 SCRA 108

DOCTRINE: An ejectment suit cannot be suspended by an action filed in the RTC based on
tenants claim that his right of preemption (or prior purchase) was violated.

FACTS:
Wilmon Auto Supply Corporation, et al. (Wilmon) were lessees of a commercial building and
bodegas standing on a registered land owned in common by Lucy Solinap, et al. The leases
were embodied in uniformly worded deeds executed by the individual petitioners wherein one of
the clauses provided for a reservation of rightsthe lessor has the right to sell, mortgage,
hypothecate or encumber the property so long as it requires the purchaser or mortgage
creditors to respect the terms of the lease contract; provided further that lessee shall be duly
informed about lessors plan to sell the property.

After the expiration of the period fixed in the lease agreements, the premises were sold to Star
Group Resources and Development Inc. (Star Group). Thereafter, Star Group instituted an
action in the Municipal Trial Court (MTC) for unlawful detainer against Wilmon. Wilmon refused
to concede, and indeed impugned, Star Group's right to eject them. Wilmon then filed an action
with the Regional Trial Court (RTC) and argued that in so selling the properties and seeking
their ejectment therefrom, the lessors and their buyer had violated their leasehold rights
because (i) they (the lessees) were not accorded the right of preemption, (b) the buyer was not
required to honor the leases, and (c) the lessees were denied the option to renew their leases
upon the expiration thereof.

ISSUE:
Whether or not an action of unlawful detainer filed in the MTC against a lessee grounded on
the expiration of the latter's lease should be abated or suspended by an action filed in the
RTC by the defendant lessee on the claim that he is entitled to a right of preemption (or prior
purchase) of the premises in question and wishes to have said right judicially enforced. -- NO

HELD:
An ejectment suit cannot be suspended by an action filed in the RTC based on tenants claim
that his right of preemption (or prior purchase) was violated.

Actions in the RTC did not involve physical or de facto possession and, on not a few occasions,
that the case in the RTC was merely a ploy to delay disposition of the ejectment proceeding, or
that the issues presented in the former could quite as easily be set up as defenses in the
ejectment action and there resolved. This is specially true in the cases at bar, where the
petitioners-lessees' claims that the lessors (and the buyer of the leased premises) had
violated their leasehold rights because (a) they (the lessees) were not accorded the right of
preemption, (b) the buyer was not required to respect their leases, and (c) the lessees were
denied the option to renew their leases upon the expiration thereof constituted their causes of
action in the suits commenced by them in the Regional Court.

JGY
Semira v. CA
230 SCRA 577

DOCTRINE: The issue of ownership can be decided provisionally for the sole purpose of
resolving priority of possession and does not bar an action between the same parties involving
title to the land.

FACTS:
Gutierrez owned a parcel of land which she sold to private respondent Buenaventura by means
of a "Kasulatan ng Bilihan ng Lupa". Aside from the estimated area of 822.5 square meters
appearing in the deed of sale, the following boundaries of the lot are also stated. Thereafter,
private respondent entered the premises observing thereby the boundaries of the property and
not the area given. Subsequently, he acquired two other parcels of land from the spouses
Hornillas.

Respondent sold the first lot to his nephew, Cipriano by means of another "Kasulatan ng Bilihan
ng Lupa" where the lot was described with the same area and boundaries mentioned in the first
"Kasulatan ng Bilihan ng Lupa" with the exception of the boundary on the east. Cipriano sold the
lot to petitioner Semira. However, the area stated in the "Kasulatan ng Bilihan ng Lupa" 4 was
2,200 square meters and not 822.5 appearing in the previous document. As delimited by its
boundaries, the lot is actually much bigger than 822.5 square meters. This was confirmed by the
Taysan Cadastral Mapping Survey.

A complaint for forcible entry was filed against petitioner by respondent. The latter claimed that
the area of Lot 4221 was 822.5 square meters only and that the excess portion forcibly
occupied by petitioner formed part of a lot which he acquired from the Hornillas. MTC dismissed
the case since the issue of prior physical possession could not be resolved without first deciding
on the ownership, dismissal was proper since forcible entry cases involve the sole issue of prior
physical possession.

ISSUE:
WON the issue of ownership can be decided for the sole purpose of resolving priority of
possesion. -- YES

HELD:
In the case at bench the issue of possession cannot be decided independently of the question
of ownership. Where land is sold for a lump sum and not so much per unit of measure or
number, the boundaries of the land stated in the contract determine the effects and scope of the
sale, not the area thereof. Hence, the vendors are obligated to deliver all the land included
within the boundaries, regardless of whether the real area should be greater or smaller than that
recited in the deed.

It should be emphasized, however, that the case is merely an action for forcible entry and that
the issue of ownership was decided for the sole purpose of resolving priority of possession.
Hence, any pronouncement made affecting ownership of the disputed portion is to be regarded
merely as provisional, hence, does not bar nor prejudice an action between the same parties
involving title to the land.

JRPA
Javier v. Veridiano
237 SCRA 565

DOCTRINE: A final judgment on forcible entry or unlawful detainer is NOT a bar against to an
action for determination of ownership. A judgment rendered in a case for recovery of possession
is conclusive only as to possession, not ownership. It is not a bar against an action for
determination of ownership
FACTS:
Petitioner bought a land on a subdivision by filing a Miscellaneous Sales Application. Pending
the approval of the sale, Ben Babol entered a portion adjacent to the land being bought by
Petitioner. Petitioner claims that the occupied land by Babol is part of his land so he filed a
forcible entry case. However, the trial court found Petitioner wrong and so sustained the
possession of Babol. Later on, Babol would sell this portion to private Respondent. In the
meantime, the application was approved and a TCT was delivered to Petitioner. This motivated
the Petitioner to demand the land again, this time from Respondent, and this time on the basis
of ownership. He filed a complaint for quieting of title and recovery of possession. Respondent
countered that the first case on forcible entry constituted res judicata against the second
complaint.

ISSUE:
W/N the first case on forcible entry was res judicata against the second case on quieting of title -
- NO
HELD:

For res judicata to arise, four requisites must concur: Final judgment, Court with competent
jurisdiction, judgment based on merits, identity of parties and cause of action.

The Court said that in the two cases, there were identity of parties because Respondent, having
acquired the contested land by sale and tradition, is a successor in interest.

A judgment in a forcible entry or detainer case disposes of no other issue than possession and
declares only who has the right of possession, but by no means constitutes a bar to an action
for determination of who has right or title of ownership.

ABB
Lim Kich Tong v. CA
195 SCRA 398

DOCTRINE: Any person deprived of possession of any land or building may file an action for
forcible entry and unlawful detainer against the person unlawfully depriving or withholding
possession from him. This relief is not only available to landlord, lessor but to lessee and tenant
as well within one year from such unlawful deprivation or withholding of possession.

FACTS:
Respondent Reginaldo Lim and his family resided in room of a building owned by Petitioner Lim
Kieh Tong until the former transferred to their new residence. However, the respondent retained
possession of the room to keep his important belongings. At one point, when respondent
wanted to go to his room, he found that his key was no longer compatible with the doors lock,
that is, the lock was changed. He asked from petitioner the new key but having failed, he filed a
writ of preliminary mandatory injunction plus damages with the MTC. Petitioner countered that
since the action is one for specific performance, the action should have been filed with the RTC.

ISSUE:
Whether or not the action is one for specific performance or forcible entry and detainer.

HELD:
The suit is actually one for forcible entry and detainer. Respondent remained in possession of
the property but petitioner prevented him from enjoying his right by depriving him of the right of
egress and ingress through the door of the building and the room. Any person deprived of
possession of any land or building may file an action for forcible entry and detainer against the
person unlawfully depriving or withholding possession from him. This relief is not only available
to landlord, lessor but to lessee and tenant as well within one year from such unlawful
deprivation or withholding of possession.

FZC
Penas v. CA
233 SCRA 744

DOCTRINE: The notice giving the lessee the alternative either to pay the increased rental or
otherwise vacate the land is not the demand contemplated by the Rules of Court in unlawful
detainer cases. When after such notice, the lessee elects to stay, he thereby merely assumes
the new rental and cannot be ejected until he defaults in said obligation and necessary demand
is first made.

FACTS:
Private respondent was the lessee of a property owned by petitioners based on a written lease
contract (on a monthly basis). On January 18, 1990, petitioners (lessors) sent a letter
terminating the lease contract with respondent and demanding that he vacate the premises. In
the same letter, respondent was given an option to sign a new lease contract but with an
increased rate (from P610 to P2,000). Respondent refused to vacate and refused to sign a new
contract. He tried to pay the rentals at the old rate but petitioners refused to accept the same.
Respondent deposited the amount with PNB in trust for petitioners but the petitioners never
withdrew the said money. In August 1992 or after 2 years, petitioners again sent a letter to
respondent demanding him to vacate the premises and pay back rentals for 2 years at the
increased monthly rate. For failure to comply with the demands, petitioners filed this unlawful
detainer case against respondent in September 1992. The MTC, RTC and the CA all ruled in
favor of respondent-lessee and dismissed the case on the ground that more than one year has
elapsed from the time petitioner sent his letter terminating the contract and demanding a new
lease contract.

ISSUE:
Whether or not the unlawful detainer case was proper. -- YES

HELD:
Yes, the unlawful detainer case was proper. The Supreme Court ruled in favor of petitioners and
ordered respondent to vacate the leased premises and pay back rentals at the increased rate.

The settled rule is that when a lessee is given an option to vacate the premises or sign a new
lease contract with increased rental, his choice to continuously stay means a new contract is
made and he should pay the increased rental. In this case, respondent deposited an amount
based on the old rental. Thus he defaulted in the payment. The demand to vacate sent in
August 1992 was proper since there was a breach of the contract on the part of the lessee. The
case was filed in September 1992, and so it was properly filed within one year from the last
letter of demand.

LNAC
Francel Realty Corporation v. CA
G.R. No. 117051

DOCTRINE: The action here is not a simple action to collect on a promissory note; it is a
complaint to collect amortization payments arising from or in connection with a sale of a
subdivision lot under PD. Nos. 957 and 1344, and accordingly falls within the exclusive original
jurisdiction of the HLURB to regulate the real estate trade and industry, and to hear and decide
cases of unsound real estate business practices.

FACTS:
Petitioner Francel Realty Corporation filed a complaint for unlawful detainer against private
respondent Francisco T. Sycip.

Petitioner alleged that it had executed a Contract to Sell to private respondent Lot 16, Building
No. 14 of the Francel Townhomes, at 22 Real Street, Maliksi, Bacoor, Cavite, for P451,000.00.
The Contract to Sell provides that in case of default in the payment of two or more installments,
the whole obligation will become due and demandable and the seller will then be entitled to
rescind the contract and take possession of the property; the buyer will vacate the premises
without the necessity of any court action and the downpayment will be treated as earnest money
or as rental for the use of the premises. Petitioner alleged that private respondent failed to pay
the monthly amortization since October 30, 1990 despite demands to update his payments and
to vacate the premises, the latest of which was the demand made in the letter dated September
26, 1992, and that because of private respondents unjust refusal to vacate, petitioner was
constrained to engage the services of counsel. Petitioner prayed that private respondent be
ordered to vacate the premises and pay a monthly rental beginning October 30, 1990 until he
shall have vacated the premises, and other expenses of litigation.

Private respondent moved to dismiss the complaint but his motion was denied by the MTC. He
filed his answer in which he alleged that he had stopped paying the monthly amortizations
because the townhouse unit sold to him by petitioner was of defective construction. He alleged
that he had in fact filed a complaint for unsound real estate business practice in the Housing
and Land Use Regulatory Board against petitioner. Private respondent prayed that petitioner be
ordered to pay damages.

The MTC ruled that the answer was filed out of time on the ground that it was filed more than
ten days after the service of summons. However, it dismissed the complaint for lack of
jurisdiction. The MTC held that the case was cognizable by the HLURB. But it also ordered
petitioner to pay private respondent damages, attorneys fees, and costs.

On appeal the RTC affirmed the decision of the MTC.

Petitioner filed a petition for review in the CA, however it was dismissed.

Thus, this petition for review on certiorari.

ISSUE:
Whether MTC correctly held itself to be without jurisdiction over petitioners complaint. -- YES

HELD:
This is, therefore, not a simple case for unlawful detainer arising from the failure of the lessee to
pay the rents, comply with the conditions of a lease agreement or vacate the premises after the
expiration of the lease. Since the determinative question is exclusively cognizable by the
HLURB, the question of the right of petitioner must be determined by the agency.

Petitioners cause of action against private respondent should instead be filed as a counterclaim
in HLURB Case in accordance with Rule 6, 6 of the Rules of Court which is of suppletory
application to the 1987 HLURB Rules of Procedure per 3 of the same. In the case of Estate
Developers and Investors Corporation v. Antonio Sarte and Erlinda Sarte, the developer filed a
complaint to collect the balance of the price of a lot bought on installment basis, but its
complaint was dismissed by the RTC for lack of jurisdiction. It appealed the order to this Court.
In dismissing the appeal, we held:

The action here is not a simple action to collect on a promissory note; it is a complaint to collect
amortization payments arising from or in connection with a sale of a subdivision lot under PD.
Nos. 957 and 1344, and accordingly falls within the exclusive original jurisdiction of the HLURB
to regulate the real estate trade and industry, and to hear and decide cases of unsound real
estate business practices. Although the case involving Antonio Sarte is still pending resolution
before the HLURB Arbiter, and there is as yet no order from the HLURB authorizing suspension
of payments on account of the failure of plaintiff developer to make good its warranties, there is
no question to Our mind that the matter of collecting amortizations for the sale of the subdivision
lot is necessarily tied up to the complaint against the plaintiff and it affects the rights and
correlative duties of the buyer of a subdivision lot as regulated by NHA pursuant to P.D. 957 as
amended. It must accordingly fall within the exclusive original jurisdiction of the said Board, and
We find that the motion to dismiss was properly granted on the ground that the regular court has
no jurisdiction to take cognizance of the complaint.

We hold that the MTC correctly held itself to be without jurisdiction over petitioners complaint.
But it was error for the MTC to grant private respondents counterclaim for damages for
expenses incurred and inconveniences allegedly suffered by him as a result of the filing of the
ejectment case.

Pursuant to Rule 6, 8 a party may file a counterclaim only if the court has jurisdiction to
entertain the claim. Otherwise the counterclaim cannot be filed.

Even assuming that the MTC had jurisdiction, however the award of damages to private
respondent must be disallowed for the following reasons: (1) The MTC decision itself stated that
the answer with its counterclaim was filed out of time or more than 10 days from private
respondents receipt of summons. In effect, therefore, private respondent did not make any
counterclaim. (2) A reading of the MTC decision showed no justification for the award of moral
and exemplary damages and attorneys fees.

TKDC
Azarcon v. Eusebio
105 Phil. 569

DOCTRINE: A person who is in possession of a parcel of land and is being ordered to leave
said land while products are pending harvest has the right to a part of the net harvest.

FACTS:
This involves a dispute over the possession of a parcel of public land; Eusebio filed a lease
application for a parcel of said laid but Azarcon occupied a portion of that land under a
homestead application. The conflict between the lessee and the homesteaders was investigated
by the Director of Lands and again by the Secretary of Agriculture and natural Resources.

Before the dispute could be settled, Eusebio filed a complaint with the Court of First Instance
alleging that he acquired that parcel of land through a lease with the Bureau of Lands and while
he is in possession of said land, Azarcon occupied a portion of it. Azarcon answered that he is
in actual possession of a portion of the land under a homestead application and that the lease
application of Eusebio was subsequent to his homestead application; he occupied the land
since 1941 with interruption during the war and again in 1950 up to the time of the filing of the
action.

The trial court ruled in favor of Eusebio; while the case was appealed, a writ of execution was
issued for the restitution of the land to Eusebio. Despite receipt of the notice of the writ of
execution, Aznar entered the land to gather palay which was then pending harvest.

ISSUE:
Whether or not Aznar acted in bad faith when he entered the land to gather palay despite
receipt of the notice of the writ of execution. -- NO

HELD:
While the court ordered Aznar to move out from the premises, it did not prohibit them from
gathering the crop existing on the land. Under Article 545 of the Civil Code, a person who is in
possession and who is being ordered to leave a parcel of land while products thereon are
pending harvest has the right to a part of the net harvest.

In this case, the order of execution did not expressly prohibit Aznar from gathering the pending
fruits, which were the result of their possession and cultivation of the land, so it cannot be said
that Aznar violated the courts' order. Their act in harvesting the pending fruits was not only
justified by law but was not expressly prohibited by the court's order, and was even ratified
when the court ordered the suspension of the execution.

AMD
Calagan v. CFI of Davao
95 SCRA 498

DOCTRINES:
1. Purpose of the law on homesteads: to conserve ownership in the hands of the
homesteader and his family

2. Under Article 547, the homesteader desiring to repurchase should be given the option to
require the vendee a retro to remove the useful improvements on the land subject of the
sale a retro, such option is not granted the vendor a retro under Article 1616.

FACTS:
In 1954, Mangulon Calagan and his wife, Takura, were granted a Homesstead Patent and
issued an Original Certificate of Title over a lot located in Digos, Davao del Sur. In 1955, Takura
died and was survived by her husband and their children.

In 1961, Calagan sold a portion of their homestead to private respondent, Petra Sandoval.
Sandoval borrowed Calagans title to the land so she could have the sale annotated. In 1963,
Calagan offered to repurchase the land but Sandoval refused such offer. Sandoval was only
willing to comply if she was reimbursed the value of the house that she had constructed on the
land.

Petitioners filed an action for reconveyance against Sandoval. The CFI ruled in favor of
petitioners. Sandoval was ordered to reconvey the land, given that petitioners pay her the value
of the house she has constructed in good faith. Petitioners appealed, maintaining that the Court
erred in ordering them to reimburse the value of the house.

ISSUE:
W/N petitioners should be ordered to reimburse the value of the house built in good faith on the
land they seek to repurchase. -- NO

HELD:
It being obvious that petitioners are not exercising the option to refund the amount of the
expenses incurred in building the house, private respondent, as the vendee a retro, may remove
her house since this can be done without damage to the principal thing, as stipulated in Article
547 of the Civil Code. Petitioners should not, as opined by the trial Court, be made to refund the
value of that house to private respondent, else, the salutary policy behind the Public Land Law
would be thwarted and rendered meaningless.

There is no provision in the Public Land Act (CA No. 141) which provides for the terms and
conditions under which repurchase may be effected by a homesteader except that it should be
made within 5 years from the date of conveyance. That Act is silent as to the nature of
expenses that should be reimbursed by a repurchasing homesteader or his heir. However,
under Art. 1616 of the New Civil Code, the vendor a retro cannot avail himself of the right of
repurchase without returning to the vendee the price of the sale, the expenses of the contract
and other legitimate payments, and the necessary and useful expenses made on the thing sold.

The provision of Article 1616 of the Civil Code on redemption, is, in general, the applicable law
to a homesteader desirous to redeem his property. However, considering the purpose of the law
on homesteads, which is to conserve ownership in the hands of the homesteader and his family,
Article 1616 of the Civil Code should be construed in conjunction with Articles 546 and 547 of
the Civil Code prescribing the rules on refund of necessary and useful expenses, inasmuch as a
vendee a retro is, as a rule, considered in good faith.

Applying Article 547, the homesteader desiring to repurchase should be given the option to
require the vendee a retro to remove the useful improvements on the land subject of the sale a
retro, which option is not granted the vendor a retro under Article 1616. Under the latter Article,
the vendor a retro must pay for the useful improvements introduced by the vendee a retro,
otherwise, the latter may retain possession of the property until reimbursement is made. To
allow a vendee a retro of a homestead, however, the right of retention until payment of useful
expenses is made by the redemptioner would be to render nugatory the right of repurchase
granted by law to a homesteader because all a vendee a retro can do to prevent repurchase is
to build something on the homestead beyond the capacity to pay of the homesteader who seeks
to repurchase
CRF
Cruz v. Pahati
98 Phil. 788

DOCTRINE: The common law principle that the one who has made the happening of fraud
possible through misplaced confidence must suffer the consequence cannot be applied since
there is an express provision covering the case. Article 559, a statutory provision, prevails over
a common law principle. (batasnatin)

FACTS:
Pahati admitted having bought the automobile from Bulahan but when it was impounded, he
returned the automobile to Bulahan who in turned surrendered the check for cancellation.
Bulahan on his part claims that he acquired the automobile from Jesusito Belizo for value and
without having any knowledge of any defect in the title of the latter.

The lower court found that the automobile in question was originally owned by the Northern
Motors, Inc. which later sold it to Chinaman Lu Dag. This Chinaman sold it afterwards to
Jesusito Belizo and the latter in turn sold it to plaintiff. Belizo was then a dealer in second hand
cars. One year thereafter, Belizo offered the plaintiff to sell the automobile for him claiming to
have a buyer for it. Plaintiff agreed. At that time, plaintiff's certificate of registration was missing
and, upon the suggestion of Belizo, plaintiff wrote a letter addressed to the Motor Section of the
Bureau of Public Works for the issuance of a new registration certificate alleging as reason the
loss of the one previously issued to him and stating that he was intending to sell his car. This
letter was delivered to Belizo on March 3, 1952. He also turned over Belizo the automobile on
the latter's pretext that he was going to show it to a prospective buyer. Subsequent facts show
that the letter was falsified by Belizo to enable him to sell the car to Bulahan for a valuable
consideration.

ISSUE:
Who has, therefore, a better right of the two over the car?

HELD:
The plaintiff has a better right to the car in question than defendant Bulahan for it cannot be
disputed that plaintiff had been illegally deprived thereof because of the ingenious scheme
utilized by Belizo to enable him to dispose of it as if he were the owner thereof. Plaintiff
therefore can still recover the possession of the car even if defendant Bulahan had acted in
good faith in purchasing it from Belizo.

ART. 559. The possession of movable property acquired in good faith is equivalent to a title.
Nevertheless, one who has lost any movable or has been unlawfully deprived thereof, may
recover it from the person in possession of the same.

If the possessor of a movable lost or of which the owner has been unlawfully deprived, has
acquired it in good faith at a public sale, the owner cannot obtain its return without reimbursing
the price paid therefor.

Article 1505 of the same Code provides that "where goods are sold by a person who is not the
owner thereof, and who does not sell them under authority or with the consent of the owner, the
buyer acquires no better title to the goods than the seller had, unless the owner of the goods is
by his conduct precluded from denying the seller's authority to sell.

MPF
Aznar v. Yapdiangco
13 SCRA 486

DOCTRINE: Ownership is not transferred by contract merely but by tradition or delivery.
Contracts only constitute titles or rights to the transfer or acquisition of ownership, while delivery
or tradition is the mode of accomplishing the same.

Art. 559. The possession of movable property acquired in good faith is equivalent to a title.
Nevertheless, one who has lost any movable or has been unlawfully deprived thereof may
recover it from the person in possession of the same.

If the possessor of a movable lost or which the owner has been unlawfully deprived, has
acquired it in good faith at a public sale, the owner cannot obtain its return without reimbursing
the price paid therefor.

FACTS:
Teodoro Santos advertised the sale of his FORD FAIRLANE 500 in a newspaper.
L. De Dios went to the house of Teodoro and talked to his son Ireneo Santos and said that
his uncle Vicente Marella is interested in buying the said car.
The next day, Ireneo went to the house of Marella and they agreed to the price of
P14,700 on the understanding that it will be paid after the car has been registered in the
latters name.
A deed of sale was executed and the registration was changed to the name of
Marella. Ireneo went to Marella to get the payment and deliver the car who informed
him that he is P2,000 short of the money and that they need to go to his sister to get
it. Ireneo, together with De Dios and an unidentified man went to a house.
Once inside, De Dios asked Ireneo to wait.. After waiting in vain, he went down
and discovered that the car was gone.
Marella was able to sell the car to plaintiff-appellant Jose Aznar and while
attending to registration; Philippine Constabulary seized the car due to the report of the
incident.

ISSUE:
Who has the better right on the property involved (car)??

HELD:
Teodoro Santos has the better right. Marella did not have any title to the property under litigation
because the same was never delivered to him. He may have the contract but he never acquired
valid title. Although the keys to the car may have been given to the unidentified companion, it
may be done only because that companion took them to the place where the sister of Marella
was supposed to live. The car was evidently stolen and that the buyer did not acquire any valid
title thereto.

Marella never had title to the car as the car wasn't ever delivered to him. While there was a
deed of sale in his favor, he was only able to obtain possession of the car since he stole it from
Santos. The applicable law is Article 559. The rule is to the effect that if the owner has lost a
thing, or if he has been unlawfully deprived of it, he has a right to recover it, not only from its
finder, thief or robber, but also from third persons who may have acquired it in good faith from
such finder, thief or robber. The said article establishes 2 exceptions to the general rule of
irrevindicabiltyto wit, the owner has lost the thing or has been unlawfully deprived thereof. In
these cases, the possessor cannot retain the thing as against the owner who may recover it
without paying any indemnity, except when the possessor acquired it in a public sale.
Furthermore, the common law principle that where one of two innocent persons must suffer a
fraud perpetrated by another, the law imposes the loss upon the party who, by his misplaced
confidence, has enable the fraud to be committed, cannot be applied in this case, which is
covered by an express provision of law.

AMDG
De Garcia v. CA
37 SCRA 160

DOCTRINE: Article 559 of the Civil Code: The possession of movable property acquired in good
faith is equivalent to a title. Nevertheless, one who has lost any movable or has been unlawfully
deprived thereof may recover it from the person in possession of the same.
If the possessor of a movable loss of which has been unlawfully deprived, has acquired it in
good faith at a public sale, the owner cannot obtain its return without reimbursing the price paid
thereof.

As stated in Cruz v Pahati, the rightful owner may not be prevented from recovering his property
by reason of good faith alone. The only exception that is acknowledged by the law is when the
property was acquired in good faith at a public sale. Still, in such case, recovery may only take
place after reimbursement.

FACTS:
Guevara purchased a diamond ring from R. Rebullida, Inc. The ring was eventually stolen from
her. More than a year after it was stolen, while she was conversing with De Garcia, she noticed
the latter wearing a ring which she recognized to be her stolen ring. Guevara asked De Garcia
where she bought it and the latter answered that she purchased it from Mrs. Miranda who
bought it from Ms. Hinahon who bought it from a certain Aling Petring. After Guevara explained
to De Garcia that the ring is hers, she borrowed it and proceeded to Mr. Rebudilla. After
examining the jewelry, Mr. Rebudilla confirmed that it was indeed the ring the Guevara bought
from him. Guevara returned the ring and later on wrote and requested De Garcia to deliver the
ring to her. Despite the written request as well as the writ of seizure delivered by the sheriff and
served against her, De Garcia did not deliver the ring.

The lower court ruled in favor of De Garcia. However, the Court of Appeals reversed the lower
courts decision and ordered that the ring be returned to Guevara or pay Guevara equivalent
price of the jewelry. One of the reasons cited by the CA was the dubious source narrated by De
Garcia as regards how she acquired the ring.

ISSUE:
Whether or not the Court of Appeals erred in ruling that the ring must belongs to Guevara and
must be returned to her NO

HELD:
Reviewing the facts and reasoning of the CA, the SC did not see any error on the part of the CA
in ruling in favor of Guevara. Applying Article 559 of the Civil Code, it is evident that Guevara
was unlawfully deprived of her diamond ring. It is only proper that the ring be returned by De
Garcia who has possession of it. Furthermore, as stated in Cruz v. Pahati, the rightful owner
may not be prevented from recovering his property by reason of good faith alone. The only
exception that is acknowledged by the law is when the property was acquired in good faith at a
public sale. Still, in such case, recovery may only take place after reimbursement. Applying this
to the case, De Garcia herself admitted that she did not acquire the ring from a public sale but
rather from Mrs. Miranda.

In addition, Anzar v. Yapdiangco, reiterated that the reason behind the recovery of the rightful
owner from a person who acquired the property in good faith was because no consent was
given by the former from the dispossession of his property. The Court also discussed the
existing common law principle which is: where one of two innocent persons must suffer by a
fraud perpetrated by another, the law imposes the loss upon the part who, by his misplaced
confidence, has enabled the fraud to be committed. However, such common law cannot be
applied if there exists an express provision of the law such as Article 559 of the Civil Code.

It follows from the above discussion that De Garcias contention as regards possession in good
faith is equivalent from a just title cannot be accepted by this Court. Looking further at the true
meaning of Article 559, the title mentioned refers to a presumptive title only and not just title.
Consequently, when the true owner has been unlawfully deprived of his property it is only just
that it be returned to him.

GCG
Dizon v. Suntay
47 SCRA 160

DOCTRINE: The possession of movable property acquired in good faith is equivalent to a title.
Nevertheless, one who has lost any movable or has been unlawfully deprived thereof may
recover it from the person in possession of the same. If the possessor of a movable lost of
which the owner has been unlawfully deprived, has/acquired it in good faith at a public sale, the
owner cannot obtain its return without reimbursing the price paid therefore. The only exception
the law allows is when there is acquisition in good faith of the possessor at a public sale, in
which case the owner cannot obtain its return without reimbursing the price.

FACTS:
Plaintiff is the owner of a three-carat diamond ring valued at P5,500 and entered into a
transaction with Clarita R. Sison. The plaintiffs ring was delivered to Clarita R. Sison for sale on
commission.

Upon receiving the ring, Clarita R. Sison executed and delivered to the plaintiff the receipt. After
the lapse of a considerable time without Clarita R. Sison having returned to the plaintiff the
latters ring, the plaintiff made demands on Clarita R. Sison for the return of her ring but the
latter could not comply with the demands. Three days after the ring above-mentioned was
received by Clarita R. Sison, the ring was pledged by Melia Sison, niece of the husand of Clarita
R. Sison, evidently in connivance with the latter, with the defendants pawnshop for P2,600.
This was done without the knowledge of the plaintiff.

The plaintiff insistently demanded from Clarita R. Sison the return of her ring, the latter finally
delivered to the former the pawnshop ticket, which is the receipt of the pledge with the
defendants pawnshop of the plaintiffs ring. When the plaintiff found out that Clarita R. Sison
pledged, she took steps to file a case of Estafa against the latter with the fiscals office.
Subsequently thereafter, the plaintiff, wrote a letter to the defendant asking for the delivery to
the plaintiff of her ring pledged with defendants pawnshop.

Since the defendant refused to return the ring, the plaintiff filed the present action with the CFI
for the recovery of said ring. The plaintiff asked for the provisional remedy writ of replevin by the
delivery of the ring to her, upon her filing the requisite bond, pending the final determination of
the action.

The lower court issued the writ of replevin prayed for by plaintiff and the latter was able to take
possession of the ring during the pendency of the action upon her filing the requisite bond. The
lower court rendered judgment declaring that Suntay had the right to the possession of the ring
in question.

Petitioner Dizon, as defendant, sought to have the judgment reversed by the CA, which ruled in
favor of Suntay.

ISSUE:

Whether or not Suntay had the right to the possession of the ring. -- YES

HELD:
Yes. The Supreme Court held that Suntay had the right to the possession of the ring.

The controlling provision is Article 559 of the Civil Code.

It reads thus: The possession of movable property acquired in good faith is equivalent to a title.
Nevertheless, one who has lost any movable or has been unlawfully deprived thereof may
recover it from the person in possession of the same. If the possessor of a movable lost of
which the owner has been unlawfully deprived, has/acquired it in good faith at a public sale, the
owner cannot obtain its return without reimbursing the price paid therefore. The only exception
the law allows is when there is acquisition in good faith of the possessor at a public sale, in
which case the owner cannot obtain its return without reimbursing the price.

VCL IV
EDCA Publishing and Distributing Corp. v. Santos
184 SCRA 614

DOCTRINE: Possession of movable property acquired in good faith is equivalent to title. There
is no need to produce a receipt.

FACTS:
EDCA Publishing sold 406 books to a certain Professor Jose Cruz who ordered these by
telephone, which was agreed to be payable on delivery. The books were subsequently delivered
to him with the corresponding invoice, and he paid with a personal check.

Cruz then sold the 120 of the books to Leonor Santos who asked for verification, and was then
showed the invoice for the books.

EDCA having become suspicious over a second order placed by Cruz even before clearing of
his first check, made inquiries with the De la Salle College where he had claimed to be a dean
and was informed that there was no such person in its employ. Further verification revealed that
Cruz had no more account or deposit with the Philippine Amanah Bank, against which he had
drawn the payment check. EDCA then went to the police, which set a trap and arrested Cruz on
October 7, 1981. Investigation disclosed his real name as Tomas de la Pea and his sale of 120
of the books he had ordered from EDCA to Santos.

On the night of the same date, EDCA sought the assistance of the police in Precinct 5 at the UN
Avenue, which forced their way into the store of the private respondents and threatened Leonor
Santos with prosecution for buying stolen property. They seized the 120 books without warrant,
loading them in a van belonging to EDCA, and thereafter turned them over to the petitioner.
Protesting this high-handed action, the Santos sued for recovery of the books after demand for
their return was rejected by EDCA. A writ of preliminary attachment was issued and the
petitioner, after initial refusal, finally surrendered the books to the private respondents.
MTC ruled that the ownership of the books of Santos. RTC and CA sustained the MTCs ruling.

ISSUE:
Whether or not the owner was unlawfully deprived of the property -- NO

HELD:
Santos was a good faith buyer after taking steps to verify the identity of the seller. When she
was showed the invoice, she reasonably believed that he was a legitimate seller.

With regard to unlawful deprivation, EDCA was not unlawfully deprived of the property by mere
failure of consideration. There was already a perfected contract of sale. Proof was even
substantiated when EDCA gave the invoice as proof of payment upon delivery of the books.
This did not amount to unlawful taking, because by the delivery of EDCA to Cruz, ownership of
the books already transferred to him.

FXRL
Ledesma v. CA
213 SCRA 195

DOCTRINE: Article 559 cannot be applied when the property was lawfully divested from the
original owner.

FACTS:
September 1977, a person representing himself to be Jojo Consuji purchased two
vehicles from Citywide Motors, Inc.
Jojo used a managers check to pay for the vehicles
The said vehicles were delivered via a contract of sale.
The check was dishonored.
It was discovered that Jojo Consuji was in fact an Armando Suarez, a known and
infamous conman.
One of the vehicles was recovered, abandoned in Quezon City. The other vehicle was
found to be sold to the petitioner.
Ledesma claims that he had acquired the vehicle in good faith and is thus entitled to the
same.
The company filed for replevin and reacquired the vehicle from Ledesma after payment
of a bond.
The RTC ruled in favor of Ledesma and awarded him with possession of the vehicle plus
damages.
The CA ruled in favor of the company.
The company relies on the strength of Article 559 of the Civil Code which provides that if
the owner has lost a thing, or if he has been unlawfully deprived of it, he has a right to
recover it not only from the finder, thief or robber, but also from third persons who may have
acquired it in good faith from such finder, thief or robber. The said article establishes two (2)
exceptions to the general rule of irrevendicability (sic), to wit: when the owner (1) has lost
the thing, or (2) has been unlawfully deprived thereof. In these cases, the possessor cannot
retain the thing as against the owner who may recover it without paying any indemnity,
except when the possessor acquired it in a public sale.

ISSUE:
W/N the private respondent (the company) was unlawfully deprived of the vehicle in the first
place so as to make Article 559 apply. -- NO

HELD:
The company was not unlawfully deprived of the vehicle in question. Therefore, the petitioner is
a buyer in good faith and Article 559 may not apply.

There was a perfected unconditional contract of sale between private respondent and the
original vendee. The former voluntarily caused the transfer of the certificate of registration of the
vehicle in the name of the first vendee even if the said vendee used a fictitious name and
likewise voluntarily delivered the cars and the certificate of registration to the vendee Title
thereto was forthwith transferred to the vendee. The subsequent dishonor of the check merely
amounted to a failure of consideration which does not render the contract of sale void, but
merely allows the prejudiced party to sue for specific performance or rescission of the contract,
and to prosecute the impostor for estafa under the Revised Penal Code.

Under the law on obligations and contracts, the parties to a contract of sale may stipulate that
ownership in the thing sold shall not pass to the buyer until full payment of the purchase price
only if there is a stipulation to that effect. Otherwise, the rule is that such ownership shall pass
from the vendor to the vendee upon the actual or constructive delivery of the thing sold even if
the purchase price has not yet been paid.

Non-payment only creates a right to demand payment or to rescind the contract, or to criminal
prosecution in the case of bouncing checks. But absent the stipulation above noted, delivery of
the thing sold will effectively transfer ownership to the buyer who can in turn transfer it to
another.

RSDM
Chua Kai v. Kapunan
104 Phil. 110

DOCTRINE:
1. That the acquirer and possessor in good faith, of a chattel or movable property is entitled
to be respected and protected in his possession, as if he were the true owner thereof,
until a competent court rules otherwise;
2. That being considered, in the meantime, as the true owner, the possessor in good faith
cannot be compelled to I surrender possession nor to be required to institute an action
for the recovery of the chattel, whether or not an indemnity bond is issued in his favo

FACTS:
Roberto Soto purchased from Youngstown Hardware, owned by Ong Shu, 700 corrugated
galvanized iron sheets and 249 pieces of round iron bar for P6,137.70, and in payment thereof
he issued a check drawn against the Security Bank and Trust Company for P7,000.00, without
informing Ong Shu that he had no sufficient funds in said bank to answer for the same. When
the check was presented for payment, it was dishonored for insufficiency of funds. Soto sold
165 sheets in Pangasinan and 535 sheets in Calapan, Mindoro. Of those sold in Pangasinan,
100 were sold to petitioner Chua Hai. When the case was filed in the Court of First Instance of
Manila against Roberto Soto, for estafa, the offended party filed a petition asking that the 700
galvanized iron sheets, which were deposited with the Manila Police Department, be returned to
him, as owner of the Youngstown Hardware. Petitioner herein opposed the motion with respect
to the 100 sheets that he had bought from Soto. Notwithstanding the opposition, the court
ordered the return of the galvanized iron sheets to Ong Shu. Petitioner then presented a motion
to reconsider the order, alleging that by the return thereof to the offended party, the court had
not only violated the contract of deposit, because it was in that concept that petitioner had
delivered the 100 sheets to the Manila Police Department, and that said return to Ong Shu
amounted to a deprivation of his property without due process of law. It is also claimed that
Article 105 of the Revised Penal Code, under whose authority the return was ordered, can be
invoked only after the termination of the criminal case and not while said criminal case is still
pending trial.

ISSUE:
Whether or not petitioner Chua has better right of the possession

HELD:
To deprive the possessor in good faith, even temporarily and provisionally, of the chattels
possessed, violates the rule of Art. 559 of the Civil Code. The latter declares that possession of
chattels in good faith is equivalent to title; i.e., that for all intents and purposes, the possessor is
the owner, until ordered by the proper court to restore the thing to the one who was illegally
deprived thereof. Until such decree is rendered (and it can not be rendered in a criminal
proceeding in which the possessor is not a party), the possessor, as presumptive owner, is
entitled to hold and enjoy the thing; and "every possessor has a right to be respected in his
possession; and should he be disturbed therein he shall be protected in or restored to said
possession established by the means established by the laws and the Rules of Court."(Art. 539,
New Civil Code).

The decision of the court below, instead of conforming to Arts. 559 and 539 of the Civil Code,
directs possessor to surrender the chattel to the claimant Ong Shu before the latter has proved
that he was illegally deprived thereof, without taking into account that the mere filing of a
criminal action for estafa is no proof that estafa was in fact committed. Instead of regarding the
possessor as the owner of the chattel until illegal deprivation is shown, the court below regards
the possessor of the chattel not as an owner, but as a usurper, and compels him to surrender
possession even before the illegal deprivation is proved. We see no warrant for such a reversal
of legal rules.




USUFRUCT (ART. 562-612)

MRAM
NHA v. CA
G.R. No. 148830

DOCTRINE: A usufruct gives a right to enjoy the property of another with the obligation of
preserving its form and substance, unless the title constituting it or the law otherwise provides.

FACTS:
By virtue of Proclamation No. 481 issued by then Pres. Marcos, a portion of land in Quezon City
owned by NHA was reserved for the site of National Government Center (NGC). Subsequently,
President Marcos issued Proclamation No. 1670, which removed a seven-hectare portion from
the coverage of the NGC. Proclamation No. 1670 gave Manila Seedling Bank Foundation, Inc.
(MSBF) usufructuary rights over this segregated portion.

MSBF occupied the area granted by Proclamation No. 1670. However, over the years, MSBFs
occupancy exceeded the seven-hectare area subject to its usufructuary rights; it occupied
approximately 16 hectares by 1987. By then the land occupied by MSBF was bounded by
Epifanio de los Santos Avenue (EDSA) to the west, Agham Road to the east, Quezon Avenue
to the south and a creek to the north.

On 18 August 1987, MSBF leased a portion of the area it occupied to Bulacan Garden
Corporation (BGC) and other stallholders. BGC leased the portion facing EDSA, which
occupies 4,590 sq. m. of the 16-hectare area.

On 11 November 1987, President Corazon Aquino issued Memorandum Order No. 127 (MO
127). This revoked the reserved status of the 50 hectares, more or less, remaining out of the
120 hectares of the NHA property reserved as site of the NGC. MO 127 also authorized the
NHA to commercialize the area and to sell it to the public.

Acting on the power granted under MO 127, the NHA gave BGC 10 days to vacate its occupied
area. Any structure left behind after the expiration of the 10-day period will be demolished by
NHA.

ISSUE:
Whether or not the premises leased by BGC from MSBF is within the 7-hectare area that
Proclamation No. 1670 granted to MSBF by way of usufruct?

HELD:
The Court held that MSBF abused their usufruct rights. Clearly, in the present case,
Proclamation No. 1670 is the title constituting the usufruct. Proclamation No. 1670 categorically
states that the 7-hectare area shall be determined by future survey under the administration of
the Foundation subject to private rights if there be any. MSBF, then, has the latitude to
determine the location of its 7-hectare usufruct portion within the 16-hectare area.

Although MSBF has the discretion to determine its 7-hectare usufruct, MSBF abused its right
when it exceeded the 7-hectare portion granted to it by Proclamation No. 1670. The Court said
that a usufruct is not simply about rights and privileges. A usufructuary has the duty to protect
the owners interests. One such duty is found in Article 601 of the Civil Code which states:

ART. 601. The usufructuary shall be obliged to notify the owner of any act of a third
person, of which he may have knowledge, that may be prejudicial to the rights of
ownership, and he shall be liable should he not do so, for damages, as if they had been
caused through his own fault.

A usufruct gives a right to enjoy the property of another with the obligation of preserving its form
and substance, unless the title constituting it or the law otherwise provides.

The Court further said that at this point, the determination of the seven-hectare portion cannot
be made to rely on a choice between the NHAs and MSBFs survey. There is a need for a new
survey, one conducted jointly by the NHA and MSBF, to remove all doubts on the exact location
of the seven-hectare area and thus avoid future controversies. This new survey should consider
existing structures of MSBF. It should as much as possible include all of the facilities of MSBF
within the seven-hectare portion without sacrificing contiguity.

It must be noted however that based on Art. 605, MSBF has only 22 years to exercise its
usufruct since the Civil Code provides that the right can be exercised only within 50 year

FMM
Gaboya v. Cui
38 SCRA 85

DOCTRINE: The reserved usufructuary right of a seller on a property doesnt include rentals
from the buildings subsequently constructed on the sale, but it entitles the usufructuary to a
reasonable rental for the portion of the land being occupied by the building.

FACTS:
Don Mariano Cui was a widower who owned three lots (Nos. 2312, 2313 and 2319) situated in
Cebu City with a total area of 2,658 square meters. He sold the three commercial lots to three
of his children (Rosario C. de Encarnacion, Mercedes C. de Ramas and Antonia Ma. Cui) pro
indiviso.

Rosario C. de Encarnacion was unable to pay her corresponding share of the purchase price
due to lack of funds. Thus, the sale to her was cancelled and the one-third of the property
corresponding to her was returned to Don Mariano Cui. Because of the sale of the lots was pro
indiviso and due to the cancellation of one of the three lots, Don Mariano and is children,
Mercedes and Antonio became co-owners of the whole property.

In the Deed of Sale, Don Mariano retained for himself the usufruct of the property, stating that
he shall enjoy the fruits and rents of the property as long as his natural life shall last.

Subsequently, a building was erected on a portion of the property and was occupied by a
Chinese businessman which paid a rent of P600 a month.

Sometime after the sale to his two daughters, the latter applied to the Rehabilitation Finance
Corporation (RFC) for a loan of P130,000 with which to construct a building presumably on a
portion of the property sold to the two daughters. To facilitate the granting of the loan, Don
Mariano executed an authority to mortgage his share of the property in favor of his two
daughters.

The loan was eventually granted and secured by a mortgage on the property. However, Don
Mariano, despite being one of the mortgagors, did not join in the construction of the building.
The building was eventually constructed and the two daughters received rents therefrom and
paid their loan.

On 19 March 1949, Rosario C. Encarnacion, one of the daughters of Don Mariano filed a
petition to declared their father incompetent and to have a guardian appointed for his property.
The petition was granted and Victorino Reynes was appointed guardian of his property.

On 15 June 1949, guardian Reynes filed a motion in the guardianship proceeding seeking
authority to collect the rentals from the property. The motion, however, was denied by the judge.

Subsequently, another complaint was filed by Don Marianos guardian which alleged that the
usufructuary right reserved in favor of Don Mariano extends to and includes the rentals of the
building constructed by his two daughters on the land sold to them by their father; that the
defendants retained those rentals for themselves; that the usufructuary rights of the vendor
were of the essence of the sale, and their violation entitled him to rescind (or resolve) the sale. It
prayed either for rescission with accounting, or for delivery of the rentals of the building with
interests, attorneys' fees and costs.

The Court of First Instance of Cebu denied the resolution of the foregoing complaint hence this
petition by the Judicial Administrator of the Estate of Mariano Cui, Jesus M. Gaboya.

ISSUE:
Whether the usufruct reserved by Don Mariano in the deed of sale, over the property that were
at the time vacant and unoccupied, gave the usufructuary the right to receive the rentals of the
commercial building constructed by the two daughters with funds borrowed from the
Rehabilitation and Finance Corporation, the loan being secured by a mortgage over the lots sold.
-- NO

HELD:
The Supreme Court (the SC) held that the usufructuary rights of the late Don Mariano
reserved in the deed of sale was over the land alone and did not entitle him to the rents of the
building later constructed thereon by his two daughters.

The SC also held that said usufructuary was entitled only to the reasonable rental value of the
land occupied by the building aforementioned.

RGGM
Vda. De Albar v. Carangdang
106 Phil. 855

DOCTRINE: Under the law, usufruct is extinguished only by the total loss of the thing subject of
the encumbrance. Any war damage payment received by the naked owner should also be
subject to usufruct for life if such payment has not been used in the construction of a new
building. A usufructuary who is the only recipient of all the benefits of the property subject of the
usufruct, and who has bound himself to pay the real estate taxes on the property in a formal
agreement approved by the court, should pay such taxes.

FACTS:
Doa Rosario Fabie y Grey was the owner of the lot in the City of Manila with a building and
improvements, and by a will left by her upon her death which was duly probated she devised the
naked ownership of the whole property to Rosario Grey Vda. de Albar, et al. but its usufruct to
Josefa Fabie for life.

During liberation, as a consequence of the fire that gutted the building in many portions of
Manila, the building on the Ongpin lot was burned, leaving only the walls and other
improvements that were not destroyed by the fire.

One Au Pit, a Chinaman, offered to lease the property for a period of five years, at the same
time agreeing to construct on the lot a new building provided the naked owners as well as the
usufructuary sign the agreement of the lease. As the usufructuary maintains that she has the
exclusive right to cede the property by lease and to receive the full rental value by virtue of her
right to usufruct while on the other hand the naked owners maintain that the right of usufruct
was extinguished when the building was destroyed, the right of the usufructuary being limited to
the legal interest on the value of the lot and the materials, in order that the agreement of lease
may be affected, the parties agreed on a temporary compromise whereby the naked owners
would receive P100.00, or 20% of the monthly rental of P500.00 and the usufructuary the
balance of 80% or P400.00 of said monthly rental. It was likewise stipulated in the agreement
that the title to the building to be constructed would accrue to the land upon it completion as an
integral part of the lot covered by the transfer certificate of title issued in the name of the naked
owners but subject to the right of usufruct of Josefa Fabie. The parties expressly reserved the
right to litigate their respective claims after the termination of the contract of lease to determine
which of said claims was legally correct.

By reason of the destruction of the building on the Ongpin property, the United States War
Damage Commission approved the claim that was presented for the damage caused to the
property, paid to and received by the naked owners. In the meantime, the usufructuary paid the
real estate taxes due on the property at Ongpin for the years 1945 to 1952.

ISSUE:
W/N the usufruct included the building and the land? W/N the usufructuary (FABIE) or naked
owner (VDA DE ALBAR) should undertake the reconstruction? W/N the usufructuary should pay
the real estate taxes?

HELD:
The usufruct for life extended to the land and the building. From the above, it is clear that when
the deceased constituted the life usufruct on the rentals "fincas situadas" in Ongpin and Sto.
Cristo streets, she meant to impose the encumbrance both the building and the land on which it
is erected for indeed the building cannot exist without the land. And as this Court well said, "The
land, being an indispensable part of the rented premises cannot be considered as having no
rental value whatsoever." Moreover, in the Spanish language, the term "fincas" has a broad
scope; it includes not only building but land as well. (Diccionario Ingles-Espaol, por Martines
Amador) Since only the building was destroyed and the usufruct is constituted not only on the
building but on the land as well, then the usufruct is not deemed extinguished by the destruction
of the building for under the law usufruct is extinguished only by the total loss of the thing
subject of the encumbrance (Article 603, old Civil Code).

FABIE, the usufructuary has the discretion to reconstruct the building. Of course, this is
addressed to the wisdom and discretion of the usufructuary who, to all intents and purposes is
deemed as the administrator of the property. This has been clarified in the case of Fabie vs.
Gutierrez David, 75 Phil., 536, which was litigated between the same parties and wherein the
scope of the same provision of the will has been the subject of interpretation.

The usufructuary should pay the taxes. We find, however, merit in the contention that the real
estate taxes paid by respondent in her capacity as usufructuary for several years previous to the
present litigation should be paid by her, as she did, instead of by petitioners not only because
she bound herself to pay such taxes in a formal agreement approved by the court in Civil Case
No. 1569 of the Court of First Instance of Manila (Fabie vs. Gutierrez David, supra). In the case,
which involved the same parties and the same properties subject to usufruct, the parties
submitted an amicable agreement which was approved by the court wherein the usufructuary,
herein respondent, bound herself to pay all the real estate taxes, special assessment and
insurance premiums, and make all the necessary repairs on each of the properties covered by
the usufruct and in accordance with said agreement, respondent paid all the taxes for the years
1945 to 1954

MCSS
Baluran v. Navarro
79 SCRA 309

FACTS:
Spouses Paraiso executed an agreement entitle BARTER whereby they agreed to barter and
exchange with spouses Baluran their residential lot with the latter's unirrigated riceland. The
documents allowed the parties to enjoy the material possession of their respective properties,
reap the fruits of the unirrigated riceland for spouses Paraiso and build a house in the residential
lot for spouses Baluran. Also, it was conditioned upon the event that if any of the children of
Natividad Obencio, daughter of spouses Paraiso, shall choose to reside in this municipality and
build a house in the residential lot, spouses Balura shall be obliged to return the lot with
damages. Lastly, neither party shall encumber, alienate or dispose of in any manner their
respective properties as bartered without the consent of the other.

Antonio Obendencio, son of Natividad, filed a complaint to recover the residential lot. Avelino
Baluran alleged that the barter agreement transferred to him the ownership of the residential lot
in exchange for the unirrigated riceland.

ISSUE:
WON there was a transfer of ownership between the parties. -- NO

HELD:
It is a settled rule that to determine the nature of a contract courts are not bound by the name or
title given to it by the contracting parties. The stipulations in said document are clear enough to
indicate that there was no intention at all on the part of the signatories to convey the ownership
of their respective properties; all that was intended, and it was so provided in the agreement,
was to transfer the material possession thereof. In the third condition, the parties retained their
right to alienate their respective properties which right is an element of ownership.

All that the parties acquired was the right of usufruct which in essence is the right to enjoy the
Property of another. Under the document, spouses Paraiso would harvest the crop of the
unirrigate riceland while Baluran could build a house on the residential lot. The mutual
agreement was subject to a resolutory condition, that the children of Natividad, shall choose to
reside in the municipality and build his house on the lot, which would terminate the right of
possession and use.

Usufruct may be constituted for any period of time and under such conditions as they may deem
convenient and beneficial subject to the provision of the Civil Code. The manner of terminating
or extinguishing the right of ususfruct is primarily determined by the stipulations of the parties
which in this case is the happening of the event agreed upon.

The right of usufruct of the parties is extinguished and each is entitled to a return of his property.

NKVS
Moralidad v. Pernes
G.R. No. 152809

FACTS:
While petitioner is in the U.S.A., where she stayed for a long time as teacher, that sometime in
1986, she received news from Arlene, petitioners niece, that Mandug was infested by NPA
rebels and many women and children were victims of crossfire between government troops and
the insurgents. She immediately sent money to Araceli, Arlenes older sister, with instructions to
look for a lot in Davao City where Arlene and her family could transfer and settle down. This was
why she bought the parcel of land in Davao.

Petitioner acquired the lot property initially for the purpose of letting Arlene move from Mandug
to Davao City proper but later she wanted the property to be also available to any of her kins
wishing to live and settle in Davao City.

Following her retirement in 1993, petitioner came back to the Philippines to stay with the
respondents on the house they build on the subject property. In the course of time, their
relations turned sour because members of the Pernes family were impervious to her
suggestions and attempts to change certain practices concerning matters of health and
sanitation within their compound. Violent confrontations meanwhile transpired, with the
petitioner narrating that, at one occasion in July 1998, she sustained cuts and wounds when
Arlene pulled her hair, hit her on the face, neck and back, while her husband Diosdado held her,
twisting her arms in the process.

Petitioner filed with the MTCC of Davao City an unlawful detainer suit against the respondent
spouses. Petitioner alleged that she is the registered owner of the land on which the
respondents built their house; that through her counsel, she sent the respondent spouses a
letter demanding them to vacate the premises and to pay rentals therefor.

In their defense, the respondents alleged having entered the property in question, building their
house thereon and maintaining the same as their residence with petitioners full knowledge and
express consent. To prove their point, they invited attention to her written declaration of July 21,
1986, supra, wherein she expressly signified her desire for the spouses to build their house on
her property and stay thereat for as long as they like.

ISSUES:
1. W/N the agreement for the use of the land constituted the defendants as usufructs.
2. W/N the usufruct is terminated.
3. W/N the defendants, as usufructs, have the right of reimbursement.

HELD:
1
st
Issue: YES, because of the written letter by the petitioner to the defendants, it created the
usufruct between the parties.

What was constituted between the parties herein is one of usufruct over a piece of land, with the
petitioner being the owner of the property upon whom the naked title thereto remained and the
respondents being two (2) among other unnamed usufructuaries who were simply referred to as
petitioners kin.

Usufruct is defined under Article 562 of the Civil Code in the following wise:

ART. 562. Usufruct gives a right to enjoy the property of another with the obligation of
preserving its form and substance, unless the title constituting it or the law otherwise provides.

Usufruct, in essence, is nothing else but simply allowing one to enjoy anothers property. It is
also defined as the right to enjoy the property of another temporarily, including both the jus
utendi and the jus fruendi, with the owner retaining the jus disponendi or the power to alienate
the same.

It is undisputed that petitioner, in a document dated July 21, 1986, supra, made known her
intention to give respondents and her other kins the right to use and to enjoy the fruits of her
property. There can also be no quibbling about the respondents being given the right "to build
their own house" on the property and to stay thereat "as long as they like."

2
nd
Issue: YES, the usufruct was terminated.

The term or period of the usufruct originally specified provides only one of the bases for the right
of a usufructuary to hold and retain possession of the thing given in usufruct. There are other
modes or instances whereby the usufruct shall be considered terminated or extinguished. For
sure, the Civil Code enumerates such other modes of extinguishment:

ART. 603. Usufruct is extinguished:
(1) By the death of the usufructuary, unless a contrary intention clearly appears;
(2) By expiration of the period for which it was constituted, or by the fulfillment of any
resolutory condition provided in the title creating the usufruct;
(3) By merger of the usufruct and ownership in the same person;
(4) By renunciation of the usufructuary;
(5) By the total loss of the thing in usufruct;
(6) By the termination of the right of the person constituting the usufruct;
(7) By prescription.

The document executed by the petitioner dated July 21, 1986 constitutes the title creating, and
sets forth the conditions of, the usufruct. Paragraph #3 thereof states "[T]hat anyone of my kins
may enjoy the privilege to stay therein and may avail the use thereof. Provided, however, that
the same is not inimical to the purpose thereof". What may be inimical to the purpose
constituting the usufruct may be gleaned from the preceding paragraph wherein petitioner made
it abundantly clear "that anybody of my kins who wishes to stay on the aforementioned property
should maintain an atmosphere of cooperation, live in harmony and must avoid bickering with
one another." That the maintenance of a peaceful and harmonious relations between and
among kin constitutes an indispensable condition for the continuance of the usufruct is clearly
deduced from the succeeding Paragraph #4 where petitioner stated "[T]hat anyone of my kins
who cannot conform with the wishes of the undersigned may exercise the freedom to look for
his own." In fine, the occurrence of any of the following: the loss of the atmosphere of
cooperation, the bickering or the cessation of harmonious relationship between/among kin
constitutes a resolutory condition which, by express wish of the petitioner, extinguishes the
usufruct.

From the pleadings submitted by the parties, it is indubitable that there were indeed facts and
circumstances whereby the subject usufruct may be deemed terminated or extinguished by the
occurrence of the resolutory conditions provided for in the title creating the usufruct.

Thus, the Court rules that the continuing animosity between the petitioner and the Pernes family
and the violence and humiliation she was made to endure, despite her advanced age and frail
condition, are enough factual bases to consider the usufruct as having been terminated.

3
rd
Issue: NO, they have no right to reimbursement.

To reiterate, the relationship between the petitioner and respondents respecting the property in
question is one of owner and usufructuary. Accordingly, respondents claim for reimbursement
of the improvements they introduced on the property during the effectivity of the usufruct should
be governed by applicable statutory provisions and principles on usufruct.

By express provision of law, respondents, as usufructuary, do not have the right to
reimbursement for the improvements they may have introduced on the property. We quote
Articles 579 and 580 of the Civil Code:

Art. 579. The usufructuary may make on the property held in usufruct such useful improvements
or expenses for mere pleasure as he may deem proper, provided he does not alter its form or
substance; but he shall have no right to be indemnified therefor. He may, however, remove such
improvements, should it be possible to do so without damage to the property.

Art. 580. The usufructuary may set off the improvements he may have made on the property
against any damage to the same.

Given the foregoing perspective, respondents will have to be ordered to vacate the premises
without any right of reimbursement. If the rule on reimbursement or indemnity were otherwise,
then the usufructuary might, as an author pointed out, improve the owner out of his property.
The respondents may, however, remove or destroy the improvements they may have
introduced thereon without damaging the petitioners property.





EASEMENTS OR SERVITUDES (ART. 613-693)

AMPS
Relova v. Lavarez
9 Phil. 149

DOCTRINE: Servitude may be created by usage for a considerable period of time.

FACTS:
The plaintiff is the owner of a tract of rice land, which is cultivated with the aid of water brought
from the River Bangcabangca, through an aqueduct which passes over the land of the
defendants. On the land of the defendants there was a dam (presa) with a small gate or
aperture in its face which was used to control the flow of the water in the aqueduct, by
permitting a greater or less quantity to escape in a drainage ditch, also on the land of the
defendants.

One of the defendants completely destroyed the dam and let all the water escape by the
drainage ditch, so that none flowed on the land of the plaintiff. At the time when the dam was
destroyed the plaintiff had some five cavanes of land prepared to plant rice, but because of the
escape of the water resulting from the destruction of the dam he was unable to raise his crop,
which was a complete failure.

The trial court granted an injunction upon the fact that the aqueduct and dam in question had
been in use by the plaintiff, as of right, for more than thirty years, and that he had an easement
in the land of the defendants for the maintenance of the said aqueduct and dam, to restrain the
defendants from interfering with the plaintiff's right to the use of the water in the aqueduct, in the
manner heretofore established by custom.

ISSUE:
The existence of servitude for the maintenance of the dam upon the land of the defendants in
favor of the land of the plaintiff.

HELD:
The aqueduct and the dam (presa) have been in existence for more than thirty years, during
which period the owner of the land in question has always exercised the right to the reasonable
use of the water in the aqueduct for irrigation purposes. (Arts. 527 and 528, Civil Code.)

Counsel for the appellants contend that under the definition of a servitude which appears in
article 530 of the Civil Code the existence of the servitude cannot be established unless it
appears that from such servitude a benefit (beneficio) was, or might be, derived by the plaintiff
landowner; and that since it appears from the testimony of the witnesses that the aperture in the
dam was used for the purpose of controlling the flow of water in the aqueduct and for preventing
damage by overflow to the lowlands over which the aqueduct runs, and since it appears that the
lands of the plaintiff are higher than the lands of the defendants, therefore the aqueduct could
never have been intended for the supply of water to the lands of the plaintiff and neither the dam
nor the aqueduct could be of any benefit to these lands.

This contention cannot be maintained in the face of the positive testimony as to the existence of
the aqueduct and its use for many years to supply water to the lands in question. It may be that
the defendants had a right to open the aperture in the face of the dam to prevent a destructive
overflow of water on their lands, but this would not give them the right to stop the flow of water
altogether; nor does it tend to establish the contention of the defendants that the plaintiff
landowner is not entitled to the benefit of the reasonable use of the water flowing in the
aqueduct, since it does not appear that such use necessarily involved destructive overflows
from the aqueduct, provided the flow of water therein was properly regulated by the opening of
the aperture in the dam.

AFFIRMED.

KGS
Solid Manila v. Bio Hong Trading
195 SCRA 748

DOCTRINES:
1. Servitudes are merely accessories to the tenements of which they form part, and even if
they are possessed of a separate juridical existence, they cannot be alienated from the
tenement or mortgaged separately.

2. The vendee of real property in which a servitude or easement exists, did not acquire the
right to close that servitude or put up obstructions thereon, to prevent the public from using it.

FACTS:
Solid Manila Corp. (petitioner) owns a parcel of land located in Ermita, Manila. The same lies in
the vicinity of another parcel owned by Bio Hong Trading (respondent). The latters title came
from a prior owner, and in their deed of sale, the parties thereto reserved as an easement of
way approx. 914sqm converted as a private alley for the benefit of the neighboring estates. As a
consequence, an annotation was entered in the respondents title. The petitioners and its
neighbors made use of the private alley and maintained and contributed to its upkeep.
Thereafter, respondent constructed steel gates that precluded unhampered use. Respondent
filed a case to remove said gates and to allow full access to the easement, which was granted.
In the Court of Appeals, it was held that since respondent has acquired title to the property,
merger brought about an extinguishment of the easement.

ISSUE:
Whether the easement still exists or had been extinguished by merger?

HELD:
The easement still exists on the property of Bio Hong Trading.

It is true that the sale did include the alley. On this score, the Court rejects the petitioner's
contention that the deed of sale "excluded" it, because as a mere right-of-way, it cannot be
separated from the tenement and maintains an independent existence. Thus: Art. 617.
Easements are inseparable from the estate to which they actively or passively belong.
Servitudes are merely accessories to the tenements of which they form part. Although they are
possessed of a separate juridical existence, as mere accessories, they cannot, however, be
alienated from the tenement, or mortgaged separately.
The fact, however, that the alley in question, as an easement, is inseparable from the main lot is
no argument to defeat the petitioner's claims, because as an easement precisely, it operates as
a limitation on the title of the owner of the servient estate, specifically, his right to use (jus
utendi). As the petitioner indeed hastens to point out, the deed itself stipulated that "a portion
thereof [of the tenement] measuring 914sqm, more or less, had been converted into a private
alley for the benefit of the neighboring estates. . ." and precisely, the former owner, in
conveying the property, gave the private owner a discount on account of the easement.
Hence, and so we reiterate, albeit the private respondent did acquire ownership over the
property including the disputed alley as a result of the conveyance, it did not acquire the
right to close that alley or otherwise put up obstructions thereon and thus prevent the public
from using it, because as a servitude, the alley is supposed to be open to the public.
The Court is furthermore of the opinion, contrary to that of the Court of Appeals, that no genuine
merger took place as a consequence of the sale in favor of the private respondent corporation.
According to the Civil Code, a merger exists when ownership of the dominant and servient
estates is consolidated in the same person. Merger then, as can be seen, requires full
ownership of both estates. One thing ought to be noted here, however. The servitude in
question is a personal servitude, that is to say, one constituted not in favor of a particular
tenement (a real servitude) but rather, for the benefit of the general public.
Personal servitudes are referred to in the following article of the Civil Code: Art. 614. Servitudes
may also be established for the benefit of a community, or of one or more persons to whom the
encumbered estate does not belong.
In a personal servitude, there is therefore no "owner of a dominant tenement" to speak of, and
the easement pertains to persons without a dominant estate, in this case, the public at large.
Merger, as we said, presupposes the existence of a prior servient-dominant owner relationship,
and the termination of that relation leaves the easement of no use. Unless the owner conveys
the property in favor of the public if that is possible no genuine merger can take place that
would terminate a personal easement.
JPOT
CID v. Javier
108 Phil. 850

DOCTRINE: "With regard to easements in the negative, from dominant to a servient, it must be
accompanied by a 'formal act.' The law is explicit in stating that it should be in an instrument
acknowledged by a notary public."

FACTS:
The review for certiorari arises with the conflict of Javier effectively blocking the easement of
light and view, although both structures are in accordance with their corresponding lots. The
conflict can be assisted by the interpretation of a formal act which has different applications in
the Spanish Civil Code as well as our Civil Code. In the former, it was discussed that any act
would suffice. As long as there is a positive act of prohibition whether orally or implied. However,
the distinction was made apparent in the latter's discussion. The court does not aver and makes
plain and distinguishes between a formal act. In this case, the effectivity of said act would be in
the negative. This act (i.e. prohibition) would be coming from the dominant directed towards the
servient. Moreover, such an act could only be validly appreciated once it is in a legal document
that is notarized. Only with such acknowledgement can an easement be binding and bar anyone
who committs to undermine and deprive the right of a land owner to his pleasures, in this
instance privacy, sunlight and a view.

ISSUE:
W/N respondents Irene P. Javier, et al., owners of a building standing on their lot with windows
overlooking the adjacent lot, had acquired by prescription an enforceable easement of light and
view arising from a verbal prohibition to obstruct such view and light, alleged to have been made
upon petitioner's predecessor-in-interest as owner of the adjoining lot, both of which lots being
covered by Torrens titles.

HELD:
NO. The requirement for an easement to be applicable would be that it be annotated and affixed
together with the registry of the Torrens title specifying the prohibition in question. Also, Article
538 provides that the requirement to be fulfilled for prescription to run is that a formal act such
as prohibition coming from a dominant estate to a servient one be in an instrument duly
acknowledged by a notary public. Otherwise, there is no prescription or easement to speak of.

MLAV
Cortes v. Yu-Tibo
2 Phil. 24

DOCTRINE: An easement of light and view is a negative easement. When easement is
negative, there should be a formal act of opposition for prescription to run.

FACTS:
Cortes wife owns a house (No. 65) in which certain windows open on the adjacent property (No.
63), a neighboring house on the same street. This setting has been in existent since 1843. The
tenant of the adjacent property raised the roof of house No. 63 in such a manner that half of the
windows of house No. 65 has been covered, thus depriving it of a large part of air and light.

Plaintiff contends that by the constant and uninterrupted use of the windows for 59 years, he
acquired from prescription an easement of light in favor of house No. 65, and as a servitude
upon house No. 63. Consequently, he has acquired the right to restrain the making of any
improvement in the latter house which may be prejudicial to the enjoyment of the easement.
Further, he contends that the easement of light is positive; and that therefore the period of
possession for the purposes of the acquisition of a prescriptive title is to begin from the date on
which the enjoyment of the same commenced, or, in other words, from the time that said
windows were opened with the knowledge of the owner of the house No. 63, and without
opposition on this part.

Defendant contends that the easement is negative, and therefore the time for the prescriptive
acquisition must begin from the date on which the owner of the dominant estate may have
prohibited, by a formal act, the owner of the servient estate from doing something which would
be lawful but for the existence of the easement.

Lower court ruled in favor of the defendant. Plaintiff appealed the case.

ISSUE:
WON the easement is positive/negative

HELD:
Easement is negative.

The easement of light which is the object of this litigation is of a negative character, and
therefore pertains to the class which cannot be acquired by prescription as provided by article
538 of the Civil Code, except by counting the time of possession from the date on which the
owner of the dominant estate has, in a formal manner, forbidden the owner of the servient
estate to do an act which would be lawful were it not for the easement.

In consequence, the plaintiff, not having executed any formal act of opposition to the right of the
owner of house No. 63 to make improvements which might obstruct the light of house No. 65, at
any time prior to the complaint, has not acquired, nor could he acquire by prescription, such
easement of light, no matter how long a time have elapsed. Because the period which the law
demands for such prescriptive acquisition could not have commenced to run, the act with which
it must necessarily commence not having been performed.

DJTV
Abellana v. CA
208 SCRA 316

DOCTRINES:
The use of a footpath or road may be apparent but it is not a continuous easement
because its use is at intervals and depends upon the acts of man.
A right of way is not acquirable by prescription.

FACTS:
The petitioners who live on a parcel of land abutting the northwestern side of the Nonoc Homes
Subdivision, sued to establish an easement of right of way over a subdivision road which,
according to the petitioners, used to be a mere footpath which they and their ancestors had
been using since time immemorial, and that, hence, they had acquired, through prescription, an
easement of right of way therein. The construction of a wall by the respondents around the
subdivision deprived the petitioners of the use of the subdivision road which gives the
subdivision residents access to the public highway. They asked that the high concrete walls
enclosing the subdivision and cutting of their access to the subdivision road be removed and
that the road be opened to them.

The private respondents denied that there was a pre-existing footpath in the place before it was
developed into a subdivision. They alleged furthermore that the Nonoc Subdivision roads are
not the shortest way to a public road for there is a more direct route from the petitioners' land to
the public highway.

After trial, the trial court ordered to demolish the subject fences or enclosures at the dead ends
of Road Lots 1 and 3 of the Nonoc Homes Subdivision at their expense and to leave them open
for the use of the plaintiffs and the general public.

However, on appeal by the defendants and intervenors (now private respondents), the appellate
court reversed the appealed judgment. It found that requisites essential for the grant of an
easement of right of way are not obtaining in this case hence no alternative presents itself
except reversal of the judgment.

The appellate court denied petitioners' motion for reconsideration of the aforesaid decision.
Hence, this petition for review.

ISSUES:

Whether or not the Court of Appeals erred:

1. 1.in not holding that the easement claimed by them is a legal easement
established by law (Art. 619. Civil Code) and acquired by them by virtue of a title under Art.
620, Civil Code and P.D. No. 957 through the National Housing Authority which has
exclusive jurisdiction to regulate subdivision and condominium projects;

2. 2.in not holding that the footpaths and passageways which were
converted into subdivision road lots have acquired the status of public streets in view of
Section 4 of Municipal Ordinance No. 1, Series of 1969 of Talisay, Cebu which provides that
subdivision roads shall be used not only for the exclusive use of the homeowners but also
for the general public, and Section 5 of Ordinance No. 5, Series of 1974, which provides that
"those subdivision road lots whose use by the public are (sic) deemed necessary by the
proper authorities shall be made available for public use"; and

3. 3.in not determining whether or not the closure of the dead ends of road
lots 1 and 3 of the Nonoc Homes Subdivision by the private respondents was legal.

HELD: NO
Petitioners' assumption that an easement of right of way is continuous and apparent and may
be acquired by prescription under Article 620 of the Civil Code, is erroneous. The use of a
footpath or road may be apparent but it is not a continuous easement because its use is at
intervals and depends upon the acts of man. It can be exercised only if a man passes or puts
his feet over somebody else's land. Hence, a right of way is not acquirable by prescription.

Neither may petitioners invoke Section 29 of P.D. 957 which provides:

Sec. 29.Right of Way to Public Road. The owner or developer of a subdivision without
access to any existing public road or street must secure a right of way to a public road or
street and such right of way must be developed and maintained according to the
requirement of the government authorities concerned.

The above provision applies to the owner or developer of a subdivision (which petitioners are
not) without access to a public highway.

The petitioners' allegation that the footpaths which were converted to subdivision roads have
acquired the status of public streets, is not well taken. In the first place, whether or not footpaths
previously existed in the area which is now known as the Nonoc Homes Subdivision, is a factual
issue which this Court may not determine for it is not a trier of facts.

The municipal ordinances which declared subdivision roads open to public use "when deemed
necessary by the proper authorities" simply allow persons other than the residents of the Nonoc
Homes Subdivision, to use the roads therein when they are inside the subdivision but those
ordinances do not give outsiders a right to open the subdivision walls so they can enter the
subdivision from the back. As the private respondents pointed out in their Comment:

The closure of the dead ends of road lots 1 and 3 is a valid exercise of proprietary rights. It is for
the protection of residents in the subdivision from night prowlers and thieves. And the public is
not denied use of the subdivision roads, only that the users must get inside the subdivision
through the open ends of the road lots that link the same to the public road. It is common to
most, if not all subdivisions in Cebu, Metro Manila and other places, that points of ingress to and
egress from the subdivision are the points where the subdivision roads intersect with public
roads. It is of judicial notice that most, if not all, subdivisions are enclosed and fenced with only
one or few points that are used as ingress to and egress from the subdivisions.

WHEREFORE, finding no merit in the petition for review, the same is DENIED with costs
against the petitioners.

JGY
Ronquillo v. Roco
103 Phil. 84

DOCTRINE: An easement of right of way being discontinuous, cannot be acquired through
prescription but only by virtue of a title.

FACTS:
Petitioners Ronquillo have been in the continuous and uninterrupted use of a road or passage
way which traversed the land of the Respondents Roco and their predecessors in interest, in
going to Igualdad Street and the market place of Naga City, from their residential land and back,
for more than 20 years.

Respondents Roco and his men started constructing a chapel in the middle of the said right of
way construction actually obstructed the continuous exercise of the rights of the plaintiffs over
said right of way. Respondents also forcibly planted wooden posts, fenced with barbed wire and
closed the road passage way.

ISSUE:
WON an easement of right of way can be acquired thru prescription -- NO

HELD:
The dismissal was based on the ground that an easement of right of way though it may be
apparent is, nevertheless, discontinuous or intermittent and, therefore, cannot be acquired
through prescription, but only by virtue of a title.

Easements may be Continuous or discontinuous, apparent or non-apparent, discontinuous
being those used at more or less long intervals and which depend upon acts of man.
Continuous and apparent easements are acquired either by title or prescription, continuous non-
apparent easements and discontinuous ones whether apparent or not, may be acquired only by
virtue of a title. Both Manresa and Sanchez Roman are of the opinion that the easement of right
of way is a discontinuous one.

JRPA
Amor v. Florentino
74 Phil. 404
FACTS:
Maria Florentino owned a house and a camarin (warehouse). By a will, she transferred the
house to Jose Florentino and the warehouse to Maria Florentino. Maria sold the warehouse to
Amor. Amor then demolished the old warehouse in order to build a new 2-storey structure. The
problem is it will shut off the light and air that come in through the window of the adjacent house
owned by Jose. Hence the latter files for prohibition claiming there is a negative easement
prohibiting Amor from constructing any structure at any height that would block the window.
Amor counters that there is no easement. Moreover, since the death of testator was before the
Civil Code took effect, the rules on easement do not apply.

ISSUES:
1. Whether or not there is an easement prohibiting Amor from doing said construction.
2. Whether or not the Civil Code may be applied
HELD:
1. Yes. Easement are established by law or by will of the owners or by title.
Under Art. 624, there is title by the doctrine of apparent sign. When the estate is
subsequently owned by two different persons and the service (it cannot be an easement
before the transfer) is not revoked in the title nor removed, an easement is established.

The Cortez case cannot be invoked by Amor because it involved acquisition by prescription.
Art. 624 is acquisition by title.

2. Amor failed to prove that the death of the testator occurred before the
effectivity of the Old Civil Code. The facts show that it happened after the effectivity of the
said code so the law on easement is already applicable. In any case, even if we assume
Amors supposition, the law on easement was already integrated into the Spanish Law and
in fact, had been established by Jurisprudence.

Therefore, Amor is prohibitied from constructing the warehouse above the level of the window.

ABB
Gargantos v. Tan Yanon
108 Phil. 888

Doctrine: Art. 624 provides that when two adjoining estates were formerly owned by one
person who introduced improvements on both such that the wall of the house constructed on
the first estate extends to the wall of the warehouse on the second estate; and at the time of the
sale of the first estate, there existed on the aforementioned wall of the house, doors, windows
which serve as passages for light and view, there being no provision in the deed of sale that the
easement of light and view will not be established, the apparent sign of easement between the
two estates is established as a title.

FACTS:
The late Francisco Sanz was the previous owner of a land which he subdivided into several lots.
One lot was sold to Guillermo Tengtio, who sold to Vicente Uy Veza. Another lot with a house
constituted thereon was sold to Tan Yanon. A third portion with a warehouse was sold to
Gargantos. The problem arose when Gargantos asked from the Municipality for a permit to
demolish the warehouse in order to construct a higher one. Tan Yanon opposed the application
for it would block his window and impair his right of light and view. Both the provincial fiscal and
district engineer of Romblon recommended granting of the building permit to Gargantos. Tan
Yanon then filed against Gargantos an action to restrain him from constructing a building that
would prevent plaintiff from receiving light and enjoying the view through the window of his
house, unless such building is erected at a distance of not less than three meters from the
boundary line between the lots of plaintiff and defendant, and to enjoin the members of
Municipal Council of Romblon from issuing the corresponding building permit to defendant.

ISSUE:
Whether the property of respondent Tan Yanon has an easement of light and view against the
property of petitioner Gargantos. -- YES

HELD:
Art. 624 provides that when two adjoining estates were formerly owned by one person who
introduced improvements on both such that the wall of the house constructed on the first estate
extends to the wall of the warehouse on the second estate; and at the time of the sale of the first
estate, there existed on the aforementioned wall of the house, doors, windows which serve as
passages for light and view, there being no provision in the deed of sale that the easement of
light and view will not be established, the apparent sign of easement between the two estates is
established as a title.

FZC
Valderama v. North Negros Sugar Central
48 Phil. 492

DOCTRINE: It is against the nature of the easement to pretend that it was established in favor
of the servient estates, because it is a well settled rule that things serve their owner by reason of
ownership and not by reason of easement.

FACTS:
Several hacienda owners in Manapla, Occidental Negros, entered into a milling contract with
Miguel Osorio wherein the latter would build a sugar central of a minimum capacity of 300 tons
for the milling and grinding of all the sugar cane to be grown by the hacienda owners who in turn
would furnish the central with all the cane they might produce in their estates for 30 years from
the execution of the contract. Later on, Osorios rights and interests were acquired by the North
Negros Sugar Co., Inc. 2 years after, the current petitioners, Catalino Valderrama, Emilio
Rodriguez, Santos Urra et. al, made other milling contracts identical to the first one with the
North Negros Sugar, Co., Inc. The hacienda owners, however, could not furnish the central
sufficient cane for milling as required by its capacity, so the North Negros made other milling
contracts with the various hacienda owners of Cadiz, Occidental Negros. This gave rise to the
plaintiffs filing their complaint, alleging that the easement of way, which each of them has
established in his respective hacienda, was only for the transportation through each hacienda of
the sugar cane of the owner thereof, while the defendant maintains that it had the right to
transport to its central upon the railroad passing through the haciendas of the plaintiffs, not only
the sugar cane harvested in said haciendas, but also that of the hacienda owners of Cadiz,
Occidental Negros.

The CFI entered 1 single judgment for all of them, ruling in Valderrama et. als favor finding that
North Negros had no right to pass through the lands of the hacienda owners for the
transportation of sugar cane not grown from their lands. Thus the appeal to the SC.

ISSUE:
WON the easement of way established was restricted to transporting only sugar cane from the
hacienda owners lands. -- NO

HELD:
The contract entered into by each of the hacienda owners contained a clause that granted the
North Negros an easement of way 7 meters wide for the period of 50 years upon their properties
for the construction of a railroad. The owners allege ambiguity since it could permit the
transportation of sugar cane which they did not produce which is contrary to their intent but the
SC held that it is clear that the easement was established for the benefit of all producers and of
the corporation as it is the intent of the milling contract.

Since the easement is a voluntary, apparent, continuous easement of way in favor of the
corporation, it is contrary to the nature of the contract that it is only limited to canes produced by
the servient estates since it is a well settled rule that things serve their owner by reason of
ownership and not by reason of easement. The owners also cannot limit its use for there is
nothing in the contract prohibiting the central from obtaining other sources.

Transporting cane from Cadiz also does not make it more burdensome since what is prohibited
in Art. 543 of the CC is that in extending the road or in repairing it, it should occupy a greater
area or deposit excavations outside the granted 7 meters. This does not happen in this case
when the North Negros transports sugar cane from Cadiz, crossing the servient estates, since it
continues to occupy the same area and the encumbrance is still the same regardless of the
number of times it passes through the estates.

Also the period of the easement is longer than the period of the milling contracts, so even if the
owners no longer desire to furnish the central canes for milling, the North Negros still has the
right to the easement for the remaining period so the contention that it should be limited to the
canes produced by the owners has no basis.

LNAC
Javellana v. IAC
172 SCRA 280

DOCTRINE: They closed the entrance of the canal and demolished portions of the main dike
thus impairing the use of the servitude by the dominant estate. And by so doing, plaintiffs
violated not only the law on easement but also Presidential Degree No. 296 which enjoins any
person, natural or juridical, to demolish structures or improvements which tend to obstruct the
flow of water through rivers, creeks, esteros and drainage channels. For this canal did not serve
merely to supply salt water to the school fishpond but also serves as drainage charged or
channel of rainwater from adjacent lands to the Iloilo River.

FACTS:
Marsal & Co., Inc., and Marcelino Florete, Sr. is the present owner of the land adjoining the Iloilo
River up to the adjacent lot where the L. Borres Elem. School is located. There existed a main
canal from the Iloilo River which passes through the Marsal property and through a canal that
traverses the school property going towards Lot 2344. Marsal & Co. closed the dike entrance
and later on demolished the portions of the main dike connecting the main canal to the canal
running through the school grounds. This closure caused flooding in the premises of the school
and its vicinity because the canal serves as outlet of rain or flood water that empties into the
river. This prompted the school and barangay officials to complain to higher authorities about
the closure of the canal. When Florete was about to bury a pipe in lieu of an open canal, he was
prevented from doing so by District Supervisor Javellana.

Florete instituted a complaint for recovery of damages for allegedly denying his access to the
use of the canal to his property.

RTC ruled in favor of Javellana.

Florete appealed to the IAC which reversed the decision.

Thus, Javellana instituted herein recourse.

ISSUE:
Whether an easement has been constituted on the subject property. -- YES

HELD:
A positive easement of water-right-of-way was constituted on the property of Florete as the
servient estate in favor of the L. Borres Elementary School and the nearby lands as dominant
estates since it has been in continuous use for no less than 15 years by the school fishpond as
well as by the nearby adjacent lands.

As a positive easement, Florete had no right to terminate the use of the canal without violating
Art. 629 of the CC which provides that The owner of the servient estate cannot impair, in any
manner whatsoever, the use of the servitude. Nevertheless if by reason of the place originally
assigned or of the manner established for the use of the easement, the same should become
very inconvenient to the owner of the servient estate, or should prevent him from making any
important works, repairs or improvements thereon, it may be charged at his expense, provided
he offers another place or manner equally convenient and in such a way that no injury is caused
thereby to the owner of the dominant estate or to those who may have a right to the use of the
easement.

Defendants' closure of the dike's entrance connecting the main canal with the canal running
through the school premises, therefore, caused the flooding of the premises of the L. Borres
Elementary School and its vicinity. This is so because during rainy season, said canal also
serves as outlet of rain or flood waters that empties to the Iloilo River. Witnesses Ignacio
Gencianeo, Francisco Regacho, Severo Maranon and Barangay Captain Antonio Sison were
unanimous in declaring so.

TKDC
Benedicto v. CA
25 SCRA 145

DOCTRINE: The easement is perpetual in character and was annotated on all the transfer
certificates of title issued to Heras and to Benedicto. Absence of anything that would show
mutual agreement to extinguish the easement, the easement persists.

FACTS:
Heras filed an action with the Court of First Instance to recover a portion of land enclosed and
walled by Benedicto and to demand the reopening of an easement of way between his and
Benedictos real property. Hendrick sold portions of her property to several personalities
including Recto and Heras. When portion of the property was sold to Herras, he closed and
walled the part of land serving as easement of way.

Trial court found that the easement of way was found entirely within Benedictos property
contrary to the stipulation in the deed of sale between Hedrick and Recto that it should be
between their properties with each contributing an equal portion of his property. Thus, trial court
directed the parties to equally contribute to the maintenance of the passageway between Herras
and Benedicto.

ISSUE:
Whether or not Benedicto may enclose his property.

HELD:
In this case, the easement is perpetual in character and was annotated on all the transfer
certificates of title issued to Heras and to Benedicto. Absence of anything that would show
mutual agreement to extinguish the easement, the easement persists.

TKDC
Benedicto v. CA
25 SCRA 145

DOCTRINE: Under Art. 624, an easement may continue by operation of law. Alienation of the
dominant and servient estates to different persons is not a ground for the extinguishment of
easements, absent a statement extinguishing it.

FACTS:
Private respondent Antonio Cardenas was the owner of 2 parcels of land situated in Cebu City.
An apartment building was constructed on Lot A, while on Lot B stands a 4-door apartment, a 2-
storey house, a bodega, and a septic tank for the common use of the occupants of Lots A and B.
A small portion of the apartment building on Lot A also stands on Lot B.

Cardenas sold Lot A to petitioner Eduardo C. Taedo. On the same day, Cardenas also
mortgaged Lot B to Taedo as a security for the payment of a loan. Cardenas agreed that he
would sell Lot B only to Taedo in case he should decide to sell it. However, Cardenas sold Lot
B to the respondent spouses Romeo and Pacita Sim.

Upon learning of the sale, Taedo offered to redeem the property from Romeo Sim, but the
latter refused. Instead, Sim blocked the sewage pipe connecting the building of Eduardo
Taedo built on Lot A, to the septic tank in Lot B. He also asked Taedo to remove that portion
of his building enroaching on Lot B.

As a result, Taedo filed an action for legal redemption and damages against Spouses Sim and
Antonio Cardenas, invoking the provisions of Art. 1622 of the Civil Code. Respondent judge,
Juanito A. Bernad, dismissed the complaint for legal redemption, as well as petitioners motion
for reconsideration. Hence, this petition for review on certiorari.

ISSUE:
W/N the alienation of Lots A and B is a ground for the extinguishment of the easement of
drainage. -- NO

HELD:
The finding of the trial court that Taedo's right to continue to use the septic tank on Lot B
ceased upon the subdivision of the land and its subsequent sale to different owners who do not
have the same interest, also appears to be contrary to law.

Article 631 of the Civil Code enumerates the grounds for the extinguishment of an easement.
From its provisions, the alienation of the dominant and servient estates to different persons is
not one of the grounds for the extinguishment of an easement. On the contrary, use of the
easement is continued by operation of law. Article 624 of the Civil Code provides:

Art. 624. The existence of an apparent sign of easement between two estates, established or
maintained by the owner of both, shall be considered, should either of them be alienated, as a
title in order that the easement may continue actively and passively, unless, at the time the
ownership of the two estates is divided, the contrary should be provided in the title of
conveyance of either of them, or the sign aforesaid should be removed before the execution of
the deed. This provision shall also apply in case of the division of a thing owned in common by
two or more persons.

In the instant case, no statement abolishing or extinguishing the easement of drainage was
mentioned in the deed of sale of Lot A to Taedo. Nor did Cardenas stop the use of the drain
pipe and septic tank by the occupants of Lot A before he sold said lot. Hence, the use of the
septic tank is continued by operation of law. Accordingly, the spouses Romeo and Pacita Sim
the new owners of the servient estate (Lot B), cannot impair, in any manner whatsoever, the use
of the servitude.

CRF
Alcantara v. Rita
372 SCRA 364

DOCTRINE: Construction of a house on the lot of another to facilitate the utilization of usufruct
may constitute as personal easement pursuant to Article 614. (batasnatin)

FACTS:
Petitioner filed a complaint against Cornelio B. Reta, Jr. for the exercise of the right of right of
first refusal under Presidential Decree No. 1517, injunction with preliminary injunction, attorneys
fees and nullity of amicable settlement. The plaintiffs claimed that they were tenants or lessees
of the land located in Barangay Sasa, Davao City, covered by Transfer Certificate of Title No. T-
72594, owned by Reta; that the land has been converted by Reta into a commercial center; and
that Reta is threatening to eject them from the land. They assert that they have the right of first
refusal to purchase the land in accordance with Section 3(g) of Presidential Decree No. 1517
since they are legitimate tenants or lessees thereof.

On the other hand, Reta claimed that the land is beyond the ambit of Presidential Decree No.
1517 since it has not been proclaimed as an Urban Land Reform Zone; that the applicable law
is Batas Pambansa Blg. 25 for failure of the plaintiffs to pay the rentals for the use of the land;
and that the amicable settlement between him and Ricardo Roble was translated to the latter
and fully explained in his own dialect.

ISSUE:
Whether or not the petitioners have the right of first refusal under Presidential Decree No. 1517.
-- NO

HELD:
None of the petitioners is qualified to exercise the right of first refusal under P. D. No. 1517. The
area involved has not been proclaimed an Urban Land Reform Zone (ULRZ).To be able to
qualify and avail oneself of the rights and privileges granted by the said decree, one must be:
(1) a legitimate tenant of the land for ten (10) years or more; (2) must have built his home on the
land by contract; and, (3) has resided continuously for the last ten (10) years. Obviously, those
who do not fall within the said category cannot be considered legitimate tenants and, therefore,
not entitled to the right of first refusal to purchase the property should the owner of the land
decide to sell the same at a reasonable price within a reasonable time. Respondent Reta
allowed petitioner only usufruct to the property of another with the obligation of preserving its
form and substance, unless the title constituting it or the law otherwise provides. Petitioner
Roble was allowed to construct his house on the land because it would facilitate his gathering of
tuba. This would be in the nature of a personal easement under Article 614 of the Civil Code.

A contract has been defined as a meeting of the minds between two persons whereby one
binds himself, with respect to the other, to give something or to render some service. Clearly,
from the moment respondent Reta demanded that the petitioners vacate the premises, the
verbal lease agreements, which were on a monthly basis since rentals were paid monthly,
ceased to exist as there was termination of the lease. Indeed, none of the petitioners is qualified
to exercise the right of first refusal under P. D. No. 1517. Another factor which militates against
petitioners claim is the fact that there is no intention on the part of respondent Reta to sell the
property. Hence, even if the petitioners had the right of first refusal, the situation which would
allow the exercise of that right, that is, the sale or intended sale of the land, has not happened.
P. D. No. 1517 applies where the owner of the property intends to sell it to a third party.

MPF
Costabella Corp. v. CA
193 SCRA 333

DOCTRINE:An easement of right of way is discontinuous and as such cannot be acquired by
prescription. Convenience of the dominant estate is not a gauge for the grant of compulsary
right of way.

While a right of way is legally demandable, the owner of the dominant estate is not at liberty to
impose one based on arbitrary choice. Under Article 650 of the Code, it shall be established
upon two criteria: (1) at the point least prejudicial to the servient estate; and (2) where the
distance to a public highway may be the shortest.

FACTS:
Petitioners owned a lot wherein they started constructing their beach hotel.
Before such construction, the private respondent, in going to and from their respective
properties and the provincial road, passed through a passageway, which traversed the
petitioners property.
As a result of the construction, this passageway, including the alternative route, was
obstructed.
Private respondent filed for injunction plus damages.
In the same complaint the private respondents also alleged that the petitioner
had constructed a dike on the beach fronting the latters property without the necessary
permit, obstructing the passage of the residents and local fishermen, and trapping debris of
flotsam on the beach.
The private respondent also claim that they have acquired the right of way
through prescription.
They prayed for the re-opening of the ancient road right of way (what they called
the supposed easement in this case) and the destruction of the dike.
Petitioner answered by saying that their predecessor in interests act of allowing
them to pass was gratuitous and in fact, they were just tolerating the use of the private
respondents. CA ruled in favor of the private respondents.

ISSUES:
1. Whether or not easement of right and way can be acquired through prescription? -- NO
2. Whether or not the private respondents had acquired an easement of right of way in the
form of a passageway, on the petitioners property? -- NO

HELD:
1. Easement of right of way is discontinuous thus it cannot be subject to acquisitive
prescription.

2. One may validly claim an easement of right of way when he has proven the: (1) the
dominant estate is surrounded by other immovables and has no adequate outlet to a
public highway; (2) proper indemnity has been paid; (3) the isolation was not due to acts
of the proprietor of the dominant estate; (4) the right of way claimed is at point least
prejudicial to the servient estate. The private respondent failed to prove that there is no
adequate outlet from their respective properties to a public highway; in fact the lower
court confirmed that there is another outlet for the private respondents to the main road
(yet they ruled in favor of the private respondents).

Apparently, the CA lost sight of the fact that the convenience of the dominant estate was
never a gauge for the grant of compulsory right of way. There must be a real necessity
and not mere convenience for the dominant estate to acquire such easement. Also, the
private respondents made no mention of their intention to indemnify the petitioners. The
SC also clarified that least prejudicial prevails over shortest distance (so shortest
distance isnt necessarily the best choice).

AMDG
Villanueva v. Velasco
346 SCRA 99

DOCTRINE: A legal easement is one mandated by law, constituted for public use or for private
interest and becomes a continuing property right. As a compulsory easement, it is inseparable
from the estate to which it belongs as provided for Article 617 of the Civil Code.

FACTS:
Villanueva is currently registered owner of land that he bought from Pacific Bank Corporation.
The bank is the mortgagee of the land and bought it from Maximo and Justina Gabriel at a
public auction. It is noted that before Villanueva purchased the property, a two-meter wide
easement of right of way was granted by the spouses Gabriel in favor of the Espinolas, the
predecessor-in-interest of private respondents Sebastian and Lorilla. In addition, Gabriel
constructed a small house that encroached one-meter of the easement right.

A Civil Case was filed by Sebastian and Lorilla against the spouses Gabriel praying for the
easement right to be enforced. The trial court and Court of Appeals ruled in their favor and
ordered the demolition of the house. Consequently, an Alias Writ of Demolition was issued. It is
noted that at this time, Villanueva is now the owner of the property. Villanueva filed a Third Party
Claim with Prayer to Quash Alias of Demolition which was later on denied. The Court of Appeals
also dismissed his petition for certiorari.

Villanueva is arguing that the civil case decision cannot be enforced against him since (1) the
easement right of way was not annotated in his title and (2) he was not a party in the civil case.

ISSUE:
1. Whether or not a right of way can be enforced against Villanueva although it was not
annotated in his title YES
2. Whether or not the civil case decision can be enforced against Villanueva even though
he was not a party to the civil case YES

HELD:
The Court of Appeals correctly identified that the contract of easement present in this case is
both voluntary and legal easement. A legal easement is one mandated by law, constituted for
public use or for private interest, and becomes a continuing property right. As a compulsory
easement, it is inseparable from the estate to which it belongs, as provided for in Article 617 of
the Civil Code.

The essential requisites for an easement to be compulsory are:
1. The dominant estate is surrounded by other immovable and has no adequate outlet to a
public highway
2. Proper indemnity has been paid
3. The isolation was not due to acts of the proprietor of the dominant estate
4. The right of way claimed is at a point at least prejudicial to the servient estate
5. To the extent consisted with the foregoing rule, where the distance from the dominant
estate to a public highway may be the shortest.

Having established that the easement right present in this case is legal in nature, it follows that
the servient estate (Villanueva) is legally bound to adhere to his obligation of providing the
dominant estate (Sebastian and Lorilla) its right of way. Applying this to the case, the one-meter
wide easement is insufficient for the needs of the private respondents which is why Villanueva is
obligated to demolish the house to be able to conform to what is stated in the contract of
easement.

As regards Villanuevas second argument, a decision in a case is conclusive and binding upon
the parties as well as to its successor in interest by title. It is clear from the facts that the civil
case was decided almost four years before Villanueva purchased the property. Being the
successor in interest, the civil case decision binds Villanueva.

GCG
Cristobal v. CA
291 SCRA 122

DOCTRINE: To be entitled to a compulsory easement of right of way, the preconditions
provided under Arts. 649 and 650 of the Civil Code must be established. These are: (1) that the
dominant estate is surrounded by other immovables and has no adequate outlet to a public
highway; (2) that proper indemnity has been paid; (3) that the isolation was not due to acts of
the proprietor of the dominant estate; (4) that the right of way claimed is at a point least
prejudicial to the servient estate and, in so far as consistent with this rule, where the distance
from the dominant estate to a public highway may be the shortest. The burden of proving the
existence of these prerequisites lies on the owner of the dominant estate.
FACTS:
Petitioners own a house and lot situated at No. 10 Visayas Avenue Extension, Quezon City.
They have already been residing there since 1961.
Respondent Cesar Ledesma, Inc., on the other hand, is the owner of a subdivision at Barrio
Culiat along Visayas Avenue. It included the disputed residential lots, Lot 1 and Lot 2.
The said lots were originally part of a private road known as Road Lot 2 owned exclusively by
Cesar Ledesma, Inc. When Visayas Avenue became operational as a national road in 1979,
Cesar Ledesma Inc., filed a petition before the RTC of Quezon City to be allowed to convert
Road Lot 2 into residential lots.
The petition was granted. Road Lot 2 was converted into residential lots designated as Lot 1
and Lot 2. Subsequently, Cesar Ledesma, Inc. sold both lots to Macario Pacione in whose favor
Transfer Certificates of Title were correspondingly issued.
In turn, Macario Pacione conveyed the lots to his son and daughter-in-law, respondent spouses
Jesus and Lerma Pacione. When the Pacione spouses, who intended to build a house on Lot 1,
visited the property in 1987, they found out that the lot was occupied by a squatter named
Juanita Geronimo. A portion was being used as a passageway by petitioners to and from
Visayas Avenue. Accordingly, the spouses complained about the intrusion into their property to
the barangay office.
At the barangay conciliation proceeding, petitioners offered to pay for the use of a portion of Lot
1 as passageway but the Pacione spouses rejected the offer. When the parties failed to arrive at
an amicable settlement, the spouses started enclosing Lot 1 with a concrete fence. Petitioners
protested the enclosure alleging that their property was bounded on all sides by residential
houses belonging to different owners and had no adequate outlet and inlet to Visayas Avenue
except through the property of the Paciones.
As their protest went unheeded, petitioners instituted an action for easement of right of way with
prayer for the issuance of a temporary restraining order (TRO). The trial court issued a TRO
directing the Pacione spouses to cease and desist from fencing the disputed property.
The trial court dismissed the complaint holding that one essential requisite of a legal easement
of a right of way was not proved. Petitioners appealed to the Court of Appeals. The appellate
court affirmed the findings of the trial court. Their motion for reconsideration having been denied,
petitioners filed the present petition together with the issue of legality or illegality of the
conversion of Road Lot 2 into two (2) residential lots by the Cesar Ledesma, Inc.
ISSUE:
Whether or not there was a compulsory easement of right of way.
HELD:
The Supreme Court denied the petition.
To be entitled to a compulsory easement of right of way, the preconditions provided under Arts.
649 and 650 of the Civil Code must be established. These are: (1) that the dominant estate is
surrounded by othe immovables and has no adequate outlet to a public highway; (2) that proper
indemnity has been paid; (3) that the isolation was not due to acts of the proprietor of the
dominant estate; (4) that the right of way claimed is at a point least prejudicial to the servient
estate and, in so far as consistent with this rule, where the distance from the dominant estate to
a public highway may be the shortest. The burden of proving the existence of these
prerequisites lies on the owner of the dominant estate.
In the present case, the first element is clearly absent. As found by the trial court and the Court
of Appeals, an outlet already exist, which is a path walk located at the left side of petitioners'
property and which is connected to a private road about five hundred (500) meters long. The
private road, in turn, leads to Ma. Elena Street which is about 2.5 meters wide and, finally, to
Visayas Avenue. This outlet was determined by the court a quo to be sufficient for the needs of
the dominants estate, hence petitioners have no cause to complain that they have no adequate
outlet to Visayas Avenue.
Further, no evidence was adduced by petitioners to prove that the easement they seek to
impose on private respondents' property is to be established at a point least prejudicial to the
servient estate. For emphasis, Lot 1 is only 164 square meters and an improvident imposition of
the easement on the lot may unjustly deprive private respondents of the optimum use and
enjoyment of their property, considering that its already small area will be reduced further by the
easement. Worse, it may even render the property useless for the purpose for which private
respondents purchased the same.
It must also be stressed that, by its very nature, and when considered with reference to the
obligations imposed on the servient estate, an easement involves an abnormal restriction on the
property rights of the servient owner and is regarded as a charge or encumbrance on the
servient estate. Thus, it is incumbent upon the owner of the dominant estate to establish by
clear and convincing evidence the presence of all the preconditions before his claim for
easement of right of way be granted. Petitioners miserably failed in this regard.
VCL IV
Quimen v. CA
257 SCRA 163

DOCTRINE: Where the easement may be established on any of several tenements surrounding
the dominant estate, the one where the way is shortest and will cause the least damage should
be chosen. However, as elsewhere stated, if these two (2) circumstances do not concur in a
single tenement, the way which will cause the least damage should be used, even if it will not be
the shortest.

FACTS:
Petitioner Anastacia Quimen together with her brothers Sotero, Sulpicio, Antonio and sister
Rufina inherited a piece of property situated in Pandi, Bulacan. They agreed to subdivide the
property equally among themselves, as they did, with the shares of Anastacia, Sotero, Sulpicio
and Rufina abutting the municipal road. Located directly behind the lots of Anastacia and Sotero
is the share of their brother Antonio designated as Lot No. 1448-B-C which the latter divided into
two (2) equal parts, now Lots Nos. 1448-B-6-A and 1448-B-6-B. The latter Lot is behind the
property of Sotero, father of private respondent Yolanda Oliveros. Yolanda purchased Lot No.
1448-B-6-A from her uncle Antonio through her aunt Anastacia who was then acting as his
administratrix.

According to Yolanda, when petitioner offered her the property for sale she was hesitant to buy
as it had no access to a public road. But Anastacia prevailed upon her to buy the lot with the
assurance that she would give her a right of way on her adjoining property.

Thereafter, Yolanda constructed a house on the lot she bought using as her passageway to the
public highway a portion of Anastacia's property. But when Yolanda finally offered to pay for the
use of the pathway Anastacia refused to accept the payment. In fact she was thereafter barred
by Anastacia from passing through her property.


Later, Yolanda purchased the other lot of Antonio Quimen, Lot No. 1448-B-6-B, located directly
behind the property of her parents who provided her a pathway between their house from the lot
of Yolanda behind the sari sari store of Sotero, and Anastacia's perimeter fence. The store is
made of strong materials and occupies the entire frontage of the lot measuring four (4) meters
wide and nine meters (9) long. Although the pathway leads to the municipal road it is not
adequate for ingress and egress. The municipal road cannot be reached with facility because
the store itself obstructs the path so that one has to pass through the back entrance and the
facade of the store to reach the road.

Finally, Yolanda filed an action with the proper court praying for a right of way through
Anastacia's property. The report was that the proposed right of way was at the extreme right of
Anastacia's property facing the public highway, starting from the back of Sotero's sari-sari store
and extending inward by one (1) meter to her property and turning left for about five (5) meters
to avoid the store.

TC dismissed Yolandas complaint, but the CA reversed the decision declaring that she was
entitled to a right of way on petitioners property and that the way proposed by Yolanda would
cause the least damage and detriment to the servient estate.

ISSUE:
Whether or not passing through the property of Yolanda's parents is more accessible to the
public road than to make a detour to her property and cut down the avocado tree standing
thereon. -- YES

HELD:
The conditions sine quo non for a valid grant of an easement of right of way are: (a) the
dominant estate is surrounded by other immovables without an adequate outlet to a public
highway; (b) the dominant estate is willing to pay the proper indemnity; (c) the isolation was not
due to the acts of the dominant estate; and, (d) the right of way being claimed is at a point least
prejudicial to the servient estate.

The criterion of least prejudice to the servient estate must prevail over the criterion of shortest
distance although this is a matter of judicial appreciation. While shortest distance may ordinarily
imply least prejudice, it is not always so as when there are permanent structures obstructing the
shortest distance; while on the other hand, the longest distance may be free of obstructions and
the easiest or most convenient to pass through. In other words, where the easement may be
established on any of several tenements surrounding the dominant estate, the one where the
way is shortest and will cause the least damage should be chosen. However, as elsewhere
stated, if these two (2) circumstances do not concur in a single tenement, the way which will
cause the least damage should be used, even if it will not be the shortest.

As between a right of way that would demolish a store of strong materials to provide egress to a
public highway, and another right of way which although longer will only require an avocado tree
to be cut down, the second alternative should be preferred.

FXRL
Floro v. Llenado
244 SCRA 713

DOCTRINE: One may not claim a legal easement merely out of convenience.

FACTS:
Floro is the owner of the Floro Park Subdivision in Bulacan
The subdivision has access roads from MacArthur Highway through road lot 4
Llenado is the owner of the Llenado Homes Subdivision, formerly known as Emmanuel
Homes Subdivision prior to his purchase of the project.
This subdivision is bounded on the south and separated from Floro Park Subdivision by
Planas Creek. To its west lies a parcel of land owned by Marcial Ipapo
Llenado Homes does not have any existing access to MacArthur Highway. However, a
proposed access road traversing the property of Ipapo had been provided for in the
subdivision plan of Emmanuel Homes which was approved by the HLURB.
Llenado sought and was granted the (oral and provisional, as they were still drafting a
formal contract) permission of Floro to use lots 4 and 5 of Floro Park Subdivision as a
passage to and from MacArthur Highway.
Several months later, Floro barricaded road lot 5 to prevent Llenado from further using
the property on account of the damage done to the property due to the passage of heavy
equipment.
Llenado filed an easement claim with the RTC but was denied.
The CA ruled in favor of Llenado and ordered Floro to remove the barricade.

ISSUE:
W/N Llenado can demand a compulsory easement of right of way over the existing roads of an
adjacent subdivision instead of developing his subdivision's proposed access road as provided
in his duly approved subdivision plan. -- NO

HELD:
A compulsory easement of right of way may be granted only upon the acquisitions of the
conditions required by Articles 649 and 650 of the Civil Code. To wit, these conditions are:

1. That the dominant estate is surrounded by other immovables and has no adequate
outlet to a public highway;
2. That proper indemnity has been paid
3. That the isolation was not due to acts of the proprietor of the dominant estate
4. That the right of way claimed is at a point least prejudicial to the servient estate and, in
so far as consistent with this rule, where the distance from the dominant estate to a
public highway may be the shortest.

In this case, the elements are incomplete. The original subdivision development plan presented
by Llenado indicates an existing and prior agreement which creates a right of way through the
abandoned Ipapo ricefield. Ipapo had long agreed to these terms but Llenado apparently
thought it too much work and cost to develop such road. It was easier for him to create an
easement via the Floro property.

Jurisprudence has dictated that one may not claim a legal easement merely out of convenience.
It was convenience motivated Llenando to abandon the Ipapo access road development and
pursue an access road through the Floro estate. He was stacking the cards in his favor to the
unnecessary detriment of his neighbor.

RSDM
Fransisco v. IAC
177 SCRA 527

DOCTRINE: The law makes it amply clear that an owner cannot, as respondent has done, by
his own act isolate his property from a public highway and then claim an easement of way
through an adjacent estate.

FACTS:
Ramos' Lot 860-A used to be a part of Lot 860 of the Malinta Estate. Lot 860 was owned by
Cornelia and Frisca Dila, and had a frontage along Parada Road measuring 51.90 meters.
Adjoining Lot 860 was Lot 226, owned by Eusebio Francisco, as aforestated; it also had a
frontage along Parada Road of 62.10 meters.

On December 3,1947, the co-owners of Lot 860 (Cornelia and Frisca Dila) executed a deed by
which an undivided one-third portion of the land was donated to a niece, Epifania Dila, and
another undivided one-third (1/3) portion to the children of a deceased sister, Anacleta Dila, and
the remaining portion, also an undivided third, was declared to pertain exclusively to and would
be retained by Cornelia Dila.

Some months later, in March, 1972, after having set up a piggery on his newly acquired
property, Ramos had his lawyer write to Eusebio Francisco owner, as above mentioned, of
the adjoining lot, Lot 266- to ask for a right of way through the latter's land. Negotiations
thereafter had however failed to bring about a satisfactory arrangement. Francisco's proposal
for an exchange of land at the rate of one (1) square meter from him to three (3) square meters
from Ramos, as was supposedly the custom in the locality, was unacceptable to Ramos.

Later that year, 1972, Ramos succeeded, through the intercession of Councilor Tongco of
Valenzuela, in obtaining a three-meter wide passageway through Lot 860-B of Epifania Dila .
Yet in August, 1973, he inexplicably put up a ten-foot high concrete wall on his lot, this was in
August, 1973, and thereby closed the very right of way granted to him across Lot 860-B. It
seems that what he wished was to have a right of passage precisely through Francisco's land,
considering this to be more convenient to him, and he did not bother to keep quiet about his
determination to bring suit, if necessary, to get what he wanted.

Francisco learned of Ramos' intention and reacted by replacing the barbed-wire fence on his lot
along Parada Road with a stone wall, also in August, 1973. Shortly thereafter, Francisco was
served with summons and a copy of the complaint in Civil Case of the Court of First Instance of
Bulacan, instituted by Ramos, as well as a writ of preliminary mandatory injunction directing him
to remove his stone fence and keep his lot open for Ramos' use.

The Court handed down its verdict, adversely to Francisco. Francisco appealed to the Court of
Appeals. In its own decision, the latter affirmed the Trial Court's judgment.

ISSUE:
Whether or not Private Respondent was entitled to an easement of right of way through the land
belonging to the Petitioner

HELD:
It is the fact already adverted to and which has never been disputed that respondent Ramos,
having already been granted access to the public road (Parada Road) through the other
adjoining Lot 860-B owned by Epifania Dila and this, at the time he was negotiating with
petitioner for the similar easement over the latter's Lot 266 that he now claims inexplicably
gave up that right of access by walling off his property from the passageway thus established.
The evidence, also uncontradicted, is that said passageway was 2.76 meters wide, or wide
enough to accommodate a truck. The surveyor who at the instance of petitioner made a survey
of the premises on September 13, 1973, shortly after Ramos had filed his complaint, verified the
existence of said passageway from the presence of tire marks found on the scene and indicated
on the sketch plan he prepared the path that it took from said respondent's Lot 860-A through
Lot 860-B to Parada Road. That there was such a passageway was also confirmed by another
witness, Parada Barrio Captain Fausto Francisco, one of those who had earlier tried to bring
petitioner and respondent to an agreement about the proposed right of way through the property
of the former. This witness declared, as already stated, that after the negotiations had been
stalled by the failure of the parties to agree on the terms of a proposed land exchange that
would have given Ramos access to Parada Road, said respondent had been able to obtain right
of passage to the same public road over a 3-meter wide portion of Lot 860-B owned by Epifania
Dila through the intercession of Councilor Tongco of Valenzuela.

The evidence is, therefore, persuasively to the effect that the private respondent had been
granted an adequate access to the public highway (Parada Road) through the adjacent estate
of Epifania Dila even as he was trying to negotiate a satisfactory agreement with petitioner
Francisco for another passageway through the latter's property. If at the time he filed suit
against the petitioner, such access (through the property of Epifania Dila) could no longer be
used, it was because he himself had closed it off by erecting a stone wall on his lot at the point
where the passageway began for no reason to which the record can attest except to
demonstrate the isolation of his property alleged in his complaint. But the law makes it amply
clear that an owner cannot, as respondent has done, by his own act isolate his property from a
public highway and then claim an easement of way through an adjacent estate. The third of the
cited requisites: that the claimant of a right of way has not himself procured the isolation of his
property had not been met indeed the respondent had actually brought about the contrary
condition and thereby vitiated his claim to such an easement. It will not do to assert that use of
the passageway through Lot 860-B was dffficult or inconvenient, the evidence being to the
contrary and that it was wide enough to be traversable by even a truck, and also because it has
been held that mere inconvenience attending the use of an existing right of way does not justify
a claim for a similar easement in an alternative location. .

MRAM
Sta. Maria v. CA
285 SCRA 163

DOCTRINE: The requirements for an estate to be entitled to a compulsory servitude of right of
way under the Civil Code are the ff.:

1. The dominant estate is surrounded by other immovables and has no adequate outlet to
a public highway;
2. There is payment of proper indemnity;
3. The isolation is not due to the acts of the proprietor of the dominant estate; and
4. The right of way claimed is at the point least prejudicial to the servient estate; and
insofar as consistent with this rule, where the distance from the dominant estate to a
public highway may be the shortest .

FACTS:
Respondent spouses Arsenio and Roslynn Fajardo are the registered owners of a piece of land
(Lot 124), containing an area of 1,043 sq. m in Obando Bulacan. They acquired said lot under a
Deed of Absolute Sale executed by the vendors Pedro M. Sanchez, et al.

Lot 124 is surrounded by Lot 1, a fishpond, on the northeast; by Lot 126, owned by Florentino
Cruz on the southeast; by Lot 6-A and a portion of Lot 6-B owned respectively by Sps Sta. Mr
and Florcerfida Sta. Maria (petitioners); on the southwest,; and by Lot 122 owned by the Jacinto
family on the northwest.

The respondents filed a complaint against the defendants for the establishment of an easement
of right of way, alleging that 1) Lot 124 is surrounded by properties belonging to other persons,
2) since they have no adequate outlet to the provincial road, an easement of a right of way
passing through either of the alternative petitioners properties which are directly abutting the
provincial road would be plaintiffs only convenient, direct and shortest access to and from the
provincial road, 3) respondents predecessors-in-interest have been passing through the
properties of petitioners in going to and from their lot, 4) petitioners mother even promised
respondents predecessors-in-interest to grant the latter an easement of right of way as she
acknowledged the absence of an access from their property to the road, and 5) despite
respondents request for a right of way and referral of the dispute to the barangay officials, the
petitioners refused to grant them an easement.

ISSUE:
Whether or not a compulsory right of way can be granted to the respondents who have two
other existing passageways other than that of petitioners and an alternative vacant lot fronting
the provincial road also adjacent to plaintiffs property, which can be used in going to and from
respondents property?

HELD:
The Court held in the affirmative. The following are the requirements for an estate to be entitled
to a compulsory servitude of right of way under the Civil Code, to wit:
1. The dominant estate is surrounded by other immovables and has no adequate outlet to
a public highway (Art. 649, par. 1);
2. There is payment of proper indemnity (Art. 649, par. 1);
3. The isolation is not due to the acts of the proprietor of the dominant estate (Art. 649, last
par.); and
4. The right of way claimed is at the point least prejudicial to the servient estate; and
insofar as consistent with this rule, where the distance from the dominant estate to a
public highway may be the shortest (Art. 650).

Anent the first requisite, there is no dispute that the respondents property is surrounded by
other immovables owned by different individuals and that the property is without adequate outlet
to a public highway.

The second requisite is also present since, as established through testimony, respondents are
willing to pay the corresponding damages provided for by law if granted the right of way.

As for the third requisite, the isolation of their lot is not due to plaintiffs acts. The property they
purchased was already surrounded by other immovables leaving them no adequate ingress or
egress to a public highway.

Lastly, as to the fourth requisite of least prejudice and shortest distance, the Court agreed
with the lower court that this twin elements have been complied with in establishing the
easement of right of way on defendants-appellants properties. Among the 3 possible servient
estates, it is clear that petitioners property would afford the shortest distance from defendants
property to the provincial road. Moreover, it is the least prejudicial since as found by the lower
court, it appears that there would be no significant structures to be injured in the defendants
property and the right-of-way to be constructed thereon would be the shortest of all the
alternative routes pointed to by the defendants

FMM
National Irrigation Authority v. CA
340 SCRA 661

DOCTRINE: When a land, originally public, is awarded to a private individual, a legal easement
may be constituted and thus no just compensation is required.

FACTS:
On 28 June 1963, a free patent over three hectares of land situated in Barrio Baybayog,
municipality of Alcala, province of Cagayan, was issued in the name of respondents
predecessor-in-interest, Vicente Manglapus and registered under a title in his name, subject to
provisions including conditions on public easements and servitudes recognized and prescribed
by law.

Subsequently, Dick Manglapus (Manglapus) acquired the lot from Vicente Manglapus by
absolute sale.

On 18 July 1974, the land was registered in Respondents name under Transfer Certificate of
Title No. T-26658 of the Register of Deeds for the Province of Cagayan.

Sometime in 1982, the National Irrigation Authority (NIA) entered into a contract with Villamar
Development Construction. Under the contract, NIA was to construct canals in Amulung,
Cagayan and Alcala, Cagayan. NIA then entered a portion of Manglapus' land and made
diggings and fillings thereon.
]


On 14 March 1991, Manglapus filed with the Regional Trial Court (the RTC), Tuguegarao,
Cagayan a complaint for damages against NIA. Manglapus alleged that NIA's diggings and
fillings destroyed the agricultural use of his land and that no reasonable compensation was paid
for its taking. Despite service of notice of the pretrial conference, NIA did not appear at the pre-
trial conference.
On 3 December 1991, the trial court declared NIA in default and received Manglapus' evidence
ex parte and on 23 December 1991, the RTC rendered a decision in favor of Manglapus.
On 27 January 1992, NIA filed a motion to lift the order of default dated 3 December 1991, and
to set aside the decision of 23 December 1991.
On 3 June 1992, the RTC issued a resolution denying the motion for lack of merit. Thus on 17
July 1992, NIA filed a notice of appeal to the Court of Appeals (the CA).
On 27 July 1992, the RTC gave due course to the appeal and ordered the transmission of the
original records to the CA. Manglapus subsequently filed a motion for execution of judgment
with the RTC which the NIA through the Solicitor General opposed. On 17 August 1992, the trial
court declared that since the notice of appeal of NIA was given due course, the motion for
execution was "moot and academic."
On 8 March 1994, the CA dismissed the appeal hence this petition. The NIA contended that the
certificate of title covering the subject parcel of land contained a reservation granting the
government a right of way over the land covered therein.

ISSUE:
Whether the NIA should pay Manglapus just compensation for the taking of a portion of his
property for use as easement of a right of way. -- NO

HELD:
The Supreme Court held that Article 619 of the Civil Code provides that, "Easements are
established either by law or by the will of the owners. The former are called legal and the latter
voluntary easements." In the present case, we find and declare that a legal easement of a right-
of-way exists in favor of the government. The land was originally public land, and awarded to
respondent Manglapus by free patent. The ruling would be otherwise if the land were originally
private property, in which case, just compensation must be paid for the taking of a part thereof
for public use as an easement of a right of way.

RGGM
Abellana v. CA
208 SCRA 316

DOCTRINE: The use of a footpath or road may be apparent but it is not a continuous easement
because its use is at intervals and depends upon the acts of man; A right of way is not
acquirable by prescription.

FACTS:
Petitioners are owners of a parcel of land on the NW side of Nonoc Subdivision, Cebu. They
sued to establish an easement of a right of way over a subdivision road, which they claim
theyve acquired through prescription since their ancestors have been using these since time
immemorial.

They pray that the concrete wall surrounding the village be taken down to allow easy access to
the public highway.

RTC found for the petitioners. CA reversed, averring that road lots in subdivisions are private
property and may only be used as a public highway once acquired by the government through
donation, purchase or expropriation.

ISSUE:
Whether or not the easement of a right of way may be acquired by prescription?

HELD:
No. Art. 620 of the Civil Code provides that only continuous and apparent easements may be
acquired by prescription. The easement of a right of way cannot be considered continuous
because its use is at intervals and is dependent on the acts of man.

MCSS
Encarnacion v. CA
195 SCRA 74

FACTS:
The servient estate of respondent heirs are what stands between the dominant estate and the
national road. When the servient estate was not yet enclosed with a concrete fence, persons
going to the national highway just crossed the servient estate at no particular point.

When a fence was constructed, a roadpath measuring 25 meters long and a meter wide was
constituted to provide access to the highway, with one-half meter taken from the servient estate
and another one-half from another lot.

Petitioner's plant nursery business through sheer hard work flourished and he bought an owner-
jeep which he could use for transporting his plants. However, the jeep could not pass through
the roadpath. He offered the servient estate owners that they sell him one and one-half meters
of their property to be added to the existing pathway. His request was turned down.

During the trial, the attention of the lower court was called to the existence of another exit, a
dried river bed, to the highway, only eighty meters away from the dominant estate.

ISSUE:
WON petitioner is entitled to a widening of an already existing easement of right-of-way. -- YES

HELD:
Just because the second egress is nearer to the highway by a difference of only 65 meters, are
not amount the conditions specified by Article 649 of the Civil Code. While there is a dried river
bed closer to the dominant tenement, that access is grossly inadequate. Generally, the right of
way may be demanded:(1) when there is absolutely no access to a public highway, and (2)
when, even if there is one, it is difficult or dangerous to use or is grossly insufficient.

The river bed route is traversed by a semi-concrete bridge and there is no ingress nor egress
from the highway. For the jeep to reach the level of the highway, it must literally jump four to five
meters up. During the rainy season, the river bed is impassable due to the floods. Thus, it can
only be used at certain times of the year. The river bed which make passage difficult, if nor
impossible, it is if there were no outlet at all. There is a real and compelling need for such
servitude in his favor.

Article 651 provides that the width of the easement of right of way shall be that which is
sufficient for the needs of the dominant estate, and may accordingly be changed from time to
time. It is the needs of the dominant property which ultimately determine the width of the
passage, and these may vary from time to time. To force petitioner to leave his jeepney in the
highway, exposed to the elements and to the risk of theft simply because it could not pass
through the improvised pathway, is sheer pigheadedness on the part of the servient estate and
can only be counter-productive for all the people concerned.

Petitioner should not be denied a passageway wide enough to accommodate his jeepney since
that is a reasonable and necessary aspect of the plant nursery business.

NKVS
Vda. De Baltazar v. CA
245 SCRA 333

DOCTRINE: The owner of an estate may claim a compulsory right of way only after he has
established the existence of four (4) requisites, namely, (1) the estate is surrounded by other
immovables and is without adequate outlet to a public highway; (2) after payment of the proper
indemnity; (3) the isolation was not due to the proprietor's own acts; and (4) the right of way
claimed is at a point least prejudicial to the servient estate, and in so far as consistent with this
rule, where the distance from the dominant estate to a public highway may be the shortest.

FACTS:
Daniel Panganiban is the owner of a parcel of residential land consisting of 117 square meters
denominated as Lot no. 1027 located at Sta. Ines, Bulacan. Immediately to the front of said land
is Lot 1026 of Loreto Vda. de Baltazar and her son Nestor Baltazar. Immediately behind is the
Sta. Ana River. On either side are Lots 1025 and 1028 owned by Ricardo Calimon and Jose
Legaspi, respectively. Braulio Street, a provincial road, runs along the frontage of Lots 1025,
1026 and 1028.

Daniel Panganiban filed a complaint against the Baltazars who are owners of Lot 1026 for the
establishment of a permanent and perpetual easement of right of way for him to have access to
the provincial road.

Petitioners opposed arguing that there exists two other rights of way adjacent to private
respondent's property. Aside from the passageway which the plaintiff seeks to be established
as a permanent easement, the property of the plaintiff is accessible to and from the provincial
road via two (2) other passageways. RTC dismissed the case.

The CA, however, found that the two passageways mentioned were mere temporary pathways,
that the existence of the two passageways was not simultaneous and was granted by
respondent's neighbors, Calimon and Legaspi only upon respondent's request when petitioner
Baltazar closed the claimed passageway is supported by the evidence on record.

In light of the above findings of the Court of Appeals, the underlying issue begging resolution is
whether or not respondent Panganiban is entitled to claim an easement of right of way over the
Baltazars' property.

ISSUE:
W/N Panganiban is entitled to the easement prayed for. -- YES

HELD:
By express provision of Articles 649 and 650 of the New Civil Code, the owner of an estate may
claim a compulsory right of way only after he has established the existence of four (4) requisites,
namely, (1) the estate is surrounded by other immovables and is without adequate outlet to a
public highway; (2) after payment of the proper indemnity; (3) the isolation was not due to the
proprietor's own acts; and (4) the right of way claimed is at a point least prejudicial to the
servient estate, and in so far as consistent with this rule, where the distance from the dominant
estate to a public highway may be the shortest.

It is not disputed that the first requisite has been established by the court a quo in its Order
dated May 22, 1990. Respondent Panganiban's property is indeed surrounded by immovables
on three sides and a river on the fourth.

As for the second requisite, Francisco v. Intermediate Appellate Court states:
There would indeed be some point in looking askance at a reading of the law which would
impute to it a strict requirement to pay "proper indemnity" in advance of a suit the purpose of
which, in addition to creating an easement, is precisely to fix the amount of the indemnity to be
paid therefor.

With respect to the third requisite, respondent Panganiban was likewise able to establish that
the isolation of his property was not due to his own act for he merely bought Lot 1027. The
respondents have been using as a right of way, has been "existing, recognized, acknowledged,
tolerated and used by the appellant as a right of way for thirty (30) years during the lifetime of
petitioner's grandfather, Fidel and his father, Onisimo Baltazar." It was also established that the
right of way was "closed and obstructed by the petitioners when they closed the gate and
placed plants across the gate of Lot 1026-B when petitioners constructed their present
residence."

As regards the fourth requirement, both parties agreed that the passage claimed by respondent
as his right of way, compared to the other passageways, is the shortest distance from
respondent's lot to Braulio Street.

Petitioners could not have been inconvenienced by the passageway for, as borne out by the
records, the same is separate and distinct from the gate used by them to enter their lot and
residence. Such being the case, we conclude that respondent is entitled to claim a compulsory
easement of right of way over petitioners' Lot 1026-B.

AMPS
David-Chan v. CA
268 SCRA 677

DOCTRINE: There are rigorous standards to be complied with by owners of the dominant
estate before they may be granted with easement of right of way. These standards must be
strictly complied with because easement is a burden on the property of another. Equity is not
applied as statutory laws on the matter are existent.

FACTS:
Petitioner filed with the trial court an amended petition with prayer for preliminary prohibitory
injunction, seeking to stop private respondent from fencing its property and depriving her of
access to the highway.

Petitioner alleged that her property was delineated on its northern and western sides by various
business establishments. Adjoining her property along its southern boundary was the land of the
Pineda family, while along the east-northeastern boundary, and lying between her property and
the MacArthur Highway, was another lot with an area of approximately 161 square meters
owned by private respondent. In short, petitioner's lot was almost completely surrounded by
other immovables and cut off from the highway. Her only access to the highway was a very
small opening measuring two feet four inches wide through the aforementioned property of
private respondent.

Petitioner believed she was entitled to a wider compulsory easement of right of way through the
said property of private respondent. The prospective subservient estate was a portion of a
bigger lot, which was formerly owned by the Singian Brothers Corporation and was sold to
private respondent without the knowledge and consent of petitioner, who was thereby allegedly
prevented from exercising her right of pre-emption or right of redemption. The petition likewise
prayed that judgment be rendered ordering private respondent to sell to petitioner the subject lot.

Private respondent denied the allegations of petitioner. The parents and relatives of petitioner
were never tenants or lessees of the former owner, Singian Brothers; rather, they were found to
be illegally occupying the property as ruled by the MTC-San Fernando, Pampanga.

As their affirmative and special defenses, Defendant Singian Brothers averred that the
complaint of petitioner stated no cause of action because, being apparent and discontinuous,
the right of way cannot be acquired by prescription. Petitioner was not a tenant of the Singian
Brothers; therefore she was not entitled to a right of pre-emption or right of redemption. Finally,
petitioner had another access to the National Highway which, however, she closed during the
pendency of the case at the trial court when she extended the construction of her fence.

ISSUES:
1. Is petitioner legally entitled to a right of way through private respondent's property?

2. Should traditional Filipino values as pakikisama be considered?

HELD:
1. Not legally entitled to a right of way for failure to satisfy the requirements.

Citing Articles 649 and 650 of the Civil Code, petitioner submits that "the owner of an estate
may claim a compulsory right of way only after he (or she) has established the existence of four
requisites, namely: (1) the estate is surrounded by other immovables and is without adequate
out-let to a public highway; (2) proper indemnity is paid; (3) the isolation is not due to the
proprietor's own acts; and (4) the right of way claimed is at a point least prejudicial to the
servient estate and, insofar as consistent with this rule, where the distance from the dominant
estate to a public highway may be the shortest.

The Supreme Court upheld the factual findings of the lower courts thus:

1) Petitioner is not "without adequate outlet to a public highway". It was plaintiff who built a
concrete fence on the southern boundary of her property to separate it from the property
of the Pineda family. Worse, during the pendency of the case, she closed the 28-inch
clearance which she could use as a means to reach the National Highway without
passing through the property of defendant.

2) The appellate court likewise found that petitioner failed to satisfy the third requirement
because she caused her own isolation by closing her access through the Pineda
property.

3) Petitioner failed to prove she made a valid tender of the proper indemnity.

2. Equity should only be applied in the absence of statutory law or judicial rules of procedure.

Petitioner is not legally entitled to a right of way on the property of private respondent. Thus,
such equitable arguments cannot prevail over the legal findings.

There are rigorous standards to be complied with by owners of the dominant estate before they
may be granted with easement of right of way. These standards must be strictly complied with
because easement is a burden on the property of another. Before such inconvenience may be
imposed by the Court, applicants must prove that they deserve judicial intervention on the basis
of law, and certainly not when their isolation is caused by their own acts. In the latter case, they
decide their detachment and must bear the consequences of such choice.

KGS
Almendras v. CA
G.R. No. 110067

DOCTRINE: When the easement may be established on any several tenements surrounding
the dominant estate, the one where the way is shortest and will cause the least damage should
be chosen.

FACTS:
Almendras owns a land that is bounded on the north and on the east by lots owned by Eng and
Yap, on the south by the lot owned by Bongo, and on the west by the properties of the Opones.
On the western boundary of Almendras land abuts an existing private road, which passes
through the several lots and leads to another private road (a permanent easement) located on
the property of Tudtud, which in turn connects to the provincial road.

Thereafter, Eng and Yap began building a concrete wall on his property on the northern and
eastern sides of petitioners lot. For this reason, Almendras wrote to them offering to buy a
portion of their lot, so that he could have access to the provincial road. But her request was
denied on the ground that there was an existing private road on the western side of Almendras
property providing adequate outlet to the provincial road. Eng and Yap claimed that granting
petitioners request would greatly reduce the value of his property, as the proposed right of way
cuts across the middle of the property. Shortly thereafter, Bongo also fenced his property, thus
closing off the southern boundary of petitioners lot. As such, Almendras brought this action for
the establishment of a right of way through Eng and Yaps land, which was granted by the trial
court. Then, Opone subsequently closed off the western side of petitioners property by erecting
a fence on his lot, with the result that petitioners property became inaccessible.

ISSUE:
Is Almendras entitled to a right of way through Eng and Yaps property? -- YES

HELD:
To begin with, the owner of a landlocked property has the right to demand a right of way through
the neighboring estates. The easement must be established at the point which is least
prejudicial to the servient estate and, whenever possible, the shortest to the highway. If these
two conditions exist on different properties, the land where establishment of the easement will
cause the least prejudice should be chosen. Thus, it has been held that where the easement
may be established on any of several tenements surrounding the dominant estate, the one
where the way is shortest and will cause the least damage should be chosen. However, . . . if
these two (2) circumstances do not concur in a single tenement, the way which will cause the
least damage should be used, even if it will not be the shortest.

In the case at bar, the trial court ruled that the easement should be constituted through the land
of private respondents on the eastern side because it would be the shortest way to the
provincial road, being only 17.45 meters long, compared to 149.22 meters if the easement was
constituted on the Opone and Tudtud roads on the western and southern sides of petitioners
land.

On the other hand, as already pointed out, the Court of Appeals, in pointing to the longer way,
considered the fact that this was already existing and does not preclude its use by other parties
than the individual owners of Lot 1-A to Lot 1-G and the owners of the land on which the
connecting Tudtud road is found.

The way may be longer and not the most direct way to the provincial road, but if the
establishment of the easement in favor of petitioner on this roads will cause the least prejudice,
then the easement should be constituted there. This seems to be reasoning of the Court of
Appeals. However, this can only be determined if the several lot owners (i.e., the Opones and
their buyers and those of Bienvenido Tudtud) are before the court, for the determination of the
point least prejudicial to the owners of servient estates (if there are two or more possible sites
for an easement) requires a comparative evaluation of the physical conditions of the estates. It
is not possible to determine whether the estates which would be least prejudiced by the
easement would be those of the owners of the Opone and Tudtud properties because they have
not been heard. Although evidence concerning the condition of their estates has been
presented by private respondents, it is impossible to determine with certainty which estate
would be least prejudiced by the establishment of an easement for petitioner until these parties
have been heard. Any decision holding them liable to bear the easement would not be binding
on them since they are not parties to this action.

The case was REMANDED to the RTC.

JPOT
Dionisio v. Ortiz
204 SCRA 745

DOCTRINE: Easement-Right of way is not applicable when the cause of its effectivity is the
proprietary's own actuations.

FACTS:
Owners of lots are contiguous and adjacent each other leading to Howmart road and EDSA.
Both parties consent to have a shared road by way of agreement involving Quezon City
Industrial Estates (QCIE). This, in order to traverse Howmart road which is private and make
known cooperation with fellow QCIE members having a shared interest. However, legal action
ensued from the Regional Trial Court and Court of Appeals until January of 1990. The said
internal agreement with QCIE only subsisted until December. Furthermore, said gate came into
fruition because of respondents' willingness to subdivide their lot. Eventually, this would lead to
unnecessary noise pollution and disturbances brought upon by the transportation business of
the respondent. The gate in question leads into Dionisios' lot so they found it best to assemble a
barricade.

ISSUE:
W/N the private respondents have an easement of right of way over Howmart Road -- NO

HELD:
The court finds it untenable to appreciate the logic of an expired contract of agreement.
Moreover, it finds it harder to take into account the claim of right of way when the land owner
necessitated to avail said easement because of their own proprietary action. It is inadequate to
make use of such a right when it is through one's own fault why there is a need for another gate
for them to be able to access Howmart road.

WHEREFORE, the petition is hereby GRANTED. The questioned decision of the Court of
Appeals and the Order of the Regional Trial Court in Civil Case No. Q-89-3949 are SET ASIDE.
The writ of preliminary injunction is hereby LIFTED.
SO ORDERED.

MLAV
Cortes v. Yu-Tibo
2 Phil. 24

DOCTRINE: An easement of light and view is a negative easement. When easement is
negative, there should be a formal act of opposition for prescription to run.

FACTS:
Cortes wife owns a house (No. 65) in which certain windows open on the adjacent property (No.
63), a neighboring house on the same street. This setting has been in existent since 1843. The
tenant of the adjacent property raised the roof of house No. 63 in such a manner that half of the
windows of house No. 65 has been covered, thus depriving it of a large part of air and light.

Plaintiff contends that by the constant and uninterrupted use of the windows for 59 years, he
acquired from prescription an easement of light in favor of house No. 65, and as a servitude
upon house No. 63. Consequently, he has acquired the right to restrain the making of any
improvement in the latter house which may be prejudicial to the enjoyment of the easement.
Further, he contends that the easement of light is positive; and that therefore the period of
possession for the purposes of the acquisition of a prescriptive title is to begin from the date on
which the enjoyment of the same commenced, or, in other words, from the time that said
windows were opened with the knowledge of the owner of the house No. 63, and without
opposition on this part.

Defendant contends that the easement is negative, and therefore the time for the prescriptive
acquisition must begin from the date on which the owner of the dominant estate may have
prohibited, by a formal act, the owner of the servient estate from doing something which would
be lawful but for the existence of the easement.

Lower court ruled in favor of the defendant. Plaintiff appealed the case.

ISSUE:
WON the easement is positive/negative

HELD:
Easement is negative.

The easement of light which is the object of this litigation is of a negative character, and
therefore pertains to the class which cannot be acquired by prescription as provided by article
538 of the Civil Code, except by counting the time of possession from the date on which the
owner of the dominant estate has, in a formal manner, forbidden the owner of the servient
estate to do an act which would be lawful were it not for the easement.

In consequence, the plaintiff, not having executed any formal act of opposition to the right of the
owner of house No. 63 to make improvements which might obstruct the light of house No. 65, at
any time prior to the complaint, has not acquired, nor could he acquire by prescription, such
easement of light, no matter how long a time have elapsed. Because the period which the law
demands for such prescriptive acquisition could not have commenced to run, the act with which
it must necessarily commence not having been performed.

DJTV
Purugganan v. Paredes
69 SCRA 69

DOCTRINE: In drainage or easement of receiving water falling from roofs, the distances
prescribed in the Decree of Registration should not correspond to the width and length of the
roof of the house but to the distance of the rainwater falling inside the land of the servient estate.

FACTS:
Plaintiff-appellee Emilio Purugganan is the owner of a piece of a residential lot subdivided as
Lot 1 and Lot 2 (servient estate), adjacent to and bounded on the North by the lot of defendant-
appellant Felisa Paredes (dominant estate). The lots of the plaintiff-appellee are subject to an
easement of drainage in favor of the defendants-appellants in the Decree of Registration of the
Court of First Instance of Abra.

Defendants-appellants constructed a house on their lot adjacent to Lots 1 and 2 of plaintiff-
appellee in such a manner that the southern side of their house is exactly on the brick wall, the
southern side of which is the demarcation line between the plaintiff-appellee and the
defendants-appellants, demolishing said brick wall and built thereon the southern wall of their
house with 3 windows. The house constructed by the defendants-appellants is 2- meters
longer than the length of roofing allowed in the Decree of Registration, and has an outer roofing
(eaves) of 1.20 meters, protruding over the property of the plaintiff-appellee which is .20 meters
wider than that allowed in the same Decree of Registration, and the rain water from the GI
roofing falls about 3 meters inside Lots 1 and 2 of the plaintiff-appellee.

Plaintiff filed a case prohibiting defendant from proceeding with the construction of the roof. Trial
court, in a summary proceeding decided in favor of the plaintiff. CA affirmed trial courts decision.
Hence this appeal.

ISSUE:
Whether or not the easement of drainage refers to the length of the roofing? -- NO

HELD:
The Supreme Court held that defendants-appellants have made a mistake in applying the
distances prescribed in the Decree of Registration to the roofing of their house. They failed to
comprehend the meaning of the phrase "servidumbre de vertiente de los tejados" constitutes on
the land of plaintiff. Translated, it means the easement of receiving water falling from the roof
which is an encumbrance imposed on the land of the plaintiff-appellee. Consequently, the
distances prescribed in the Decree of Registration should not correspond to the width and
length of the roof of the defendants-appellants' house but to the distance of the rain water falling
inside the land of the plaintiff-appellee because the encumbrance is not the roof itself but the
rain water falling inside the property of the plaintiff-appellee.

The summary judgment appealed from is affirmed with costs against defendants-appellants.

JGY
Valisno v. Adriano
161 SCRA 398

DOCTRINE: Water rights, such as the right to use a drainage ditch for irrigation purposes,
which are appurtenant to a parcel of land, pass with the conveyance of the land, although not
specifically mentioned in the conveyance

FACTS:
Plaintiff Valisno file against the Defendant Adriano an action for damages. The complaint
alleged that the plaintiff is the absolute owner and actual possessor of a parcel of land in Nueva
Ecija. Plaintiff bought the land from the defendants sister, Honorata. Both parcels of land had
been inherited by defendant and her sister from their father. At the time of the sale of the land to
Valisno, the land was irrigated by water from the Pampanga River through a canal about 70
meters long, traversing the appellee's land.

Adriano levelled a portion of the irrigation canal so that Valisno was deprived of the irrigation
water and prevented from cultivating his 57-hectare land. Plaintiff Adriano filed in the Bureau of
Public Works and Communications a complaint for deprivation of water rights. A decision was
rendered in favor of the plaintiff. Defendant asked for a reinvestigation of the case and was
granted. Meanwhile, plaintiff Valisno rebuilt the irrigation canal at his own expense because of
urgency. He also filed a complaint for damages in the CFI against respondent.

Defendant Adriano claims that he merely allowed his sister to use his water rights when she still
owned the adjacent land. According to the appellant, the water right was the primary
consideration for his purchase of Honorata's property, for without it the property would be
unproductive.

ISSUE:
WON the water rights pass with the conveyance of the land. -- YES

HELD:
Water rights, such as the right to use a drainage ditch for irrigation purposes, which are
appurtenant to a parcel of land, pass with the conveyance of the land, although not specifically
mentioned in the conveyance. The purchaser's easement of necessity in a water ditch running
across the grantor's land cannot be defeated even if the water is supplied by a third person. The
fact that an easement by grant may also have qualified as an easement of necessity does
detract from its permanency as property right, which survives the determination of the necessity.

As an easement of waters in favor of the plaintiff has been established, he is entitled to enjoy it
free from obstruction, disturbance or wrongful interference such as the appellee's act of levelling
the irrigation canal to deprive him of the use of water from the Pampanga River.

JRPA
Trias v. Araneta
15 SCRA 241

DOCTRINE: Sellers of land may validly impose reasonable easements and restrictions as
conditions for contracts of sales; the same may not be overturned by courts merely on the
ground that it impacts dominical rights.

FACTS:
JM Tuason and Co. owned a piece of land that was part of a subdivision. Thru broker Araneta
Inc (of Araneta Coliseum fame), this civic-minded company sold the land to Mr Lopez with the
condition that said lot should never be used to erect a factory. This imposition was annotated to
the TCT.
A series of transfers and conveyances later, the lot ended up in the hands of the gorgeous Ms.
Rafael Trias. She was dismayed with the annotation that stated 5. That no factories be
permitted in this section.

Ms. Trias felt that the annotation impaired her dominical rights and therefore illegal and existed
as mere surplusage since existing zoning regulations already prevented the erection of factories
in the vicinity. Worse, the annotation possibly hindered her plans to obtain a loan. She
accordingly raised the issue to the court and received relief.

Later on, Gregorio Araneta moved for reconsideration stating that the imposition resulted from a
valid sales transaction between her predecessors in interest. He alleged that the court held no
authority to overrule such valid easement and impaired the right to contract.

ISSUE:
Whether or not the imposition was valid.

HELD:
The imposition was valid. The prohibition is an easement validly imposed under art 594 which
provides that every owner of a piece of land may establish easements he deems suitable xxx
and not in contravention to the law, public policy and public order

The court ruled that the easement existed to safeguard the peace and quiet of neighboring
residents. The intention is noble and the objectives benign. In the absence of a clash with public
policy, the easement may not be eroded.

The contention of surplusage is also immaterial. Zoning regulations may be repealed anytime,
allowing the erection of factories. With the annotation, at the very least, the original intent to bar
factories remains binding.

ABB
La Vista v. CA
278 SCRA 498

DOCTRINE: Like any other contractual stipulation, a voluntary easement cannot be
extinguished except by voluntary recession of the contract establishing the servitude or
renunciation by the owner of the dominant lots.

FACTS:
Mangyan road is a 15-meter wide road abutting Katipunan Avenue on the west, traverses the
edges of La Vista Subdivision on the north and of the Ateneo de Manila University and
Maryknoll College on the south. The said road was originally owned by the Tuasons who sold a
portion of their land to Philippine Building Corporation. Included in such sale was half or 7.5
meters width of the Mangyan road. The said corporation assigned its rights, with the consent of
the Tuasons, to Ateneo through a Deed of Assignment with Assumption of Mortgage. Ateneo
later on sold to Maryknoll the western portion of the land. Tuason developed their land which is
now known as La Vista. On January, 1976, Ateneo and La Vista acknowledged the voluntary
easement or a mutual right of way wherein the parties would allow the other to use their half
portion of the Mangyan road (La Vista to use Ateneos 7.5 meters of the Mangyan road and also
the other way around.) Ateneo auctioned off the property wherein Solid Homes Inc., the
developer of Loyola Grand Villas, was the highest bidder.

Ateneo transferred not only the property, but also the right to negotiate the easement on the
road. However, La Vista did not want to recognize the easement thus they block the road using
6 cylindrical concrete and some guards over the entrance of the road blocking the entrance of
the residents of Loyola Grand Villas. Solid Homes Inc. filed for injunction and La vista in turn
filed a third party complaint against Ateneo. Some of the arguments of the petitioner were that
Loyola residents had adequate outlet to a public highway using other roads and also that
Ateneo has not yet finalized the negotiation of the easement.

ISSUE:
Whether or not there is an easement of right of way -- YES

HELD:
There was a voluntary easement of right of way which was acknowledged on January 1976 by
the Tuasons and Ateneo. Being a voluntary easement, the four requisites for a compulsory
easement need not be satisfied. Also, like any other contractual stipulation, the same cannot be
extinguished except by voluntary recession of the contract establishing the servitude or
renunciation by the owner of the dominant lots. In the case at bar, all the predecessors-in-
interest of both parties recognized the existence of such easement and there was no agreement
yet to revoke the same. The free ingress and egress along Mangyan Road created by the
voluntary agreement is thus demandable.



NUISANCE (ART.694 TO 707)

FZC
Sitchon v. Aquino
98 Phil. 720

DOCTRINE: Houses constructed, with governmental authority, on public streets, and waterways
obstruct at all times the free use by the public of said streets and waterways, and, accordingly,
constitute nuisance per se, aside from public nuisances. As such, the summary removal thereof,
without judicial process or proceedings may be authorized by the statute or municipal ordinance,
despite the due process clause.

FACTS:
This decision stems from six (6) different suits. All of the petitioners implead Aquino (the City
Engineer of Manila) as respondent so that he may be enjoined from causing the demolition of
their respective houses situated in different areas along public streets in Manila inasmuch as
these
constitute public nuisances. All of the petitioners occupied the subject parcels of land initially
entirely without consent. However, all of them subsequently paid concession fees or damages
for the use of the land with the agreement that such payment and consent shall be without
prejudice to an order to vacate. The time came when the City Engineer demanded that
petitioners vacate the occupied streets. Unheeded, he threatened to demolish the houses.
Petitioners contend that by virtue of arts. 700 and 702, the power to remove public nuisances is
vested in the District Health Officer, not in the City Engineer.

ISSUES:
1. Is there a public nuisance?
2. Does the City Engineer have authority to cause the abatement of the nuisance?

HELD:
There is a public nuisance. This case falls on article 694(4), classifying as a nuisance the
obstruction of free passage of any public highway or street. It is public because it affects a
community or neighbourhood. The constructions in fact constitute nuisances per se, obstructing
at all times the streets. As such, the summary removal of these may be authorized by statute or
ordinance.

Aquino, as City Engineer, is vested with authority to effect the abatement of the nuisances
through demolition. By virtue of the Revised Charter of Manila, such duty, among others, was
placed upon him. Arts. 700 and 702 must yield to this provision not only because it is later law
but also because of the principle that special provisions prevail over general ones. Moreover, an
ordinance authorized the action sought to be taken by respondent.


LNAC
Velasco v. Manila Electric
40 SCRA 342

DOCTRINE: A noise may constitute an actionable nuisance but it must be a noise which affects
injuriously the health or comfort of ordinary people in the vicinity to an unreasonable extent.
Injury to a particular person in a peculiar position or of specially sensitive characteristics will not
render the noise an actionable nuisance. In the conditions of present living, noise seems
inseparable from the conduct of many necessary occupations. Its presence is a nuisance in the
popular sense in which that word is used, but in the absence of statute noise becomes
actionable only when it passes the limits of reasonable adjustment to the conditions of the
locality and of the needs of the maker to the needs of the listener.

FACTS:
Appellant Velasco bought from the People's Homesite and Housing Corporation three (3)
adjoining lots. These lots are within an area zoned out as a "first residence" district by the City
Council of Quezon City. Subsequently, the appellant sold two (2) lots to the Meralco, but
retained the third lot, which was farthest from the street-corner, whereon he built his house.

Appellee company started the construction of the sub-station in question and finished it the
following November, without prior building permit or authority from the Public Service
Commission. The facility reduces high voltage electricity to a current suitable for distribution to
the company's consumers, numbering not less than 8,500 residential homes, over 300
commercial establishments and about 30 industries. The substation has a rated capacity of "2
transformers at 5000 Kva each or a total of 10,000 Kva without fan cooling; or 6250 Kva each or
a total of 12,500 Kva with fan cooling". It was constructed at a distance of 10 to 20 meters from
the appellant's house. The company built a stone and cement wall at the sides along the streets
but along the side adjoining the appellant's property it put up a sawale wall but later changed it
to an interlink wire fence. It is undisputed that a sound unceasingly emanates from the
substation.

Velasco contends that the sound constitutes an actionable nuisance under Article 694 of the
Civil Code of the Philippines, reading as follows:

A nuisance is any act, omission, establishment, business condition of property or anything else
which:

(1) Injures or endangers the health or safety of others; or
(2) Annoys or offends the senses;

X X X

Subjection to the sound since 1954 had disturbed the concentration and sleep of said appellant,
and impaired his health and lowered the value of his property. Wherefore, he sought a judicial
decree for the abatement of the nuisance and asked that he be declared entitled to recover
damages.

Court dismissed the claim of the plaintiff.

Plaintiff then appealed to this Court.

ISSUE:
Whether this sound constitutes an actionable nuisance. -- YES

HELD:
The general rule is that everyone is bound to bear the habitual or customary inconveniences
that result from the proximity of others, and so long as this level is not surpassed, he may not
complain against them. But if the prejudice exceeds the inconveniences that such proximity
habitually brings, the neighbor who causes such disturbance is held responsible for the resulting
damage, being guilty of causing nuisance.

While no previous adjudications on the specific issue have been made in the Philippines, our
law of nuisances is of American origin, and a review of authorities clearly indicates the rule to be
that the causing or maintenance of disturbing noise or sound may constitute an actionable
nuisance.

By jurisprudence, there can be no doubt but commercial and industrial activities which are lawful
in themselves may become nuisances if they are so offensive to the senses that they render the
enjoyment of life and property uncomfortable. It is no defense that skill and care have been
exercised and the most improved methods and appliances employed to prevent such result. Of
course, the creation of trifling annoyance and inconvenience does not constitute an actionable
nuisance, and the locality and surroundings are of importance. The fact that the cause of the
complaint must be substantial has often led to expressions in the opinions that to be a nuisance
the noise must be deafening or loud or excessive and unreasonable. Usually it was shown to be
of that character. The determining factor when noise alone is the cause of complaint is not its
intensity or volume. It is that the noise is of such character as to produce actual physical
discomfort and annoyance to a person of ordinary sensibilities, rendering adjacent property less
comfortable and valuable. If the noise does that it can well be said to be substantial and
unreasonable in degree; and reasonableness is a question of fact dependent upon all the
circumstances and conditions. There can be no fixed standard as to what kind of noise
constitutes a nuisance. It is true some witnesses in this case say they have been annoyed by
the humming of these transformers, but that fact is not conclusive as to the nonexistence of the
cause of complaint, the test being the effect which is had upon an ordinary person who is
neither sensitive nor immune to the annoyance concerning which the complaint is made. In the
absence of evidence that the complainant and his family are supersensitive to distracting noises,
it is to be assumed that they are persons of ordinary and normal sensibilities.

As can be anticipated, character and loudness of sound being of subjective appreciation in
ordinary witnesses, not much help can be obtained from the testimonial evidence. That of
plaintiff Velasco is too plainly biased and emotional to be of much value. The estimate of other
witnesses on the point of inquiry are vague and imprecise, and fail to give a definite idea of the
intensity of the sound complained of.

TKDC
Iloilo Cold Storage v. Municipal Council
24 Phil. 471

DOCTRINE: City Council cannot, by a mere resolution or motion, declare any particular thing a
nuisance which has not theretofore been pronounced to be such by law, or so adjudged by
judicial determination.

FACTS:
Plaintiff is the owner of an ice and cold storage plant. Nearby residents made complaints
regarding the smoke that the plant emits saying that it was very injurious to their health and
comfort. The defendant made investigations and later on passed a resolution which demands
that the smokestacks of the said factory be elevated or else the factory operations will be closed
or suspended. Plaintiff opposed by filing for injunction.

Iloilo Ice constructed an ice and cold storage plant in Iloilo City, after being granted authority by
theMunicipal Council.After the plant was completed, residents who lived near the plant
complained to the Council that smokecoming from the plant was injurious to their health and
comfort.The Council formed a committee to investigate the complaints. The committee found
that the complaintswere well founded, so the Council passed a resolution ordering Iloilo Ice to
construct smokestacks; failureto comply with the same would force the municipal president to
execute an order closing or suspendingthe operations of Iloilo Ice.Iloilo Ice commenced an
action to enjoin the Council from carrying into effect the resolution, and apreliminary injunction
was issued. They maintained that they were not obliged to comply with thedirective to elevate
the smokestacks one hundred ft and that the Council threatened to require compliancewith the
resolution without the intervention of the court.Council answered, denying the allegations in the
complaint, asking that they be absolved from thecomplaint and that Iloilo Ice be declared to
have no right to the remedy asked for. Iloilo Ice demurred to Councils answer. Demurrer was
sustained so the Council appealed before the SC.

ISSUE:
Whether or not the resolution alone issued by the municipal council is sufficient to label and
abate the supposed nuisance in this case? -- NO

HELD:
There are two kinds of nuisances: nuisances per se and per accidens. The former are
recognized as nuisances under any and all circumstances. The latter are such only because of
the special circumstances and conditions surrounding them. The former may be abated even by
private individuals however the latter is different; it needs a determination of the facts which is a
judicial function.

The question of nuisance can conclusively be decided, for all legal uses, by the established
courts of law or equity alone, and that the resolution of officers, or of boards organized by force
of municipal charters, cannot, to any degree, control such decision. City Council cannot, by a
mere resolution or motion, declare any particular thing a nuisance which has not theretofore
been pronounced to be such by law, or so adjudged by judicial determination.

In the present case it is certain that the ice factory of the plaintiff is not a nuisance per se. It is a
legitimate industry, beneficial to the people and conducive to their health and comfort. The
resolution is obviously not enough to abate the property of the plaintiff.

AMD
Hidalgo Enterprises v. Balandan
91 Phil. 488

DOCTRINE: Doctrine of Attractive Nuisance One who maintains on his premises dangerous
instrumentalities or appliances of a character likely to attract children in play, and who fails to
exercise ordinary care to prevent children from playing therewith or resorting thereto, is liable to
a child of tender years who is injured thereby, even if the child is technically a trespasser in the
premises.

The attractive nuisance doctrine, generally, is not applicable to bodies of water, artificial as well
as natural, in the absence of some unusual condition or artificial feature other than the mere
water and its location.

FACTS: Petitioner Hidalgo Enterprises, Inc. was the owner of an ice-plant in San Pablo, Laguna.
Installed therein were two 9-feet-deep tanks full of water, with edges barely a foot high from the
surface of the ground. While the compound was fenced, the tanks themselves were without any
kind of fence or top cover. The plant had a wide gate entrance, usually left open for motor
vehicles, customers, and anyone else to pass and enter the premises. There was no guard
assigned on the gate. At about noon of April 16, 1948, plaintiff's son, Mario Balandan, an 8-year
old boy, while playing with and in company of other boys of his age, entered the plant through
the gate to take a bath in one of said tanks. While bathing, Mario sank to the bottom of the tank,
only to be fished out later, already a cadaver, having died of "asphyxia secondary to drowning."

Both the CFI of Laguna and the CA ruled in favor of Spouses Balandan. They both took the
view that the petitioner maintained an attractive nuisance (the tanks), and neglected to adopt
the necessary precautions to avoid accidents to persons entering its premises. It applied the
doctrine of attractive nuisance, of American origin, recognized in this jurisdiction in Taylor vs.
Manila Electric. The principle reason for the doctrine is that the condition or appliance in
question, although its danger is apparent to those of age, is so enticing or alluring to children of
tender years as to induce them to approach, get on or use it, and this attractiveness is an
implied invitation to such children.

ISSUE:
W/N a water tank is an instrumentality or appliance considered as an attractive nuisance. -- NO

HELD:
The great majority of American decisions say no. There are numerous cases in which the
attractive nuisance doctrine has not been held to be applicable to ponds or reservoirs, pools of
water, streams, canals, dams, ditches, culverts, drains, cesspools or sewer pools.

The reason why a reservoir of water is not considered an attractive nuisance was lucidly
explained by the Indiana Appellate Court as follows:
Nature has created streams, lakes and pools which attract children. Lurking in their waters is
always the danger of drowning. Against this danger children are early instructed so that they are
sufficiently presumed to know the danger; and if the owner of private property creates an
artificial pool on his own property, merely duplicating the work of nature without adding any new
danger (he) is not liable because of having created an "attractive nuisance." Anderson vs.
Reith-Riley Const. Co.

Therefore, as petitioner's tanks are not classified as attractive nuisance, the question whether
the petitioner had taken reasonable precautions becomes immaterial. And the other issue
submitted by petitioner that the parents of the boy were guilty of contributory negligence
precluding recovery, because they left for Manila on that unlucky day leaving their son under the
care of no responsible individual needs no further discussion.



DONATION (ART. 725-772)

CRF
Republic v. Guzman
326 SCRA 90


DOCTRINE: There are three (3) essential elements of a donation: (a) the reduction of the
patrimony of the donor; (b) the increase in the patrimony of the donee; and, (c) the intent to do
an act of liberality or animus donandi. When applied to a donation of an immovable property, the
law further requires that the donation be made in a public document and that there should be an
acceptance thereof made in the same deed of donation or in a separate public document. In
cases where the acceptance is made in a separate instrument, it is mandated that the donor
should be notified thereof in an authentic form, to be noted in both instruments.

FACTS:
David Rey Guzman, a natural-born American citizen, is the son of the spouses Simerican citizen.
In 1968 Simeon died leaving to his sole heirs Helen and David an estate consisting of several
parcels of land located in Bulacan.

On 29 December 1970 Helen and David executed a Deed of Extrajudicial Settlement of the
Estate of Simeon Guzman dividing and adjudicating to themselves all the property belonging to
the estate of Simeon. The taxes due thereon were paid and the parcels of land were accordingly
registered in their names in undivided equal shares.

On 10 December 1981 Helen executed a Quitclaim Deed assigning, transferring and conveying
to her son David her undivided one-half (1/2) interest on all the parcels of land subject matter of
the Deed of Extrajudicial Settlement of the Estate of Simeon Guzman. On 18 October 1989
David executed a Special Power of Attorney where he acknowledged thaeon Guzman, a
naturalized American citizen, and Helen Meyers Guzman, an Amt he became the owner of the
parcels of land subject of the Deed of Quitclaim executed by Helen on 9 August 1989 and
empowering Atty. Lolita G. Abela to sell or otherwise dispose of the lots. On 1 February 1990
Atty. Lolita G. Abela, upon instruction of Helen, paid donors taxes to facilitate the registry of the
parcels of land in the name of David.

The ownership of David was assailed and on the basis thereof, the Government filed before the
Regional Trial Court of Malolos Bulacan a Petition for Escheat praying that one-half (1/2) of
David's interest in each of the subject parcels of land be forfeited in its favor. On 9 August 1994
David Rey Guzman responded with a prayer that the petition be dismissed.

Thus as a rule, only a Filipino citizen can acquire private lands in the Philippines. The only
instances when a foreigner can acquire private lands in the Philippines are by hereditary
succession and if he was formerly a natural-born Filipino citizen who lost his Philippine
citizenship. Petitioner therefore contends that the acquisition of the parcels of land by David
does not fall under any of these exceptions. It asserts that David being an American citizen
could not validly acquire one-half (1/2) interest in each of the subject parcels of land by way of
the two (2) deeds of quitclaim as they are in reality donations inter vivos.

ISSUE:
Whether or not the quitclaim executed by Helen tantamount to a donation in favor of her son
David? -- NO

HELD:
Not all the elements of a donation of an immovable property are present in the instant case. The
language of the deed of quitclaim is clear that Helen merely contemplated a waiver of her rights,
title and interest over the lands in favor of David, and not a donation. That a donation was far
from Helen's mind is further supported by her deposition which indicated that she was aware
that a donation of the parcels of land was not possible since Philippine law does not allow such
an arrangement. There are three (3) essential elements of a donation: (a) the reduction of the
patrimony of the donor; (b) the increase in the patrimony of the donee; and, (c) the intent to do
an act of liberality or animus donandi. When applied to a donation of an immovable property, the
law further requires that the donation be made in a public document and that there should be an
acceptance thereof made in the same deed of donation or in a separate public document. In
cases where the acceptance is made in a separate instrument, it is mandated that the donor
should be notified thereof in an authentic form, to be noted in both instruments.

Art. 633 of the Civil Code from whence Art. 749 came Manresa said: "If the acceptance does not
appear in the same document, it must be made in another. Solemn words are not necessary; it
is sufficient if it shows the intention to accept x x x x it is necessary that formal notice thereof be
given to the donor, and the fact that due notice has been given must be noted in both
instruments. Then and only then is the donation perfected. "

Therefore, the provisions of the law not having been complied with, there was no effective
conveyance of the parcels of land by way of donation inter vivos. There is no valid repudiation of
inheritance as Helen had already accepted her share of the inheritance when she, together with
David, executed a Deed of Extrajudicial Settlement of the Estate of Simeon Guzman on 29
December 1970 dividing and adjudicating between the two (2) of them all the property in
Simeons estate. By virtue of such extrajudicial settlement the parcels of land were registered in
her and her sons name in undivided equal share and for eleven (11) years they possessed the
lands in the concept of owner.

MPF
Heirs of Velasquez v. CA
325 SCRA 552

DOCTRINE: An action for partition will not lie if the claimant has no rightful interest over the
subject property. A donation as a mode of acquiring ownership results in an effective transfer of
title over the property from the donor to the donee and the donation is perfected from the
moment the donor knows of the acceptance by the donee. And once a donation is accepted. the
donee becomes the absolute owner of the property donated.

FACTS:
Spouses Leoncia de Guzman and Cornelio Aquino died intestate sometime in
1945 and 1947, respectively and were childless.
Leoncia de Guzman was survived by her sisters Anatalia de Guzman (mother of the
plaintiffs) and Tranquilina deGuzman (grandmother of the defendants). During the
existence of their marriage, spouses Aquino were able to acquire real properties.
The plaintiffs alleged that Leoncia de Guzman, before her death, had a talk with
the plaintiffs mother, Anatalia de Guzman, with plaintiff Santiago Meneses and
Tranquilina de Guzman and his son Cesario Velasquez in attendance; that in the
conference Leoncia told Anatalia de Guzman, Tranquilina de Guzman and Cesario
Velasquez that the documents of donation and partition which she and her husband earlier
executed were not signed by them as it was not their intention to give away all the
properties to Cesario Velasquez because Anatalia de Guzman who is one of her sisters
had several children to support; Cesario Velasquez together with his mother allegedly
promised to divide the properties equally and to give the plaintiffs one-half (1/2) thereof;
that they are entitled to of each of all the properties in question being the children of
Anatalia de Guzman, full blood sister of Leoncia de Guzman.
Plaintiffs also claim that after the death of Leoncia, defendants forcibly took
possession of all the properties and despite plaintiffs repeated demands for partition,
defendants refused. Plaintiffs pray for the nullity of any documents covering the properties
in question since they do not bear the genuine signatures of the Aquino spouses, to order
the partition of the properties between plaintiffs and defendants in equal shares and to
order the defendants to render an accounting of the produce of the land in question from
the time defendants forcibly took possession until partition shall have been effected.

ISSUE:
Whether or not the action for partition should be sustained. -- NO
HELD:
No. In actions for partition, the court cannot properly issue an order to divide the property
unless it first makes a determination as to the existence of co-ownership. The court must
initially settle the issue of ownership, the first stage in an action for partition. Needless to state,
an action for partition will not lie if the claimant has no rightful interest over the subject property.
In fact, Section 1 of Rule 69 requires the party filing the action to state in his complaint the
nature and the extent of his title to the real estate. Until and unless the issue of ownership is
definitely resolved, it would be premature to effect a partition of the properties.

In this case, the properties sought to be partitioned by private respondents have already been
delivered to petitioners and therefore no longer part of the hereditary estate which could be
partitioned. After finding that no co-ownership exists between private respondents and
petitioners, the court found no reason to discuss the other arguments raised by the petitioners in
support of their petition.

AMDG
Aluad v. Aluad
G.R. No. 176943

DOCTRINE: The donation being mortis causa, the formalities of a will must be observed.
Absent that, the donation is void. The following are the characteristics of a donation mortis
causa:

1. It conveys no title or ownership to the transferee before the death of the transferor; or
what amounts to the same thing, that the transferor should retain the ownership (full or
naked) and control of the property while alive;

2. That before the death of the transferor, the transfer should be revocable by the
transferor at will, ad nutum; but revocability may be provided for indirectly by means of a
reserved power in the donor to dispose of the properties conveyed; and

3. That the transfer should be void if the transferor should survive the transferee.


FACTS:
Matilde and Crispin Aluad raised Maria and Zenaido Aluad (respondent). Crispin was the owner
of Lot Nos. 674, 675, 676, 677, 680 and 682 and when he died Matilde adjudicated the said
lands. Even before TCTs were issued to Matilde, she executed a Deed of Donation of Real
Property Inter Vivos over the six lots in favor of Maria. The deed of donation will become
effective upon the death of Matilde and until then Matilde may use, encumber or dispose any of
the six lands. The deed of donation also mentioner that if Maria dies first, the deed of donation
will be automatically be rescinded.

After the TCTs were issued, Matilde sold Lot No. 676 to Zenaido and later on executed a last
will and testament wherein Lot Nos., 675, 677, 682 and 680 were given to Maria while Lot No.
674 to Zenaido. It is noted that both Matilde and Maria are now deceased.

The heirs of Maria (petitioners) filed an action to recover Lot No. 674 and 676 from Zenaido. In
his answer, one of the contentions of Zeniado was that the deed of donation does not exist and
that if it did, it was already revoked by Matilde. The trial court ruled on favor of the heirs however,
the decision was reversed by the Court of Appeals in so far as Lot No. 676 is concerned. The
Court of Appeals ruled that Deed of Donation that was executed was not Inter Vivos but one of
Mortis Causa. Being a Donation Mortis Causa, its formalities under Article 805 were not
complied since the donation was only witnessed by two persons and it had no attestation clause.
Lot No. 674 cannot be awarded to Zenaido because the last will and testament was not
probated. It is noted that Lot No. 674 was not awarded to the heirs.

ISSUES:
1. Whether or not the Deed of Donation is one of Inter Vivos or Mortis Causa? -- MORTIS
CAUSA
2. Whether or not Matilde (DONOR) could still sell to Zenaido Lot. 676 even after a deed of
donation has been constructed? -- YES

HELD:
The SC agrees to the appellate court that the donation present in this case is a donation mortis
causa. The deed of donation expressly stated that the donation will only take effect after the
death of the Donor; it is clear that Matilde never intended to transfer the ownership of the lots to
Maria during the lifetime of the former. To further support this, the deed also made an express
mention that Matilde reserved the right to dispose the properties. Also, even after the deed of
donation was constituted, Matilde still paid for the taxes, applied for free patents and the lots
remained under her name.

Having established that it is a donation mortis causa, it is important that the formalities under
Article 805 must be followed. In this case, it was established that the donation was only
witnessed by two persons and they only signed the attestation clause which is a separate
requirement from signing the left pages of the will. The court emphasizes the distinction
between these two signature requirements:

x x x Article 805 particularly segregates the requirement that the instrumental witnesses
sign each page of the will from the requisite that the will be attested and subscribed by
[the instrumental witnesses]. The respective intents behind these two classes of
signature[s] are distinct from each other. The signatures on the left-hand corner of every
page signify, among others, that the witnesses are aware that the page they are signing
forms part of the will. On the other hand, the signatures to the attestation clause
establish that the witnesses are referring to the statements contained in the attestation
clause itself. Indeed, the attestation clause is separate and apart from the disposition of
the will. An unsigned attestation clause results in an unattested will. Even if the
instrumental witnesses signed the left-hand margin of the page containing the unsigned
attestation clause, such signatures cannot demonstrate these witnesses undertakings in
the clause, since the signatures that do appear on the page were directed towards a
wholly different avowal.

Since the formalities were not observed, the donation mortis causa is void and transmitted no
right to Maria. Even if they were observed, the deed was not probated hence, it still cannot be
transmitted to her. Therefore, Matilde validly sold and transmitted Lot Nos. 674 and 676 to
Zenaido with the condition that the will must be probated.

GCG
Danguilan v. IAC
168 SCRA 22

DOCTRINE: In order to determine whether or not said donation is valid and effective it should
be sufficient to demonstrate that, as a contract, it embraces the conditions the law requires and
is valid and effective, although not recorded in a public instrument.
Donation of real property should be in a public instrument.
FACTS:
The respondent filed a complaint against the petitioner in the then Court of First Instance of
Cagayan for recovery of a farm lot and a residential lot. She claimed that she had purchased
from Domingo Melad the land now being unlawfully withheld by the defendant. In his answer,
the petitioner denied the allegation and averred that he was the owner of the said lots of which
he had been in open, continuous and adverse possession, having acquired them from Domingo
Melad. The case was dismissed for failure to prosecute but was refilled.
Danguilan presented three other witnesses to corroborate his statements and to prove that he
had been living in the land since his marriage to Isidra and had remained in possession thereof
after Domingo Melad's death. Two of said witnesses declared that neither the plaintiff nor her
mother lived in the land with Domingo Melad.
The trial court ruled based mainly on the issue of possession. Weighing the evidence presented
by the parties, the judge held that the defendant was more believable and that the plaintiff's
evidence was "unpersuasive and unconvincing." It was held that the plaintiff's own declaration
that she moved out of the property and left it in the possession of the defendant was
contradictory to her claim of ownership. She was also inconsistent when she testified first that
the defendant was her tenant and later in rebuttal that he was her administrator. The decision
concluded that where there was doubt as to the ownership of the property, the presumption was
in favor of the one actually occupying the same, which in this case was the defendant.
The appellate court merely affirmed the factual findings of the trial court except for an irrelevant
modification, and it was only toward the end that it went to and resolved what it considered the
lone decisive issues.
ISSUE:
Whether or not there was a valid donation. -- NO

HELD:
The donation being of real property, it is void for not complying with the requirements given by
law. Donation of real property should be in a public instrument. In this case, it wasnt.
The Supreme Court ruled, considering the language of the two instruments, that Domingo
Melad did intend to donate the properties to the petitioner, as the private respondent contends.
We do not think, however, that the donee was moved by pure liberality. While truly donations,
the conveyances were onerous donations as the properties were given to the petitioner in
exchange for his obligation to take care of the donee for the rest of his life and provide for his
burial.
The case at bar comes squarely under the doctrine laid down in Manalo v. De Mesa where the
Court held:
There can be no doubt that the donation in question was made for a valuable
consideration, since the donors made it conditional upon the donees' bearing the
expenses that might be occasioned by the death and burial of the donor Placida Manalo,
a condition and obligation which the donee Gregorio de Mesa carried out in his own
behalf and for his wife Leoncia Manalo; therefore, in order to determine whether or not
said donation is valid and effective it should be sufficient to demonstrate that, as a
contract, it embraces the conditions the law requires and is valid and effective, although
not recorded in a public instrument.
The deed of sale was allegedly executed when the respondent was only three years old and the
consideration was supposedly paid by her mother, Maria Yedan from her earnings as a wage
worker in a factory. This was itself a suspicious circumstance, one may well wonder why the
transfer was not made to the mother herself, who was after all the one paying for the lands. The
sale was made out in favor of Apolonia Melad although she had been using the surname Yedan
her mother's surname, before that instrument was signed and in fact even after she got married.
The averment was also made that the contract was simulated and prepared after Domingo
Melad's death. It was also alleged that even after the supposed execution of the said contract,
the respondent considered Domingo Melad the owner of the properties and that she had never
occupied the same.

VCL IV
Aldaba v. CA
27 SCRA 263

DOCTRINE: A letter showing an intention to donate is not sufficient to prove donation; and most
certainly not the form required by law in donations.

FACTS:
Two lots owned by Belen Aldaba are being disputed in this case. Petitoners Dr. Vicente Aldaba
and Jane Aldaba, father and daughter, lived with Belen Aldaba for 10 years and took care of her
until her death. Belen had presumptive heirs her surviving husband Estanislao Bautista, and her
brother Cesar Aldaba (represented as the respondents in this case.) After the death of Belen,
the respondents asked the petitioners to leave the premises and upon their refusal, the former
instituted an ejectment case.

The petitioners argue that Belen really intended to donate the property to them as evidence by
the note written by Belen to them which reads;

June 18, 1953
Jane,
Huag kayong umalis diyan. Talagang iyan ay para sa inyo. Alam nila na iyan ay sa inyo.
Belen A. Bautista.
They also argue that the property was for compensation of their services which amounted to
P53,000. The respondents contend that the letter no way proves a donation.

TC ruled that Estanislao Bautitsta is the absolute owner of the property in question. CA affirmed
TCs ruling.

ISSUE:
Whether or not there was a disposition of property by Belen in favor of the petitioners -- NO

HELD:
For the following reasons: (1) The note was insufficient conveyance, and hence could not be
considered as evidence of a donation with onerous caus. The note can be considered, at most,
as indicative of the intention to donate. (2) no notarial document was executed by Belen to the
petitioners during those 10 years. (3) P53,000 worth of services made by the petitioners no way
proves the alleged donation. If at all, the petitioners believed that the gratuitous use of the
property was not sufficient to compensate them for their services, they could have presented
their claims in the intestate proceedings, which they themselves could have initiated, if none
was instituted.

The SC emphasized that there was no express agreement between the parties and that
respondents Jane did not even expect to be compensated.

FXRL
Jutic v. CA
153 SCRA 269

DOCTRINE: An affidavit is an inadequate means of transferring title by donation.

FACTS:
The petitioners are the heirs Melquiades Seville, sibling of Arsenio Seville.
Prior to his death, Arsenio executed an affidavit stating that he was the
owner of two parcels of land and that upon his death, he desires to donate these parcels of
land in favor of his sibling, Melquiades.
With no spouse nor children, it was the siblings of Arsenio who served to
be the heirs of his estate including the two lots allegedly donated in the affidavit.
It is contended by the heirs of Melquiades that Arsenio designated the
former to be the sole inheritor of the two parcels of land in question on the basis of the
affidavit executed in favor of their father.
The private respondents are his other siblings seeking to assert their
rights over the said properties.

ISSUE:
W/N there was a valid donation from Arsenio to Melquiades Seville.

HELD:
No. The affidavit is inadequate to render the donation valid.

A close reading reveals that Exhibit 4 is not a donation inter vivos or mortis causa but a mere
declaration of an intention and a desire. It is not a concrete and formal act of giving or donating.

Further, the petitioners did not even assert any actions of ownership over the property in
question. This is illustrated by the fact that the property was mortgaged by Arsenio with the
knowledge of the Melquiades shows that ownership has not yet transferred. Additionally, when
Arsenio died, payments to the loan for which the property was mortgaged stopped and were not
continued by the petitioners. Consequently, the property was even foreclosed but was later on
redeemed by Zoilo, one of the brothers of Arsenio who is also among the respondents.

Petitioners have a rightful claim over the property based on the fact that they are heirs of
Arsenio and not due to the alleged affidavit executed in favor of their father, Melquiades.

It is also notable that the signed affidavit is a forgery because Arsenio Sevile was illiterate
during his lifetime. The man was unable to write his name and only affixed his thumbmark in the
aforementioned real estate mortgage.

RSDM
Howard v. Padilla
96 Phil. 983

DOCTRINE: Donations must be in accordance with the law..

FACTS:
The property that was donasted is a conjugal in nature. The CA ruled that the donation was inter
vivos, not mortis causa. It is valid and irrevocable. It is valid, however, only up to the extent of
the share of the donor in the property.

ISSUE:
Is the donation inter vivos or mortis causa?

HELD:
The donation is mortis causa which takes effect upon the death of the donor. Therefore, the
donation not having conformed with the formalities of the law, the same is void.

MRAM
Puig v. Penaflorida
16 SCRA 136

DOCTRINE: The reservation by the donor of the right to dispose of the property during her
lifetime in the deed does not indicate that title had passed to the donee in her lifetime but that
the donor merely reserves power to destroy the donation at any time.

FACTS:
On April 10, 1953, Carmen Ubalde Vda. de Parcon died in the City of Iloilo, without forced heirs,
leaving certain properties in the City and province of Iloilo. She left a will and was survived by
nephews and nieces, children of her predeceased brother, Catalino Ubalde, and sister, Luisa
Ubalde, married to Ariston Magbanua.

It also appears that besides her will, the deceased had executed two notarial deeds of donation.
One, entitled Donacion Mortis Causa, was executed in favor of her niece, Estela Magbanua,
married to Mariano Peaflorida, purporting to convey to the donee the properties covered by
TCT Nos. 2338 and 18951 of the Registry of Deeds of Iloilo. Two, the deceased executed
another deed of donation, also entitled "Escritura de Donacion Mortis Causa" in favor of the
same donee, Estela Magbanua Peaflorida, conveying to her three parcels of land covered by
TCT Nos. 925, 927 and 11042 of the Register of Deeds of Iloilo .

There was a condition in the instrument saying that if at the date of her death the donor had not
transferred, sold, or conveyed one-half of lot 58 of the Pototan Cadastre to other persons or
entities, the donee would be bound to pay to Caridad Ubalde, married to Tomas Pedrola, the
amount of P600.00, and such payment was to be made on the date the donee took possession
of Lot No. 58.

ISSUES:
Are the two donations inter vivos or mortis causa? (It being admitted that in the latter event the
donations are void for not being executed with testamentary formalities.)

HELD:
The Court held that the first donation is a valid donation inter vivos while the second one is a
donation mortis causa.

An essential characteristic of dispositions mortis causa is that the conveyance or alienation
should be (expressly or by necessary implication) revocable ad nutum, i.e., at the discretion of
the grantor or so-called "donor," simply because the latter has changed his mind. In
consequence, the specification in a deed of the causes whereby the act may be revoked by the
donor indicates that the donation is inter vivos, rather than a disposition mortis causa

The Court further said that the designation of the donation as mortis causa, or a provision in the
deed to the effect that the donation is "to take effect at the death of the donor" are not
controlling criteria ; such statements are to be construed together with the rest of the instrument,
in order to give effect to the real intent of the transferor.

In case of doubt, the conveyance should be deemed donation inter vivos rather than mortis
causa, in order to avoid uncertainty as to the ownership of the property subject of the deed.

Tested by the foregoing principles, the donation of November 24, 1948, while somewhat
ambiguous, should be held inter vivos in character. Admittedly, it is designated as "mortis
causa," and specifies that it will take effect upon the death of the donor; but, as previously
stated, these expressions are not controlling, and, in the instance, before us, are contradicted
by other provisions indicating a contrary intent. Thus,

The conveyance of the properties described in the deed appears made in consideration
of the undertaking of the donee, Estela Magbanua, to bear "all expenses for medical
treatment, hospital expenses and/or burial of the Donor," without limiting the time when
such expenses are to be incurred. In fact, the use of the words "y/o entierro" (and/or
burial) strongly suggests that the illness and hospitalization expenditures to be borne by
the donee may or may not be connected with the donor's last illness.
Emphasizing the onerous character of the transaction is the requirement that if the
donee should predecease the donor, Governor Peaflorida shall assume the obligations
of the donee, "especialmente" (but not exclusively) "los gastos de ultima enfermedad y
entierro de la donante" (par. 4), and this undertaking was assumed even if the properties
donated would not go to Peaflorida but to the donee's children and descendants (par.
3). It was evidently because of this liability, unconnected with the conveyance, that
Peaflorida had to sign the document together with the donee. It is easy to see that
unless the conveyance were to be effective before the death of donor,, the obligations
assumed by the donee and Governor Peaflorida (her husband) would be without
consideration (causa). Such conditions (consent, subject matter, causa or consideration
and observance of the formalities or solemnities required by law) are all present in the
deed of November 24, 1948.
Again, while there is a clause that the donor reserved her right "to mortgage or even sell
the donated property, when and if she should need funds to meet her own needs," this
last sentence of the stipulation appears incompatible with the grantor's freedom to
revoke a true conveyance mortis causa, a faculty that is essentially absolute and
discretionary, whether its purpose should be to supply her needs or to make a profit, or
have no other reason than a change of volition on the part of the grantor-testator. If the
late Carmen Ubalde Vda. de Parcon wished or intended to retain the right to change the
destination of her property at her sole will and discretion, there was no reason for her to
specify the causes for which she could sell or encumber the property covered by her
bounty.
It is no objection to our view that the donation of November 24, 1948 should provide that
it is not to be recorded until after the donor's death (par. 5), since the absence of
registration would affect only subsequent purchasers, without denying the validity and
obligatory effects of the conveyance as between the parties thereto.

As for the second deed of donation, the text thereof is clear that no proprietary right was
intended to pass to the alleged "donee" prior to the "donor's" death, and that the same was a
true conveyance mortis causa, which by law is invalid because it was not executed with the
testamentary formalities required by the statutes in force at the time. Here, unlike in the previous
donation the designation is donation mortis causa is confirmed by the fact that no signs
contradict or limit the unqualified and unrestricted right of the donor to alienate the conveyed
properties in favor of other persons of her choice at any time that she should wish to do so; it is
so expressed in the deed, and it indirectly recognizes the donor's power to nullify the
conveyance to the alleged "donee" whatever the "donor" wished to do so, for any reason or for
no particular reason at all. As we have seen, this faculty is characteristic of conveyances post
mortem or mortis causa: for the right of the transfer or to alienate the "donated" property to
someone else necessarily imports that the conveyance to the "donee" will not become final and
definite in favor of the latter until the death of the "donor" should exclude every possibility that
the property maybe alienated to some other person.

FMM
Puig v. Penaflorida
294 SCRA 183

DOCTRINE: The real nature of a deed is to be ascertained by both its language and the
intention of the parties as demonstrated by the circumstances attendant upon its execution.

FACTS:
On 11 December 1979, a deed entitled Deed of Donation Inter Vivos was executed by the late
Aurora Virto Vda. De Montinola (Montinola) in favor of her grandchildren, Catalino, Judy
Cristina and Jesus Antonio, all surnamed Valderrama (the Valderramas) over a property
located in Panay, Capiz covered by Transfer Certificate of Title (TCT) No. T-16105. The deed
contained signature of the Valderramas in acknowledgment of their acceptance of the donation.

Montinolas secretary, thereafter, presented the deed for recording in the Property Registry and
the Register of Deeds cancelled TCT No. T-16105, the title under Montinolas name, and
replaced it with TCT No. T-16622 in the name of the Valderramas. However, Montinola retained
TCT No. T-16622 as well as the property until she transferred the same ten years later, on 10
July 1990, to Spouses Ernesto and Evelyn Sicad (the Sicads).

On 12 March 1987, Montinola drew up a deed of revocation of the donation and caused it to be
annotated as an adverse claim to TCT No. T-16622.

On 24 August 1990, she then filed a petition with the Regional Trial Court (RTC) of Roxas City
for the cancellation of TCT No. T-16622 and the reinstatement of TCT No. T-16105. Her
petition was founded on the theory that the donation she executed was one of mortis causa
which had to comply with the formalities of a will and since it had not, the donation was void and
cannot be the basis for the cancellation of TCT No. T-16105 and the issuance of TCT No. T-
16622.

In an opposition dated 29 August 1990, the Valderramas argued that the donation was one inter
vivos which, having complied with the requirements set out in the Civil Code, was perfectly valid
and efficacious.

On 27 March 1991, the RTC rendered judgment holding that the donation was one inte rvivos
and dismissed Montinolas petition for lack of merit.

Montinola elevated the case to the Court of Appeals (CA). She however died on 10 March
1993 while the case was pending.

Thereafter, on 31 March 1993, a Manifestation and Motion was filed by the Sicads in which
they alleged that pursuant to a Deed of Definite Sale dated 25 May 1992, they had become
owners of the property covered by TCT No. T-16622.

On 30 June 1995, the CA affirmed the RTCs decision.

Hence, this petition to the Supreme Court (SC). The Sicads contended that lower courts erred
in their judgment in ruling the donation as one inter vivos because the circumstances
surrounding the execution of the deed of donation, and the subsequent actions of Montinola
incontrovertibly signify the latters intent to transfer the property only after her death (mortis
causa) and that Montinola did not intend to give effect to the donation.

ISSUE:
Whether the donation is mortis causa or inter vivos

HELD:
The donation is mortis causa.

The SC held that the real nature of a deed is to be ascertained by both its language and the
intention of the parties as demonstrated by the circumstances attendant upon its execution.

The SC found the following circumstances signifies that Montinola never intended the donation
to take effect within her lifetime: (a) she expressed her wish that the donation take effect 10
years after her death; (b) she intercalated a new provision which states that however, the
donees shall not sell or encumber the properties herein donated within 10 years after the death
of the donor.; (c) she continued, as explicitly authorized in the deed itself, to possess he
property, enjoy its fruits and otherwise exercise the rights of dominion, paying the property taxes
as they fell due. All these she did until she transferred the property to the Sicad Spouses on
July 10, 1990. The SC set aside the decisions of the lower courts.

RGGM
David v. Sison
76 Phil. 118

DOCTRINE: When the donor maintains the essential rights of ownership over the property
during his lifetime, the donation is mortis causa.

FACTS:
The Administrator of deceased Ms. David was ordered by the court to pay the lawyer of
deceased the amount of P18,000 for legal services rendered. The lawyer, on the other hand,
wanted about P81,000. which was equivalent to 5% of the inventoried estate. Administrator
thought that too high and said hed be happy to pay P3,000.

Both parties based their claims on the central issue of the donation by Ms. David. If the donation
was mortis causa, then the donation should be inventoried with the estate and therefore the
lawyer can get his lofty demand. Should the donation have been intervivos, then the donation
wouldnt be inventoried and therefore the administrator can pay the lower sum contended.

The lower court analyzed a few paragraphs of the deed (in tagalog) and concluded that the
deed partook of the nature mortis causa.

"Na ang naturang "donor," Margarita David y Puato, alang-alang sa malaki niyang pagtigin,
pagligap at pagmamahal sa mga nabanguit na "donees" Narcisa de la Fuente at Priscila de la
Fuente, sa pamamagitan nang kasulatang ito, malayang ibinigay at ipinagkakaloob sa mga
naturang Narcisa de la Fuente at Priscila de la Fuente, at sa kanilang mga tagapagmana,
"albacea" at "Administradores", sa habang panahon, ang kanyang mga titulo, interes at
participacion sa mag sumusunod na ari-arian na pawang malines sa lahat nang mga
pananagutan: (Rec. on Appeal, pp. 209, 210.)

Datapwa't ang lahat nang mga tubo at pakinabangan nang nagbibigay o "donor" na si Margarita
David y Puato hanggang siya ay hindi binabawian nang buhay nang maykapal; at ang mga
pinagbibigyan na si Narcisa de la Fuente at Priscila de la Fuente ay hindi maaaring maipagbili,
maisangal, a maipagpalit o sa ano pa man paraan, kung walang kaalaman at pahintulot nang
naturang Margarita David y Puato.

ISSUE:
Whether or not the Deed of Donation is Mortis Causa -- YES

HELD:
The deed of donation makes it clear that all rents, proceeds, fruits, of the donated properties
shall remain for the exclusive benefit and disposal of the donor, Margartia David, during her
lifetime and that, without the knowledge consent of the donor, the donated properties could not
be disposed of in any way, whether by sale, mortgage, barter, or in any other way possible, thus
making the donees just as paper owners of the properties.

The court then concluded that the donation in question is a donation mortis causa, because the
combined effect of the circumstances surrounding the execution of the deed of donation and of
the above-quoted clauses thereof could not have taken effect before the death of Margarita
David. According to the terms of the deed, the most essential elements of ownership the right
to dispose of the donated properties and the right to enjoy the products, profits, possession
remained with Margarita David during her lifetime, and would accrue to the donees only after
Margarita David's death. While the donation in question is a donation mortis causa, the court
declined to rule that the donated properties should be included in the inventory of the estate and
should follow the same proceedings as if they were not donated at all.

The court then awarded the lawyer P10,000 instead of the P81,000 he craved.

MCSS
Maglasang v. Heirs of Cabatingan
383 SCRA 6

FACTS:
Conchita Cabatingan executed in favor of her borhter, Nicolas, a Deed of Conditional Donation
Inter Vivos for House and Lot, covering two parcels of land. The document stated that, in
consideration of the love and affection of the DONOR, Conchita transfers, conveys, by way of
donation the properties. However, the same was bound by the condition that, in the even that
the DONEE should die before DONOR, the present donation shall be deemed automatically
rescinded.

Respondent heirs filed an action to declare the document void for failing to comply with the
formalities of wills and testaments, considering donations mortis causa.

ISSUE:
WON the donations made are inter vivos or mortis causa. -- NO

HELD:
In a donation mortis causa, the right of disposition is not transferred to the donee while the
donor is still alive. In determining whether a donation is one of mortis causa, the following
characteristics must be take into account: (1) It conveys no title or ownership before death or
the transferor should retain the ownership (full or naked) and control of the property while alive;
(2) Before death, the transfer should be revocable by the transferor at will, even if the same may
be done by indirect means of a reserved power; and (3) That the transfer should be void if the
transferor should survive the transferee.

If the donation is made in contemplation of the donor's death, then it is at the time that the
donation takes effect, and it is a donation mortis causa which should be embodied in a last will
and testament.

The said deed must be executed in accordance with the requisites on solemnities of wills and
testaments under Articles 805 and 806 of the Civil Code.

NKVS
Bonsato v. CA
95 Phil. 481

FACTS:
Respondents complaint charged that on December, 1949, Domingo Bonsato, then already a
widower, had been induced and deceived into signing two notarial deeds of donations in favor of
his brother Juan Bonsato and of his nephew Felipe Bonsato, respectively, transferring to them
several parcels of land situated in the municipalities of Mabini and Burgos, Province of
Pangasinan, both donations having been duly accepted in the same act and documents.
Plaintiffs likewise charged that the donations were mortis causa and void for lack of the requisite
formalities. The defendants, Juan Bonsato and Felipe Bonsato, answered averring that the
donations made in their favor were voluntarily executed in consideration of past services
rendered by them to the late Domingo Bonsato; that the same were executed freely without the
use of force and violence, misrepresentation or intimidation; and prayed for the dismissal of the
case and for damages in the sum of P2,000.

After trial, the Court of First Instance rendered its decision on November 13, 1949, finding that
the deeds of donation were executed by the donor while the latter was of sound mind, without
pressure or intimidation; that the deeds were of donation inter vivos without any condition
making their validity or efficacy dependent upon the death of the donor; but as the properties
donated were presumptively conjugal, having been acquired during the coverture of Domingo
Bonsato and his wife Andrea Nacario, the donations were only valid as to an undivided one-half
share in the three parcels of land described therein.

ISSUE:
W/N the late Domingo Bonsato made donations inter vivos or dispositions post mortem in favor
of the petitioners herein. -- DONATION INTER VIVOS.

HELD:
If the donation is a dispositions post mortem in favor of the defendants, then the documents
should reveal any or all of the following characteristics:

(1) Convey no title or ownership to the transferee before the death of the transferor; or, what
amounts to the same thing, that the transferor should retain the ownership (full or naked) and
control of the property while alive (Vidal vs. Posadas, 58 Phil., 108; Guzman vs. Ibea, 67 Phil.,
633);

(2) That before his death, the transfer should be revocable by the transferor at will, ad nutum;
but revocability may be provided for indirectly by means of a reserved power in the donor to
dispose of the properties conveyed (Bautista vs. Sabiniano, G. R. L-4326, November 18, 1952);

(3) That the transfer should be void if the transferor should survive the transferee.

None of these characteristics is discernible in the deeds of donation, executed by the late
Domingo Bonsato. The donor only reserved for himself, during his lifetime, the owner's share of
the fruits or produce a reservation that would be unnecessary if the ownership of the donated
property remained with the donor. Most significant is the absence of stipulation that the donor
could revoke the donations; on the contrary, the deeds expressly declare them to be
"irrevocable", a quality absolutely incompatible with the idea of conveyances mortis causa
where revocability is of the essence of the act, to the extent that a testator can not lawfully
waive or restrict his right of revocation.

It is true that the last paragraph in each donation contains the phrase "that after the death of the
donor the aforesaid donation shall become effective". However, said expression must be
construed together with the rest of the paragraph, and thus taken, its meaning clearly appears
to be that after the donor's death, the donation will take effect so as to make the donees the
absolute owners of the donated property, free from all liens and encumbrances; for it must be
remembered that the donor reserved for himself a share of the fruits of the land donated. Such
reservation constituted a charge or encumbrance that would disappear upon the donor's death,
when full title would become vested in the donees.

AMPS
Alejandro v. Geraldez
78 SCRA 245

DOCTRINE: A transfer mortis causa should be embodied in a last will and testament (Art. 728).
It should not be called donation mortis causa. It is in reality a legacy. If not embodied in a valid
will, the donation is void.

FACTS:
Andrea Diaz sued her brother, Angel Diaz, in the Court of First Instance for the partition of Lot
2502 donated to them by their parents. The Alejandros (compulsory heirs husband and
children of sister Olimpia Diaz) intervened in the said case, claiming one-third of Lot No. 2502
and that the donation of this lot to Andrea and Angel was a void mortis causa disposition.

The trial court held that the said deed of donation was a donation mortis causa because the
ownership of the properties donated did not pass to the donees during the donors' lifetime but
was transmitted to the donees only "upon the death of the donors". However, it sustained the
division of Lot No. 2502 into two equal parts between Angel Diaz and Andrea Diaz on the theory
that the said deed of donation was effective "as an extra-judicial partition among the parents
and their children. Consequently, the Alejandro intervenors were not given any share in Lot No.
2502.

On appeal before the SC, the Alejandro intervenors contend that the said donation is mortis
causa; that they are entitled to a one-third share in Lot No, 2502, and that the trial court erred in
characterizing the deed as a valid partition.

ISSUE:
Whether the Alejandro intervenors should be awarded one-third of Lot No. 2502 square meters
thereof, as intestate heirs of the Diaz spouses.

HELD:
Not entitled. Questioned donation valid.

To resolve that issue, it is necessary to determine whether the deed of donation is inter vivos or
mortis causa.

The Code prescribes different formalities for the two kinds of donations. An inter vivos donation
of real property must be evidenced by a public document and should be accepted by the donee
in the same deed of donation or in a separate instrument. In the latter case, the donor should be
notified of the acceptance in an authentic form and that step should be noted in both
instruments. (Art. 749, Civil Code. As to inter vivos donation of personal property, see art. 748).

On the other hand, a transfer mortis causa should be embodied in a last will and testament (Art.
728, supra). It should not be called donation mortis causa. It is in reality a legacy (5 Manresa,
Codigo Civil, 6th Ed., p. 107). If not embodied in a valid will, the donation is void (Narag vs.
Cecilio, 109 Phil. 299; Aznar vs. Sucilla 102 Phil. 902; Tuazon vs. Posadas, 54 Phil. 289;
Serrano vs. Solomon, 105 Phil. 998, 1002).

From articles 728 to 732, it is evident that it is the time of effectivity (aside from the form) which
distinguishes a donation inter vivos from a donation mortis causa . And the effectivity is
determined by the time when the full or naked ownership (dominum plenum or dominium
directum) of the donated properties is transmitted to the donees. (See Lopez vs. Olbes, 15 Phil.
540; Gonzales and Fuster Fabra vs. Gonzales Mondragon, 35 Phil. 105). The execution of a
public instrument is a mode of delivery or tradition (Ortiz vs. Court of Appeals, 97 Phil. 46).

The donation in the instant case is inter vivos because it took effect during the lifetime of the
donors. It was already effective during the donors' lifetime, or immediately after the execution of
the deed, as shown by the granting, habendum and warranty clause of the deed.

In that clause it is stated that, in consideration of the affection and esteem of the donors for the
donees and the valuable services rendered by the donees to the donors, the latter, by means of
the deed of donation, wholeheartedly transfer and unconditionally give to the donees the lots
mentioned and described in the early part of the deed, free from any kind of liens and debts.
Thus, the habendum and warranty clause is the donors' declaration that they donate Lot No.
2502, the property in litigation, in equal shares to their children Angel Diaz and Andrea Diaz.

The acceptance clause is another indication that the donation is inter vivos. Donations mortis
causa, being in the form of a will, are never accepted by the donees during the donors' lifetime.
Acceptance is a requirement for donations inter vivos. In the acceptance clause herein, the
donees declare that they accept the donation to their entire satisfaction and, by means of the
deed, they acknowledge and give importance to the generosity and solicitude shown by the
donors and sincerely thank them.

In the reddendum or reservation clause of the deed of donation, it is stipulated that the donees
would shoulder the expenses for the illness and the funeral of the donors and that the donees
cannot sell to a third person the donated properties during the donors' lifetime but if the sale is
necessary to defray the expenses and support of the donors, then the sale is valid. The limited
right to dispose of the donated lots, which the deed gives to the donees, implies that ownership
had passed to them by means of' the donation and that, therefore, the donation was already
effective during the donors' lifetime. That is a characteristic of a donation inter vivos.

However, paragraph 3 of the reddendum in or reservation clause provides that "also, while we,
the spouses Gabino Diaz and Severa Mendoza, are alive, our administration, right, and
ownership of the lots mentioned earlier as our properties shall continue but, upon our death, the
right and ownership of the donees to each of the properties allocated to each of them shall be
fully effective." Evidently, the draftsman of the deed did not realize the discordant and
ambivalent provisions thereof. The habendum clause indicates the transfer of the ownership
over the donated properties to the donees upon the execution of the deed. But the reddendum
clause seems to imply that the ownership was retained by the donors and would be transferred
to the donees only after their death.

We have reflected on the meaning of the said contradictory clauses. All the provisions of the
deed, like those of a statute and testament, should be construed together in order to ascertain
the intention of the parties. Our conclusion is that the reddendum or reservation clause refers to
the beneficial ownership (dominium utile) and not to the naked title and that what the donors
reserved to themselves, by means of that clause, was the management of the donated lots and
the fruits thereof. But, notwithstanding that reservation, the donation, as shown in the habendum
clause, was already effective during their lifetime and was not made in contemplation of their
death because the deed transferred to the donees the naked ownership of the donated
properties.

That conclusion is further supported by the fact that in the deed of donation, out of the eight lots
owned by the donors, only five were donated. Three lots, Lots Nos. 4168, 2522 and 2521 were
superflously reserved for the spouses or donors in addition to one- third of Lot No. 2377. If the
deed of donation in question was intended to be a mortis causa disposition, then all the eight
lots would have been donated or devised to the three children and daughter-in-law of the donors.

The trial court's amended decision is reversed insofar as it pronounces that the deed of
donation is void. That donation is declared valid as a donation inter vivos. The disputed lot
should be partitioned in accordance with that deed between Andrea Diaz and Angel Diaz.

KGS
Reyes v. Masqueda
187 SCRA 661

DOCTRINES:
1. The title given to a deed of donation is not the determinative factor which makes the
donation inter vivos or mortis causa.

2. Characteristics of a donation inter vivos and mortis causa distinguished in Bonsato et al. v.
Court of Appeals et al.

3. Whether a donation is inter vivos or mortis causa depends upon the nature of the disposition
made.

FACTS:
Dr. Emilio Pascual died intestate and without issue. As such, his sister, Ursula, and the children
of his late sisters became Dr. Pascuals heirs. The heirs filed Special Proceedings for the
administration of the estate. Thereafter, Ursula filed a motion to exclude some properties from
the inventory alleging that Dr. Pascual during his lifetime or in 1966 executed a Donation Mortis
Causa in her favor, which was granted by the court. Among the properties included in said
donation was a lot in Tondo, Manila. Records show that in 1969 said property was donated inter
vivos to Ofelia Parungao, who is then a minor. The latters mother accepted it. Accordingly, said
donation was duly registered. Subsequently, Ursula sold said property to the Reyes (involving 4
persons). The CFI ruled that the donation to Ursula is actually a donation inter vivos and that
properties covered be excluded from the inventory of the estate of Dr. Pascual. The donation to
Ofelia was declared null and void.

ISSUE:
Is the donation made by Dr. Pascual to Ursula a donation inter vivos although the title given to
the deed is donation mortis causa? -- YES

HELD:
It is, now a settled rule that the title given to a deed of donation is not the determinative factor
which makes the donation "inter vivos" or "mortis causa" As early as the case of Laureta v.
Manta, et al., this Court ruled that the dispositions in a deed of donation-whether "inter vivos" or
"mortis causa" do not depend on the title or term used in the deed of donation but on the
provisions stated in such deed. This Court explained in Concepcion v. Concepcion ...But, it is
a rule consistently followed by the courts that it is the body of the document of donation and the
statements contained therein, and not the title that should be considered in ascertaining the
intention of the donor. Here, the donation is entitled and called donacion onerosa mortis causa.
From the body, however, we find that the donation was of a nature remunerative rather than
onerous. It was for past services rendered, services which may not be considered as a debt to
be paid by the donee but services rendered to her freely and in goodwill. The donation instead
of being onerous or for a valuable consideration, as in payment of a legal obligation, was more
of remuneratory or compensatory nature, besides being partly motivated by affection.

We should not give too much importance or significance to or be guided by the use of the
phrase 'mortis causa in a donation and thereby to conclude that the donation is not one of inter
vivos. In the case of De Guzman et al. v. Ibea et al., this Court through Mr. Chief Justice
Avancena said that if a donation by its terms is inter vivos, this character is not altered by the
fact that the donor styles it mortis causa.

In the case of Laureta v. Mata, et al., the court held that the donation involved was inter vivos.
There, the donor Severa Magno y Laureta gave the properties involved as ... a reward for the
services which he is rendering me, and as a token of my affection toward him and of the fact
that he stands high in my estimation, I hereby donate 'mortis causa to said youth all the
properties described as follows: xxx xxx xxx I also declare that it is the condition of this donation
that the donee cannot take possession of the properties donated before the death of the donor,
and in the event of her death the said donee shall be under obligation to cause a mass to be
held annually as a suffrage in behalf of my sold, and also to defray the expenses of my burial
and funerals.

It will be observed that the present case and that of Laureta above cited are similar in that in
both cases the donation was being made as a reward for services rendered and being rendered,
and as a token of affection for the donee; the phrase 'mortis causa was used; the donee to take
possession of the property donated only after the death of the donor; the donee was under
obligation to defray the expenses incident to the celebration of the anniversary of the donor's
death, including church fees. The donation in both cases were duly accepted. In said case of
Laureta this Court held that the donation was in praesenti and not a gift in futuro.

In the later case of Bonsato et al. v. Court of appeals, et al. this Court, distinguished the
characteristics of a donation inter vivos and "mortis causa" in this wise: Did the late Domingo
Bonsato, make donations inter vivos or dispositions post mortem in favor of the petitioners
herein? If the latter, then the documents should reveal any or all of the following characteristics:
(1) Convey no title or ownership to the transferee before the death of the transferor; or, what
amounts to the same thing, that the transferor should retain the ownership (fun or naked) and
control of the property while alive; (2) That before his death, the transfer should be revocable by
the transferor at will, ad nutum; but revocability may be provided for indirectly by means of a
reserved power in the donor to dispose of the properties conveyed; (3) That the transfer should
be void if the transferor should survive the transferee.

These principles were repeated in the case of Castro v. Court of Appeals, to wit: Whether a
donation is inter vivos or mortis causa depends upon the nature of the disposition made. 'Did
the donor intend to transfer the ownership of the property donated upon the execution of the
donation? If this is so, as reflected from the provisions contained in the donation, then it is inter
vivos; otherwise, it is merely mortis causa, or made to take effect after death.'

Applying the above principles to the instant petitions, there is no doubt that the so-called
DONATION MORTIS CAUSA is really a donation inter vivos. The donation was executed by Dr.
Pascual in favor of his sister Ursula Pascual out of love and affection as well as a recognition of
the personal services rendered by the donee to the donor. The transfer of ownership over the
properties donated to the donee was immediate and independent of the death of the donor. The
provision as regards the reservation of properties for the donor's subsistence in relation to the
other provisions of the deed of donation confirms the intention of the donor to give naked
ownership of the properties to the donee immediately after the execution of the deed of donation.

JPOT
Gestopa v. CA
342 SCRA 105

DOCTRINE: Acceptance makes a donation inter vivos. There cannot be acceptance mortis
causa in the lifetime of a donor because this would be in a form of a will.

FACTS:
The Danlags own six properties of unregistered land in Cebu. They donated four parcels of land
mortis causa with reservations to Mercedes Pilapil who is an illegitimate child of the husband.
After four years, the donors then again donated to Pilapil only this time it was inter vivos
including the remaining two parcels. Afterwhich, the spouses sold two parcels to the Gestopas.
It was first held in favor of the Gestopas and eventually reversed by the CA. It should be noted
also that the husband of Mercedes was forced to purchase two parcels even though it was
contrary to their interest. The Pilapils assumed responsibility over these lands and have tax
declarations which were appreciated but put aside for reasons of them being easily obtained
from the municipal offices, which after all, does not prove ownership. Some of the terms
included in the Danlags issuance of donation was for them to enjoy the fruits of said parcels
during their lifetime. Unfortunately, there were incidents when they were denied from getting
coconuts.

ISSUE:
W/N the donation was inter vivos or mortis causa is the determination of whether the donor
intended to transfer the ownership over the properties upon the execution of the deed.

RULING:
Said donation was absolutely made inter vivos. Spouses knew of the difference between the
two modes and made use of them in remarkably distinguishing actions. To stress the court
continues:

"Note first that the granting clause shows that Diego (father) donated the properties out
of love and affection for the donee. This is a mark of a donation inter vivos. Second, the
reservation of lifetime usufruct indicates that the donor intended to transfer the naked ownership
over the properties. As correctly posed by the Court of Appeals, what was the need for such
reservation if the donor and his spouse remained the owners of the properties? Third, the donor
reserved sufficient properties for his maintenance in accordance with his standing in society,
indicating that the donor intended to part with the six parcels of land. Lastly, the donee accepted
the donation. In the case of Alejandro vs. Geraldez, 78 SCRA 245 (1977), we said that an
acceptance clause is a mark that the donation is inter vivos. Acceptance is a requirement for
donations inter vivos. Donations mortis causa, being in the form of a will, are not required to be
accepted by the donees during the donors' lifetime."

The only recourse best available for the spouses were to revoke said donation on
grounds of officiousness or ingratitude, both of which were not invoked.

WHEREFORE, the instant petition for review is DENIED. The assailed decision of the
Court of Appeals dated August 31, 1993, is AFFIRMED.
Costs against petitioners.



MLAV
Quijada v. CA
299 SCRA 645

DOCTRINE: Donation is perfected once the acceptance by the donee is made known to the
donor. Accordingly, ownership is immediately transferred and that ownership will only revert to
the donor if the resolutory condition is not fulfilled.

FACTS: Petitioners are the children of the late Trinidad Corvera Vda. de Quijada. Trinidad was
one of the heirs of a two-hectare parcel of land. Trinidad Quijada together with her sisters and
brother executed a conditional deed of donation of the two-hectare parcel of land in favor of the
Municipality of Talacogon, the condition being that the parcel of land shall be used solely and
exclusively as part of the campus of the proposed provincial high school in Talacogon.

Trinidad remained in possession of the parcel of land despite the donation. She sold one (1)
hectare of the subject parcel of land to respondent Regalado Mondejar. Subsequently, she
verbally sold the remaining to respondent Mondejar without the benefit of a written deed of sale
and evidenced solely by receipts of payment.

In 1980, the heirs of Trinidad, who at that time was already dead, filed a complaint for forcible
entry against respondent Mondejar. It was dismissed for failure to prosecute.

Because the proposed provincial high school having failed to materialize, the municipality of
Talacogon enacted a resolution reverting the land donated back to the donors. In the meantime,
respondent Mondejar sold portions of the land to respondents Fernando Bautista, Rodolfo
Goloran, Efren Guden and Ernesto Goloran.

In their complaint, petitioners alleged that their deceased mother never sold the property. At the
time of the alleged sale, the land belongs to the Municipality of Talacogon by virtue of the
conditional deed of donation. Therefore, it renders the sale null and void.

Respondents, in their answer, claimed that the land was sold to Mondejar. And that petitioners
action is already barred by prescription.

The lower court ruled in favor of the petitioners. It held that Trinidad Quijada had no legal title or
right to sell the land to respondent Mondejar since the ownership at the time of sale belongs to
the municipality.

On appeal, the CA reversed and set aside the lower courts judgment ruling that the sale made
by Trinidad Quijada to respondent Mondejar was valid as the former retained an inchoate
interest on the lots by virtue of the automatic reversion clause in the deed of donation.

ISSUE:
WON the sale to Mondejar is void -- NO

HELD
The donation was subject to the condition that the donated property shall be "used solely and
exclusively as a part of the campus of the proposed Provincial High School in Talacogon." It
further provides that should the proposed high school be discontinued or in the future be closed,
the donated property shall automatically revert to the donor. Such condition was validly imposed
in the donation. The donation is perfected once the acceptance by the donee is made known to
the donor. Accordingly, ownership is immediately transferred to the latter and that ownership will
only revert to the donor if the resolutory condition is not fulfilled.

At the time of the sales made, the alleged seller (Trinidad) could not have sold the lots since
ownership was already transferred. So long as the resolutory condition subsists and is capable
of fulfillment, the donation remains effective and the donee continues to be the owner. Such
period, however, became irrelevant when the donee-Municipality manifested through a
resolution that it cannot comply with the condition of building a school and the same was made
known to the donor. Only then that ownership of the donated property reverted to the donor as
provided in the automatic reversion clause of the deed of donation.

The donor may have an inchoate interest in the donated property during the time that ownership
of the land has not reverted to her. Such inchoate interest may be the subject of contracts
including a contract of sale. In this case, however, what the donor sold was the land itself which
she no longer owns.

As to laches, petitioners' action is not yet barred by it. When petitioners initiated the suit, it
cannot be said that they had slept on their rights for a long time.

Be that at it may, there exists a valid sale. Sale, being a consensual contract, is perfected by
mere consent, which is manifested the moment there is a meeting of the minds as to the offer
and acceptance thereof on three (3) elements: subject matter, price and terms of payment of the
price. Ownership by the seller on the thing sold at the time of the perfection of the contract of
sale is not an element for its perfection. What the law requires is that the seller has the right to
transfer ownership at the time the thing sold is delivered.

The consummation, however, occurs upon the constructive or actual delivery of the subject
matter to the buyer when the seller or her successors-in-interest subsequently acquires
ownership thereof. Such circumstance happened in this case when petitioners became the
owners of the subject property upon the reversion of the ownership of the land to them.
Consequently, ownership is transferred to respondent Mondejar and those who claim their right
from him.

DJTV
Lagazo v. CA
287 SCRA 18


DOCTRINES:
A simple or pure donation is one whose cause is pure liberality (no strings attached),
while an onerous donation is one which is subject to burdens, charges or future services
equal to or more in value than the thing donated. Under Article 733 of the Civil Code,
donations with an onerous cause shall be governed by the rules on contracts; hence, the
formalities required for a valid simple donation are not applicable.
Acceptance of the donation by the donee is indispensable, its absence makes the
donation null and void.

FACTS:
Petitioner filed an action seeking to recover from private respondent Cabanlit a parcel of land
which the former claims to have acquired from his grandmother by donation. Private respondent,
on the other hand, put up the defense that when the alleged donation was executed, he had
already acquired the property by a Deed of Assignment from a transferee of plaintiff-appellee's
grandmother.

After trial, the lower court decided in favor of plaintiff-appellee and against defendant-appellant,
rationalizing that the evidence presented by the former is more credible than that of the latter.
Respondent Court of Appeals reversed trial courts decision and anchored its ruling upon the
absence of any showing that petitioner accepted his grandmother's donation of the subject land.
Citing jurisprudence that the donee's failure to accept a donation whether in the same deed of
donation or in a separate instrument renders the donation null and void, Respondent Court
denied petitioner's claim of ownership over the disputed land. The appellate court also struck
down petitioner's contention that the formalities for a donation of real property should not apply
to his case since it was an onerous one he paid for the amortizations due on the land before
and after the execution of the deed of donation reasoning that the deed showed no burden,
charge or condition imposed upon the donee; thus, the payments made by petitioner were his
voluntary acts.

Petitioner contends that the burdens, charges or conditions imposed upon a donation need not
be stated on the deed of donation itself. Thus, although the deed did not categorically impose
any charge, burden or condition to be satisfied by him, the donation was onerous since he in
fact and in reality paid for the installments in arrears and for the remaining balance of the lot in
question. Being an onerous donation, his acceptance thereof may be express or implied, as
provided under Art. 1320 of the Civil Code, and need not comply with the formalities required by
Art. 749 of the same code. His payment of the arrearages and balance and his assertion of his
right of possession against private respondent clearly indicate his acceptance of the donation.

ISSUES:
1. Whether or not the acceptance of a donation made in a separate instrument but not
formally communicated to the donor may be considered complete, valid and subsisting. -
- NO
2. Whether or not the deed of donation which did not expressly impose any burden (the
expressed consideration being purely one of liberality and generosity) but the recipient
actually paid charges imposed on the property like land taxes and installment arrearages
may be deemed onerous and thus governed by the law on ordinary contracts. -- NO

HELD.
A simple or pure donation is one whose cause is pure liberality (no strings attached), while an
onerous donation is one which is subject to burdens, charges or future services equal to or
more in value than the thing donated. Under Article 733 of the Civil Code, donations with an
onerous cause shall be governed by the rules on contracts; hence, the formalities required for a
valid simple donation are not applicable.

The Supreme Court ruled that the donation was simple, not onerous. Even conceding that
petitioner's full payment of the purchase price of the lot might have been a burden to him, such
payment was not however imposed by the donor as a condition for the donation.
It is clear that the donor did not have any intention to burden or charge petitioner as the donee.
Supreme Court agrees with the respondent Court that the payments made by petitioner were
merely his voluntary acts.

As a pure or simple donation, the donation is perfected from the moment the donor knows of the
acceptance by the donee (Art. 734 of the Civil Code). Acceptance of the donation by the donee
is, therefore, indispensable; its absence makes the donation null and void. Furthermore, if the
acceptance is made in a separate instrument, the donor shall be notified thereof in an authentic
form, and this step shall be noted in both instruments.

WHEREFORE, the petition is DENIED and the assailed Decision is AFFIRMED.

JGY
Cagaoan v. Cagaoan
43 Phil. 554

DOCTRINE: Applying the rules on double sale, if the one who first registered the property is in
bad faith, the one who first took possession of the property shall have the better claim.

FACTS:
Plaintiff Eugenio and the defendant Felix are brothers, the sons of Gregorio Cagaoan. Gregorio
executed a deed of gift of four parcels of land in Pangasinan, in favor of Felix. He executed a
similar deed in favor of Eugenio for a parcel of land which, apparently, is the same as that
described as parcel No. 4 in the deed of gift executed in favor of Felix. Both of the deeds of gift
are free from formal defects and were duly accepted by the donees.

Eugenio went into possession of the parcel donated to him but he failed to get the donation
recorded with the register of deeds. The deed given to Felix was duly recorded but he has never
had possession of parcel No. 4. Gregorio died.

Plaintiff Eugenio filed an action to be declared as the owner of the parcel donated to him. On the
other hand, Felix asks that he be awarded the possession of the land. The trial court rendered
judgment for the defendant

ISSUE:
WON the rules on double sale should be applied -- YES

HELD:
The case seems to use to be analogous to one where the same real property has been sold by
the same vendor to two difference vendees. In such cases, under article 1473 of the Civil Code,
the property goes to the vendee who first records his title in the registry of property. If the sale is
not recorded by either vendee, the property goes to the one who first takes possession of its in
good, faith, and in the absence of both record and possession, to the one who present oldest
title, provided there is good faith.

It clearly appears that Felix Cagaoan had full notice of the plaintiff's claim to the land before he
had his deed of gift recorded with the register of deeds. The plaintiff Eugenio Cagaoan having
first taken possession in good faith must therefore be considered to have the better right to the
land in question.

JRPA
JLT Agro v. Balasag
453 SCRA 211

DOCTRINE:
Well-entrenched is the rule that all things, even future ones, which are not outside the
commerce of man may be the object of a contract. The exception is that no contract may be
entered into with respect to future inheritance, and the exception to the exception is partition
inter vivos referred to in Article 1080.

FACTS:
Don Julian Teves contracted two marriages, first with Antonia Baena and had two kids namely
Josefa and Emilio. After her death, he married Milagros Teves and they had four children
namely: Maria Teves, Jose Teves, Milagros Teves and Pedro Teves. Thereafter, the parties to
the case entered into a Compromise Agreement.

When Antonia died an action for partition was instituted where the parties entered into a
Compromise Agreement which embodied the partition of all the properties of Don Julian. On the
basis of the compromise agreement, the CFI declared a tract of land known as Hacienda
Medalla Milagrosa as property owned in common by Don Julian and his two children of the first
marriage. The property was to remain undivided during the lifetime of Don Julian. Josefa and
Emilio likewise were given other properties at Bais, including the electric plant, the movie
property, the commercial areas, and the house where Don Julian was living. The remainder of
the properties was retained by Don Julian.

On 16 November 1972, Don Julian, Emilio and Josefa executed a Deed of Assignment of
Assets with Assumption of Liabilities in favor of J.L.T. Agro, Inc. (petitioner). Later, Don Julian,
Josefa and Emilio also executed an instrument entitled Supplemental to the Deed of
Assignment of Assets with the Assumption of Liabilities (Supplemental Deed) dated 31 July
1973. This instrument transferred ownership over Lot No. 63, among other properties, in favor of
petitioner. The appellate court ruled that the supplemental deed, conveying ownership to JLT
agro is not valid because the Compromise Agreement reserved the properties to Don Julians
two sets of heirs their future legitimes. The two sets of heirs acquired full ownership and
possession of the properties respectively adjudicated to them and Don Julian himself could no
longer dispose of the same. The appellate court in holding that the Supplemental Deed is not
valid, added that it contained a prohibited preterition of Don Julians heirs from the second
marriage.

ISSUE:
1. Was there preterition in the case?
2.
3. Whether or not the future legitime can be determined, adjudicated and reserved prior to
the death of Don Julian

HELD:

1. None. Manresa defines preterition as the omission of the heir in the will. In the
case at bar, Don Julian did not execute a will since what he resorted to was a partition inter
vivos of his properties, as evidenced by the court approved Compromise Agreement. Thus,
it is premature if not irrelevant to speak of preterition prior to the death of Don Julian in the
absence of a will depriving a legal heir of his legitime. Besides, there are other properties
which the heirs from the second marriage could inherit from Don Julian upon his death.

2. As a general rule, No. Well-entrenched is the rule that all things, even future
ones, which are not outside the commerce of man may be the object of a contract. The
exception is that no contract may be entered into with respect to future inheritance, and the
exception to the exception is partition inter vivos referred to in Article 1080. The partition
inter vivos of the properties of Don Julian is undoubtedly valid pursuant to Article 1347.
However, considering that it would become legally operative only upon the death of Don
Julian, the right of his heirs from the second marriage to the properties adjudicated to him
under the compromise agreement was but a mere expectancy. It was a bare hope of
succession to the property of their father. Being the prospect of a future acquisition, the
interest by its nature was inchoate. Evidently, at the time of the execution of the
supplemental deed in favor of petitioner, Don Julian remained the owner of the property
since ownership over the subject lot would only pass to his heirs from the second marriage
at the time of his death.

ABB
Roman Catholic Archbishop of Manila v. CA
198 SCRA 300

DOCTRINE: There is no need for prescription to be applied where a stipulation for automatic
reversion is expressly provided for in the terms of the deed of donation. Hence, there is no need
for a judicial declaration for the rescission of a contract because the law of the contract governs.

FACTS:
Private respondents spouses Eusebio de Castro and Martina Rieta executed a deed of donation
in favor of the Roman Catholic Archbishop of Manila covering a parcel of land wherein a
resolutory condition was imposed that donee shall not dispose or sell the property within a
period of one hundred (100) years from the execution of the deed of donation, otherwise would
render ipso facto null and void and such deed and property would revert back to donors.

However, prior to the exhaustion of the period of one hundred (100) years, the Bishop of Imus
executed a deed of absolute sale to spouses Florencio and Soledad Ignao for P114,000.00.
Rieta then filed a complaint for the nullification of the deed of donation, reconveyance of the
property with damages, and for the rescission of the contract.

Ignao, in his answer said that the action for the rescission of the contract and reconveyance of
the property has already prescribed.

ISSUE:
Whether or not the cause of action in the case at bar has already prescribed. -- NO

HELD:
As a general rule, article 764 of the New Civil Code provides that "(t)his action shall prescribe
after 4 years from the non-compliance with the condition, may be transmitted to the heirs of the
donor, and may be exercised against the donee's heirs. But in the case at bar, there is no need
for prescription to be applied where a stipulation for automatic reversion is expressly provided
for in the terms of the deed of donation. Hence, there is no need for a judicial declaration for the
rescission of a contract because the law of the contract governs. Judicial action is proper only
when there is absence of a special provision granting the power of cancellation.

However, the resolutory condition is held to be an undue restriction on the rights of ownership
and is contrary to public policy. A donation is an effective transfer of title over the property from
the donor to the donee. Once a donation is accepted, the donee becomes the absolute owner of
the property donated. Although the donor may impose certain conditions in the deed of donation,
the same must not be contrary to law, morals, good customs, public order and public policy. The
condition imposed must not be perpetual or for an unreasonable period of time.

FZC
Genato v. Lorenzo
23 SCRA 618

DOCTRINE: The delivery by the donor and the acceptance by done must be simultaneous and
the acceptance by a person other than the true done must be authorized by a proper power of
attorney set forth in a public document

FACTS:
The property under dispute in this case is the 530 shares of stocks of Genato Commercal
Corporation, which has P100 par value, of the deceased Simona B. De Genato (Director and
secretary-treasurer of the said company). The petitioners herein, 2 heirs of Simona, are claiming
that they own 530 shares of stocks of Genato Commercal Corporation because of the donation
made by Simona to them.
Respondents (other remaining heirs), however, are trying to recover from the petitioners, their
co-heirs, the said stocks so they can include it in the intestate estate which should later be
distributed among all the surviving children of the decedent.

Four or five days after having Florentino Genato elected and designated as Assitant Secretary-
Treasurer of the Corporation, 265 shares were issued in favour of Florentino Genato and
another 265 were issued in favour of Francisco G. Genato. These were not presented as
evidence in the course of the trial; they were merely mentioned by Florentino Genato in the
course of his testimony as a witness.

ISSUE:
Whether or not there was a valid donation? -- NO

HELD:
There was no valid donation for lack of proper acceptance. Incontestably, one of the two donees
was not present at the delivery, and there is no showing that Francisco Genato had authorized
his brother, Florentino to accept for both of them. The delivery by the donor and the acceptance
by done must be simultaneous and the acceptance by a person other than the true done must
be authorized by a proper power of attorney set forth in a public document. None has been
claimed to exist in this case.


LNAC
Cruz v. Court of Appeals
140 SCRA 245

DOCTRINE: In the case of the subsequent adoption of a minor by one who had previously
donated some or all of his properties to another, the donor may sue for the annulment or
reduction of the donation within four years from the date of adoption, if the donation impairs the
legitime of the adopted, taking into account the whole estate of the donor at the time of the
adoption of the child. Of course, the burden of proof is on the plaintiff-donor, who must allege
and establish the requirements prescribed by law, on the basis of which annulment or reduction
of the donation can be adjudged.

FACTS:
Eduvigis J. Cruz, a childless widow, donated a 235.5 sq.m residential lot in San Isidro, Taytay
Rizal together with the two-door apartment erected thereon to her grandnieces private
respondents herein, in a deed of donation entitled "Kasulatan Sa Kaloobpala". The property was
accordingly transferred to the names of private respondents.

Cruz judicially adopted Cresencia Ocreto, a minor, after which she extrajudicially tried to revoke
the donation, but the donees resisted, alleging that: (a) the property in question was co-owned
by Eduvigis Cruz and her brother, the late Maximo Cruz, grandfather of the donees, hence the
latter own 1/2 of the property by inheritance; and (b) Eduvigis Cruz owns another property, an
agricultural land of more than two hectares situated in Barrio Dolores, Taytay, Rizal, hence the
donation did not impair the presumptive legitime of the adopted child.

Petitioner filed a complaint against the donees for revocation of donation in the CFI.

Trial court rendered a decision revoking the donation.

On appeal, the CA reversed the trial court and dismissed the complaint.

Thus, prompted herein petition for review.

ISSUE:
Whether the CA correctly dismissed the complaint to annul the subject donation. -- YES

HELD:
In the case of the subsequent adoption of a minor by one who had previously donated some or
all of his properties to another, the donor may sue for the annulment or reduction of the donation
within four years from the date of adoption, if the donation impairs the legitime of the adopted,
taking into account the whole estate of the donor at the time of the adoption of the child. (Civil
Code, Articles 760, 761 and 763). Of course, the burden of proof is on the plaintiff-donor, who
must allege and establish the requirements prescribed by law, on the basis of which annulment
or reduction of the donation can be adjudged.

Unfortunately, in the case at bar, the complaint for annulment does not allege that the subject
donation impairs the legitime of the adopted child. Indeed it contains no indication at all of the
total assets of the donor. Nor is there proof of impairment of legitime. On the contrary, there is
unrebutted evidence that the donor has another piece of land (27,342 sq. m.) situated in
Dolores, Taytay, Rizal worth P273,420.00 in 1977, although then subject to litigation.

The legal situation of petitioner-donor, as plaintiff, is made worse by the factual finding of the CA
that the grandfather of the donees was the owner pro indiviso of one-half of the donated land,
the effect of which is to reduce the value of the donation which can then more easily be taken
from the portion of the estate within the free disposal of petitioner.

TKDC
De Luna v. Abrigo
181 SCRA 150

FACTS:
De Luna donated a portion of a 75 sq. m. lot to the Luzonian University Foundation. The
donation was embodied in a Deed of Donation Intervivos and was subject to certain terms and
conditions. In case of violation or non-compliance, the property would automatically revert to the
donor. When the Foundation failed to comply with the conditions, de Luna revived the said
donation by executing a Revival of Donation Intervivos with the following terms and conditions:

1) The Donee shall construct on the land and at its expense a Chapel, Nursery, and
Kindergarten School to be named after St. Veronica
2) Construction shall start immediately and must be at least 70% completed three years from
the date of the Deed unless the Donor grants extensions
3) Automatic reversion in case of violation

The Foundation accepted and the donation was registered and annotated in the TCT. By a
Deed of Segregation, the foundation was issued a TCT for area the lot donated while the
remaining area was retained by the De Luna.

The children and only heirs of the late De Luna (died after the donation) filed a complaint with
the RTC for the cancellation of the donation on the ground that the terms were violated. The
Foundation defended itself by saying that it had partially and substantially complied with the
conditions and that the donor granted it an indefinite extension of time to complete construction.

The RTC dismissed the petition on the ground of prescription (for being filed after 4 years). The
heirs did not file an MR and went straight to the SC.

ISSUE:
Whether the action prescribes in 4 years (based on art. 764 NCC-judicial decree of revocation
of the donation) or in 10 years (based on art. 1144 enforcement of a written contract)

HELD:
10 years
The donation subject of this case is one with an onerous cause.

Under the old Civil Code, it is a settled rule that donations with an onerous cause are governed
not by the law on donations but by the rules on contract. On the matter of prescription of actions
for the revocation of onerous donation, it was held that the general rules on prescription apply.
The same rules apply under the New Civil Code as provided in Article 733 thereof which
provides:

Donations with an onerous cause shall be governed by the rules on contracts, and
remuneratory donations by the provisions of the present Title as regards that portion which
exceeds the value of the burden imposed.

It is true that under Article 764 of the New Civil Code, actions for the revocation of a donation
must be brought within four (4) years from the non-compliance of the conditions of the donation.
However, said article does not apply to onerous donations in view of the specific provision of
Article 733 providing that onerous donations are governed by the rules on contracts. The rules
on prescription and not the rules on donation applies in the case at bar.

AMD
Central Phil. University v. CA
246 SCRA 511

DOCTRINE:
Onerous Donation: one executed for a valuable consideration which is considered the
equivalent of the donation itself, e.g., when a donation imposes a burden equivalent to the value
of the donation.

When a person donates land to another on the condition that the latter would build upon the
land a school, the condition imposed was not a condition precedent or a suspensive condition
but a resolutory one.

FACTS:
In 1939, Don Ramon Lopez, Sr., then a member of the Board of Trustees of CPU, executed a
deed of donation in favor of CPU over a parcel of land, with the following annotations

1. The land described shall be utilized by the CPU exclusively for the establishment and use
of a medical college with all its buildings as part of the curriculum;
2. The said college shall not sell, transfer or convey to any third party nor in any way
encumber said land;
3. The said land shall be called "RAMON LOPEZ CAMPUS", and the said college shall be
under obligation to erect a cornerstone bearing that name. Any net income from the land
or any of its parks shall be put in a fund to be known as the "RAMON LOPEZ CAMPUS
FUND" to be used for improvements of said campus and erection of a building thereon.

50 years later, private respondents, the heirs of Don Ramon Lopez, Sr., filed an action for
annulment of donation, reconveyance and damages against CPU alleging that the latter had not
complied with the conditions of the donation. They also argued that CPU negotiated with the
National Housing Authority (NHA) to exchange the donated property with another land. In its
answer CPU alleged that the right of private respondents to file the action had prescribed and it
denied any violation of the conditions in the deed of donation.

In 1991, the RTC ruled against CPU and ordered it to reconvey the property in favor of the heirs
of the donor. Petitioner appealed to the CA, which reversed the RTC decision. It ruled that the
annotations at the petitioner's certificate of title were resolutory conditions, breach of which
should terminate the rights of the donee thus making the donation revocable. The CA also ruled
that while the first condition mandated the use of the donated property for the establishment of a
medical school, the donor did not fix a period within which the condition must be fulfilled, hence,
until a period was fixed for the fulfillment of the condition, petitioner could not be considered as
having failed to comply with its part of the bargain. Hence, this petition for review on certiorari.

ISSUE:
W/N the donation was onerous and the conditions therein resolutory, although such conditions
were given no fixed period. -- YES

HELD:
Based on the conditions in the deed of donation, the donation was onerous. When Don Ramon
Lopez donated the parcel of land but imposed an obligation upon CPU to establish a medical
college thereon, the donation must be for an onerous consideration.

Under Art. 1181 of the Civil Code, in conditional obligations, the acquisition of rights, as well as
the extinguishment or loss of those already acquired, shall depend upon the happening of the
event which constitutes the condition. Thus, when a person donates land to another on the
condition that the latter would build upon the land a school, the condition imposed was not a
condition precedent or a suspensive condition but a resolutory one. It is not correct to say that
the school had to be constructed before the donation became effective, that is, before the donee
could become the owner of the land, otherwise, it would be invading the property rights of the
donor. The donation had to be valid before the fulfillment of the condition. If there was no
fulfillment or compliance with the condition, such as what obtains in the instant case, the
donation may now be revoked and all rights which the donee may have acquired under it shall
be deemed lost and extinguished.

The claim of petitioner that prescription bars the instant action of private respondents is
unavailing. When the obligation does not fix a period but from its nature and circumstances it
can be inferred that a period was intended, the general rule provided in Art. 1197 of the Civil
Code applies, which provides that the courts may fix the duration because the fulfillment of the
obligation itself cannot be demanded until after the court has fixed the period for compliance
and such period has arrived.

This general rule, however, cannot be applied considering the different set of circumstances
existing in the instant case. More than a reasonable period of 50 years has allowed petitioner to
avail of the opportunity to comply with the condition even if it be burdensome, to make the
donation in its favor forever valid. But, unfortunately, it failed to do so. Hence, there is no more
need to fix the duration of a term of the obligation when such procedure would be a mere
technicality and formality and would serve no purpose than to delay or lead to an unnecessary
and expensive multiplication of suits.

Records are clear and facts are undisputed that since the execution of the deed of donation up
to the time of filing of the instant action, petitioner has failed to comply with its obligation as
donee. Petitioner has slept on its obligation for an unreasonable length of time. Hence, it is only
just and equitable now to declare the subject donation already ineffective and, for all purposes,
revoked so that petitioner as donee should now return the donated property to the heirs of the
donor by means of reconveyance.

DISSENTING OPINION (Davide):
I agree with the view in the majority opinion that the donation in question is onerous considering
the conditions imposed by the donor on the donee which created reciprocal obligations upon
both parties. Beyond that, I beg to disagree.

First of all, may I point out an inconsistency in the majority opinion's description of the donation
in question. In one part, it says that the donation in question is onerous. Yet, later on it states
that the donation is basically a gratuitous one.

Second, the discussion on conditional obligations is unnecessary. There is no conditional
obligation to speak of in this case. It seems that the "conditions" imposed by the donor and as
the word is used in the law of donations is confused with "conditions" as used in the law of
obligations. In his annotation of Article 764 of the Civil Code on Donations, Arturo M. Tolentino
states clearly the context within which the term "conditions" is used in the law of donations, to
wit:
The word "conditions" in this article does not refer to uncertain events on which the birth or
extinguishment of a juridical relation depends, but is used in the vulgar sense of obligations or
charges imposed by the donor on the donee. It is used, not in its technical or strict legal sense,
but in its broadest sense.

Clearly then, when the law and the deed of donation speaks of "conditions" of a donation, what
are referred to are actually the obligations, charges or burdens imposed by the donor upon the
donee and which would characterize the donation as onerous. In the present case, the donation
is, quite obviously, onerous, but it is more properly called a "modal donation." A modal donation
is one in which the donor imposes a prestation upon the donee. The establishment of the
medical college as the condition of the donation in the present case is one such prestation.

The conditions imposed by the donor Don Ramon Lopez determines neither the existence nor
the extinguishment of the obligations of the donor and the donee with respect to the donation. In
fact, the conditions imposed by Don Ramon Lopez upon the donee are the very obligations of
the donation to build the medical college and use the property for the purposes specified in
the deed of donation. It is very clear that those obligations are unconditional, the fulfillment,
performance, existence or extinguishment of which is not dependent on any future or uncertain
event or past and unknown event, as the Civil Code would define a conditional obligation.

It is incorrect to say that the "conditions" of the donation in the present case are resolutory
conditions because, applying Article 1181 of the Civil Code, that would mean that upon
fulfillment of the conditions, the rights already acquired will be extinguished. Obviously, that
could not have been the intention of the parties. What the majority opinion probably had in mind
was that the conditions are resolutory because if they are not complied with, the rights of the
donee as such will be extinguished and the donation will be revoked. To my mind, though, it is
more accurate to state that the conditions here are not resolutory conditions but, for the reasons
stated above, are the obligations imposed by the donor.

Third, I cannot subscribe to the view that the provisions of Article 1197 cannot be applied here.
The conditions/obligations imposed by the donor herein are subject to a period. I draw this
conclusion based on our previous ruling in Barretto vs. City of Manila, in which we said that
when the contract of donation has no fixed period in which the condition should be fulfilled, the
provisions of what is now Article 1197 are applicable and it is the duty of the court to fix a
suitable time for its fulfillment. Indeed, from the nature and circumstances of the
conditions/obligations of the present donation, it can be inferred that a period was contemplated
by the donor. Don Ramon Lopez could not have intended his property to remain idle for a long
period of time when in fact, he specifically burdened the donee with the obligation to set up a
medical college therein and thus put his property to good use. There is a need to fix the duration
of the time within which the conditions imposed are to be fulfilled.

It is also important to fix the duration or period for the performance of the conditions/obligations
in the donation in resolving the petitioner's claim that prescription has already barred the
present action. I disagree once more with the ruling of the majority that the action of the
petitioners is not barred by the statute of limitations. The mere fact that there is no time fixed as
to when the conditions of the donation are to be fulfilled does not ipso facto mean that the
statute of limitations will not apply anymore and the action to revoke the donation becomes
imprescriptible.

Admittedly, the donation now in question is an onerous donation and is governed by the law on
contracts (Article 733). Accordingly, the decision of the Court of Appeals must be upheld, except
its ruling that the conditions of the donation are resolutory.

CRF
Parks v. Province of Tarlac
49 Phil. 142

DOCTRINE: A condition which cannot be complied with except after giving effect to the
donation is not a condition precedent. (batasnatin)

FACTS:
On October 18, 1910, Concepcion Cirer and James Hill donated a parcel of land perpetually to
the municipality of Tarlac, Province of Tarlac, under certain conditions(erection of a public
school within six months) specified in the public document in which they made this donation
which was accepted through its municipal president and was transferred in the name of the
done. Later on, the donor sold this parcel to the herein plaintiff George L. Parks. On August 24,
1923, the municipality of Tarlac transferred the parcel to the Province of Tarlac.
The plaintiff, George L. Parks, alleging that the conditions of the donation had not been
complied with prayed that he be declared the absolute owner entitled to the possession of this
parcel, that the transfer of the same by the municipality of Tarlac to the Province of Tarlac be
annulled, and the transfer certificate issued to the Province of Tarlac cancelled.

ISSUE:
Whether or not the donation was coupled with a condition precedent? W/N the action to revoke
has prescribed? -- NO

HELD:
Appellant contends that a condition precedent having been imposed in the donation and the
same not having been complied with, the donation never became effective. We find no merit in
this contention. The appellant refers to the condition imposed that one of the parcels donated
was to be used absolutely and exclusively for the erection of a central school and the other for a
public park, the work to commence in both cases within the period of six months from the date
of the ratification by the parties of the document evidencing the donation. It is true that this
condition has not been complied with. The allegation, however, that it is a condition precedent is
erroneous. The characteristic of a condition precedent is that the acquisition of the right is not
effected while said condition is not complied with or is not deemed complied with. In the present
case the condition that a public school be erected and a public park made of the donated land,
work on the same to commence within six months from the date of the ratification of the
donation by the parties, could not be complied with except after giving effect to the donation.
The donee could not do any work on the donated land if the donation had not really been
effected, because it would be an invasion of another's title, for the land would have continued to
belong to the donor so long as the condition imposed was not complied with.

MPF
Austria-Magat v. CA
375 SCRA 556

DOCTRINE: Whether the donation is inter vivos or mortis causa depends on whether the donor
intended to transfer ownership over the properties upon the execution of deed. When the deed
of donation provides that the donor will not dispose or take away the property donated, he in
effect is making a donation inter vivos.

FACTS:
Basilisa Comerciante is a mother of five (5) children, namely, Rosario Austria,
Consolacion Austria, herein petitioner Apolinaria Austria-Magat, Leonardo(deceased), and
one of herein respondents, Florentino Lumubos. In 1953, Basilisa bought a parcel of
residential land together with the improvement thereon covered and described in Transfer
Certificate of Title No. RT-4036 (T-3268). On December 17, 1975, Basilisa executed a
document designated as Kasulatan sa Kaloobpala (Donation). The said document which
was notarized by Atty. Carlos Viniegra, reads as follows:
Na ako, si BASELISA COMERCIANTE, may sapat na gulang, Filipina, balo, at
naninirahan sa blg. 809 L. Javier Bagong Pook, San Antonio, Lungsod ng Kabite,
Filipinas, sa pamamagitan ng kasulatang itoy. xxx Na alang-alang sa mabuting
paglilingkod at pagtingin na iniukol sa akin ng apat kong mga tunay na anak na sila:
Kusang loob na ibinibigay ko at ipinagkakaloob ng ganap at hindi na mababawi sa
naulit ng apat na anak ko at sa kanilang mga tagamagmana (sic), ang aking isang
lupang residential o tirahan sampu ng aking bahay nahan ng nakatirik doon na nasa
Bagong Pook din, San Antonio, Lungsod ng Kabite, at nakikilala bilang Lote no. 7,
xxx Na ang Kaloob palang ito ay magkakabisa lamang simula sa araw na akoy
pumanaw sa mundo, at sa ilalim ng kondision na: Magbubuhat o babawasin sa
halaga ng nasabing lupa at bahay ang anumang magugul o gastos sa aking libing at
nicho at ang anumang matitira ay hahatiin ng APAT na parte, parepareho isang
parte sa bawat anak kong nasasabi sa itaas nito upang maliwanang (sic) at walang
makkalamang sinoman sa kanila; At kaming apat na anak na naakalagda o nakadiit
sa kasulatang ito ay TINATANGGAP NAMIN ang kaloob-palang ito ng aming
magulang na si Basilisa Comerciante, at tuloy pinasasalamatan namin siya ng taos
sa (sic) puso dahil sa kagandahan look (sic) niyang ito sa amin. Xxx (sgd.)
Basilisa and her said children likewise executed another notarized document
denominated as Kasulatan which is attached to the deed of donation. On February 6, 1979,
Basilisa executed a Deed of Absolute Sale of the subject house and lot in favor of herein
petitioner Apolinaria Austria-Magat for Five Thousand Pesos (P5,000.00). As the result of
the registration of that sale, TCT No. RT-4036 in the name of the donor was cancelled and
in lieu thereof TCT No. T-10434 was issued in favor of petitioner Apolinaria Austria-Magat
on February 8, 1979.
On September 21, 1983, herein respondents (representing their deceased
mother Consolacion Austria), Ricardo, Mamerto and Segunda, all surnamed Sumpelo
(representing their deceased mother Rosario Austria) and Florentino Lumubos filed before
the Regional Trial Court of Cavite an action against the petitioner for annulment of TCT No.
T-10434 and other relevant documents, and for reconveyance and damages.
ISSUE:
Whether or not the donation was inter vivos. -- YES

HELD:
The decisive proof that the deed is a donation inter vivos is in the provision that: Ibinibigay ko at
ipinagkakaloob ng ganap at hindi mababawi sa naulit na apat na anak ko at sa kanilang mga
tagapagmana, ang aking lupang residential o tirahan sampu ng aking bahay nakatirik doon xxx.
This is a clear expression of the irrevocability of the conveyance. The irrevocability of the
donation is a characteristic of a donation inter vivos. By the words hindi mababawi, the donor
expressly renounced the right to freely dispose of the house and lot in question. The right to
dispose of a property is a right essential to full ownership. Hence, ownership of the house and
lot was already with the donees even during the donors lifetime. xxx

In the attached document to the deed of donation, the donor and her children stipulated that:
Gayon din ang nasabing titulo ay hindi mapapasangla o maipagbibili ang lupa habang may
buhay ang nasabing Basilisa Comerciante.xxx The stipulation is a reiteration of the irrevocability
of the dispossession on the part of the donor. On the other hand, the prohibition to encumber,
alienate or sell the property during the lifetime of the donor is a recognition of the ownership
over the house and lot in issue of the donees for only in the concept of an owner can one
encumber or dispose a property. The express irrevocability of the same (hindi na mababawi) is
the distinctive standard that identifies that document as a donation inter vivos. The other
provisions therein which seemingly make the donation mortis causa do not go against the
irrevocable character of the subject donation.

Thus, the court arrived at no other conclusion in that the petitioners cited provisions are only
necessary assurances that during the donors lifetime, the latter would still enjoy the right of
possession over the property; but, his naked title of ownership has been passed on to the
donees; and that upon the donors death, the donees would get all the rights of ownership over
the same including the right to use and possess the same.

Furthermore, it also appeared that the provision in the deed of donation regarding the
prohibition to alienate the subject property is couched in general terms such that even the donor
is deemed included in the said prohibition (Gayon din ang nasabing Titulo ay hindi
mapapasangla o maipagbibili ang lupa habang maybuhay ang nasabing Basilisa Comerciante).
Both the donor and the donees were prohibited from alienating and encumbering the property
during the lifetime of the donor. If the donor intended to maintain full ownership over the said
property until her death, she could have expressly stated therein a reservation of her right to
dispose of the same. The prohibition on the donor to alienate the said property during her
lifetime is proof that naked ownership over the property has been transferred to the donees. It
also supports the irrevocable nature of the donation considering that the donor has already
divested herself of the right to dispose of the donated property. On the other hand, the
prohibition on the donees only meant that they may not mortgage or dispose the donated
property while the donor enjoys and possesses the property during her lifetime. However, it is
clear that the donees were already the owners of the subject property due to the irrevocable
character of the donation.

JPOT
Eduarte v. CA
253 SCRA 391

DOCTRINE: All crimes which offend the donor are considered manifests of ingratitude and are
cause for revocation of donation.

FACTS:
Pedro Calapine donates half a parcel of his land to his niece, Helen Doria. For this benevolent
act, he is blessed tenfold by greed and disloyalty. He willingly and knowingly gave only half of
said land. However, there materialized a donation from him supposedly as well giving the other
half to make whole said parcel. Moreover, these lands "donated" were made for profit and
ironically enough, spirituality. This is when the Eduartes Romulo and Sally fall prey also.
Furthermore, a certain Calauan Christian Reform Church (CCRC) enters the fray. The niece,
Helen, had the audacity to falsify said donation and donate yet again to attain false pretense of
forgiveness. This, by way of giving the land to a church except of course the residence. The
Eduartes, unknowingly trust Doria and purchase the lot altogether. Pedro then moves to
reprimand his niece by setting forth machinations to effectively revoke his donation.

ISSUE:
W/N the act of Helen Doria of falsification of documents is tantamount to ingratitude towards
Pedro Calapine which would lead to the effective revocation of donation? -- YES

HELD:
This act is of pure treason. Any action that the donor takes offense to is equal to ingratitude. It is
saddening that an individual can have such a shameful display of thanks relating to family. The
mere act of falsifying documents to reserve the right to half the land to which you have already
been granted half of is appaling to say the least.

WHEREFORE, the instant petition for review is DENIED. The assailed decision of the Court of
Appeals dated August 31, 1993, is AFFIRMED.
Costs against petitioners.

JPOT
Noceda v. CA
313 SCRA 504

DOCTRINE: Usurpation with regard to donee towards donor is a definite act of ingratitude and
neecessitates only to be proved to effect revocation.

FACTS:
Aurora Directo, Rodolfo Noceda and Maria Arbizo extra-judicially partitioned a land in Zambales.
Aurora coincidentally donated on the same day of the partition to Rodolfo who happens to be
her nephew. After several surveys conducted by a certain Geodetic Engineer Eugene Quejada
of the Bureau of Lands and specific boundary adjustments, Aurora opted to fence her property
accordingly. This, for reasons of making known and presuppose which of the vast land is indeed
in her rightful possession. Even though there was an apparent exercise of giving and safety
measures so as to avoid conflict, it appears to have been averred by Rodolfo. It was made true
by his actions of passing through said fences and intentionally staying at nipa huts designated
inside Aurora's land. What comes into contention is the manner Rodolfo makes his action.

ISSUE:
W/N usurpation of Rodolfo is enough to suffice revocation of donation? --

RULING:
In this case, Rodolfo is directly showing his disregard of Aurora's wishes intentionally going
beyond the boundaries Directo so painstakingly made plain. Usurpation is the undermining of
one's authority. Noceda not only negates her will but makes it obvious in the sense of flaunting it
even with the presence of Maria Arbizo.

We find that both the trial court and the respondent Court had carefully considered the
questions of fact raised below and the respondent Courts conclusions are based on the
evidence on record. No cogent reason exists for disturbing such findings. We also note that
petitioner in this petition merely rehashed the same issues and arguments raised in the
respondent Court in whose decision we find no reversible error. Clearly, petitioner failed to
present any substantial argument to justify a reversal of the assailed decision.
WHEREFORE, the petition for review is hereby DENIED. Costs against appellant.
SO ORDERED.

Das könnte Ihnen auch gefallen